Other Exam Questions

Pataasin ang iyong marka sa homework at exams ngayon gamit ang Quizwiz!

Dun cardio In older patients, beta blockers have decreased effect. Which of the following mechanisms best explains this change in effect? A. Decreased density of beta adrenergic receptors. B. Decreased endogenous noradrenaline. C. Decreased liver metabolism. D. Decreased renal excretion. E. Reduced intracellular cyclic adenosine monophosphate (cAMP).

A

Dun cardio The hyponatraemia of heart failure is predominantly due to: A. increased anti-diuretic hormone. B. diuretic induced sodium loss. C. secondary hyperaldosteronism. D. renin-induced thirst. E. increased atrial natriuretic peptide.

A

Dunedin Haem Which is the following is true about haemophilia A. Haemophilia A is sex-linked; haemophilia B is autosomal B. Allo-antibody inhibitors develop in about 30% of severe haemophiliacs C. Mild haemophilia is defined when Factor level = 1-10% D. FEIBA stands for Factor eight inhibitor binding assay E. FVIII gene therapy is unlikely because the FVIII gene is too long for an AAV vector

B

Dunedin Resp In the establishment of low dose CT screening program for lung cancer which of the following statements is most correct? A. People with previous lung cancers should be included. B. Adenocarcinoma is the most common tumour detected. C. Overdiagnosis is eliminated by use of CT measurement of nodule volume D. The reduction in lung cancer mortality is seen in stage III & IV disease.

B

Dunedin endo IN PREGNANCY THE FOLLOWING ARE CONSIDERED TO BE NORMAL CHANGES IN THYROID FUNCTION? A. βHCg will reduce levels of free thyroid hormones B. βHCg acts like TSH in early pregnancy C. fully suppressed TSH with marginally elevated free thyroid hormones D. TSH will elevate in early pregnancy E. There is no expected change

B

Dunedin endo Question 6: Which of the following is a marker of bone resorption? A. Alkaline phophatase B. Urine N-telopeptide C. Osteocalcin D. Procollagen C-terminal propeptide E. Procollagen N-terminal propeptide

B

Dunedin endo Which of the following genetic loci is associated with the greatest future risk of Type 1 diabetes? A: PTPN22 B: HLA Class II loci C: INS D: CTLA-4 E:HLA Class I loci

B

DM Mock 45. Which of the following is LEAST consistent with subclinical adrenal Cushing's Syndrome? A. Presence of a 2 cm adrenal adenoma with Hounsfield units of 5 B. Increased DHEAS C. Decreased ACTH D. Serum Cortisol of 120 nmol/L following ingestion 1 mg dexamethasone at 10 pm the evening before the test

45. ANS: B Given ACTH stimulates DHEAS and given that ACTH tends to be suppressed in subclinical adrenal Cushings' syndrome, DHEAS is typically low.

DM Mock 49. A 42-year-old female presents to pre-operative assessment clinic for preparation for an elective total knee replacement. Her history is significant for obesity, for which she underwent intensive dietary change and bariatric surgery 3 years prior. She remains on antihypertensives and statin for metabolic syndrome though successfully stopped oral hypoglycaemics A coagulation profile is performedPlatelets 160x109/L (150-400) INR 1.2 (0.9-1.2) APTT 49 secs (24-35) Fibrinogen 2.1 (2-4) APTT 50:50 42 secs What is the most likely cause? A. Vitamin K dependent factor deficiency post bariatric surgery B. Mild Haemophilia A C. Von Willebrand's disease D. Lupus anticoagulant

49. ANS: D The APTT examines the intrinsic and common pathways of the coagulation cascade by introduction of phospholipids to platelet poor plasma (PPP). In the presence of vitamin K dependant factor deficiency or factor VIII/vWF deficiency, the APTT will be prolonged, and the addition of normal plasma should correct this. The lupus anticoagulant is an antibody against phospholipid, leading to prolonged APTT that does not correct on introduction of normal plasma.

DM Mock 56. Vijay is 20 years old and complaining of breathlessness. An echocardiogram is performed and shows a dilated right heart and pulmonary hypertension. A right heart catheterization is performed, and the following pressures and oxygen saturations are obtained from multisite sampling. Pressure (mmHg) 02 saturation SVC 10 67% IVC 10 73% RA 10 84% RV 30/10 80% Main PA 30/12 80% What is the diagnosis? A. Atrial septal defect B. Post-capillary pulmonary hypertension C. Chronic thromboembolic pulmonary hypertension D. Persistent left superior vena cava

56. ANS: A There is an obvious step-up in O2 saturations from the SVC and IVC to the RA. This implies there must be a shunt supplying oxygenated blood present.

DM Mock 6. Which of the following is NOT a supportive feature for a diagnosis of Dementia with Lewy Bodies (DLB)? A. Autonomic dysfunction B. Fluctuating cognition C. Poor cognitive response to cholinesterase inhibitor D. Hypersensitivity to typical antipsychotics

6. ANS: C. Meta-analysis revealed that donepezil and rivastigmine were superior to placebo in improving cognitive function in DLB, and in fact DLB pts may even have a better response than AD pts

DM Mock 62. A 65-year-old female is being treated with combined immunochemotherapy (rituximab, cyclophosphamide, vincristine, dexamethasone) for low grade lymphoma. After an episode of febrile neutropenia post cycle 2, pegylated filgrastim is added for growth factor support. After four cycles of chemotherapy, she develops numbness and tingling in the tips of all digits of her hands. Which agent is most likely to be the cause? A. Dexamethasone B. Cyclophosphamide C. Filgrastim D. Vincristine

62. ANS: D Peripheral neuropathy is a known and common side effect of vinca based chemotherapy agents.

DM Mock 64. A deficiency in the classical components of the complement pathway is associated with autoimmunity and lupus, due to which of the following? A. Increased deposition of C3 on tissue B. Reduced immune complex clearance C. Development of antibodies to C1q D. Reduced opsonisation, phagocytosis and killing of organisms

64. ANS: B The classical complement pathway is triggered by antibodies (IgM/IgG1/IgG3) and is important in immune complex clearance. Deficiency thus is associated with lupus.

DM PT R9 9. A 34-year-old s 95 kg smoker is noted to have microscopic haematuria during a work health check. His blood pressure is 118/72mmHg, and otherwise examination is unremarkable. His creatinine is 80 µmol/L, albumin 41g/L and no proteinuria and ACR is normal. The rest of his glomerulonephritis screen is normal. Renal ultrasound shows 2 normal sized kidneys without obstruction. What is the most appropriate next step? A. Commence RAS blockade B. Discharge from clinic C. Refer for cystoscopy D. Renal biopsy

9. ANS: C • Microscopic hematuria smoker, normal renal function, no proteinuria, screens normal need to exclude underlying malignancy - bladder Glomerular causes of hematuria Thin membrane disease Positive family history, autosomal dominant IgA nephropathy • RBC morphology - glomerular RBC may be helpful

Deltamed stats A test has a sensitivity of 95% and a specificity of 80%. It is used to screen for a condition with a prevalence of 1 in 100 What will the positive predictive value be nearest too? A. 0.2% B. 0.5% C. 1% D. 2% E. 5%

?

Dunedin Gastro A 50-year old woman is found to have the lesions (divert) as seen below as a screening colonoscopy for colo-rectal cancer. She questions the significance of this finding. What can you tell her? A. Her annualized risk of diverticulitis is higher than a 60-year old found to have the same lesions B. Data consistently indicate that a high fiber diet can reduce her risk of diverticulitis C. Data indicate that avoiding nuts and seeds can reduce her risk of diverticulitis D. Compared to those without these lesions, she is at an increased risk of developing Crohn's disease.

?

Dunedin Gastro A 61-year old woman presents with diarrhea for 1 year. She reports 5-6 watery stools daily, episodic incontinence and night time stools. While she also reports some mild abdominal cramping, she denies significant abdominal pain, weight loss or blood in her stools. Past medical history is significant for hypertension for with she is treated with Bendrofluazide. No relevant travel or family history. Examination is normal. Anti-TTG antibodies, CRP, FBC are unremarkable. Colonoscopy is normal but biopsies are taken: H&E stain Trichrome stain Management: A. Prednisone B. Budesonide C. Mesalamine D. Cholestyramine E. Bismuth salicylate

??

CLS ID A 25 year old male presents with acute onset diarrhoea passing 10 stools per day. Stool microscopy shows leukocytes and erythrocytes. He has not had any recent travel. The most likely infectious agent is: A. Campylobacter jejuni B. Clostridium perfringens C. Enterotoxogenic E.Coli D. Giardia E. Rotovirus

??A Noro > NSTEC E.coli > campylo

CLS Rheum A 52 year old woman with RA has inability to extend her 4th and 5th fingers A. Extensor tendon rupture B. Radial nerve palsy C. Dupuytren's contracture D. Metacarpal joint deformity

A

CLS ID Which of the following antimicrobial agents involves binding to bacterial ribosome? A. vancomycin B. moxifloxacin C. meropenem D. metronidazole E. linezolid

Linezolid (E) moxiflox - DNA gyrase vancomycin - D-A-DA in cell wall meropenem - cell wall synthesis metronidazole - inhibits protein synthesis linezolid - inhibits protein synthesis

Dunedin gastro 77 yr man. Warfarin for non-valvular AF. One previous TIA. Hypertension with CKD - frusemide, bisoprolol, cilazapril. Presents with increasing dyspnoea, mild chest pain. Gives a 1 week history of melaena, following naproxen use for recent shoulder injury. Hb 78 (previous month HB 138) INR 5.8 Clinically AF 130 BP 110/56, some postural dizziness, e-GFR 21. Stopped warfarin and NSAID, transfused. Best ongoing pre-endoscopy management? a. IV vitamin K, scope once INR ≤2.5 b. PO vitamin K + PCC (prothrombin complex concentrate) - scope once INR <1.5 c. IV vitamin K + PCC (prothrombin complex concentrate)- scope immediately d. PO vitamin K and await INR <1.5 e. PO vitamin K + PCC (prothrombin complex concentrate) + tranexamic acid, scope when INR ≤ 2.5

A

Dunedin gastro Normal portal pressure is: a)< 5 mm Hg b)15 mm Hg c) 25 mm Hg d)30 mm Hg e)60 mm Hg

A

Which of the following is an absolute contraindication to the use of rivaroxaban in the treatment of pulmonary embolism? A. Pregnancy. B. Antiphospholipid syndrome. C. Recent (within 6 weeks) haemorrhagic stroke. D. Malignancy.

A

Dun Neuro In complex partial seizures of temporal lobe type, the most common finding at surgery for the epilepsy is: A. mesial temporal sclerosis B. neuronal migration disorder C. old infarction D. normal brain E. cavernous angioma

A (selected for surgery meed abnormality that can be reversed)

Dunedin Haem An adult from South East Asia with Hb 110 g/L and MCV 68 fL is most likely to have: A. Alpha thalassaemia trait B. HbH disease C. Homozygous HbD D. Beta thalassaemia major E. Beta thalassaemia minor

A A more common in Asia

Dunedin gastro A 45 year old man has Duke's C adenocarcnioma of the colon. Strong family history of colon cancer in the absence of polyps and sister with cancer of the ovary. Most likely genetic basis is? A. mismatch repair gene B. Ras C. chromosomal deletion D. P glycoprotein gene E. APC gene

A Lynch

DM Resp A 69 year old male with a history of stable chronic lymphocytic leukemia presents to the clinic complaining of 5 year history of cough, dyspnea and production of copious amounts of foul smelling sputum. Physical examination reveals crackles at the lung bases. Chest xͲray shows dilated and thickened bronchi that appear as ringͲlike markings. Which of the following organisms is most likely to be identified on sputum culture? A. Haemophilus influenzae B. Mycobacterium avium complex C. Staphylococcus aureus D. Stenotrophomonas maltophilia

A.

Deltamed onc 29 y/o M presents with painless testicular lump with bulky R retroperitoneal lymphadenopathy Which is most consistent with pure seminoma A. normal AFP, high hCG, high LDH B. high, normal, normal C. high, normal, high D. high, high, high

A.

CLS Haem 36 healthy man with no siblings. In addition to telling pt Dx, which is most appropriate? Haemoglobin 111 g/dL 128-175 RCC 5.6 x 10^12/L 4.5-6.5 MCV 62fL 80-96 Total WCC 6.4 x 10^9/L 4-8 Platelets 350 x 10^12/L 150-450 Ferritin 29ng/ml 20-300 Serum Iron 6 ug/dL 9-27 Transferrin 3.1mg/dL 2.3-3.9 Transferrin saturation 23% 15-50% HPLC Confirms B thalassaemia trait A. Globin gene analysis. B. Provide information on partner screening. C. Commence iron therapy. D. Haemoglobinopathy screening for parents E. Gastrointestinal endoscopy.

B

CLS Haem Infusion of cryoprecipitate is principally indicated for replacement of: A. Factor IX. B. Fibrinogen. C. Immunoglobulin A. D. Antithrombin. E. Protein C.

B

CLS ID 42 y/o recently diagnosed with HIV after 6m sx. CD4 0.03, viral load >100,000. Blood cx positive for MAC. Commences ART and rifabutin, ethambutol and clarithromycin. Sx settle then 4 weeks later = fevers, malaise, LNs. Why? A. NHL B. IRIS C. Barotenalla D. Syphilis E. Toxo

B

Dunedin gastro H. pylori infection is associated with a decreased risk of which one of these conditions? a. Idiopathic iron deficiency anaemia b. Oesophageal adenocarcinoma c. Gastric MALT (mucosa-associated lymphoid tissue) lymphoma d. Idiopathic/immune thrombocytopaenia e. Gastric adenocarcinoma

B

Dunedin gastro A 35 year patient known to be HIV positive and who now has a CD4 count of 150/ml has had pain on swallowing for the past week. Sharply circumscribed 0.3-0.8cm ulcers are seen on upper GI endoscopy in the lower oesophagus. These ulcer are probably due to an infection with: a. Helicobacter pylori b. Candida albicans c. Herpes simplex d. Mycobacterium avium-complex e. Cytomegalovirus

C

Dunedin gastro What is the leading cause of death in a person who has had a liver transplant? a) Rejection b) Cardiovascular disease c) Infection d) Recurrence of primary liver disease e) Malignancy

C

CLS ID An unwell 72 year old patient with NHL treated with R-CHOP a week ago presents to the ED. The patient is observed to have a temperature of 38.7, tachypnoea, tachycardia and hypotension. A FBC performed yesterday was abnormal; Hb 110, PLT 80 and Neut 0.2. Which of the following empiric antibiotics is most appropriate? A. Amoxicillin clavulanate B. Ceftriaxone + roxithromycin C. Ciprofloxacin D. Piperacillin tazobactam E. Vancomycin + gentamicin

D

Dunedin Resp Which of the following is not expected in idiopathic pulmonary fibrosis? A. Honeycomb changes in high resolution CT chest scan. B. Low pulmonary gas transfer factor (DLco) on lung function tests. C. Basal predominant disease on chest x-ray. D. Predominant lymphocyte infiltrate on lung biopsy.

D

Dunedin endo Question 6: A 63-year-old woman presents with weight gain, reduced energy and depression, five years after pituitary surgery and radiotherapy for a non-functioning pituitary adenoma. Current medications are thyroxine 0.15 mg/day, hydrocortisone 20 mg/day (equivalent to cortisone acetate 25 mg/day) in divided doses, conjugated oestrogen 0.625 mg/day, medroxyprogesterone 2.5 mg/day, and fluoxetine 20 mg/day. Examination is unremarkable except for truncal obesity. Her body mass index (BMI) is 30 kg/m2 [20- 25] and her blood pressure is 145/85 mmHg. Which one of the following is the most likely cause of her weight gain? A. Inadequate thyroxine replacement dose. B. Excess hydrocortisone replacement dose. C. Excess oestrogen replacement dose. D. Growth hormone deficiency. E. Fluoxetine therapy.

D (on appropriate doses of thyroxine, hydrocort etc)

Dun ID A 36 yo HIV+ male with CD4 count 250, VL 800,000 is seen and is ready to start ART. He is HLAB5701 +. He has a significant PMH for psychiatric illness with several suicide attempts. Which of the following is the best treatment option: A. tenofovir/emtricitabine/efavirenz B. abacavir/lamivudine + dolutegravir C. lamivudine + dolutegravir D. tenofovir/emtricitabine + dolutegravir

D 5701 so can't use abacavir high VL so can't do double rx TEE = suicidality

Dun Neuro Pred is not a recognised rx for which of the following? A MG B Temporal arteritis C Polymyositis D Neurosarcoid E Guillian-Barre

E CIDP = yes to roids

Deltamed rheum Which of the following is not associated with Psoriatic Arthritis? a) Isolated DIP joint involvement b) Sacro-iliitis c) Family history of psoriasis d) Dactylitis e) Mid-point Achilles tendinitis

E. not midpoint. lower point (where it attaches)

Deltamed resp Which of the following findings has the best negative predictive value for a ruling out a diagnosis of asthma in a patient with current symptoms of dyspnoea and cough • Negative mannitol bronchoprovocation • Lack of significant response to salbutamol on spirometry • Lack of significant response to salbutamol on PEFR • Low (<10ppb) fraction of exhaled nitric oxide • Negative methacholine bronchoprovocation

Negative metacholine = sensitive (e for met) Negative mannitol = specific Low FEno = only eosinophilic asthma

Deltamed Rheum All of the following have been assoc with the risk of developing RA except DRB1 PTPN22 STAT3 STAT4 PADU

Stat3

Deltamed Rheum 65 y/o. 10 yrs RA on MTX humira, pred. Elective THJR. Which statement is true? a. All DMARDS except pred should be ceased 2w preop b. Pre + periop renal function should be checked + MTX reduced or wittheld if any deterioration if eGFR c. Humira should be w/h for 3ms preop d Pred dose should be doubled before surgery e Risk of periop infection is increased

b + E

Dun cardio How do the following change with age ie. icnrease or decrease? Aortic diameter, aortic distensibility, aortic velocity

increased decreased (stiffer) faster

Deltamed Rheum Which of the following is a chimeric Ab to TNF etanercept rituximab adalimumab infliximab anakinra abatacept

infliximab. I for IV fusion chimeric, CD20 humanised, TNF chimeric, TNF

Haemodynamics CLS lecture Case: The following oxygen saturations were obtained during cardiac catheterization from a patient with a suspected shunt. Superior vena cava 62% Inferior vena cava 65% Right atrium 65% Right ventricle 78% Pulmonary artery 80% Pulmonary vein 95% Aorta 95% Which of the following is the correct location of the shunt? A. ASD B. VSD C. PDA D. Coronary artery fistula E. Lung AVM

vsd

Deltamed gastro lecture 1 The most common precipitant of oesophageal sq cell cancer is A. smoking B achalasia C. Barrett's D. Low fibre diet E. reflux oesophagitis

A. smoking SqC: smoking, achalasia, alcohol Adeno: M, white, GORD, smoking, Barret's H.pylori protective for adeno

Deltamed immuno Which of the following is associated with HLA‐DR antigens A Examples of Class II antigens of the MHC complex B Normally associated with 2 microglobulin on the cell surface C Normally expressed on most body cells D Expressed on resting T lymphocytes E Examples of Class I antigens of the MHC complex

A 2 letters = MHCII Normally expressed on APCs only Not on resting lymphocytes, only active

DM Resp Which of the following treatmentshas been demonstrated in RCT's to assist with steroid weaning in ABPA? A. Itraconazole • Antifungal B. Lebrikizumab • AntiͲILͲ13 C. Mepolizumab • AntiͲILͲ5 D. Pirfenidone • AntiͲfibrotic E. Rituximab • Anti CDͲ20 (BͲcell)

A. itraconazole

The least likely medication to contribute to the development of diabetes after transplantation is: a. mycophenolate b. rapamycin c. tacrolimus d. Cyclosporin e. prednisone

A. mycophen tac worse than cyclo tic tacs are surgary

deltamed renal What is the main factor responsible for systolic hypertension in the elderly? A. Impaired renal perfusion. B. High cardiac output. C. Sympathetic activation. D. Reduced arterial compliance. E. High peripheral resistance.

D.

CLS ID Your local hospital guideline for empiric treatment of acute pyelonephritis (after cultures) recommends IV Gentamicin 5mg/kg LBW every 24 hours. Which of the following is the most relevant pharmacodynamic parameter to this dosing regimen of Gentamicin? 1. Cmax/MIC 2. T>MIC 3. AUC/MIC 4. 0.7 x (Vd/CL) 5. All of the above

1.

Deltamed Onc Which of the following endocrine drugs and side effects are not commonly associated 1. Tamoxifen - Osteoporosis 2. Palbociclib - neutropenia 3. Letrozole - arthralgia 4. Goserelin - Hot flushes

1. tamoxifen = ER agonist on bone + uterus. antag for others

DM PT H10 10. A 72-year-old male with multiply relapsed multiple myeloma presents with severe fatigue and nose bleeds to the Emergency Department. The patient has recently returned from an overseas trial and recalls having received an 'immune therapy through the vein' at the treatment facility. You do not have immediate access to patient records regarding this trial. A routine FBE reveals he has anaemia at 64g/L and thrombocytopenia at 14x109 /L. You request a crossmatch for 2 units of packed red blood cells and a pool of platelets. The Transfusion Medicine scientist calls you. They report the patient has a panagglutinating antibody screen with a negative auto-control and negative DAT. What medication is the patient taking? A. Carfilzomib B. Pomalidomide C. Rituximab D. Daratumumab

10. ANS: D • This patient has the typical blood screen panel findings of an anti-CD38 monoclonal antibody such as daratumumab. • CD38 is expressed on red blood cells- in the presence of an anti-CD38 monoclonal antibody in the patient's plasma there will be pan-agglutination of an antibody screen. • Carfilzomib and pomalidomide are not monoclonal antibodies (they are related to thalidomide) • Rituximab is an anti-CD20 monoclonal antibody used in the treatment of lymphoid malignancies.

Deltamed Rheum What cell is most critical for erosions to develop in RA? T cell B cell Plasma cell Osteoclast Osteoblast Fibroblast

Osteoclast

Deltamed HF lecture 64yo male, idiopathic dilated cardiomyopathy. Admitted with acute decompensated HF, 3rd admission in the last year. BP 96/70, HR 90/min, AF. JVP +5, PSM at apex. Bibasal creps. Mild SOA. Na 136 K 4.6 eGFR 80. LVEF 30%. Currently taking perindopril 8mg, bisoprolol 2.5mg, frusemide 80mg. What medication change is the best first option on admission? 1. Increase bisoprolol 2. Start spironolactone 3. Start sacubitril/valsartan (Entresto) 4. Start ivabradine

2. 1. Increase bisoprolol No, patient is congested/fluid overloaded 2. Start spironolactone Yes, adding an MRA is indicated (relatively preserved renal function and normal K+) 3. Start sacubitril/valsartan (Entresto) No, patient is on perindopril, need at least 36 hour washout period (risk of angioedema) 4. Start ivabradine This reduces the sinus node rate (If channel inhibitor), a pure heart rate reduction agent. Not indicated in AF.

DM Mock 27. Which of the following statements is the most correct regarding changes in respiratory physiology in healthy pregnancy? A. Oxygen consumption remains constant B. Minute ventilation is predominantly achieved by increasing respiratory rate C. Minute ventilation is predominantly achieved by increasing tidal volume D. Breathlessness reflects inadequate gas exchange

27. ANS: C. During pregnancy oxygen consumption is increased by approximately 15-20% (answer A is incorrect). This is achieved by increasing minute ventilation by 40% above the normal 7L/min. Increase in minute ventilation is predominantly achieved by increasing tidal volume (answer C is correct) rather than respiratory rate (answer B is incorrect). Breathlessness is perceptual rather than a reflection of inadequate gas exchange in the healthy pregnancy (answer D is incorrect).

DM Mock 47. A couple have a child affected with Tuberous Sclerosis (TS). Genetic testing reveals a germline mutation in the TSC1gene in the child. Both parents are tested via a blood sample and neither carries the mutation identified in the child. What recurrence risk would you give the parents regarding having another child affected with TS? A. 50%, because TS is inherited in an autosomal dominant manner B. 25% because TS is inherited in an autosomal recessive manner C. 2% because of the risk of one of the parents being gonadal mosaic for the TSC1 mutation identified in the child D. 0% because neither parent carries the TSC1 mutation identified in the child

47. ANS: C Although the parents have both tested negative for the TSC1 mutation, there is still a small residual risk that one of the parents is mosaic for the mutation within their germ cell tissue (gonadal or germinal mosaicism). Empiric data suggests this risk is approximately 2% for TS

DM Mock 57. In superiority, parallel-group clinical trials, which of the following statements is correct regarding intention-to-treat analysis? A. It leads to an under-estimation of a treatment effect, if this is present B. It is undertaken to reduce information bias C. It is undertaken to reduce confounding D. It increases the power of the study

57. ANS: A ITT analysis is undertaken to reduce selection bias. By assuming that people remained in the groups to which they were randomised, even if there was cross-over, ITT analysis leads to an under-estimation of the treatment effect, if this is present.

DM Mock 58. A 20-year-old presents with new onset diabetes. Genetic analysis reveals a HNF1A mutation. There is a strong family history of early onset diabetes. What is the best treatment? A. Insulin B. Sulfonylurea C. Metformin D. Exercise

58. ANS: B Maturity-Onset Diabetes of the Young (MODY) is an autosomal dominant heterogenous condition often misdiagnosed as type 1 (or 2) diabetes. HNF1A mutations are a common cause of MODY and respond well to sulfonylureas.

DM Mock 7. A 22-year-old female is referred to you by her local doctor for antenatal anticoagulation planning. She is G3P2, 12/40. She developed gestational diabetes in both previous pregnancies, without other serious complications. Due to a family history of thrombosis, a thrombophilia screen was performed which showed the following: Factor V Leiden Mutation Not detected Prothrombin Gene Mutation Not detected Antithrombin III 82 (80-120) Protein C 92 (65-135) Protein S 83 (>65) Lupus Anticoagulant Not detected by two methods Anticardiolipin IgG 32 (<10) Anticardiolipin IgM 35 (<10) Beta2 glycoprotein IgG <10 (<10) Beta2 glycoprotein IgM <10 (<10) What is correct regarding the clinical and laboratory criteria for antiphospholipid syndrome? A. An extensive clot of the great saphenous vein meets clinical criteria B. Anticardiolipin IgG level >100 is a disease defining criteria without need for clinical features C. One pregnancy loss > 10 weeks' gestation of a morphologically normal fetus meets pregnancy criteria D. Detected lupus anticoagulant or anticardiolipin/b2 glycoprotein requires repeat at 4 weeks to exclude transient antibody

7. ANS: C Explanation: Antiphospholipid syndrome is a clinico-pathological diagnosis of the presence of the lupus anticoagulant and/or anticardiolipin/beta2 glycoprotein antibodies and thrombosis. The great saphenous vein is superficial, which is excluded from the diagnosis. Anticardiolipin antibodies in itself are not diagnostic for the syndrome regardless of level. Tests need to be repeated at 12 weeks to confirm persistence

DM ID Flatmate of IVDU presents with multiple recent boils. Surprisingly is not blaming whitetail spiders. Picture shown. Best treatment? • A. Incision & drainage; no antis • B. Flucloxacillin • C. Vancomycin • D. Clindamycin • E. Linezolid

A

R 26 Which of the following is NOT included in ACR diagnostic criteria for SLE? A. alopecia B. Non-erosive arthritis C. Painless oral ulcer D. lymphopenia <1.5 E. warm AIHA

A serositis, oral ulcers (usually painless, often palatal), arthritis (non-erosive 2+ peripheral joints), photosens, low WCCs or lymphoctes, haemolytic anaemia (mostly warm type), proteinuria fixed erthrma, doscoid rash. ANA Alopecia is common but not included in ACR diagnostic criteria as of 2019: non-scarring alopecia + low complement now included

Dunedin Resp Which of the following is not an indication for non-invasive ventilatory support in the setting of acute hypercapnic respiratory failure? A. Asthma. B. COPD. C. Neuromuscular disease. D. Obesity-hypoventilation syndrome.

A • COPD Moderate to severe respiratory distress Respiratory rate >23 breaths/min pH<7.35 AND PaCO 2 >49 mmHg (>6.5 kPa) • Neuromuscular disease • Obesity-hypoventilation

CLS Resp Which of the following factors is associated with the worst prognosis in acute pulmonary thromboembolism? A. Hemodynamic instability B. High respiratory rate C. High brain natiuretic peptide (BNP) D. High troponin E. Right ventricular dysfunction of transthoracic echocardiogram

A • RV dysfunction on TTE - associated with increased mortality and particularly when severe is more useful predictor of poor prognosis than BNP and troponin (which are surrogate markers) In haemodynamically stable group has not been shown to consistently predict death (unlike haemodynamic instability) • BNP and troponin - both identify a higher risk group (are additive to risk) but are a poor predictor of death in haemodynamically stable PE • Respiratory rate - not included in any of the severity scores for PE (PESI, sPESI, other composite models) Only haemodynamic instability is an indication for thrombolysis = worth the risk = worst prognostic marker

CLS Rheum A 49-year-old female presents with a five week history of symmetrical polyarthralgia. Laboratory results are: Antinuclear antibody (ANA) positive, titre: 1/80, pattern: speckled Antibodies to extractable nuclear antigens (ENAs) negative Anti-double stranded DNA antibody (anti-dsDNA) 7 IU/mL [0-5] Anti-cyclic citrullinated peptide (anti-CCP) positive. Rheumatoid factor (RF) < 20 [0-20] Which of the following is the most likely diagnosis? A. Rheumatoid arthritis B. Systemic lupus erythematosus C. Mixed connective tissue disease D. Viral polyarthritis E. Osteoarthritis

A Anti-CCP antibodies potentially important surrogate markers for diagnosis and prognosis in RA • More sensitive and specific than IgM RF in early and established disease • May predict development of RA • Are a marker of erosive disease in RA • May be detected in healthy individuals years before onset of clinical RA

Dun Rheum The following symptoms are features associated with Ehlers Danlos Syndrome (EDS), especially hypermobileEDS except: a. Lens dislocation b. Mast cell activation syndrome c. Constipation d. Raynaud's e. Postural Orthostatic Tachycardia Syndrome

A (Marfans)

CLS Renal What is the mechanism of action of calcimimetics (Cinacalcet)? A. Decrease PTH level B. Decrease phosphate level C. Decrease calcium/phosphate product D. Increase Ca level E. Increase vit D level

A - main action - "calcimimetic" to trick the parathyroid • Rx for SHPT in dialysis pts - parathyroidectomy rate • Limiting factors in its use: • GI side effects • Hypocalcaemia - increase calcitriol ± calcium-based PO4 binders • PTH • Ca - unlike calcitriol • PO4 / - unlike calcitriol • EVOLVE NEJM 12 - RCT inconclusive on CV events & mortality (due to "cross-over" from placebo arm?) => now off PBS

deltamed renal 49 y.o. on haemodialysis for ESRF due to IgA has: Hb 95 Ferritin 80 (15-180ug/L) Transferrin saturation 18% (15-46%) What is the most appropriate treatment? A. Daily oral iron tablet B. Intravenous iron infusion C. Weekly Erythropoiten D. Fortnightly Erythropoiten E. Increased dialysis duration

B.

Deltamed 9/4 BRCA 2 mutations are associated with an increase in the lifetime risk of which of the following tumours? A. Pancreatic cancer B. Retinoblastoma C. Endocrine cancer D. Colorectal cancer

A Pancreatic Cancer BRCA2 associated cancers include 1. breast (male and female) 2. ovarian cancer 3. prostate 4. laryngeal 5. pancreatic

CLS Rheum Which of the following is most likely to exacerbate gout? A. Beer B. Dairy products C. High purine vegetables D. Red meat E. Fish

A beer

deltamed renal A patient develops fever, abnormal LFTs and diarrhoea on day 60 post-renal transplant - the most likely cause would be: a. acute rejection b. graft versus host disease c. tacrolimus toxicity d. mycophenolate toxicity e. CMV disease

CMV disease

CLS Rheum Please examine this hand X ray. What is the likely diagnosis? bitten out bits A. Psoriasis B. Rheumatoid arthritis C. Systemic lupus erythematosus D. Scleroderma E. Gout F. Osteoarthritis

A psoriasis Enthesitis and marginal bone erosions • Joint subluxation or interphalangeal ankylosis may be present. • Irregular, "fuzzy" appearance of bone around affected joint • Periostitis may appear as periosteal layer of new bone, or as irregular thickening of cortex • Dactylitis - 'sausage finger' = soft tissue swelling of a whole digit ; US → synovitis & tenosynovitis • Sacroilitis is often asymmetrical • Spondylitis - asymmetric paravertebral ossifications and relative sparing of the facet joints

Deltamed onc A 65 year old male with an 80 pack-year smoking history presents with a rapidly progressive shortness of breath, cough, haemoptysis and weight loss. CT chest shows superior vena cava obstruction with a large right lower lobe lung mass confluent with mediastinal lymph nodes. Bronchoscopy shows a right lower lobe bronchus mass. What subtype of lung cancer is this patient most likely to have? A. Small cell carcinoma B. Adenocarcinoma C. Squamous cell carcinoma D. Adenosquamous carcinoma

A. rapidly progressing. Central: could be SC or Sq

Deltamed onc 73 y/o admitted with 3w malaise, fevers. B/G of HTN and smoker. New systolic murmur. Mild anaemia, WCCs. Strep bovis in blood. and echo showing vege. What additional evaluation indicated? a. colonoscopy b. CT brain c. CTPA d. renal bx e. toxicology screen

A.

Deltamed endocrine 7d after commencing thyroxine, presents to ED very unwell with worsening fatugye, vomiting, SBP 80 What is the most likely cause of his deteriation? A. Apoplexy B. coeliac crisis C. adrenal failure D. thyroid storm

Adrenal failure If you diagnose primary hypothyroidism but miss coexisting AI then commencing thyroxine can precipitate life threatening adrenal crisis since thyroxine accelerates metabolic clearance of cortisol

CLS Resp Bronch lecture What epidemiological feature is most in keeping with a patient with EGFR activating mutations? - Asian, smoker, female - Non-Asian, smoker, female - Non-Asian, non-smoker, male - Asian, non-smoker, male - Asian, non-smoker, female

Asian, non-smoker, F

Dun Rheum Regarding JAKi therapy with tofacitinib, which of the following are true? A) tofacitinib inhibits JAK1 and JAK2 B) Serious infections occur at 3.1/ 100 pt-yrs (5mg bd) C) In tofacitinib users, HZ occurs at a rate of 4% per year and is further doubled with glucocorticoidS D) Japanese patients taking tofacitinib have a lower risk of HZ compared with European pts e) Administration of "Zostavax" is recommended in patients escalating from TNFi/MTX to tofa /MTX

B+C

Dun stroke A 57‐year‐old man with a long history of poorly controlled hypertension and smoking presents with sudden onset of right‐sided weakness. On examination, his speech is slurred but content and comprehension are normal. There is a right facial weakness, with the right side of the soft palate elevating poorly. He has a right hemiparesis, power grade 2‐3/5, in a pyramidal distribution with normal sensation to all modalities. Right‐sided reflexes are enhanced with an extensor right plantar response. The most likely cause of this stroke syndrome is left: A. Middle cerebral artery occlusion. B. Middle cerebral penetrating artery occlusion. C. Posterior inferior cerebellar artery occlusion. D. Hemispheric cortically based haemorrhage. E. Internal carotid artery occlusion.

B (nil sensory signs)

Deltamed renal 10. A 71-year-old woman with hypertension and a remote smoking history of 20 pack years is found to have abnormal kidney function on a routine blood test. Her current medication is enalapril 10 mg daily and amlodipine 10 mg daily and her last blood pressure was 138/86 mmHg. Her results are as follows: Normal values Serum creatinine 168 ȝmol/L [49-90] Estimated glomerular filtration rate (eGFR) 31 mL/min/1. Sodium (Na) 138 mmol/L Potassium (K) 4.7 mmol/L Haemoglobin (Hb) 116 g/L Calcium (Ca) 2.45 mmol/L [2.20-2.55] Phosphate (PO4) 1.3 mmolL [0.78-1.43] Urine albumin:creatinine ratio (ACR) 11 mg/g [< 3.5] In addition to monitoring kidney function, what additional strategy should be the focus of her management? A. Dietary potassium restriction. B. Modification of cardiovascular risk. C. Planning for dialysis access. D. Reduction of proteinuria. E. Repletion of haematinics.

B. modification Already on ACE and protein not too bad

CLS Geris Which of the following classes of medication is most likely to cause falls in the older person? 1. Anticholinergics 2. Benzodiazepines 3. Beta blockers 4. NSAIDs 5. Thiazide diuretics

Benzos

Dun cardio A 68-year-old female presents with headache, nausea and vomiting. She is noted to have retinal haemorrhages and papilloedema. Her blood pressure is 250/150 mmHg, creatinine 200 μmol/L [50 - 120 μmol/L] and urea 28 mmol/L [<8.0 mmol/L]. The computed tomography (CT) scan of her brain shows abnormalities in the parieto-occipital regions. Which is the most appropriate initial treatment? A. Captopril 12.5 mg three times daily orally. B. Metoprolol 5 mg intravenously. C. Sodium nitroprusside 0.25 microgram/kg/min intravenously. D. Nifedipine 10 mg sublingually. E. Glyceryl trinitrate 10 mg/24h topically.

C Hypertensive emergnecy/crisis Nitruprusside = short acting, can be titrated. risk of cyanide poisoning.

DM Mock 10. A 65-year-old man is admitted to intensive care following out-of-hospital VF arrest with prolonged bystander resuscitation. Which finding on examination is NOT compatible with brain death? A. Absent cough with tracheal suctioning B. Absent sucking reflex C. Seizure D. Subtle, semi-rhythmic movements of facial nerve-innervated muscles

C Movements originating from the spinal cord or peripheral nerve may occur in brain death. These are common (33 to 75 percent) and may be triggered by tactile stimuli or occur spontaneously. Examples include: ●Subtle, semirhythmic movements of facial nerve-innervated muscles can arise from the denervated facial nerve. ●Finger flexor movements. ●Tonic neck reflexes - Passive neck displacements, especially flexion, may be accompanied by complex truncal and extremity movements, including adduction at the shoulders, flexion at the elbows, supination or pronation at the wrists, flexion of the trunk ("sitting up" type movements), and neck-abdominal muscle contraction or head turning to one side. These might be quite dramatic, often called a "Lazarus sign." ●Triple flexion response with flexion at the hip, knee, and ankle with foot stimulation (eg, testing for a Babinski sign). ●Other truncal movements including asymmetrical opisthotonic posturing of the trunk and preservation of superficial and deep abdominal reflexes. ●Alternating flexion-extension of the toes with passive displacement of the foot (undulating toe sign), or flexion of the toes after foot percussion, or a Babinski sign. ●Upper limb pronation extension reflex. ●Widespread fasciculations of trunk and extremities

DM Resp Regarding idiopathic pulmonary arterial hypertension, which of the following is most correct? A. All patients with BMPR2 mutations develop pulmonary hypertension B. Calcium channel antagonists are effective in 90% of patients with pulmonary arterial hypertension C. Endothelin A and Endothelin B receptors are present on smooth muscle and produce vasoconstriction D. Endothelin A receptors on vascular epithelium release nitric oxide which results in vasodilatation E. Patients with elevated BNP levels have a better prognosis than patients with low BNP levels

C.

Deltamed neuro A 55 year old lady presents with fatiguable ophthalmoplegia, with elevated AChRAbs and positive sfEMG: What should be the first treatment offered? A. low dose prednisolone B. plasma exchange C. pyridostigmine D. thymectomy E. IVIg

C (increases ACh)

Book R13 Which of the following statements about PAN is correct? A. Hep C has the strongest assoc as a causative pathogen of this condition B. it is a multisystem disease affecting the large arteries C. All organ systems can pot be affected by PAN except pulmonary system D. Renal bx recommended as 1st choice for dx E. ANCAs are usually positive

C - Medium or small vessels. Constitutional symptoms + multisystem involvement. Idiopathic, secondary (most freq hep B). Others: GAS, Hep C, CMV, EBV, Parvovirus. Skip lesions, ischaemic regions in skin, kidneys, GI tract, testes. Gold standard: focal, segmental, panmural necrotising inflammation of medium arteries. Multiple aneurysms and left often stenosis. can cause renal infarct, renin HTN Affects all organs other than pulmonary Orchitis, ANCA -ve, abnormal renal or mesenteric angio + nil GN A B - medium arteries C D - not 1st choice E - not

DM PT Neuro9 9. A 78-year-old woman is reviewed with recurrent focal seizures associated with impaired awareness. She has a background of type 2 diabetes and mild cognitive impairment. Which would be the most appropriate choice of antiepileptic drug (AED)? A. Carbamazepine B. Lamotrigine C. Levetiracetam D. Phenytoin

C - less discontinuation rate in elderly

Dunedin gastro A 43 year old man, 18 weeks post resection of 50cm of terminal ileum, presents with diarrhoea and abdominal pain. On examination he is afebrile with a well healed scar and no focal abdominal findings. Investigations: FBC, ESR - N Faeces - N Colonoscopy: Normal mucosa, well healed anastomoses What is the best management for this man? A. Codeine B. High fibre diet C. Cholestyramine D. Sulphasalzine E. Prednisilone F. Loperamide

C bile salts

CLS Rheum Which of the following cells is most likely to cause bony erosions in patients with rheumatoid arthritis? A. T-Lymphocyte B. B-Lymphocyte C. Osteoclast D. Osteoblast E. Plasma cells

C clasts

DeltaMed 1/1 1. Which of the following periods represents ventricular diastole? A. S to P wave B. P to Q wave C. end of T to QRS D. end of QRS to end of T

C end of T (ventricular repol/AV closure) to QRS

Book R12 A 35 y/o F in rheum clinic. Fever, proximal muscle weakness, Raynauds, swollen fingers + hands. Multiple joint pains and reflux. Which serological test is most useful to dx MCTD? A ANCA B. hypocomplement C. anti-U1 small nuclear - antiRNP D. antijo1 E. ANA

C: MCTD: Fs in 3rd decade. Raynaud, polyarthralgias, swollen hands, sausage fingers, fatigue, myalgias. B - lupus D - antisynthetase E - lupus

Dunedin endo Case:18 year old patient on long term steroids • Transfer from paediatric clinic • Born with ambiguous genitalia • Height 160 cm, weight 90 kg • Cushingoid appearance • Irregular menstrual periods • Meds: hydrocortisone 15mg tds and fludrocortisone 200mcg daily • Lab results: 17OH progesterone 75 • Gynaecology review - vaginal dilators Dx?

CAH

Deltamed Resp A 57yo man has had dyspnoea for 6 months.Respiratory function tests were performed with the results below: PreͲBD: PostͲBD: TLCO 50% predicted FEV1 2.3L (65% predicted) FEV1 2.4L KCO 55% predicted FVC3.6L (62% predicted) FVC 3.6L TLC 110% predicted FER 0.64 (lower limit 0.69) FER 0.67 RV/TLC 0.55 (upper limit 0.46) What is the most likely diagnosis?: • COPD • Restrictive lung disease • Asthma • Mixed obstructive and restrictive lung disease • Poor effort

COPD Non-reversible obstruction (200mL AND 12%) Gas trapping High TLC excludes restriction

DM PT Gen5 5. Spinal Muscular Atrophy (SMA) has a disease incidence of 1 in 6,400. What is the carrier frequency for SMA in the general population? A. 1/40 B. 1/80 C. 1/120 D. 1/160

Chance of a couple having a child affected by an autosomal recessive disease = disease incidence (if no family history of the disease and no consanguinity) Chance that father is a carrier Chance that mother is a carrier = x x 1/4 1/6,400 = 1/a x 1/a x ¼ = 1/a2 x ¼ a2 = 1600 a = 40 Therefore, carrier frequency in general population is 1 in 40

Dunedin pharm Which of the following drugs is most likely to have the slowest rate of clearance from plasma? A. amiloride. Vd 1190. 1/2 21 B. amiodarone 4620. 600 C. dapsone 70. 22 D. tolbutamide 7. 6 E. trimethoprim 126. 11

Cl = 0.69 x Vd // 1/2 ie. Vd / t 1/2 approxmate 1200 / 20 = 60 4600 / 600 = 7 75 / 25 = 3 7 / 6 = 1.2 ish 130 / 10 = 13 answer D

Dun neuro MG characterised by impaired neuromusc transmission WHat change is seen in post-synaptic terminal? A. increased latency B. prolonged duration C. prolonged recovery D. reduced amplitude E> reduced freq

D Reduced receptors = released quantum of ACh = smaller MEPP

DM ID A 25 year female develops cystitis symptoms shortly after backpacking around Greece. Despite GP-prescribed Augmentin Duo Forte, she gets worse and has symptoms of pyelonephritis. She is admitted and put on meropenem 500mg q8/24 but does not respond by 72 hours. Culture of blood then comes back with Klebsiella resistant to meropenem. Best option: • A - Meropenem increased to 2000mg q8/24 • B - Tigecycline • C - Fosfomycin • D - Colistin • E - Ciprofloxacin

D

Deltamed renal A 60-year-old man with a five-year history of type 2 diabetes presents for a routine check-up. The most appropriate screening test for diabetic nephropathy is: A. serum creatinine level. B. spot urine albumin concentration. C. 24 hour urine protein excretion. D. spot urine albumin/creatinine ratio. E. 12 hour urinary albumin excretion rate.

D. 24hrs no good for screening

Deltamed 4/3. Autoimmunity is the LEAST prominent clinical manifestation in which of the following proteins deficiencies: A. Foxp3 B. AIRE C. CTLA4 D. CD40 ligand

D. AIRE: failure of neg selection in thymys FoxP3: loss of Treg cells CTLA4: loss of inhibt signals to T cells CD40 = hyperIgM/immunodeficiency is predomiannt

DM ID Moxifloxacin is appropriate for which of the following? • A - Acute exacerbations of chronic bronchitis • B - Enterococcus faecium infection • C - Stenotrophomonas maltophilia infection • D - Moderate to severe pneumonia in a patient with severe penicillin allergy • E - Cough with haemoptysis in a patient from Somalia

D

Deltamed Onc 60 y/o M with nil significant hx presents for BP meds. What is most appropriate strategy for CRC? A. dietary advice B. CT colo C. DR exam D. FOBT annual E. 5 yrly colo

D. lowest risk. nil risk of harm. asymptomatic

DM ID Which of the following statements concerning the systemic antifungal agent caspofungin is true? • A. it inhibits ergosterol synthesis • B. it is not active against Candida krusei • C. it can be used to treat cryptococcal meningitis in patients intolerant of amphotericin • D. it has activity against Aspergillus fumigatus • E. dosage reduction is required in patients with renal failure

D

Deltamed Immuno A type 2 T lymphocyte (Th2 cell) differs most from a type 1 T lymphocyte (Th1) in which one of the following? A. Surface expression of CD4 B. Surface expression of major histocompatibility gene complex (MHC) class II C. Production of interleukin 2 (IL‐2) D. Production of interleukin 5 (IL‐5) E. Responsiveness to antigen presented by macrophages

D

Deltamed Onc In which of the following is RT least likely to be of clinical benefit to the pt? A. bleeding malignant GI ulcer B. brain mets with raised ICP C. painful bone mets from prostate D. intrahepatic mets causing cholestasis E. NSCLC causing bronc obstruction

D

Deltamed Onc Patients with familial retinoblastoma carry a germline mutation in one copy of the RB1 gene. Potential mechanisms for inactivation of the other allele in a retinoblastoma arising in one of these patients include: • A. An independent point mutation B. Loss of the normal chromosome 13 C. Mitotic crossing over D. all of the above E. A and C

D

Deltamed Onc This patient's daughter develops a breast cancer in her early 30s, and is found to carry a BRCA1 mutation. What type of breast cancer is the patient's daughter most likely to have? A. Luminal A B. Luminal B C. HER2 positive D. Triple negative

D

Deltamed renal With regards to haemodialysis patients, survival is improved by? A. Tight BP control B. Tight weight control C. Dialysis dose D. Dialysis duration E. Tight cholesterol control

D

Deltamed stats If the pre‐test probability of a condition is known, which of the following is also needed to be able to estimate the post‐test probability? A. Sensitivity B. Specificity C. Accuracy D. Likelihood ratio E. Odds

D

Dun Pharm A male patient is commenced on quinidine, a specific CYP (cytochrome P450) 2D6 inhibitor. He is also being treated with paroxetine which is a CYP1A2, 2D6 and 3A4 substrate. The greatest increase in serum concentrations of paroxetine will occur with which one of the following profiles of enzyme activity? CYP1A2 CYP2D6 CYP3A4 A. Poor Poor Poor B. Extensive Poor Extensive C. Extensive Poor Poor D. Poor Extensive Poor E. Extensive Extensive Extensive Poor = poor metaboliser Extensive = extensive metaboliser

D

Dun cardio A 30-year-old woman presents with sustained hypertension (170/110 mmHg) while taking amlodipine as monotherapy. Her only other medication is a combined oral contraceptive. There is no radiofemoral delay or abdominal bruit. Investigation reveals the following: Plasma renin activity <40 fmol/L/s [130-2350] Plasma aldosterone 320 pmol/L [80-1040] Serum potassium 3.9 mmol/L [3.5-5.5] Urinary metanephrines 1.3 μmol/24h [<1.5] Which of the following secondary causes of hypertension is most likely? A. ACTH-producing tumour. B. Oestrogen-containing contraceptive. C. Phaeochromocytoma. D. Primary hyperaldosteronism. E. Renal artery stenosis

D

Dun cardio In a patient with rheumatic mixed mitral valve disease, for which of the following procedures is there the strongest indication for antibiotic prophylaxis? A. Flexible bronchoscopy. B. Cardiac catheterisation. C. Colonoscopy without biopsy. D. Oesophageal dilatation. E. Gastroscopy.

D

Dun pharm Cyclosporin = very low bioav Lipid soluble. Mainly met by CYP3A4 in liver. Removal of liver accounts for only 50%. Why is bioav so low? A poor dissolution of tablets B neutrophilic attack by stomach acid C. destruction by colioform bacteria D metabolism in S.I E. circulating esterases in portal blood

D

Dunedin Haem Which of the following is a cause of microcytic anaemia? A. Myeloma B. Chronic renal failure C. Hypothyroidism D. Lead poisoning E. Aplastic anaemia

D

Delta med Onc Which of the following interventions is not used in patients with BRCA2 mutations A. Bilateral prophylactic mastectomy B. Annual mammograms C. Bilateral salpingo-oophorectomy D. Annual screening with pelvic ultrasound and Ca125

D. nil evidence BRCA2 = ER+ often so do mastectoym + BSO

Dunedin Resp A 22 year old university student presents with sudden onset of left sided chest pain whilst playing a video game. By the time he reaches the Emergency Department the pain has largely settled and he appears comfortable at rest. He is a non-smoker and has no other medical problems. Vital signs are normal and chest examination is unremarkable. Chest x-ray shows a 2 cm apical pneumothorax. SaO2 on room air is 97%. Which one of the following is your next step? A. Undertake needle aspiration. B. Insert intercostal tube drain. C. Request CT scan of the chest. D. Discharge home with early outpatient review.

D

Dunedin Resp Regarding treatment of COPD, which of the following is false? A. Adherence and inhaler technique need to be checked on a regular basis. B. Vaccination reduces the risks associated with influenza and pneumococcal infection. C. Systemic corticosteroids reduce the severity of, and shorten recovery from exacerbations. D. Long-term oxygen therapy has been demonstrated to improve survival for COPD patients with a resting PaO2 of 60-69mmHg. E. Consider pulmonary rehabilitation at any time, including during the recovery phase following an exacerbation.

D

Deltamed 6/8 A 23-year-old female has developed lower motor neuron pattern left facial weakness over the course of 4 days. On examination, she demonstrates moderate to severe weakness of the left forehead, with minimal voluntary elevation of the left eyebrow, and incomplete eye closure at rest. She has voluntary lateral movement at the angle of the mouth on the left. She describes a sense of dryness of the eye, and on direct questioning some altered taste. Which is the best management option? A. Antiviral plus corticosteroid B. Antiviral alone. C. Corticosteroid alone D. No medication

D.

Dunedin endo A 35 year old man is referred for evaluation of muscle and bone pain, fatigue, weakness, fractures and difficulty walking. DEXA scanning shows Z scores of between -3 and -4 at all sites. Laboratory results: • Serum calcium 2.3 mmol/L Serum phosphate 0.4mmol/L PTH 98pg/ml (10-65) 25(OH) vitamin D 69 nmol/L (62-199) 1,25 dihydroxyvitamin D 31 pmol/L (41-169) • Maximum tubular phosphate reabsorption - low Which of the following is the best next test to order? A. Serum protein electrophoresis B. Sestamibi scan C. 24,25 dihydroxyvitamin D level D. Fibroblast growth factor 23 measurement E. 24 urine collection for calcium, electrolytes, amino acids, glucose and creatinine

D

Dunedin endo Metabolic memory refers to which of the following? A: Ability of glucose meters to recall BSL's B: The effect of hypoglycaemia on short term memory C: The effect of the in-utero metabolic environment on future risk of Type 1 diabetes D: The effect of prior glucose control on risk of diabetes complications E: The time delay between interstitial and plasma glucose levels when using CGMS

D

Deltamed Onc Epidermal Growth Factor Receptor Therapy ( eg. Cetuximab) improves disease free survival & overall survival in patients with what type of metastatic colorectal cancer • A. Both EGFR and KRAS mutant • B. EGFR mutant • C. EGFR wildtype • D. KRAS wildtype

D.

CLS CLL Lecture A 75 year old man with no comorbidities presents with a lymphocyte count of 50, Hb 110 and platelet count of 160. He has cervical lymphadenopathy (1.5cm) and no splenomegaly. His weight is stable and he denies fevers and sweats. Peripheral blood flow cytometry shows a population of lymphocytes CD5+, CD19+, CD23+ and lambda light chain restriction. The most appropriate initial approach is to A. commence therapy with an anti-CD 20 monoclonal antibody B. confirm the diagnosis with an excisional lymph node biopsy C. confirm the diagnosis with a core biopsy of a lymph node D. do nothing immediately and observe E. commence multi-agent chemotherapy combined with an anti-CD20 monoclonal antibody

D

CLS Quiz Resp Recognised methods for managing ARDS include Select one: a. Use of high frequency oscillation b. Low PEEP and high tidal volumes c. High Driving pressures d. Prone ventilation e. All of the above

D

CLS Rheum 28 yo woman who is otherwise well presents with Raynauds of fingers. ANA +ve 1/80 titer. The most likely diagnosis is: A. SLE B. CREST C. Systemic sclerosis D. Primary Raynaud's

D

CLS Rheum 47 year old male, HIV for past 20 yrs, presents with gradual onset right hip pain over last 3 weeks. He is on emcitrabine, tenefavir, enviranz. Known to have lipodystrophy. No history AIDS defining illness. XR shown right hip/pelvis showing collapse of superior surface femoral head ANA negative Rh factor negative urate 0.52 (normal < 0.44) What is most likely diagnosis? A. Gout B. OA C. Psoriatic arthritis D. Avascular necrosis E. RA

D

CLS Rheum In which part of the vertebral column are syndesmophytes first seen in ankylosing spondylitis? A. Cervical B. Cervical/thoracic C. Thoracic D. Thoraco-lumbar E. Lumbar

D

CLS genetics The DNA approach to genetic disorders means that individuals who are at risk, but clinically asymptomatic, can be tested to determine if they will develop the disease. In this circumstance, which one of the following is the most appropriate reason for testing a nineyear-old child? A. Informed consent can be obtained from the parents. B. Parental anxieties can be relieved. C. In the case of autosomal recessive disorder, the child's carrier status can be determined. D. Medical intervention can alter the disorder's natural history. E. The child wants to be tested.

D

DM ID 37 yo female diagnosed with DLBCL. Starts treatment with R-CHOP. Recognised side effects of rituximab include all of the following except: • A. Progressive multifocal leucoencephalopathy • B. Infusion reactions with death within 24/24 • C. Severe bacterial or fungal infections • D. Reactivation of hepatitis C • E. CMV disease

D

A 23 Y.O. female with cystic fibrosis and recurrent chest infections is diagnosed with ulcerative colitis and requires multiple induction courses of corticosteroids despite mesalazine. Which is the most appropriate maintenance agent for this patient? A) Azathioprine B) Methotrexate C) Adalimumab D) Vedolizumab E) Prednisolone

D. nil increased risk of infections

DM ID A 24 year old man with known HIV has been developing weight loss and night sweats over the past 3 weeks. He is found to have a CD4 count of 40 and a viral load of 140,000. He confesses that he is having trouble recalling the dosage and timing of his current ART. He has a number of investigations including FBC, UEC and cultures all of which are essentially within normal limits. His therapy is changed to a new triple therapy regime and is dispensed in a Dosette box. 2 weeks later he presents complaining of a swollen neck and lumps in the groin. In addition, he is still getting night sweats and is continuing to lose weight, albeit at a slower rate than previously. On examination he has generalised lymphadenopathy and splenomegaly. His CD4 count is 184 and his viral load 15,000. • • Which of the following statements is most correct ? • A the symptoms and lymphadenopathy are most likely due to lymphoma • B the persisting symptoms are due to the ART and it should be changed again • C the symptoms and lymphadenopathy are due to progression of his HIV • D he has immune reconstitution disease most likely due to MAC infection • E the patient has developed a CMV infection

D

Deltamed renal 71. In a patient requiring urgent dialysis which of the following parameters would favour continuous renal replacement therapy (CRRT) rather than intermittent haemodialysis (IHD)? A. Active bleeding. B. Coagulopathy. C. Hyperkalaemia. D. Hypotension. E. Hypoxia.

D.

Deltamed renal In a 50-year-old man undergoing a CT for investigation of abdo pain, which of the following is the most likely cause of an incidentally discovered 3cm adrenal mass lesion? A. Adrenal metastasis. B. Cortisol secreting adrenal adenoma. C. Phaeochromocytoma. D. Non-functioning adrenal adenoma. E. Adrenal carcinoma.

D.

Deltamed genetics John and Rita are having their first child together. They are unrelated. John's niece and Rita's brother both had cystic fibrosis. What is the risk to their child of developing cystic fibrosis? A. 1 in 36 B. 1 in 18 C. 1 in 16 D. 1 in 12 E. 1 in 8

D 1 in 12 John has 1/2 chance of being affected (parents AA and Aa). Rita has 2/3 chance If both carriers = 1/4 so 1/2 x 2/3 x 1/4 = 1/12

Dun cardio What is the main factor responsible for systolic hypertension in the elderly? A. Impaired renal perfusion. B. High cardiac output. C. Sympathetic activation. D. Reduced arterial compliance. E. High peripheral resistance.

D = higher PP (more compliant = cushions = less variability)

Dun pharm which of the following drugs newly prescribed with Li is most likely to increase drug level? A. fluox B> thephylline Ccaptopril D. thiazide E. aspirin

D. thiazide. Na depletion = high Li

Deltamed endocrine 44 y/o man. HTN. Nil regular meds. K 3. Renin 0.1 (low). aldo 20 (low). Best next step A. dietary hx B. renal artery doppler C. urine diuretic screen D. fine cut dedicated adrenal CT

Dietary hx Licquorice = appararent mineralocorticoid excess. Low renin and aldo. ALso exogenous. Cushings. CAH. Liddle

CLS ID Vibrio cholerae causes diarrhoea primarily by what mechanism? A. Hypermotility B. Inflammatory C. Malabsorption D. Osmotic E. Secretory

E

Dun Rheum In ANCA associated vasculitis, all of the following are correct except? a) Plasma exchange does not reduce the incidence of death in patients with kidney injury ( eGFR<50ml/min) b) Plasma exchange does not reduce the incidence of end stage kidney disease (ESKD) in patients with kidney injury ( eGFR <50ml/min) c) Plasma exchange should not be included in standard induction therapy for patients with ANCA associated vasculitis who have kidney dysfunction d) High dose corticosteroid use increases complications related to infection e) Reduced corticosteroid dosing increases relapse rate and progression to ESKD

E Lower dose roids NEJM Feb

Dun Rheum With respect to contraception in lupus patients, the following are correct except: a. Low dose oestrogen oral contraceptive pills (OCPs) do not increase SLE flare b. Progestin only pills(POPs) do not increase SLE flare c. Implants are safe in SLE patients and do not increase flare d. Oestrogen containing OCPs should be avoided in patients with history of lupus nephritis e. Mycophenolate may decrease the efficacy of progestin only pills.

E (reduces estrogen pills) Low dose estrogens OK (nil increase in flares) If high risk of VTE ie. APLS = don't use low dose estrogen

CLS Resp Which of the following statements regarding non-invasive ventilation (NIV) is the most correct? A. Use of NIV during an acute hypercapnic exacerbation of asthma can reduce intubation rates and mortality B. NIV during hypercapnic exacerbations of COPD is similarly effective regardless of arterial pH C. Continuing long-term NIV in those with COPD and persistent hypercapnia will not improve mortality D. NIV is superior to CPAP as first line treatment for obesityhypoventilation syndrome E. CPAP and NIV have similar efficacy for acute pulmonary oedema in the absence of hypercapnia

E Meta-analyses suggest similar efficacy with a preference for NIV in those with hypercapnia Asthma = Meta-analysis of 5 small RCTs has not shown an impact on intubation rates or mortality. = off-label use currently From meta-analysis: Arterial pH predicts the value of NIV Benefits over standard care demonstrated only when pH<7.30 or if mortality in the control group was >10% No difference in outcomes in milder exacerbations Long-term NIV shown in two studies to improve mortality in stable hypercapnic COPD Australian RCT - demonstrated worsening HRQoL measures German/Austrian RCT - very large mortality benefit but concerns about methodology A local RCT showed similar rates of treatment failure using NIV vs CPAP as first line therapy Current practice is for CPAP as first line (simpler, cheaper) Take home message - HFNC first line for acute hypoxaemic respiratory failure with the exception of acute cardiogenic pulmonary oedema (-> CPAP or NIV)

Deltamed Onc 59 y/o. Newly diagnosed adeno of desc colon on colonoscopy after +ve FOBT. Undergoes L hemicolectomy Mod diff adeno arising from adenoma. 38mm in max dimesnion + invading muscularis propria but not thru. Nil mets in 8 regional LNs Which of the following features is associated with increased risk of recurrence? A. L sided tumour B. size C. invasion of muscularis D. tumour arising from adenoma E. inadequate LNs sampled

E need 12

Dun Neuro Which of the following anticonvulsant drugs is most likely to cause a clinically important drug interaction with lamotrigine? A. Clonazepam B. Gabapentin C. Vigabatrin D. Carbamazepine E. Sodium valproate

E rash

CLS ID 77. A 32-year-old man has returned from 8 weeks in India 16 days previously. Since his arrival home, he has had fevers, abdominal pain, constipation and lethargy. Gram negative bacilli have grown in both aerobic and anaerobic blood culture samples 13 hours after being drawn. What is the most likely diagnosis? A. Dengue. B. Escherichia coli O157. C. Leptospirosis. D. Malaria. E. Typhoid.

E typhoid

Deltamed Onc A 60 year old Korean female presents with an exudative pleural effusion. Cytology shows cancer cells that stain for TTF1. What additional test would you request? Her2 staining KRAS mutation testing MRI head Bronchoscopy with biopsy EGFR mutation testing

EGFR (asian, F) TTF1 = adeno NSCL so don't need brain

Deltamed Onc 61 y/o man completes neoadjuvant chemo-RT + APR for low rectal ca followed by adj chemo with 5FU. Most appropriate Mx? A. Aromatase0I B. tamoxifen C. prophylactic cranial irrad D. surveill with 3monthly exam E. surveill with 3 monthly CEA + CT CAP F. surveill with 3 monthyl CEA + annual CT G. referral to familial ca clinic H obs

F Probably 6m

Dunedin endo Case:16 year old with primary amenorrhoea • No breast development • Female external genitalia • Investigations: - LH 34 u/L, FSH 72 u/L, oestradiol < 70 pmol/L - Testosterone 1.0 nmol/L - Karyotype 46XY - Ultrasound pelvis - bilateral streak gonads, small pre-pubertal uterus • Laparoscopy & removal streak gonads • Commenced on HRT Dx?

Gonadal dysgenesis External genitalia F with normal uterus and tubes SRY mutation Rx with HRT to induce F secodnary sex characteristics Gonads not functioning Normal adrenals

Dunedin pharm New anticonvulsant. Vd 2L, hepatic clearance 5L/hr, renal clearance of 8L/hr, resp clearance of 2L/hr, 98% protein bound. Usual half life is 6hrs with usual dose 100mg QDS Most appropriate dosing regimen for efficacy, safety and compliance with end stage dialysis on HD? A. 100mg OD B. 50mg QDS C. 100mg TDS D. 100mg BD E. 50mg QDS, 100mg after dialysis

HD doesn't change anything clearance = 15 renal clearance = 0.5 approx Usually 400mg total so want 200mg total For ease of taking then 100mg BD

Dunedin pharm A drug has a fraction excreted unchanged of 0.1 and a total body clearance of 50L/hr. Liver blood flow is 90L/hr. Its maximum oral bioavailability is likely to be: A. 90% B. 75% C. 50% D. 25% E. 10%

Hepatic extraction ratio = 1 - bioavailability bioavailability = 1 - HER HER = 50 / 90 = approx 0.5 x 0.9 metabolised (as FU = 0.1) approx 0.5 x 1 = 50%

Deltamed gastro lecture 1 Which of the following is most likely to be deficient in a pt poorly compliant to diet for Coeliac disease? A. iron B. Vit B12 C. folate D. calcium E. vitK

Iron (absorbed in prox small bowel) Folate rare.

What is the most common type of mutation in non-small cell lung cancer - EGFR - KRAS - ROS1 - P53 - EML4-ALK rearrangement

KRAS Most common targetable is EGFR

Dunedin endo Case: 17 year male with delayed puberty • Surgery for a cleft palate as a child • Absent secondary sexual characteristics • Height 178cm • Small testes • Anosmia • Lab results: FSH 0.1, LH 0.1, testosterone 1.0 nmol/L • Mother had delayed puberty Dx?

Kallmanns. Anosmia. Hypogonadotrophic hypogonadism

Regarding primary GN recurrence in Tx. Which is most common? MCGN vs FSGS? Is it specific to a time frame (FSGS in 1st year vs MCGN every year thereafter)? This was a previous MCQ.

MCGN (now called MPGN) - as this recurs in >50% over time. FSGS recurs in <20% but occurs early and causes graft failure in about 10%

56 y/o man with RA complicated by serositis, vasculitis, nodules and ILD presents with suddeon onset C/P and SOB. The most likely dx is a, tamponade b. severe pHTN C. PE D. Coronary vasculitis E. dissecting aneurysm E. MI

MI CRP directly associated with risk.

Deltamed Rheum A 20 year old man presents with one year of low back and alternating buttock pain that is worse in the morning, relieved by activity and NSAIDs with associated limited range of movement of the lumbar spine. X-ray of Sacro-iliac joints is normal ɵ What is the next most appropriate diagnostic test? a) CT Sacro-iliac joints b) Assessment of inflammatory markers c) MRI Sacro-iliac joints d) HLA B27 e) XR Lumbar spine

MRI need imaging

Deltamed Renal Serum creatinine increases from 100 to 130 umol/L from day 7 to day 8 post-renal transplant - the most unlikely cause would be: a. acute rejection b. tacrolimus toxicity c. sepsis d. mycophenolate toxicity e. ureteric obstruction

Mycophenolate - not nephrotoxic

Deltamed Onc Which is not a common side effect of checkpoint inhibitors • Neutropenia • Diarrhoea • Hypothyroidism • Arthralgia

Neutropenia

Deltamed rheum All of the following support osteoclast development in RA except? Monocyte/macrophage T cell derived factors RANKL RANK TNF IL6 OPG

OPG protects bone

Deltamed rheum The following cells are predominantly found in RA synovial fluid Macrophages CD4T cells Synovial fibroblasts TH17 cells PMN

PMN

Deltamed Resp A 60 year old nonͲsmoker has been recently diagnosed with idiopathic pulmonary fibrosis. You conduct a literature search to determine treatment options for him. Which of the following would not be appropriate? • Pirfenidone • Referral to lung transplant team • Nintendanib • Prednisolone • Oxygen therapy

Pred PANTHER Study

DM PT RFs and rx of C.auris

Risk factors for C. auris acquisition include underlying immunocompromised state, intensive care unit (ICU) admission and increased length of stay, presence of invasive medical devices (particularly intravascular lines and urinary catherters), and previous antibiotic or antifungal exposure. First-line treatment of C. auris is with an echinocandin antifungal agent (e.g. caspofungin), given the susceptibility profile is most favourable for this class when compared to -azole and amphotericin antifungal agents.

Deltamed liver The optimal management for a patient with alcoholic hepatitis. Maddrey's DF = 35? Corticosteroids Pentoxifylline Corticosteroids + Acetylcysteine (NAC) Corticosteroids + Pentoxifylline

Roids and NAC if >32

Deltamed rheum 36 t/o RA pat stable for 3yrs on MTX/SZP/HCQ wants to get pregnant. You would advise all the following except? Cease MTX 3-9m preconception No MTX or leflunomide while b/F May be safe to continue SZO.HCQ with low dose pred Imuran may be safe if flare Switch to antiTNF for 2nd trimester Her RA may settle in preg Her RA may flare in preg +/- b/f

Switch to anti-TNF is incorrect

CLS Resp Quiz A 50 year old male smoker presents with dyspnoeaand the attached chest x-ray. An intercostal catheter is inserted. A CT is performed. The presence of which cells in a lung biopsy would confirm this man's diagnosis? Select one: a. Fibroblasts b. LAM cells c. Macrophages d. CD1a+ Langerhans cells e. Pneumocystis

The answer is (d) This man has Langerhans cell histiocytosis as the cause of his cystic lung disease. Langerhans cells are dendritic cells which is diagnostic of this condition. The CT demonstrates multiple irregularly shaped cysts within the lung. Sparing of the costophrenic recesses (not shown) is classic of this condition. Whereas in LAM the cysts are typically found in the costophrenic recesses -'LAMbsfall to the gullies'. The cysts in LAM typically have a smooth outline.

CLS Resp Quiz A 68 year old male presents with chronic cough and dyspnoea. His CT scan is reported as demonstrating bilateral lower zone 'crazy-paving'. A bronchoalveolar lavage has returned 'milky' fluid which is periodic acid Schiff positive. Presence of which of the below antibodies would be suspected? Select one: a. Anti-GM-CSF b. MPO-ANCA c. PR3-ANCA d. Anti-Jo1 e. Anti-MDA5

The correct answer is (a) The features in this case are suggestive of Pulmonary Alveolar Proteinosis (PAP). PAP results from the accumulation of surfactant in alveoli due to impaired clearance by alveolar macrophages. There are primary and secondary forms. The auto-immune (primary) form accounts for 90% of cases and is characterized by antibodies to GM-CSF. The treatment options include whole lung lavage, immunomodulation and inhaled GM-CSF.

CLS Resp Quiz Which of the following is a good prognostic feature in hypersensitivity pneumonitis? Select one: a. Presence of honeycombing on CT b. Identifiable antigen exposure c. Shortened telomeres d. Biopsy pattern of UIP e. Negative serum precipitins

The correct answer is (b) An identifiable exposure allows that patient to avoid/remove the exposure which is causing inflammation and fibrosis. An exposure is not identifiable in ~25% of patients with chronic hypersensitivity pneumonitis. The lack of an identifiable exposure is a poor prognostic feature. The presence of honeycombing, UIP and shortened telomeres are poor prognostic features. Serum precipitins may be useful in diagnosing HP, but are not contributory to prognostication. Serum precipitins may assist in defining an exposure.

CLS Resp Quiz A 35 year old female with a history of previous pneumothorax and the attached CT has had progressive decline in her FEV1 over the last 5 years. Which of the below therapies has proven efficacy in this condition to slow lung function decline? Select one: a. Pirfenidone/p> b. Sirolimus c. Inhaled corticosteroids d. Mycophenolate e. Oestrogen

The correct answer is (b) This patient has lymphangioleiomyomatosis. The combination of pulmonary cysts and renal angiomyolipoma in a young woman is a classic presentation. There are sporadic and tuberous sclerosis associated forms. High VEGF-D levels may assist in diagnosis in difficult cases. Rapamycin (mTOR inhibitor) has been shown to slow lung function decline in this condition

CLS Resp A 45 year old lady presents with type 1 respiratory failure and the CT scan attached. She has no known medical history. Examination of the hands reveals erythematous lesions symmetrically over the meta-carpophalangealjoints. Her CK is not elevated. She is started on high flow oxygen therapy at 60% FiO2 and stabilized on the ward. What is the most important next step in management of this patient? Select one: a. Await ENA result b. Await myositis panel result c. Bronchoscopy with bronchoalveolar lavage d. Commence immunosuppression e. Start broad spectrum antibiotic therapy

The correct answer is (d) This lady has a severe dermatomyositis associated interstitial lung disease. In this particular case the patient has an anti-MDA5 antibody which is typically amyopathic (hence the negative CK) and a pneumomediastinum (a common complication of this disease). The constellation of skin and lung findings is sufficient for the diagnosis and therefore the antibody panel is not necessary before the commencement of treatment. A bronchoscopy is not required to make the diagnosis and would also be highly risky in the patient's current clinical circumstance.

CLS Resp Quiz An 80 year old lady presents to outpatients with chronic cough and dyspnoea and the attached chest CT. She has a past history of hypertension, dyslipidaemia, type 2 diabetes, chronic kidney disease and recurrent urinary tract infections. Which of the below medications is the most likely cause? Select one: a. Perindopril b. Atorvastatin c. Nitrofurantoin d. Metoprolol e. Metformin

This is a classic scan for nitrofurantoin induced lung disease. The RACP candidate would not be expected to know this classic scan appearance but should be aware of common causes of drug induced ILD (e.g. amiodarone, nitrofurantoin, methotrexate, newer biologic therapies).

Deltamed endocrine 67 year-old man presents with low grade fever, night sweats, weight loss z S Ca 2.96 mmol/L z PTH <3 ng/L z Hb 106 g/L z What test is not indicated? A. Urinary calcium B. CTchest/abdomen/pelvis C. TSH D. IFE, EPG E. PTHrP

Urinary calcium irrelevant if PTH independent

CLS Quiz Resp Non-invasive ventilation in neuromuscular disease Select one: a. Is most effective when commenced electively for incipent respiratory failure as opposed for critically ill presentations in respiratory failure b. Is most effective in motor neurone disease when commenced in patients with normal or mildly impared bulbar function c. Is most effective in patients with chest wall deformities d. a,b,c above e. None of the above

abc above

CLS GI Quiz Which one of the following statements about the epidemiology of coeliac disease is correct? Select one: a. Prevalence is rising primarily due to increased awareness and testing b. Infectious agents, such as Pseudomonas and rotavius, are linked to the development of coeliac disease c. Higher socioeconomic status has been associated with a lower risk of coeliac disease development d. The global prevalence of coeliac disease is approximately 0.3% e. It is now most commonly diagnosed in childhood

b

CLS GI Quiz Which one of the following statements about the diagnosis of coeliac disease is correct? Select one: a. Symptomatic improvement on a gluten-free diet is diagnostic b. Gluten-specific IgE levels are useful to screen for coeliac disease c. A high titre of anti-transglutaminase antibodies has strong positive predictive value for coeliac disease d. HLA-DQ genotyping is unreliable if a person has been on a gluten-free diet for several months or more e. Small intestinal villous atrophy is pathognomonic for coeliac disease

c

CLS ID Which of the following antibiotics is most likely to be equally effective administered orally at similar doses to intravenous administration? A) Flucloxacillin B) Penicillin VK C) Cephalexin D) Clindamycin E) Erythromycin

clindamycin

Dun Pharm 5-10% of caucasians have reduced capacity to metabolise drugs via CYP2D6 system. This is most likely to be relevant in rx using which drugs? A. cyclosporin (renal tox) B. codeine (failure of analgesia) C. hydralazine (drug lupus) D. metoprolol (severe brady) E. panadol (hep tox)

codeine

Deltamed Rheum An 80 year old woman is admitted to hospital with a 4 week history of new onset, unilateral, temporal headache, jaw claudication and fatigue. Today she had a self-limiting episode of monocular visual loss lasting 30 minutes that has now improved. ESR is 95, CRP 140. CT Brain is normal. ɵ The next appropriate step in her management should be: a) Organise Temporal Artery Biopsy (TAB) and await result before treating b) Organise TAB and commence high dose oral Prednisolone 50mg daily c) Commence aspirin and organise carotid Doppler, echocardiogram d) Organise TAB and commence intravenous methylprednisolone 500mg-1g daily for 3 days e) MRI Brain before initiating any further treatment

d.

Deltamed rheum A 45 year old man presents with three months of malaise, fatigue, arthralgia and weight loss. In the past 2 months he has developed nasal crusting, epistaxis and hearing loss. Today he has had two episodes of haemoptysis. ɵ On examination there is a saddle nose deformity ɵ Investigation results: - CXR - diffuse infiltrate with two large cavitating nodules - Urine - >100 dysmorphic erythrocytes with 4+proteinuria - CRP 180, ESR 110, Creatinine 200 micromol/L ɵ What is his auto-antibody screen most likely to show? a) pANCA/MPO positive b) Normal/ no abnormality c) dsDNA positive d) cANCA/PR-3 positive e) SS-A/SS-B positive on ENA

d.

CLS GI Quiz Which one of the following statements about the pathogenesis of coeliac is INCORRECT? Select one: a. Intra-epithelial lymphocytes can adopt a cytolytic phenotype that contributes to villous atrophy b. Intestinal B cells and plasma cells amplify inflammation in the coeliac lesion by presenting gluten to T cells c. Gluten-specific CD4+ T cells are HLA-restricted, long-lived and exhibit a pro-inflammatory phenotype d. Gluten is modified by the enzyme transglutaminase to become more immunogenic e. Transglutaminase antibody mediated complement activation and complex formation is a key cause of damage in the small intestine

e

CLS Quiz Resp Management of respiratory failure should include Select one: a. Use of cough assist devices when sputum clearance is impaired b. Use of extracorporeal support when PEEP is greater than 10 cm H2O c. Routine use of oxygen to improve mobility in exercise induced hypoxaemia d. Regular use of oxygen therapy for nocturnal episodic hypoxaemia e. a and c above

e

Dunedin Pharm Drug X has linear pharmacokinetics and a narrow therapeutic index and is cleared by both renal filtration (80%) and hepatic metabolism (20%). The standard daily dose required by an individual with normal renal and hepatic function to achieve a therapeutic response is 100 mg. Which one of the following is the best estimate of the daily maintenance dose that would be required in an elderly person with a creatinine clearance one quarter of normal? A. 20 mg. B. 25 mg. C. 30 mg. D. 35 mg. E. 40 mg.

(1 - Fu) + Fu x (pt gfr/reference) (0.2) + (0.8 x 0.25) = 0.2 + 0.2 = 0.4 x 100mg 40mg

DM Resp For a patient with severe COPD and pulmonary hypertension, which of the following is indicated? A. Bosentan B. Epoprostenol C. Pulmonary rehabilitation D. Riociguat E. Sildenafil

C

CLS Geris The timed get up and go test in the elderly is used to quantify which of the following? 1. Falls risk 2. Balance 3. Mobility 4. Urinary incontinence 5. Dementia

1. Falls risk

DM PT ID3 3. The '5 Moments for Hand Hygiene' approach has been adopted by the World Health Organisation and defines the key moments when healthcare workers should perform hand hygiene. Which of the following would NOT be a moment for hand hygiene? A. Before performing a bedside procedure in which sterile gloves are worn B. After examining a patient when hand hygiene was performed prior to examining the patient C. After handing patient urine bottle D. Entered patient's room but did not touch environment

. ANS: D • The 5 moments of hand hygiene are: - before touching a patient - before clean/aseptic procedures - after body fluid exposure/risk - after touching a patient - after touching patient surroundings • Hand hygiene is vitally important in preventing healthcare-associated infections and should be practiced both before and after seeing a patient. • Hands should also be cleaned before and after wearing gloves. • Whenever in doubt you should perform hand hygiene.

CLS Geris 82 year old man presents with difficulty walking due to osteoarthritis in his right hip. When prescribing a single point stick as a walking aide for him, the most appropriate instruction regarding use of the cane is: Hold the cane: 1. In the right hand and advance the SPS with the right leg 2. In the right hand and advance the SPS with the left leg 3. In the left hand and advance the SPS with the right leg 4. In the left hand and advance the SPS with the left leg 5. In the right hand and advance the SPS after both feet have advanced

. In the left hand and advance the SPS with the right leg ← hold the SPS in the "good" hand and advance the SPS with the "bad" leg

CLS ID A 50 year old woman undergoes kidney transplant with ATG induction . She is CMV seronegative but her donor is CMV seropositive and she is commenced on valganciclovir prophylaxis at an appropriate dose of 450mg a day based on a eGFR of 50ml/min. After two months she becomes quite neutropenic. What is the best method of prevention moving forward? 1. Stop valganciclovir and switch to preemptive monitoring, with a plan to start treatment when the VL climbs above 1000 IU/ml (plasma) 2. Dose reduce the valganciclovir to 450mg alternate days 3. Switch prophylaxis therapy to marabivir 4. Commence GCSF 5. Check CMV interferon gamma release assay to guide duration of prophylaxis

1

CLS Resp Bronch lecture • The risk of death occurring as a result of bronchoscopy is - 1 in 1000 cases - 1 in 2500 cases - 1 in 500 cases - 1 in 300 cases

1 in 2500

DM PT H1 1. A 24-year-old female presents to the Emergency Department generally unwell with symptoms of abdominal pain, headaches, diarrhoea and fevers. She has no past medical history and is not on any regular medications. She has not travelled overseas recently. Her initial observations show she is hypotensive with a blood pressure of 90/50mmHg, tachycardic at 130bpm and febrile at 38.3C. Her blood tests show: hb 103, plt 60, macrocytosis, creat 120 Soon after arrival her conscious state declines and she develops seizures and is intubated. An ADAMTS13 level is performed and returns at <10%. What is the most appropriate initial treatment? A. Therapeutic plasma exchange B. Eculizumab C. Rituximab D. Fibrinogen concentrate

1. ANS: A • ADAMTS13 <10% is diagnostic of TTP (compared to other causes of microangiopathic haemolysis eg. Atypical Haemolytic Uraemic Syndrome) • ADAMTS13 cleaves ultralarge vW multimers into smaller components, decreasing activity. • Reduction ADAMTS13 through auto-antibodies (autoimmune disorder) • Circulating ultralarge vW multimers • Unrestricted platelet aggregation and thrombi formation- especially in the microvasculature due to high shear stress, but ± also large vessels (organ dysfunction) • The most appropriate initial management is plasma exchange (remove ULvwF monomers)

DM PT Resp1 1. A 60-year-old woman admitted with acute pancreatitis has a MET (Medical Emergency Team) call because of dyspnoea and falling room air SpO2 to 82%. Her respiratory rate is 32 breaths per minute. She is commenced on supplemental oxygen with a FiO2 0.35. ABG's taken on oxygen with FiO2 0.35 demonstrate pH 7.36, PaO2 60mmHg, PaCO2 38mmHg. Her CT chest is shown below: Which of the following initial therapies is the most appropriate? A. Continue oxygen via nasal prongs B. High flow oxygen therapy C. Invasive ventilation D. Non-invasive ventilation using BiPAP

1. ANS: B • This scenario describes acute hypoxaemic respiratory failure/ARDS pattern in a 60- year-old patient with a recent precipitating event for ARDS (PaO2/FiO2 < 200mmHg). • The patient may yet require intubation and invasive ventilation and whilst this should not be unnecessarily delayed high flow oxygen therapy is the appropriate initial therapy. • In a randomised open label study comparing standard oxygen, high-flow oxygen therapy and NIV via full face mask in Type 1 respiratory failure 90-day mortality was significantly lower in patients treated by HFOT alone (12%) than with NIV (28%). • Patients with hypercapnoea were excluded • Inclusion criteria PaO2/FiO2 < 300mmHg

DM PT Rh1 1. A 63-year old man is admitted to hospital with pneumonia. He has a past history of ischaemic heart disease, diet controlled type II diabetes, hypertension and hypercholesterolaemia. As part of his admission blood tests his uric acid is tested and is found to be elevated at 570umol/L The most appropriate treatment of the uric acid result is: A. Allopurinol B. Colchicine C. No therapy D. Prednisolone

1. ANS: C NB: 10-20% of people in the industrialised nations have hyperuricaemia McGill N/. Treating to target in gout: simple steps towards effective long-term control. Medicine Today 2016; 17(8): 27 - 36

DM PT P1. A 70kg man is on stable therapy with a drug administered 100mg daily orally. The drug is eliminated via first-order kinetics and has the following pharmacokinetic properties: Oral bioavailability: 80% Volume of distribution: 0.5L/kg Clearance: 3L/hr Fraction excreted unchanged: 30% Which of the following will decrease the half-life of the drug most? A. Change the regimen to 50mg daily orally. B. Change the regimen to 100mg daily intravenously. C. The man's renal function reduces by 10%. D. The man gains 10% in weight.

1. ANS: C The man's renal function reduces by 10% t 1/2 = Vd x 0.693 / clearance

DM PT R1 1. A 72-year-old man is admitted to hospital with biliary colic. He has a history of type 2 diabetes mellitus, hypertension and hyperlipidaemia. His medications include aspirin, metformin, empagliflozin, perindopril, atenolol and simvastatin. He undergoes a laparoscopic cholecystectomy the next day. That evening, he develops abdominal pain and nausea and then becomes progressively drowsy. He is afebrile, blood pressure is 132/78mmHg and his blood glucose level is 6.4mmol/L. What is the most appropriate initial investigation? A. CT abdomen with contrast B. ECG C. Bloods for gases to assess pH and ketones D. Serum lipase

1. ANS: C • Euglycemic ketoacidosis • SGLT2 inhibitors common association with euglycemic ketoacidosis • ?? Related to inhibition of insulin release • Metabolic acidosis, high anion gap - ketones • Risk - fasting, surgery, infection • Cease sGLT2 inhibitors 72 hours prior to planned surgery, restart once normal oral intake resumed

DM PT Neu1 1. A 32-year-old woman with a history of migraine with aura presents with three episodes over 5 days of new sudden, excruciating headaches, reaching peak intensity within seconds. The first episode occurred at orgasm, the second apparently spontaneously, the third with sneezing. She describes a diffuse headache, without localization, accompanied with nausea and photosensitivity, lasting around 30 minutes in total with gradual spontaneous resolution over that time. She reports the character of these headaches is distinctly different from her prior migraine headaches. What is the most likely diagnosis? A. Aneurysmal subarachnoid haemorrhage B. Primary angiitis of the nervous system C. Recurrent migraine D. Reversible cerebral vasoconstriction syndrome

1. ANS: D More common women 20-50ys • reversible multifocal narrowing of the cerebral arteries • thunderclap headache, focal neurologic deficits, seizure Usually benign (major stroke/death in minority) Avoid vasoconstrictors eg triptans RCVS associated with: • recurrent thunderclap headaches • superficial location and small quantity of subarachnoid blood • widespread, symmetric vasoconstriction RCVS angiography: • smooth, tapered narrowing followed by abnormal dilated segments of secondand third-order branches (sausage) RCVS presentation: • rarely recurs • Angiographic abnormalities persist for several weeks. Aneurysmal subarachnoid haemorrhage • major consideration in the differential of RCVS • Thunderclap headaches, subarachnoid blood, and cerebral artery narrowing • Primary angiitis (PACNS): • insidious progressive clinical course • chronic headaches - thunderclap headache rare • angiographic appearance of irregular arterial narrowing • Migraine: • differential diagnosis of RCVS • Normal angiography • Thunderclap uncommon • Beware misdiagnosis of migraine (vasoconstrictors à stroke). •

Deltamed Rheum Which of the following are predictors of severity/erosive damage of RA? A. High titre RF B. serum TNF levels C. presence of antiCCP D. erosions E. CRP F. ESR G. swollen joint count H. High disability HAQ score I. Rheumatoid nodules J. HLA DR4 Which of the following indicate RA activity?

1. D (erosions) > C (anticcp) > A (RF) 2. CRP, ESR, swollen joint count

DM Mock 12. Which of the following is not associated with increased risk of progression to end stage kidney disease in IgA nephropathy? A. Degree of proteinuria B. Blood pressure C. Degree of haematuria D. Estimated GFR

12. ANS: C Degree of haematuria https://www.ncbi.nlm.nih.gov/pmc/articles/PMC3962373/

DM PT EBP5 10. The following is the modified abstract from a randomised controlled trial recently published in the New England Journal of Medicine. Background: The timing and indications for surgical intervention in asymptomatic patients with severe aortic stenosis remain controversial. Methods: In a multicenter trial, we randomly assigned 145 asymptomatic patients with very severe aortic stenosis to early surgery or to conservative care. The primary end point was a composite of death during or within 30 days after surgery Results: In an intention-to-treat analysis, a primary end-point event occurred in 1 patient in the early-surgery group (1%) and in 11 of 72 patients in the conservative-care group (15%) (hazard ratio, 0.09; 95% confidence interval, 0.01 to 0.67; P = 0.003). Which of the following statements is CORRECT? A. The primary result was statistically significant at the 5% level of confidence. B. The primary result was highly precise. C. The probability of a Type I statistical error was 0.003%. D. All of the above

10. ANS: A A. The primary result was statistically significant at the 5% level of confidence. B. The primary result was highly precise. [wide 95% confidence interval] C. The probability of a Type I statistical error was 0.003%. [0.3%] D. All of the above.

DM PT Neuro10 10. Which of the following is a recognised risk factor for Sudden Unexpected Death in epilepsy (SUDEP)? A. 3 or more generalised tonic clonic seizures per year B. Carbamazepine or phenytoin levels that are above the reference range C. Duration of epilepsy D. Monotherapy vs polytherapy

10. ANS: A • "Sudden unexpected death in epilepsy incidence rates and risk factors," Neurology April 24, 2017, • https://www.aan.com/Guidelines/home/GetGuidelineContent/852

DM PT ID10 10. A 42-year-old woman presents with acute onset of right thigh and buttock pain and high fevers. She has no significant past history. On examination, extensive oedema, erythema and bullae formation are noted in the region of the pain. She is taken for urgent surgery and blood cultures drawn on admission are positive for Clostridium septicum. What is the most likely original source of the infection? A. Seeding from colon B. Local inoculation C. Ingestion from contaminated food D. Endometritis

10. ANS: A • Clostridial species are Gram positive bacilli. • Infections caused by these organisms can cause a variety of clinical syndromes: - C. septicum is associated with spontaneous myonecrosis/gas gangrene resulting from haematogenous spread, typically from colonic seeding from an undiagnosed adenocarcinoma. - C. perfringens also causes myonecrosis/gas gangrene (more usually secondary to trauma), foodborne gastroenteritis and endometritis. - C. botulinum is responsible for causing botulism, while C. tetanii causes tetanus and both can be associated with ingestion of contaminated food or inoculation into wounds. - C. sordelli is classically associated with endometritis. - C. difficile causes a colitis in the context of antibiotic exposure and can rarely have extra-intestinal manifestations. • Therapy for C. septicum and C. perfringens causing myonecrosis is high-dose penicillin and clindamycin. • Patients with C. septicum bacteraemia should be evaluated for an underlying gastrointestinal malignancy

Deltamed PT10 10. Which of the following is an indication for dose reduction of apixaban in a patient with non-valvular atrial fibrillation? A. Age > 75 years old B. Cr >100umol/L C. Weight <60kg D. History of falls

10. ANS: C Criteria for dose reduction of apixaban are 2 or more of: age >80, Cr>133umol/L, weight <60kg.

DM PT Resp10 10. A 50-year-old patient presents with a 5cm left upper lobe mass. EBUS-TBNA sampling of the contralateral hilar lymph nodes confirms a diagnosis of squamous non-small cell lung cancer. Tumour PD-L1 burden is 60%. Which of the following treatments is most likely to be offered in the first instance? A. Docetaxel B. Gemcitabine and carboplatin C. Pembrolizumab D. Pneumonectomy

10. ANS: C • Gemcitabine and carboplatin are offered as second line therapy (answer b is incorrect) and docetaxel as third line therapy (answer a is incorrect) in these patients. • Surgical management is not recommended in patients with advanced non-small cell lung cancer (answer d is incorrect). • Pembrolizumab is recommended in those patients with advanced (stage IIIB and above) squamous cell lung cancer with PD-L1 (programmed cell death ligand 1) burden above 50% (answer c is correct). Pembrolizumab for advanced NSCLC with PD-L1>50% • RCT of pembrolizumab monotherapy versus standard platinum-based chemotherapy • Median PFS, 10.3 versus 6 months; HR 0.50, 95% CI 0.37-0.68 • Objective response rates 45% with pembrolizumab vs 28% with chemotherapy • At 25-month follow-up improved median OS with pembrolizumab • 30 versus 14.2 months with chemotherapy; HR 0.63, 95% CI 0.47-0.86 J Clin Oncol. 2019;37(7):537. Epub 2019 Jan 8

DM PT Geris 10 10. A 92-year-old man with end stage Parkinson's Disease Dementia is brought in from his nursing home with reduced level of consciousness. He is conversing at times but confused and requesting ice cream. Breathing is shallow, with oxygen saturation 85-90% on room air. A CT brain reveals a large subdural haematoma, with midline shift. His wife of 50 years agrees with medical recommendations for palliative goals of care. However, his daughter from his first marriage has arrived from the UK and requests active treatment of his condition. There is no pre-existing Advanced Care Directive. What is the MOST appropriate next step? A. Commence intravenous fluids to allow time to come to a decision B. Commence oral antibiotics to alleviate his respiratory symptoms C. Commence a supervised diet and allow him to eat and drink D. Commence total parenteral nutrition (TPN) as he cannot maintain nutrition otherwise.

10. ANS: C • In this situation, we are obliged to provide palliative and not medical care • Palliative care includes the reasonable provision of food and water. Medical care is any treatment that needs to be delivered by a trained person

DM PT Onc10 For each of the following scenarios (Q9 and Q10), choose the most appropriate answer from the following options. A. Loperamide B. Metronidazole C. Ciprofloxacin D. Ganciclovir E. Atropine F. Infliximab G. Mycophenolate H. Budesonide 10. A 45-year male with metastatic colorectal cancer is receiving FOLFIRI (5FU, irinotecan, leucovorin) every 2 weeks. He experiences diarrhea shortly after the chemotherapy infusion which is sudden in onset. Which anti-diarrheal medication should he receive prophylactically before his next cycle of chemotherapy?

10. ANS: E Atropine is often useful for the early diarrhea associated with irinotecan. It can be given IV or subcutaneously prophylactically. It is typically given as premedication. Loperamide is also reasonable for irinotecan induced diarrhea, but in this case, the diarrhea is acute, immediately post the infusion. Loperamide would not be used prophylactically prior to the chemotherapy. Metronidaozole is for C-diff colitis. It is important to exclude this. Ganciclovir is for CMV colitis which is not the case. Mycophenolate is an immunosuppressant.

DM Mock 11. Which cancer's incidence is significantly elevated by BRCA1/2 mutation? A. Glioblastoma B. Testicular cancer C. Renal cell carcinoma D. Prostate cancer

11. ANS: D BRCA 1/2 related malignancies include cancers of the breast, pancreas, ovary, prostate.

DM Mock 13. All of the following patients should be tested for Alpha1-Antitrypsin deficiency EXCEPT A. A 30-year-old male with asthma that is well controlled on inhaled corticosteroid and salbutamol therapy B. A first-degree relative of a person with confirmed Alpha1-Antitrypsin deficiency C. A 50-year-old female with c-ANCA vasculitis who is a current smoker D. A 60-year-old male with COPD and a 50-pack-year smoking history

13. ANS: A AAT deficiency (proteinase inhibitor genotype MZ) has been associated with other clinical conditions including ANCA-associated vasculitis (answer C is incorrect), gallstone disease, emphysema (answer D is incorrect) and liver cirrhosis. Patients with poorly responsive asthma should be considered for AAT deficiency testing (answer A is correct as this patient has well controlled asthma). It is recommended that first-degree relatives of people with AAT deficiency should undergo testing (answer B is incorrect). Reference: Strnad, P, McElvaney, N, Lomas, D. Alpha1-Antitrypsin Deficiency. N Engl J Med 2020; 382:1443-55.

DM Mock 15. Proton pump inhibitors such as esomeprazole may interact with which of the following cardiac medications? A. Aspirin B. Beta blockers C. Clopidogrel D. Diuretics

15. ANS: C Both clopidogrel and PPIs are prodrugs that use the same metabolic pathway (CYP2C19) for metabolism - hence they may compete. Multiple systematic reviews have shown that when PPIs and clopidogrel are routinely co-administered, rates of major adverse cardiovascular outcomes (stent thrombosis, requirement for revascularization) increase. NB that pantoprazole has the lowest CYP2C19 affinity and has been shown to not increase rates of MACE when assessed on its own. It is typically the go-to PPI when there is an indication for both DAPT and PPI.

DM Mock 16. The usual dose of a drug given orally is 50mg daily for people with normal renal function. Its oral bioavailability is 0.50 and its fraction excreted unchanged is 0.80. In a person with renal impairment whose eGFR has halved, what is the ideal renallyadjusted dose for once daily oral dosing? A. 10mg B. 20mg C. 30mg D. 40mg

16. ANS: C 80% of the usual dose that is renally excreted needs to be halved. Hence the renally-adjusted dose = (40mg*50%) + 10mg = 30mg

DM Mock 17. During pregnancy, which maternal condition confers the highest risk of foetal thyrotoxicosis? A. Gestational thyrotoxicosis B. Toxic multinodular goitre C. Graves' disease D. Subacute thyroiditis

17. ANS: C Due to the transplacental transfer of TRABs which can stimulate the foetal thyroid gland, just as they do the mothers gland.

DM Mock 18. A 60-year-old female attends the emergency department with abdominal distention and early satiety. She undergoes a bedside ultrasound which reveals ascites. Diagnostic paracentesis reveals an albumin of 11g/L, total protein of 22g/L (reference <30) and 150 white cells (50% polymorphonucleocyte cells, PMNs). She has a serum albumin of 28g/L. What is the next step in management? A. Assess LDH and arrange a PET scan to assess for peritoneal malignancy B. Arrange an echocardiogram to assess for cardiogenic ascites C. Commence therapeutic antimicrobial cover for spontaneous bacterial peritonitis (SBP) D. Workup for chronic liver disease

18. ANS:D. This patient has a serum albumin to ascites gradient (SAAG) of >11. The presence of a total protein <25g/L is indicative of portal hypertension, and the finding of <250 PMNs makes SBP (option C) unlikely. Option A is ruled out by a SAAG >11. In terms of option B, a SAAG >11 is seen in cardiogenic ascites, but protein is often above the reference level (i.e., >25g/L).

DM Mock 19. Which of the following variants would most likely result in a single amino acid change? A. Single nucleotide deletion B. Substitution of the 2nd nucleotide in a codon C. Substitution of the 3rd nucleotide in a codon D. 6 nucleotide deletion

19. ANS: B A single nucleotide deletion will interrupt the downstream reading frame and cause a protein truncating change. A 6-nucleotide deletion will disrupt 2 amino acids. Substitution of the 2nd nucleotide in a codon will result in a single amino acid change. Substitutions of the 3rd nucleotide is less likely to cause an amino acid change than substitutions of the 1st or 2nd nucleotide because of redundancy for amino acid codes with the 3rd nucleotide being more variable in its coding for the same amino acid.

Dun renal 77 yr old woman underwent a renal biopsy for renal insufficiency and proteinuria. She has a past history of a gastric lymphoma treated with radiation and chemotherapy (CHOP) with no evidence of relapse. She had received 2 units of blood. She has a long history of treated hypertension. Creatinine 0.19 mmol/l, urinalysis 4+ protein, 2+ blood and dysmorphic RBC on microscopy, ANCA ‐ve, normal LFTs but Hep C +ve. Cryoglobulins were negative. Haemoglobin 102 gmll with schistocytes evident on the blood film. The renal biopsy is shown. IF was weakly positive for focal capillary wall IgM, C3 and fibrinogen 1.Therapy with an ACEI is contra indicated due to hypertensive nephrosclerosis. 2.Endothelial injury has contributed to the renal lesion and anaemia. 3.This is a paraneoplastic phenomenon. 4.The proteinuria is due to a drug induced interstitial nephritis. 5.Hypertensive nephrosclerosis produces a microangiopathic anaemia.

2 IF supports diagnosis of either chronic thrombotic microangiopathy or less likely HCV associated MPGN • Focal capillary aneursyms with entrapped RBC plus severe and focal obliterative arteriopathy favours chronic thrombotic microangiopathy

Deltamed Haem2 A 42‐year‐old female has pre‐operative coagulation studies performed prior to elective bariatric surgery. She has a BMI of 35 and has undergone a dietary and exercise program prior to consideration. Her past medical history includes hypertension on an ACE inhibitor, type 2 diabetes on oral hypoglycaemics and gout. Her bleeding history is significant for menorrhagia. Results return‐ Platelets 160x109/L (150‐400) INR 1.2 (0.9‐1.2) APTT 52 secs (24‐35) Fibrinogen 2.1 (2‐4) 50:50 mix or patient and normal plasma does not correct the APTT. What is the most likely cause? 1. Vitamin K dependent factor deficiency from poor dietary intake 2. Lupus anticoagulant 3. Laboratory artefact 4. Type 2N Von Willebrand's disease

2. Would expect higher INR with 1

CLS Neuro LEcture (enc) 62 yo man brought to ED after a witnessed tonic clonic seizure - Preceding 3 days - fever, headache (severe holocranial ache, mild neck pain), myalgias and lethargy - Past hx: healthy; social: retired electrician, lives with wife; recent travel for a camping trip to Colorado Exam: Awake but confused, oriented only to person Temp: 37.8̐ᴼC; pulse 98/ min: BP: 134/92 mm Hg; RR: 22/ min Cardia, abdomen, respiratory systems: NAD Mild pain on deep palpation of calf and thigh muscles Cranial nerves: intact Muscle strength: markedly diminished in lower limbs, somewhat diminished in upper limbs DTRs: absent; plantar response - flexor bilaterally FBC: mild leukocytosis 11800/uL; urine 1+ proteinuria Normal liver and renal function tests Non enhanced CT brain: normal LP - CSF analysis OP: 310 cm H2O Cells: 90/mm3, lymphocytes Protein: 105 mg/ dL; Glucose: 3.27 mmol/L Gram stain: no organisms seen; Culture - no growth Patient is admitted to ward Empiric Acyclovir is begun Which of the following is the most likely diagnosis? 1. Progressive Multifocal Leukoencephalopathy (PML) 2. West Nile virus encephalitis 3. Pneumococcal meningitis 4. Dengue fever 5. Herpes Simplex virus Encephalitis (HSE)

2. West Nile Virus Encephalitis - most common cause of epidemic viral encephalitis in the USA

DM PT Geris 2 2. Which of the following is most likely to be found in late-onset Alzheimer's dementia? A. Decreased CSF Aβ42 B. Hippocampal hypermetabolism C. Positive family history D. Decreased CSF tau

2. ANS: A Findings on investigation in AD MRI changes - most classic is hippocampal atrophy, out of proportion with atrophy in other parts of brain. FDG-PET - hypometabolism in precuneus, lateral temporal lobes and hippocampi. Lumbar puncture (not done routinely in Au) - Low cerebrospinal fluid (CSF) Aβ42 (or Aβ42:Aβ40 ratio), increased Tau

DM PT G2 2. Hepatocellular carcinoma can be diagnosed non-invasively due to characteristic features on imaging. What is the underlying process that causes this appearance? A. Angiogenesis resulting in unpaired hepatic arteries B. Nodular regeneration C. Focal fatty sparing D. Calcification

2. ANS: A HCC - angiogenesis and imaging criteria • HCC is a hypervascular tumour mainly supplied by hepatic arteries (normal liver primarily supplied by portal vein) Hepatocarcinogenesis characterized by angiogenesis and the development of unpaired hepatic arteries (not accompanied by portal tract) This results in characteristic findings on multiphase imaging with early arterial enhancement and early "washout" in portal venous phase or delayed phase (appearing indistinct or hypoattenuating) • Rim enhancement may be seen on delayed post-contrast images • Larger lesions can be heterogenous due to fibrosis, necrosis or calcification • Focal fatty change or sparing may be seen due to perfusion abnormalities from arterioportal shunts

DM PT Onc2. You are reviewing a 53-year-old man who completed radical chemoradiotherapy for left tonsillar squamous cell carcinoma five days ago. He has a 50 pack-year smoking history and history of type 2 diabetes for which he is currently on metformin, and stable ischaemic heart disease. His blood sugars are between 4-8 mmol/L. On examination, he has ulcers under his tongue and over the oropharynx. He is unable to tolerate solids and is meeting 50% of his nutritional requirement on a liquid diet. He has lost > 10% of his body weight over the last 3 weeks. What treatment would you recommend? A. Morphine mouth wash PRN and titrate opiate dose, gastrostomy or nastrogastric tube and commence enteral feeding. B. Morphine mouth wash PRN, amphotericin B lozenges, gastrostomy and commenced enteral feed. C. Morphine mouth wash PRN and titrate opiate dose, sucralfate mouthwash amphotrecicin B lozenges. D. Morphine mouth wash PRN, chlorexadine mouth wash PRN.

2. ANS: A Incidence of World Health Organization (WHO) grade 3 or 4 oral mucositis in patients receiving head and neck radiation (e.g. 60-70 Gy) to the oral cavity approaches 85%, but all treated patients have some degree of oral mucositis. Grade 0 = no oral mucositis Grade 1 = erythema and soreness Grade 2 = ulcers, able to eat solids Grade 3 = ulcers, requires liquid diet (due to mucositis) Grade 4 = ulcers, alimentation not possible (due to mucositis) Mucositis is one of the prime limiting factors of chemoradiation for advanced head and neck carcinoma. The oral pain associated with the lesions frequently leads to the need for enteral nutritional support with or without use of a feeding tube or gastrostomy, as well as use of opioids, with the objective of maintaining dose intensity throughout the entire radiation regimen. Examination does not reveal any evidence of oral thrush so the use of antibiotics or antifungals is not recommended. There is insufficient evidence to suggest the use of sucrafate mouth wash

DM PT R2 2. Which of the following is not a factor when considering commencement of tolvaptan in a patient with autosomal dominant polycystic kidney disease? A. Frequency of cyst infections B. Liver function tests C. Rate of decline of renal function D. Renal volume, as assessed on MRI

2. ANS: A • Tolvaptan orally active V2 Vasopressin R antagonist • Trials have shown a decrease in the decline of renal function in moderate to severe APCKD • Response is sustained • Hepatotoxicity (idiosyncratic), thirst and polyuria common side effects • Factors used to assess indication of Tolvaptan Total kidney volume, increase kidney volume over time, eGFR, decline of eGFR, genetics and family history • PBS criteria - Clinical criteria: Patient must have an estimated glomerular filtration rate (eGFR) between 30 and 89 mL/min 1.73 m2 at the initiation of treatment with this drug for this condition, AND Patient must have or have had i)rapidly progressing disease at the time of initiation of this drug for this condition. OR ii) an average decline in eGFR of greater than or equal to 2.5 mL/min/1.73 m2 per year over a five-year period

DM PT E2 A 77-year-old frail woman with a history of early dementia, well controlled hypertension (average 130/80 mg/Hg), osteoarthritis and previous cholecystectomy presents with selflimiting abdominal pain. Medications include verapamil, vitamin D, and paracetamol osteo. CT abdomen shows a 2 cm adrenal lesion (see below). Attenuation was minus 10 Hounsfield units. Biochemical evaluation reveals normal plasma free metanephrines, serum cortisol of 20 nmol/L following administration of 1 mg dexamethasone the evening prior, serum potassium 3.9 mmol/L (normal range 3.5-5.0) plasma renin activity is 3.0 ng/mL/hour (0.6-4.3), and serum aldosterone 152 pmol/L (110-580). What is the best next step? A. Repeat CT and hormonal studies in 6-12 months B. No further follow-up C. Repeat CT in 6-12 months, and repeat hormonal studies only if change in clinical picture D. Change verapamil to low dose prazosin, and repeat renin/aldosterone 2 weeks later.

2. ANS: B Adrenal Incidentaloma • Pre-contrast Hounsfield units < 10, '100%' benign (systematic review)1 • 95% of malignant lesions are > 4cm • Smooth contour, homogenous=benign • European Endocrine Society guidelines1 : No further follow-up for non-functioning adrenal adenoma <10 HU & < 4cm 1 Fassnacht et al, European Journal of Endocrinology 2016

DM PT P2 2. Of relevance to pharmacokinetics, which of the following statements is TRUE regarding pglycoprotein? A. Co-administration of an inhibitor will decrease levels of substrates. B. It restricts drug absorption via the gastrointestinal epithelium. C. It facilitates drug excretion through bile. D. It facilitates transport of drugs from the portal to the systemic circulation

2. ANS: B It restricts drug absorption via the gastrointestinal epithelium

DM PT Resp2 2. A 28-year-old depressed male is found unconscious by paramedics in his petrol fuelled car after being alerted by his family who have not been able to make contact with him. On examination he is flushed and has a Glasgow coma scale score of 11, his respiratory rate is 12 breaths per minute and his pulse oximetry is 98% on oxygen via Hudson mask. Which of the following statements is the most correct? A. Supplemental oxygen should be reduced to target SpO2 88-92% to avoid hypercapnia B. Supplemental oxygen should be administered at 15L/min via reservoir mask C. PaO2 is likely to be low D. Carboxyhaemoglobin is likely to be low

2. ANS: B • This scenario describes a case of likely carbon monoxide poisoning rather than hypercapnia (answer a is incorrect). • In cases of carbon monoxide poisoning, an apparently 'normal' oximetry reading may be produced by carboxyhaemoglobin, so it is advised to aim at an oxygen saturation of 100% and use a reservoir mask at 15 L/min (answer b is correct irrespective of the oximeter reading • PaO2 which may appear normal (answer c is incorrect). • The measurement of carboxyhaemoglobin (HbCO) on blood gas may confirm a suspected diagnosis however low levels do not rule out exposure to carbon monoxide gas (answer d is incorrect).

DM PT Neuro2 2. While at lunch, a 75-year-old man presents with sudden onset of left sided hemiparesis, with associated dysarthria and drowsiness. On examination, he has GCS 9, BP 170/110 mmHg, HR 86 bpm in sinus rhythm, afebrile, RR 16, with left hemiparesis, power 3/5 left upper and lower limbs. His CT brain is shown (below) - lobar haemorrhage What is pathologic examination in this patient LEAST likely to reveal? A. Cerebral microhaemorrhages: remnants of haemorrhage as 2 to 10 mm focal or multifocal areas of hemosiderin deposition. B. Cerebral vascular deposits biochemically similar to the material comprising senile plaques in Alzheimer disease C. Positive amyloid staining by Congo red in abdominal fat pad aspirate D. Superficial haemorrhage in the cortical sulci: 'superficial siderosis'

2. ANS: C Cerebral Amyloid Angiopathy Lobar haemorrhage Vascular amyloid deposits favour cortical vessels and largely spare white matter, deep gray matter, and the brainstem. Cerebellum may be a site of CAA-related haemorrhage. • CAA haemorrhages are significantly more likely to occur in the temporal and occipital than the frontal and parietal lobes. CAA-related haemorrhages: superficial location often extend into the subarachnoid space less frequently rupture into ventricles. basal ganglia, thalamus, or pons -> hypertensive microangiopathy CMB detected in 5-23% of older individuals. Higher risk of haemorrhage recurrence. Superficial haemorrhage in cortical sulci (superficial siderosis) Acute outcome may be better than in hypertensive haemorrhage

DM PT ID2 2. Antimicrobial stewardship aims at optimising our use of antimicrobials in order to improve patient outcomes, reduce adverse events and decrease costs. Which of the following is NOT an essential strategy for effective antimicrobial stewardship? A. Establish formulary restriction and approval systems to govern use of broad-spectrum / novel antimicrobials B. Ensure that microbiology laboratories use selective reporting of susceptibility results, in accordance with local antimicrobial prescribing guidelines C. Review antimicrobial prescribing and provide feedback to prescribers D. Adjust antimicrobial prescribing guidelines to minimise cost

2. ANS: D • Antimicrobial stewardship has become a central activity that now forms a mandatory part of safety and quality procedures in Australian hospitals, both in the public and private healthcare systems. • This push has been driven by the need to preserve the activity of antimicrobials, especially broad-spectrum agents such as carbapenems, and aiming to improve patient outcomes (e.g. through decreasing rates of Clostridium difficile infection), not cost considerations. • For example, daily ceftriaxone costs significantly less than QID benpen.

DM PT H2. An 83-year-old male presents to the Emergency Department from a nursing home with nausea and vomiting for several days and a new onset gastrointestinal bleed. Although his nursing home medication chart shows he is written up for apixaban 5mg BD, it is unclear whether his most recent doses have been dispensed, refused or not taken due to his current illness. Which coagulation test is the most useful to determine whether an anticoagulation effect from apixaban is present? A. Dilute thrombin time B. Prothrombin time (PT) C. Activated partial thromboplastin time (APTT) D. Anti-Factor Xa

2. ANS: D • Apixaban is a Factor Xa inhibitor. • Traditional coagulation profile testing (PT, APTT, Fibrinogen, TCT) with NOACs have variable effect from therapeutic drug levels, though can be normal. • A calibrated anti-Factor Xa can give reliable drug levels in the presence of the Xa inhibitors and heparins. • Thrombin time examines disorders of fibrin and the presence of heparin and dabigatran.

Deltamed Neuro The patient has been asked to make an "OK" sign with both hands. Can't make 1 with R hand. The likely lesion is in the: 1. Right posterior interosseous nerve 2. Right anterior interosseous nerve 3. Right deep branch ulnar nerve at Guyon's canal 4. Right median nerve at the wrist 5. Right C8 radiculopathy

2. RIght AIN Flexor pollicis longus

DM Mock 22. In which of the following patient populations is the measles, mumps and rubella vaccine safe to be administered? A. Received recent chemotherapy B. Solid organ transplant C. Asymptomatic HIV with CD4+ count of 250 cells/uL D. Receiving 25mg daily prednisolone for the last 16 days

22. ANS: C As per Australian immunization handbook, "use of live vaccines in people with immunocomprising conditions", https://immunisationhandbook.health.gov.au/vaccination-forspecial-risk-groups/vaccination-for-people-who-are-immunocompromised Option C - MMR vaccine is allowed to be given if "asymptomatic HIV" and CD4+ count equal or greater than 200 cells/uL (assume they are on therapy with virologic control and do not have OI if asymptomatic). MMRV is not recommended. Option D - ≥20 mg per day of prednisolone for ≥14 days - not safe, must wait at least 1 month before giving vaccine.

DM Mock 20. Which of the following is LEAST likely to be found in late-onset Alzheimer's dementia (LOAD)? A. Increased CSF Aβ42 B. Increased CSF tau C. Hypometabolism in the medial temporal lobe D. Amyloid deposition on PET Amyloid imaging

20. ANS: A CSF Aβ42 is usually decreased.

Dunedin Pharm In a subject taking part in a bioavailability study, the area under the plasma concentration versus time curve (AUC) achieved after an intravenous dose of 30 mg is 150 mg/hr/L, compared with an AUC of 200 mg/hr/L after an oral dose of 50 mg. The oral bioavailability of the drug in this subject is: A. 50%. B. 60%. C. 70%. D. 80%. E. 90%.

200/5 = 40 ________________ 150/30 = 50 = 4/5 = D

DM Mock 21. A 76-year-old male with metastatic lung cancer on single agent pembrolizumab presents with significant epistaxis requiring cauterisation and packing. His background is significant for gout on allopurinol and diabetes - not currently on medication. His investigations return Haemoglobin 92g/L (135 - 150) MCV 110fL (80-100) Platelets 60 x 109/L (150- 400) White cell count 11.2 x 109/L (4-11) Lymphocytes 9.9 x 109/L (1.2-4) Sodium 136 meq/L Potassium 4.5 meq/L Bicarbonate 25 Urea 10 Creatinine 65 umol/L (60-110) INR 2.3 (0.9-1.2) APTT 52 sec (28 - 38) Fibrinogen 0.9 g/L (2.0 - 4.0) The lab calls through a blood film showing 10 schistocytes per HPF. An urgent ADAMTS13 is performed which returns at 42% (70-160%). What is the most likely diagnosis? A. Immune mediated thrombocytopenia purpura (ITP) secondary to pembrolizumab B. Haemolytic uraemic syndrome C. Disseminated intravascular coagulopathy (DIC) from metastatic malig D. Malignancy associated thrombotic thrombocytopenic purpura (TTP)

21. ANS: C The low fibrinogen is consistent with DIC. The ADAMTS13 is >10% excluding TTP. ITP from pembrolizumab would not be accompanied by features of haemolysis. Renal function is normal making HUS less likely, and not associated with a low fibrinogen.

DM Mock 23. Serum sickness, due to ceclor, is due to antibodies to the ceclor-protein carrier; which results in the formation of immune complexes and activation of which part of the immune system? A. T-cells B. Complement C. Neutrophils D. Macrophages

23. ANS: B This is a bit of trick - it is complement activation, which is activated by the immune complexes, leading to deposition in small vessels an recruitment of neutrophils by the Fc-IgG receptor resulting in the tissue damage/inflammation. So technically, even though neutrophils is right, it's because the complement activation first. This is not specific to ceclor - but any drug causing serum sickness

DM Mock 24. Which risk factor has the least population-attributable risk for ischaemic stroke? A. Diabetes mellitus B. Hypertension C. Psychosocial factors: stress, depression D. Regular physical activity (moderate to strenuous, >= 4h/week)

24. ANS: A A. Diabetes mellitus (7.5%) B. Hypertension (45.7%) C. Psychosocial factors: stress, depression (15.1%) D. Regular physical activity (moderate to strenuous, >= 4h/week) (33.4%)

CLS Resp Bronch lecture When should low molecular weight heparin (LMWH) be discontinued before bronchoscopy - LMWH does not need to be discontinued - LMWH should be discontinued 48 hrs before procedure - LMWH should be discontinued on the day of procedure - LMWH should be discontinued 24 hrs before procedure

24hrs before

Deltamed Resp A 70yo man with severe COPD is admitted to hospital with an exacerbation of COPD requiring 24hrs of nonͲinvasive ventilation, oxygen therapy, oral steroids and oral antibiotics.After four days, he is discharged home with outpatient followͲup.What is his approximate 12Ͳmonth mortality risk after this episode? • 5% • 15% • 25% • 35% • 50%

25%

DM Mock 25. Which of the following regarding high grade osteosarcoma is correct? A. The peak incidence is between 20-30 years of age B. Multimodal treatment improves survival by 20 % compare to surgery alone. C. Curative treatment includes neoadjuvant chemotherapy, surgery, and chemoradiotherapy D. The limb will be amputated in the setting of a pathologic fracture

25. ANS: B The peak incidence of high-grade osteosarcomas is between 10-20.

DM Mock 26. A 45-year-old smoker is referred to the renal clinic for investigation of resistant hypertension. Despite daily combination medication with olmesartan 40mg, amlodipine 10mg and hydrochlorothiazide 12.5mg, his blood pressure on examination is 160/90, with no postural drop. Investigations show: Creatinine 113 Potassium 3.1 Urine protein:creatinine ratio 0.12 After correcting his hypokalaemia, which of the following is the most appropriate next step in diagnosing the cause of his hypertension? A. Measure serum catecholamines B. Renal doppler ultrasound of the renal arteries C. Measure plasma aldosterone and plasma renin activity D. Renal biopsy

26. ANS: C Measure plasma aldosterone and plasma renin activity The Management of Primary Aldosteronism: Case Detection, Diagnosis, and Treatment: An Endocrine Society Clinical Practice Guideline, Funder et al. 2016

DM Mock 28. In patients presenting with myositis, which of the following antibodies has the greatest association with malignancy? A. Anti-HMGCR B. Anti-MDA5 C. Anti-SRP D. Anti-TIF1y

28. ANS: D Reference: McHugh NJ and Tansley SL. Antibodies in myositis. Nature Reviews Rheumatology 2018; 14: 290 - 302. Rationale: The presence of Anti-HMGCR or anti-SRP is usually seen in patients with severe myositis with very raised creatinine kinase levels. Patients with Anti-MDA5 often have severe and progressive ILD, as well as severe cutaneous manifestations. Anti- TIF1g has the strongest association with malignancy.

DM Mock 29. You are undertaking a pulmonary hypertension workup on Cathy, a 40-year-old woman with a new and unexplained finding of severe pulmonary hypertension on echocardiogram. Which of the following represents the BEST set of investigations in terms of identifying a cause and staging for therapy? A. CTPA, connective tissue blood screen, right heart catheterisation, 6-minute walk test B. V/Q scan and CT chest, connective tissue blood screen, right heart catheterization, 6- minute walk test C. Transoesophageal echocardiogram, CTPA, connective tissue blood screen, right heart catheterization, 6-minute walk test D. CT chest, coronary angiogram, connective tissue blood screen, right heart catheterization, 6-minute walk test

29. ANS: B You will clearly do a connective tissue blood screen, right heart catheterization, and 6-minute walk test! The differentiator in the options is the chest imaging, and contrary to general perception, CTPA is not the gold standard here. The best selection is V/Q (to screen for chronic thromboembolic pulmonary hypertension) and CT chest (looking for interstitial lung disease).

Dun ID A 25 year old patient with AML developed neutropenic fever. One of the erythematous nonblanching nodules is biopsied and cultured and reveals Cryptococcus gattii. • Which of the following would likely not be effective? 1. liposomal amphotericin 2. amphotericin deoxycholate B 3. caspofungin 4. high dose fluconazole

3 Echinocandins NO cryptococcus or trichosporon, dimorphic coverage CCC Echinocandins - no activity against Cryptococcus • Inhibit 1,3 β glucan synthase • Cryptococcus (Zygomycetes)lack significant β-glucan in their cell walls - (1-3)-β-D-glucan test CANNOT screen for Cryptococcus - >Cryptococcal antigen

CLS ID An 85-year-old female presents following a series of falls. There is a history of cognitive decline over several months, on a background of macular degeneration, osteoarthritis, previous TIAs, urinary incontinence and recurrent UTI. She denies any urinary tract or other localising symptoms. On examination she is unkempt and slightly muddled, but orientated and answering questions appropriately. Temperature and other vital signs are normal, and there are no specific findings other than minor injuries. Basic blood tests, chest x-ray and ECG are unremarkable. Urinalysis shows the following: • Dipstick: Prot 1+, Leukocytes 2+, Blood 1+, Nitrite + • Microscopy: White cells 120 x 106 /ml, Red cells 10 x 106 /ml, Epithelial cells 40 x 106 /ml • Culture: E.coli 108 CFU/ml, Sensitivities pending What is the best management? 1. Stat dose of IV gentamicin 5mg/kg, followed by Cefuroxime 1.5g q8h, pending sensitivities. 2. Oral trimethoprim 300mg daily, pending sensitivities. 3. Disregard the result; observe without antibiotics. 4. Repeat the urinalysis, ensuring strict midstream collection technique; observe without antibiotics. 5. Repeat the urinalysis, ensuring strict midstream collection technique; commence empiric treatment with IV amoxicillinclavulanate 1.2g q8h pending further results.

3.

Dun renal •57 year male with a 3 month history of back pain for which he is taking diclofinac. Xray and bone scan were normal. Now found to have renal impairment, creatinine 0.2 mmol/l, previously normal; and proteinuria of 18 gm/24 hours without hypoalbuminuria or oedema. Serum protein electrophoresis showed no monoclonal spike. Renal ultrasound normal. A renal biopsy was performed. 1. This is NSAID drug‐induced interstitial nephritis producing nephrotic range proteinuria. 2. Management should consist of a low protein diet and an ACEI. 3. A bone marrow aspirate and trephine is indicated. (overflow proteinuria from myeloma) 4. His Hb1ac will be elevated. 5. He will be HLA B27 positive

3. Glomeruli essentially normal (cf AL and MIDD) • Prominent tubular casts ‐ brittle and fragmented, surrounded by tubular epithelium, neutrophils, and multinucleated giant cells. • Tubular basement membrane discontinuous allowing communication with interstitium

CLS Neuro lecture (enc) 20 yo woman was admitted for management of short-term memory deficits and generalized tonic-clonic seizures She initially reported partial motor seizures in the left lower extremity and dystonic movements Over the course of 1/52, she developed agitation, hallucinations and catatonia. Over the subsequent fortnight, she became further ill with decrease of level of consciousness and hypoventilation, requiring mechanical ventilation Past medical hx: healthy; no medications; no recent travel; no reported stress; no alcohol or illicit drug use; regular menstrual cycles Exam: Temp: 39ᴼC; pulse 132/ min: BP: 128/84 mm Hg Normal fundi; dilated pupils; pharmacological paralysis EEG diffuse slowing, no epileptiform abnormalities MRI B bitemporal signal changes CSF 150 cells; 68 mgs/dl; HSV PCR -ve; no organisms on Gram stain, OCB+ Which of the following is the next step in management? 1. Add empiric broad spectrum antibiotics, awaiting culture 2. Substitute acyclovir with Foscarnet 3. Consider steroids, immune globulins/ plasma exchange 4. Add oral thyroxine 5. Start Olanzepine, summon mental health team

3. Rx - IV methylprednisolone followed by IVIg; exploratory laparotomy - mature right ovarian teratoma resection performed NMDA receptor encephalitis • multistage illness; progresses from psychosis, memory deficits, seizures, and language disintegration state of unresponsiveness with catatonic features often with abnormal movements, autonomic and breathing instability predominantly affects children & young adults occurs with or without tumour association responds to treatment but can relapse • presence of a tumour (ovarian teratoma usually) is dependent on age, sex & ethnicity; more frequent in women older than 18 years, and slightly more predominant in black women than it is in white women • MRI brain - unremarkable in 50%; non specific abnormalities in 50%; EEG - abnormal in most patients; may show EDB; CSF - lymphocytic pleocytosis, OCB, NMDAR antobodies • >75% of patients have substantial recovery that occurs in inverse order of symptom development and is associated with a decline of antibody titres

DM PT R3 3. Which of the following is most likely to reduce the risk of development of contrast-induced nephropathy? A. Haemodialysis after contrast administration B. Intravenous normal saline C. Oral N-acetyl cysteine D. Intravenous sodium bicarbonate

3. ANS: B Data conflicting • Data supports use of N saline hydration pre and post • No added benefit with sodium bicarbonate above normal saline hydration • Oral N-acetyl cysteine conflicting results • Hemodialysis inconsistent may consider using if ESRF with good residual function • Prophylaxis - low dose, non ionic contrast agents, CO2, stop NSAID and metformin • High risk groups eGFR < 60 proteinuria, diabetes, AMI, myeloma, liver failure All eGFR < 45

DM PT Geris3 3. A 56-year old airport worker has presented with 6 months of increasing difficulty at work. He reports finding it harder to complete his usual tasks. Colleagues have also complained that he has 'changed'. Previously shy, he is now overly friendly. Which of the following is least suggestive of a diagnosis of behavioural variant FrontoTemporal Dementia (bvFTD)? A. His age B. Auditory hallucinations C. Functional decline D. Excessive eating

3. ANS: B Symptoms of Behavioural Variant FTD? • Apathy • Disinhibition • Hyper-orality • Stereotyped/compulsive behaviours Patients with bvFTD generally lack cranial nerve, sensory, cerebellar, pyramidal, and extrapyramidal motor findings, at least initially. Frontal release signs may be seen, but the diagnostic utility of these signs is low, as they are not specific to FTD.

DM Mock 31. Which of the following is NOT a cause of hypercalcaemia? A. Addison's disease B. Hyperthyroidism C. Phaeochromocytoma D. Growth hormone deficiency

31. ANS: D All listed causes are correct, except for D. On the contrary acromegaly is associated with hypercalcaemia

DM PT H3 3. A 34-year-old male presents with right leg swelling and pain. He has a history of Hepatitis C infection related to prior intravenous drug use. He was initially treated with ribavirin and interferon, but this was ceased due to intolerance. There is no previous or family history of thrombosis. Doppler ultrasound confirms an occlusive thrombus extending from the distal popliteal vein to femoral vein. She is commenced on rivaroxaban. His thrombophilia screen shows the presence of Factor V Leiden heterozygosity with no other abnormalities. Which of the following is true regarding the Factor V Leiden gene mutation? A. It causes enhanced binding and degradation of Factor V by Activated Protein C leading to a prothrombotic state B. The increased risk of thrombosis manifests as an autosomal recessive genetic condition C. The increased risk of first thrombosis in heterozygous state is 3-5 times general population D. The risk of thrombosis is mitigated by the oral contraceptive pill

3. ANS: C • Factor V Leiden is an autosomal dominant condition • Factor V resistance to the anticoagulant Protein C. • It is the most common inherited thrombophilia. • The risk of thrombosis is increased by the oral contraceptive pill. • Large scale studies show an OR of approximately 4 of first thrombosis compared with control group.

DM PT Onc3. A 32-year-old man is receiving BEP (bleomycin, etoposide & cisplatin) chemotherapy for metastatic testicular cancer. He is an ex-smoker of 5 pack-years and does not have any comorbidities. You are reviewing him on Cycle 3 day 20 and he reports a dry cough, fatigue, and exertional dyspnoea. Chest X-ray demonstrates the following. Which of the following is the cause of his symptoms? A. Cisplatin-induced lung injury B. Etoposide-related interstitial pneumonitis C. Bleomycin related lung injury D. Fluid overload related to hydration during chemotherapy

3. ANS: C • The patient is at risk of bleomycin lung toxicity. • Cisplatin and etoposide don't cause this pattern of presentation. • The patient is young and doesn't have any cardiac history so fluid overload is incorrect.

DM PT Resp3 3. A 74-year-old non-smoker has a CT angiogram to investigate thoracic pain. It demonstrates a 4mm solid nodule in the right lower lobe. Which of the following would you recommend? A. Repeat CT chest in 3 months' time B. Repeat CT chest in 12 months' time C. Repeat CT chest at in 24 months' time D. No routine follow-up required

3. ANS: D Pulmonary Nodules • A pulmonary nodule is defined on imaging as a small (≤30 mm), well defined lesion completely surrounded by pulmonary parenchyma. • Lesions that measure >30 mm are considered masses, rather than nodules, harbor a much higher likelihood of being malignant. • Morphologically, nodules are classified as: • solid or subsolid; • subsolid nodules are subdivided into • pure ground-glass nodules (ie, no solid component) and • part-solid nodules (ie, both ground-glass and solid components) • Solid lesions are more common, but part-solid lesions have a higher likelihood of being malignant Incidental Pulmonary Nodules Consistently among studies, size is an independent predictor for malignancy • Nodules <5 mm: <1 percent are malignant • Nodules 5 to 9 mm: 2 to 6 percent • Nodules 8 to 20 mm: 18 percent • Nodules >20 mm: >50 percent Solid nodules -Solid nodules ≤8 mm are less likely to be malignant, are difficult to biopsy, not reliably characterized by PET, and, hence, best characterized with CT surveillance. In contrast, solid nodules >8 mm have a greater likelihood of malignancy, can be more reliably characterized by PET, and are more likely to be successfully diagnosed by biopsy. Subsolid nodules - Compared with solid nodules, subsolid nodules are often less amenable to PET imaging and biopsy. • The risk of malignancy in ground-glass lesions that persist beyond three months by CT ranges from 10 to 60% and depends upon the size and presence of a part-solid component. • Malignancy is rare in small (≤10 mm) nodules that are pure ground-glass • Malignancy is more common (10 to 50 percent) in larger lesions (>10 mm) • In contrast, malignancy will be identified in at least half of ground-glass lesions that have a large (>50 percent) or newly developed solid component • The Fleischner Society Guidelines were developed to guide surveillance of incidentally detected pulmonary nodules in non-immunocompromised adults over the age of 35. • According to size (6-8mm) of the nodule, initial follow-up with CT chest at 6-12 months is recommended Nodule Features on CT • Size • Nodule density (solid nodules are less likely to be malignant than subsolid nodules) • Rate of growth • Calcification - usually indicates benign nodules (NB: carcinoid tumours may be calcified) • Borders - smooth more commonly benign, spiculated more commonly malignant • Location - upper lobe nodules more likely to be malignant • Enhancement (only for nodules >5mm) - increased enhancement more likely malignant • PET only suitable for solid nodules 8+mm size

DM Mock 3. A 25-year-old woman presents for pregnancy advice. Thyroid gland is normal to palpation. Her TSH is 1.8 IU/L [reference range 0.27-4.20], and anti-TPO antibody titre is 10 IU/ml [<34]. A maternal aunt is on thyroxine treatment for hypothyroidism. What is the next best step? A. Measure anti-Thyroglobulin antibodies B. Advise her to commence thyroxine 50 mcg daily once pregnancy is confirmed and schedule review with thyroid function testing 4 weeks after thyroxine commenced C. Advise to repeat thyroid function testing once pregnancy is confirmed and schedule review as soon as possible D. Advise routine pregnancy care with no need for further thyroid function testing unless clinically indicated

3. ANS: D TSH is normal, as are anti-TPO antibodies. Consideration of thyroxine treatment in pregnant women without a thyroid history is only considered if preconception TSH is > 4.0 IU/L or if both TSH >2.5 IU/L and anti-TPO antibody titre is positive.

DM PT P3 3. In clinical practice, why is a loading dose given for some drugs? A. To establish steady state concentration immediately. B. To shorten the half-life of the drug. C. To reduce the total dose of drug required. D. To attain therapeutic concentrations of the drug rapidly.

3. ANS: D To attain therapeutic concentrations of the drug rapidly. Loading Dose • aim: reach therapeutic drug levels rapidly • Cpss can be immediately established. Eg: - loading dose = 2x dose of dosing regimen - dosing interval = t1/2 of drug • if not, then time to Cpss 4-5 half-lives after dosing regimen started • immediate establishment of Cpss not clinically relevant Loading Dose - Immediate Cpss

DM Mock 30. A validity study was undertaken of a new sputum test for lung cancer, with validation of its results against bronchoscopy (with biopsy as required), in a randomly selected 100 people from the population. A second study was then undertaken, but this time on 100 people who were heavy smokers. Compared to the first study, which of the following parameters in the second study would be higher? A. Sensitivity B. Specificity C. Positive predictive value D. Negative predictive value

30. ANS: C Sensitivity and specificity are inherent to a test and stay constant. In contrast, PPV and NPV are dependent on the sensitivity and specificity of the test, as well as the underlying prevalence (pre-test probability) of the condition being tested for.

DM Mock 32. A 63-year-old female is admitted to hospital with 6x episodes of bloody diarrhoea per day. She is iron deficient (Ferritin 9 mcg/L) and mildly anaemic (Hb 110g/L). She has COPD with a good function baseline, though has previously required ICU admission for ventilatory support in the context of pneumonia. An infectious aetiology is excluded. Endoscopy reveals Mayo 2 colitis, and histology reveals crypt abscesses and crypt architectural distortion without evidence of inclusion bodies. A decision is made to commence a biological therapy. What is the most preferable option for this patient? A. Adalimumab B. Infliximab C. Vedolizumab D. Tofacitinib

32. ANS: C. This patient has moderate-to-severe ulcerative colitis. Options A and B (anti-TNF therapies) are reasonable, though the risk of systemic infection makes Vedolizumab (a gut-selective antiintegrin targeting alpha-4-beta-7 integrin) preferable given this patient's age and comorbidities. Furthermore, Vedolizumab may be more effective than Adalimumab at inducing clinical and endoscopic remission in moderate-severe UC (NEJM. 2019; 381(13):1215-26). Tofacitinib (option D) is not approved as a first-line therapy. As to whether a biological therapy should be commenced at all in this patient might be a source of debate and would require careful consideration

DM Mock 33. Nadine has an uncle (mother's brother) and a brother affected with Duchenne Muscular Dystrophy (DMD). Nadine's daughter is pregnant. What is the risk to Nadine's future grandchild of developing DMD? A. 1 in 2 B. 1 in 4 C. 1 in 8 D. 1 in 16

33. ANS: D DMD follows X-linked recessive inheritance. Nadine's mother is an obligate carrier for DMD as she has an affected brother and son. Therefore, Nadine has a 1 in 2 chance of being a carrier. Nadine's daughter thus has a 1 in 4 chance of being a carrier. Nadine's future grandchildren would have a 1 in 8 chance of inheriting the mutation, but only grandsons would be affected, so 1 in 16 chance of a grandchild actually developing DMD.

DM Mock 34. In the ageing cardiovascular system, which of the following statements is FALSE? A. Vascular stiffening in older age leads to a gradual decrease in blood pressure B. Impaired beta receptor responsiveness limits the ability to increase cardiac output by increasing heart rate C. Pulse pressure widens with age D. There is increased myocardial fibrosis

34. ANS: A. Will lead to an increase in BP.

DM Mock 35. A 72-year-old male presents with lethargy and increased bruising. A full blood picture identified pancytopenia with circulating blast cells. Bone marrow biopsy confirmed acute myeloid leukaemia with 65% blasts with monocytic morphology. Which of the following molecular profiles is considered favourable risk in normal cytogenetic AML? A. NPM1 mutated/FLT3-ITD mutated B. NPM1 mutated/FLT3-TKD mutated C. NPM1 wildtype/FLT3-ITD wildtype D. NPM1 mutated/FLT3-ITD wildtype

35. ANS: D ELN 2017 AML risk classification, in normal cytogenetics AML the favourable molecular combination is NPM1 mutated, FLT3 (TKD or ITD) unmutated.

DM Mock 36. A 32-year-old female presents to the outpatient clinic for evaluation of a skin lesion. It began as a pink papule on her right arm and has progressively enlarged over several weeks. The lesion is not pruritic or painful and has not been associated with fever or other systemic symptoms. The patient returned from Costa Rica 5 weeks ago. During her travel, she stayed at a beachside resort and hiked in the rainforest. Skin examination findings are shown below. Biopsy of the lesion reveals intracellular, round-oval protozoa with rod-shaped kinetoplasts. A photo of the lesion is shown Which of the following most likely led to this patient's current condition? A. Bite from infected sand fly B. Contact with infected person C. Handling of an infected animal D. Swimming in contaminated water

36. ANS: A Leishmania species are obligate intracellular protozoa that mature in macrophages and can be identified on biopsy by the presence of rod-shaped kinetoplasts. They are transmitted to humans by infected sand flies and cause the clinical syndrome of cutaneous leishmaniasis, characterised by a chronic, pinkish papule that evolves into a nodule or plaque.

DM Mock 37. A 20-year-old woman was diagnosed with CTLA-4 haploinsufficiency, manifesting over the years as multiple sclerosis, haemolytic anaemia, colitis and hypogammaglogulinaemia. Other than immunoglobulin replacement therapy, what is the next treatment to consider? A. Abatacept B. Rituximab C. sirolimus D. Iplimumab

37. ANS: A Abatacept is a fusion protein which in essence binds to CD86/80, thus replacing what the missing CTLA-4 and providing that missing checkpoint. Rituximab/sirolimus are reasonable options, however not as targeted. Iplimumab is a CTLA-4 antibody and thus not suitable.

DM Mock 38. A 48-year-old man, previously well, is seen on day 4 after development of horizontal diplopia, in the setting of an upper respiratory illness one week prior. Three days ago, diplopia was worse, and he noted staggering when walking. Two days ago, he developed tremor and arm incoordination. Now, on examination on day 4: - Bilateral sixth nerve palsies, mild ptosis, gait ataxia, an upper limb sensory ataxic tremor and slight vibration sense loss. There is no limb weakness. CSF protein is elevated at 0.67 g/L, without elevated cell counts, and electrodiagnostic testing shows low amplitude sensory responses, with normal motor findings. What is the most appropriate management at this stage? A. Intravenous immunoglobulin B. Intravenous methylprednisolone C. Plasma exchange D. Supportive care alone

38. ANS: D The patient improved without active treatment over 2-3 months with complete recovery Management: - Miller Fisher Syndrome MFS seldom progresses beyond 1 week and has an excellent prognosis for full recovery - No RCT evidence of benefit of IVIg or PLEX in MFS - Supportive care alone reasonable for MFS - Consider IVIg for MFS/GBS overlap with weakness

Dun Rheum A 26yr old woman has had 2 previous miscarriages Which of the following antibodies is associated with the highest future risk of miscarriages? A. Anti‐cardiolipin B. Anti‐Ro C. ANA D. Anti‐La E. Anti‐ Sm F. Anti‐ histone

A

DM Mock 39. Which of the following confers good prognosis in patients with breast cancer? A. Low Erk5 B. HER2 amplification C. Inflammatory features D. Oestrogen receptor positivity

39. ANS: D ER positivity is a good prognostic factor in breast cancer. Inflammatory breast cancer is a rare and highly aggressive form of breast cancer that blocks lymph nodes in the skin of the breast. Patients with high ERK5 expression have worse disease-free survival compared to patients with low ERK5 expression. HER2 amplification has been shown to increase the risk of recurrence and risk of death, however, advancements in HER2 targeted therapies have significantly improved the outcomes of these patients; their prognosis, however, is not better than those without HER2 amplification.

CLS Geris A 79yo woman living in her own home has had falls every week for the last 6 months and as a result has fractured her radius and a lumbar vertebra. She has a past history of hypertension, cataracts, depression and ischaemic heart disease and now mobilises with a pick up frame. Which of the following interventions will reduce her future risk of falling the most? 1. Vitamin D supplementation 2. Withdrawal of antihypertensives medications 3. Excision of cataracts 4. Exercise programme 5. Home visit and modifications by occupational therapist

4 1. Vitamin D supplementation 2. Withdrawal of antihypertensives medications 3. Excision of cataracts 4. Exercise programme ← biggest benefit. 1, 3 and 5 also have benefit. Withdrawal of antihypertensive is useful if there is postural hypotension. 5. Home visit and modifications by occupational therapist

DM Mock 4. A 37-year-old lady undergoes an abdominal ultrasound for right upper quadrant pain. A 3 x 2 x 2.5cm hyperechoic lesion is identified. A multiphase CT scan is arranged. This demonstrates nodular peripheral involvement in the arterial phase with progressive central enhancement in the portal phase. The lesion remains isointense in the delayed phase. What should the next step in management be? A. Reassurance B. Urgent referral for bacterial cultures and empiric gram negative antimicrobial therapy C. Urgent referral for liver workup and consideration of liver mass resection D. Ca19.9 and upper gastrointestinal surgical referral

4. ANS: A This is typical imaging for a typical hepatic haemangioma. In terms of option B, hepatic abscess often have peripheral enhancement with central hypoattenuation. A double target sign is characteristic on contrast-enhanced studies. A cluster sign can be seen in pyogenic abscess. In terms of option C, HCCs fill rapidly on arterial phase (due to hepatic artery supply) and wash out in the portal phase. Option D relates to cholangiocarcinoma. These are heterogenous in imaging presentation but are often hypodense in the arterial and portal venous phase with some peripheral enhancement. Additionally, in mass-forming cholangiocarcinoma, there is often capsular retraction.

DM PT H4 4. A 24-year-old female is referred for persistent microcytic anaemia refractory to oral iron replacement. She has a history of menorrhagia for which she takes the oral contraceptive pill and has chronic abdominal pain that has recently been diagnosed as irritable bowel syndrome. Her haematological investigations show: Microcytic anaemia, normal RDW and ferritin

4. ANS: C HbH disease is 3 (out of 4) alpha globin deletions, leading to a thalassaemia intermedia phenotype. • Patients have microcytic anaemia and marked red cell anisopoikilocytosis. Testing includes supravital staining for HbH inclusions, HPLC/capillary electrophoresis and definitive genetic testing. • Deletion of all 4 alpha globin genes is Hb Barts: a condition not typically considered compatible with life though now able to be supported with intrauterine blood transfusion. • Deletion of 2 alpha globin genes can result in a form of thalassaemia minor which is usually asymptomatic with normal haemoglobin and microcytosis. Alpha Thalassemia - a, --, aaT mutations • -a / aa, -a / -a, -- / aa -a / -- Hb H disease -- / -- Hb Bart's Hydrops syndrome

DM PT P4 4. A 100kg man is started on a drug at steady regimen of 50mg twice daily, at 0800hr and 2000hr. The volume of distribution is 0.5L/kg and clearance is 3.5L/hour. Measured from the time of starting the regimen, which of the following is the best estimate of the time to steady-state concentration? A. 10 hours B. 20 hours C. 50 hours D. 80 hours

4. ANS: C 50 Hours Steady State Concentration Assuming 1st-order kinetics and constant clearance: at the same dosing regimen (dose and interval): Cpss is reached in 4 to 5 half-lives Cpss then stays constant Cpss is proportionally (predictably) changed by changing dose and/or dosing interval = (50L*0.693) ÷ 3.5L/hr = 10hr

DM PT Geris4 4. Acetylcholinesterase inhibitors are least likely to be effective in... A. Alzheimer's Disease B. Dementia with Lewy Bodies C. Huntington's Disease D. Vascular Dementia

4. ANS: C Cholinesterase inhibitors are effective in Alzheimer's Disease and subtypes, Dementia with Lewy Bodies, Vascular Dementia. They are ineffective in Frontotemporal Dementia, Huntingtons disease and other dementias.

DM Mock 44. Which of the following is commonly undertaken in cohort studies to reduce confounding? A. Recruitment of subjects from the same population sources B. Randomisation C. Blinding/masking D. Multivariable analyses

44. ANS: D In observational studies, common methods used to adjust for confounding are matching (by key confounders), stratified analyses (analyses stratified by the categories of a confounder, such as sex) and multivariable analyses. Randomisation is undertaken only in interventional studies

DM PT Onc4 4. A 72-year-old women presents with headaches, dyspnea, and insomnia. Her symptoms are worse when bending over and the patient has been sleeping upright for the last week. She has a history of type 2 diabetes, hypertension, previous myocardial infection in her 60s and a drug-eluting coronary stent in situ. Until a month ago, she was playing lawn bowls regularly. You review her in your rooms and her JVP is elevated. She is plethoric and appears short of breath. She has some mild swelling around her neck. What is the cause of her symptoms? A. Thyrotoxicosis due to multinodular goiter B. Congestive cardiac failure C. Lymphoma D. Cardiac tamponade

4. ANS: C • The patient has symptoms and signs of SVC obstruction. Common causes of SVC obstruction are lung cancer and lymphoma. A. Thyrotoxicosis shouldn't cause symptoms to be worse when bending over. B. There is no specific cause for congestive cardiac failure in the acute setting. D. Cardiac tamponade should not cause positional dyspnea

DM PT E4 4. A 65-year-old man is admitted to the hospital for an infective exacerbation of COPD. He has type 2 diabetes treated with Gliclazide MR 60 mg/daily. BMI is 32kg/m2 . HbA1c on admission is 8.7%. In addition to inhalers, and antibiotics, prednisolone 50 mg mane is commenced. Fasting glucose before breakfast is 8.0 mmol/L. He is eating and drinking normally. In addition to a prn s.c. insulin top-up scale, what is the best strategy to maintain glycaemic control in hospital? A. Increase gliclazide MR to 120 mg mane. B. Increase gliclazide MR to 120 mg mane and add an SGLT2 inhibitor. C. Commence long acting insulin (e.g. glargine) 24 units nocte D. Commence mixed insulin (e.g. Novomix 30, Mixtard 30/70) 12 units mane and 10 units at lunch

4. ANS: D Glucocorticoids given in the morning will lead to hyperglycaemia peaking around early afternoon to evening. With poor control, insulin is required and best matched to the expected excursions in sugar levels.

DM PT R4 4. An 83-year-old man presents to his GP feeling tired for a few weeks, with associated anorexia and weight loss of 4kg. Blood tests show haemoglobin of 92g/L, creatinine 296µmol/L, urea 24.5mmol/L and potassium 5.4mmol/L. He has a positive ANCA, with MPO specificity. Renal biopsy showed cellular crescents in 80% of glomeruli, with moderate interstitial fibrosis. Which of the following is most strongly associated with this patient's long-term renal prognosis? A. ANCA specificity: MPO or PR3 B. ANCA titre C. Severity of renal failure at presentation D. Histological features of activity and chronicity

4. ANS: D • Histological features more accurate estimation of LT prognosis • Renal function also important but not as much impact • Clinical presentation, ANCA titre or ANCA type less correlation with outcome • PR3 ANCA more likely to have relapsing disease ? Better response with Rituximab

DM Mock 40. Which of the following statements is true? A. eGFR measurement is sufficient for diagnosis of chronic kidney disease B. eGFR >90 rules out any underlying kidney disease C. eGFR should not be used to predict risk of contrast-induced acute kidney injury D. eGFR measurement is not affected by dietary protein intake

40. ANS: A eGFR measurement is sufficient for diagnosis of chronic kidney disease. Especially in patients with risk factors for CKD and repeated eGFR measurements, no further confirmatory investigations (such as creatinine clearance) are indicated. KDIGO CKD guidelines 2012

DM Mock 41. A 35-year-old female presents with difficulty falling asleep, which is impacting on her work as an accountant during the day. All of the following are components of cognitive behavioural therapy for insomnia EXCEPT... A. Sleep restriction B. Relaxation C. Stimulus control D. Scheduled naps

41. ANS: D Cognitive behavioural therapy for insomnia is recommended as first-line treatment for insomnia. The core components include cognitive therapy, stimulus control (answer C is incorrect), sleep restriction (answer A is incorrect), sleep hygiene and relaxation (answer B is incorrect). Scheduled naps are sometimes used in combination with pharmacotherapy for narcolepsy (answer D is correct). Reference: Ree, M, Junge, M, Cunningham, D. Australasian Sleep Association position statement regarding the use of psychological/behavioural treatments in the management of insomnia in adults. Sleep medicine. 2017(36) S43-S47.

DM Mock 42. A 69-year-old woman presents with knee pain and the x-ray shown below. (OA) Which of the following therapies has the greatest evidence in managing this condition? A. Hydroxychloroquine tablets B. Intraarticular hyaluronic acid injection C. Oral nonsteroidal antiinflammatory drugs D. Platelet rich plasma injection

42. ANS: C Reference: Kolasinski SL, Neogi T, Hochberg MC. et. al. 2019 American college of rheumatology/arthritis foundation guideline for the management of osteoarthritis of the hand, hip and knee. Arthritis and Rheumatology 2020; 72(2): 220 - 233. Rationale: The x-ray shows classic osteoarthritis changes. Unfortunately, there is no data to support the use of DMARDs used to treat rheumatoid arthritis in patients with osteoarthritis. Despite much interest in hyaluronic injections and platelet rich plasma injections, there is still insufficient data to support their routine use.

DM Mock 43. Which of the following is not a cause of pulmonary arterial (precapillary) hypertension? A. Chronic effect of pulmonary embolus B. Mitral stenosis C. Systemic scleroderma D. Idiopathic pulmonary arterial hypertension

43. ANS: B Mitral stenosis is a left-sided valvular lesion that will cause pulmonary hypertension via back pressure. It is an important cause of post-capillary pulmonary hypertension.

DM Mock 46. A 68-year-old male presents to Emergency with abdominal pain. He has a lipase rise of 5x the upper limit of normal. A CT scan demonstrates a sausage-shaped pancreas with low attenuation surrounding the pancreas. What is the most helpful test in delineating the diagnosis? A. ANA level B. Ca-19.9 C. IgG4 level D. Serum ethanol level

46. ANS: C IgG4 disease is an increasingly recognised condition which can be multi-system in nature, affecting the pancreas, biliary tree, retroperitoneum and salivary glands (amongst other manifestations). The disease can be focal (and occasionally confused with pancreatic cancer) or diffuse in nature. Recurrent symptomatic pancreatitis is less likely in diffuse IgG4 pancreatic disease, but certainly can occur. A positive serum IgG4 level, together with characteristic imaging and steroid-responsiveness are characteristic of IgG4 pancreatitis. Histology can also be very beneficial in delineating auto-immune disease versus malignancy

DM Mock 48. Management of osteoporosis is an important part of orthogeriatric care. Which of the following statements is TRUE? A. Positive family history is not a risk factor for osteoporosis B. Selective oestrogen receptor modulators (SERMs) are often recommended in postmenopausal women with osteoporosis C. Denosumab can cause hypocalcaemia D. A T-score between -1 and -2.5 indicates osteoporosis

48. ANS: C. FHx is a RF; SERMS not recommended in orthogeriatrics (guidelines recommend for <60, <10 years past menopause, with low risk of deep vein thrombosis and low cardiovascular risk); T score <2.5 indicates osteoporosis (the above score indicates osteopaenia).

Deltamed liver A patient with decompensated cirrhosis presents with acute renal insufficiency. Possible provocative factors include all but which of the following 1. NSAIDS 2. Culture negative neutrocytic ascites 3. LVP without albumin replacement 4. RecentTIPS 5. Acute hepatic encephalopathy

5.

Dun Renal 62 yr male was found to have progressive proteinuria reaching 3.2gm/24hr proteinuria when referred. He has a one year history of mild congestive heart failure controlled with diuretics. His blood pressure was 132/80 and there is no evidence of systemic disease. Plasma creatinine was 0.1 mmol/l. Creatinine clearance 96 ml/min. Electrolytes, glucose and LFTs including albumin are normal. ANA, cryoglobulins and serum protein electrophoresis & light chains were normal. 1. His Hb1AC will be elevated 2. This is an immune complex mediated GN 3. Complement levels will be low 4. Urine protein electrophoresis will be positive 5. Congo red staining will be positive

5. •Histologically ‐ lobular expansion of mesangial matrix with large sclerotic nodules obliterating surrounding capillaries. So called "Nodular glomerulosclerosis". •Kimmelstiel Wilson Nodules ‐ advanced diabetic nephropathy. •Non specific pattern of injury which can arise via several different pathogenic mechanisms. •Additional information required to assist diagnosis clinical history. Congo red staining, IF staining pattern for IgG, IgA, IgM, complement or light chains normal glucose so not diabetic

DM PT E5 5. A 48-year-old man presents with difficult to control diabetes diagnosed 1 year ago. HbA1c at diagnosis was 8.1%, and BMI 28 kg/m2 . Despite weight loss (3 kg) and treatment with metformin and a sulfonylurea, HbA1c has increased to 9.0%. He is otherwise well, and his only other medication is thyroxine for hypothyroidism. What is the most appropriate next step? A. Add a SGLT2 inhibitor. B. Conduct genetic testing for maturity onset diabetes of the young C. Measure coeliac serology D. Measure glutamic acid decarboxylase antibodies

5. ANS D Recognize when to exclude type 1 diabetes/ Latent adult onset diabetes (LADA). A Clinical Screening Tool Identifies Autoimmune Diabetes in Adults Younger age of onset (<50 years) Acute symptoms BMI < 25 kg/m2 Personal or family history of auto-immune disease No family history of type 2 diabetes Failure to respond to OHG Measure GAD antibodies, C-peptide When to suspect MODY Strong family history of diabetes (ie AD inheritance) • Onset before 25 years BMI < 30 kg/m2 Negative antibodies: GAD, Islet cell, insulin ABs Consider genetic testing in such individuals Will impact treatment: certain subtypes respond to Sulfonylureas

DM PT Onc5 5. A 35-year-old woman is concerned about her risk of breast cancer. She reports a family history of breast cancer with her paternal grandmother being diagnosed with breast cancer at the age of 46 and her maternal grandmother also being diagnosed with breast cancer at the age of 44. She is not aware of any other family history of breast or ovarian cancer. She is not of Jewish Ancestry. How should she be screened for breast cancer? A. Breast awareness and mammogram every 2 years from the age of 50. B. Breast awareness and mammogram every 1 years from the age of 50. C. Breast awareness and mammogram every 2 years from the age of 40. D. Breast awareness and mammogram every 1 years from 5 years before the age of diagnosis of the youngest relative affected.

5. ANS: A Current guidelines recommend standard procedures, with mammogram and breast awareness, every two years from 50 to 74 years of age for women with: One first-degree relative diagnosed with breast cancer aged ≥50 years One second-degree relative diagnosed with breast cancer at any age Two second-degree relatives on the same side of the family diagnosed with breast cancer aged ≥50 years Two first-degree or second-degree relatives diagnosed with breast cancer, aged ≥50 years, but on different sides (ie on each side) of the family For women at moderately increased risk, current guidelines recommend at least every two years from 50 to 74 years of age: One first-degree relative diagnosed with breast cancer aged <50 years (without the additional features of the potentially high-risk group) Two first-degree relatives, on the same side of the family, diagnosed with breast cancer (without the additional features of the potentially high-risk group) Two second-degree relatives, on the same side of the family, diagnosed with breast cancer, at least one aged <50 years (without the additional features of the potentially high-risk group) Annual mammograms from 40 years of age may be recommended if the woman has a first-degree relative aged <50 years diagnosed with breast cancer

DM PT P5 5. Dose-effectiveness curves are illustrated for Drugs A, B and C below. Which of the following statements is CORRECT? A. Drugs A and B are equipotent. B. Drugs A and B are equally efficacious. C. Drugs A and C are equipotent. D. Drug A is more efficacious than Drug C.

5. ANS: A Drugs A and B are equipotent

DM PT Resp 5 5. Which of the following is correct regarding ventilation after high spinal cord injury? A. A complete spinal cord injury at C5 results in diaphragm paralysis B. There is a reduction in lung compliance within a month of injury C. The abdominal wall is poorly compliant in spinal cord injury D. Forced vital capacity is unlikely to improve 90 days after injury

5. ANS: B Injuries about the level of the phrenic motorneurons (C3, C4 and 5) cause virtually complete paralysis of both the muscles of inhalation and exhalation resulting in dependence on mechanical ventilation or phrenic-nerve stimulation Compliance is the ability of the lungs to expand under pressure • Within one month of injury there appears to by a reduction in lung compliance • Reduced lung volume • Changes in surfactant which occur with low lung volumes • Reduced chest wall compliance due to spasticit The abdominal compartment of the chest wall is highly compliant in spinal cord injury FVC has been shown to increase 90 days to 210 days after injury due to improvement in diaphragm performance

DM PT Rh5 5. A 35-year-old Caucasian woman attends the rheumatology outpatient clinic complaining of fatigue, polyarthritis to her hands and feet, and an intermittent facial rash. Her ESR is raised at 76, her full blood examination shows a mild normochromic, normocytic anaemia and also lymphopaenia. Which of the following investigations would be specific for her likely diagnosis? A. Anti-centromere antibodies B. Anti-double stranded DNA antibodies C. Anti-Jo-1 antibodies D. Anti-neutrophil cytotoplasmic antibodies

5. ANS: B ANA is positive in virtually all patients with SLE. If ANA is positive, then specific antibody testing is indicated; Square brackets indicate prevalence: - dsDNA (Specific, not sensitive - 70%) Anti-Smith (Specific and sensitive - 30%) Ro/SSA (More in Sjogrens - 30%) La/SSB (More in Sjogrens - 20%) U1 ribonucleoprotein (RNP) also seen in Mixed Connective Tissue Disease - 25%) NB: 15% of the population will have a positive ANA of at least 1:80 by immunofluorescence; Only 10% will have a true autoimmune disease

DM Mock 5. Phenylketonuria (PKU) has a disease incidence of 1 in 10,000. What is the carrier frequency for PKU in the general population? A. 1/25 B. 1/50 C. 1/100 D. 1/200

5. ANS: B PKU is autosomal recessive. So risk of disease is carrier risk of 1 parent x carrier risk of other parent x 1 in 4. If population disease incidence is 1 in 10,000. Carrier x carrier x 1 in 4 = 1 in 10,000. So, reverse this: 1 in 10,000 divided by 4 = 1 in 2500 and then square root of 1 in 2500 equals 1 in 50.

DM PT Geris P5 5. Geriatric perioperative care is an increasing area of interest. In the ageing cardiopulmonary system, which of the following statements is FALSE? A. A-a gradient increases B. Pulse pressure narrows C. Alveolar ducts enlarge D. There is increased myocardial fibrosis

5. ANS: B • Pulse pressure increases as a normal part of ageing, mainly due to reduced vascular compliance. • Refer back to notes on physiological changes in aging.

DM PT H5 5. A 65-year-old male is referred by his GP with lymphadenopathy for investigation. His past history is significant for inflammatory bowel disease for which he takes mesalazine. He has not received any biological therapy for his inflammatory bowel disease. Examination reveals 1cm bilateral cervical lymphadenopathy, which has been confirmed on ultrasound. Histology from an ultrasound guided lymph node core biopsy shows grade 2 follicular lymphoma. Which of the following is NOT a Groupe d'Etude des Lymphomes Folliculaires (GELF) criteria for immediate treatment of follicular lymphoma? A. Involvement of ≥3 nodal sites ≥3cm in size B. A single lymph node site ≥7cm in size C. B symptoms D. Stage IV disease

5. ANS: D • Stage is not a treatment indication by GELF criteria • Groupe d'Etude des Lymphomes Folliculaires : High Tumour Burden Nodal or extranodal mass >7cm 3+ nodes >3cm high LDH Hb <10, ANC 1.5, plt <100 Spleen >16cm Risk or organ compression or compromise Sigificant serous effusions

CLS Neuro Lecture (enc) 65 yo man brought to ED after for recurrent focal seizures involving the right arm and leg - Preceding week - low grade fever, 'funny behaviors' (aggressions, hallucinations); no head trauma/ headache/ vomiting - Past hx: healthy; no medications; no recent travel; stress related to domestic matters Exam: Awake, confused, visual hallucinations Temp: 38.2ᴼC; pulse 82/ min: BP: 134/84 mm Hg; RR: 18/ min Normal fundi; gait and reflexes normal CSF studies Cells: 150/mm3, lymphocytes Protein: 150 mg/ dL; Glucose: 3.1 mmol/L Gram stain and India ink: no organisms seen Which of the following diagnostic studies should be performed next? 1. Acute and convalescent serology for HSV 2. Ratio of CSF to serum HSV antibodies 3. Brain biopsy 4. CSF PCR 5. EEG

5. EEG HSV encephalitis (HSE) HSV 1 90% of all cases in adults and children HSV 2 10% neonates and immunocompromised patients Commonest fatal sporadic encephalitis in humans An early diagnosis & timely treatment with acyclovir are very important in optimizing the outcome Acyclovir Given IV 10 mgsa/ Kg/ dose every 8 hours (adjusted for renal insufficiency) Class B medication - HSE during pregnancy can be treated with Acyclovir

DM Mock 50. You are consulting on a patient who has been tested for hepatitis B virus infection before immunosuppression. The serology results return: HBSAg negative, HBSAb negative, antiHBc Ab positive. Hepatitis B Virus DNA is not detected in blood. Which of the following conditions does not require antiviral therapy to prevent hepatitis B virus reactivation? A. Haematopoietic stem cell transplantation B. Docetaxel chemotherapy for metastatic prostate cancer C. ANCA-associated vasculitis receiving rituximab induction D. Acute leukaemia receiving chemotherapy

50. ANS: B As per recently published ASID 2019 guidelines - "Hepatitis B management during immunosuppression" - option B would be considered a "lower risk chemotherapy" option where no HBV antivirals are recommended. All other options are high risk and do require HBV antiviral. https://www.asid.net.au/documents/item/1741

DM Mock 51. Autoimmune polyendocrinopathy, chronic mucocutaneous candidiasis and ectoderma dysplasia (APECED) is due to defects in a gene responsible for which of the following processes? A. Somatic hypermutation B. Expression of class II MHC C. Generation of receptor diversity D. Thymic T-cell selection

51. ANS: D APECED is due to defects in AIRE responsible for regulation of t-cell selection within the thymus, within cortical thymic epithelial cells (tolerance)

DM Mock 52. A 57-year-old man with hypertension and diabetes was diagnosed with de novo metastatic castration resistant prostate cancer with bone metastases. He was commenced on enzalutamide and denosumab. At his two-month review, he reported dizziness and muscle cramps. What is the cause of his symptoms? A. Hypercalcemia due to enzalutamide-resistant prostate cancer B. Hypocalcaemia due to denosumab C. Hypercalcemia due to paraneoplastic syndrome D. Hypocalcaemia due to paraneoplastic syndrome

52. ANS: B The patient has symptoms of hypocalcaemia. Denosumab is a RANKL inhibitor which can cause hypocalcaemia in patients with bone metastases, particularly if there is underlying renal impairment. The patient has hypertension and diabetes which are both risk factors for renal impairment. The stem does not provide information about whether the patient has underlying vitamin D deficiency.

DM Mock 53. Which of the following is a contraindication to renal transplantation? A. Obesity, with BMI 32kg/m2 and waste circumference 104cm B. Age 74 years C. Gleeson score 4 prostate cancer D. End stage kidney disease due to light chain deposition disease, without systemic features of multiple myeloma

53. ANS: D ESKD due to LCDD LCDD implies underlying monoclonal gammopathy (either monoclonal gammopathy of renal significance, or multiple myeloma). Without treatment for the underlying disease, LCDD is likely to recur in the transplant. Age and weight in themselves are not contraindications to transplant. Low grade prostate cancer does not need to be treated prior to transplant or can be treated with no subsequent waiting time. KDIGO Transplant Candidate Guidelines 2020 https://kdigo.org/wpcontent/uploads/2018/08/KDIGO-Txp-Candidate-GL-FINAL.pdf

DM mock 54. A 52-year-old male with severe acute respiratory distress, secondary to aspiration, has an arterial blood gas performed whilst on mechanical ventilation. The fraction of inspired oxygen delivered is 0.8. pH 7.30 PaO2 68mmHg PaCO2 60mmHg What is the best explanation for the patient's hypoxia? A. Intrapulmonary shunt B. Hypoventilation C. Reduced pulmonary diffusion D. Reduced inspiratory oxygen pressure

54. ANS: A Acute respiratory distress syndrome (ARDS) and aspiration can result in the effected "lung units" receiving no ventilation and unable to participate in gas exchange. Blood flow through the affected lung is unchanged, creating an intrapulmonary shunt. End-capillary blood flow from this area is similar to mixed venous blood. With shunt, increasing the FiO2 is less effective at improving arterial oxygenation as the raised FiO2 fails to improve the PAO2 (alveolar partial pressure) of the affected lung units.

DM Mock 55. A 36-year-old lady with psoriatic arthritis presents for routine review. Her disease is well controlled with the combination of Sulfasalazine 1g BD, Paracetamol 1g Daily as required, Meloxicam 15mg mane. Which of these medications would have the greatest impact on her ability to conceive? A. Meloxicam B. None of these medication C. Paracetamol D. Sulfaslazine

55. ANS: A Reference: Sammaritano LS, Bermas BL, Chakravarty EE. et. al. 2020 American College of Rheumatology guideline for the management of reproductive health in rheumatic and musculoskeletal disease. Arthritis & Rheumatology 2020; 72(4): 529 - 556.

DM Mock 59. Which of the following is NOT a risk factor for Coeliac Disease? A. Down Syndrome B. IgA deficiency C. Autoimmune thyroiditis D. Eczema

59. ANS: D Unlike Dermatitis herpetiformis which occurs in around 10-15% of people with Coeliac disease, eczema is thought to be unrelated. People with Down Syndrome are at higher risk of Coeliac disease as are people with IgA deficiency. Regarding the latter, an IgG-containing serological test such as deamidated peptide gliadin (DPG) should be used to avoid false negative from IgA-based tests (like IgA tissue transglutaminase serology). Autoimmune thyroiditis and type 1 diabetes are other conditions that are associated with Coeliac Disease

DM PT Geris6 6. Pharmacokinetics and pharmacodynamics changes with ageing. In the elderly, compared to a younger population, which of the following statements is TRUE? A. Digoxin has a lower Volume of Distribution (VD) B. Diazepam has a lower Volume of Distribution (VD) C. First pass metabolism is increased D. Drug absorption is reduced

6. ANS: A • Digoxin is excreted by the kidneys and distributes in the heart and muscles - total body water and muscle is decreased in older people compared with younger people. • The VD of Diazepam is increased in older people (increased fat mass) • First pass metabolism is reduced due to decreased hepatic mass and blood flow • Drug absorption is roughly the same in older compared with younger people.

DM PT R5 5. Cinacalcet is not associated with which of the following? A. Reduction in parathyroid hormone level (PTH) B. Reduction in serum calcium C. Reduction in cardiovascular risk D. Reduction in serum phosphate

6. ANS: B Decrease PTH • Decrease serum calcium and phosphate • Reduction in Ph via reduction in PTH • No evidence reduction in CV mortality reason taken off PBS

Dun cardio A 54-year-old woman is receiving chemotherapy with oxaliplatin, 5 fluorouracil (5FU) and folinic acid as adjuvant therapy for newly diagnosed node positive colon cancer. During the continuous infusion of 5FU, she experiences retrosternal chest pain. She is not known to have a history of coronary artery disease, and has not had chest pain previously. What is the most likely cause of her chest pain? A. Coronary artery spasm. B. Oesophageal spasm. C. Anaphylaxis. D. Aortic dissection. E. Mucositis.

A

DM PT Onc6 6. A 67-year-old man with a metastatic nasopharyngeal carcinoma was found to have widespread bony metastases. He received 20Gy (5 fractions) of radiotherapy to a field spanning the lower thoracic and upper lumber spine. Several weeks later, he received palliative chemotherapy (docetaxel and cisplatin) for the management of his systemic disease and presented 11 days later with rash, pruritis, erythema over this site. What is the cause of his presentation? A. Radiation induced dermatitis B. Radiation recall C. Cisplatin-related rash D. Docetaxel-related rash

6. ANS: B Radiation recall is an acute inflammatory reaction confined to previously irradiated areas that can be triggered when chemotherapy agents are administered after radiotherapy. It remains a poorly understood phenomenon, but increased awareness may aid early diagnosis and appropriate management. A diverse range of drugs used in the treatment of cancer has been associated with radiation recall, but commonly include taxanes, anthracyclines, antimetabolites, vinca alkaloids. The area of the skin affected clearly corresponds to previously irradiated area but can become more generalised. It should be differentiated from incomplete healing from RT (which tends to be towards the end of the course of radiotherapy or in the 1-2 weeks after completion). In this case, the rash occurred several weeks after the end of radiotherapy, and the stem doesn't mention severe rash during radiotherapy or towards the end of radiotherapy. C/D. Whilst chemotherapies can cause rashes, one would expect it to be more generalised. This rash is extremely well demarcated.

DM PT R6 6. Serum anti-phospholipase receptor 2 antibodies (PLA2R) Ab are found in membranous nephropathy. Which of the following statements is not true? A. Serum PLA2R Ab are found 70% of patients with primary membranous nephropathy B. Serum PLA2R Ab are not seen in secondary causes of membranous nephropathy i.e. Hepatitis B, malignancy and sarcoidosis C. Serum PLA2R Ab levels can be used as a biomarker for treatment of membranous nephropathy D. Increasing levels may predict relapse of membranous glomerulonephritis

6. ANS: B • IgG 4 • PLA2 R ab on renal biopsy • 70% primary membranous • 20% secondary membranous ? Staining on biopsy may be less intense • Levels correlate with treatment (fall) with remission and increase levels may signify relapse • Treatment immunosuppression - CNI, Rituximab, Cyclophoshamide • Plasma exchange not effective form of treatment

DM PT EBP1 6. A new point-of-care device has been developed which tests blood for presence of the HLAB*15:02 allele. This genetic profile is associated with a high risk of carbamazepineassociated severe skin reactions. A study was undertaken to assess the validity of the device, with ascertainment of 'true' HLA-B*15:02 status via laboratory testing. A sample of 1000 Caucasian people were recruited into the study. The results are displayed in the table below, with 'negative' and 'positive' results pertaining to presence of the HLA-B*15:02 allele lab result - lab result + device result - 800 1 device result + 190 9 In the validation study, what was the sensitivity of the device for HLA-B*15:02? A. 5% B. 81% C. 90% D. 99%

6. ANS: C 90%

DM PT E6 6. A 68-year-old woman presents with nausea and new onset headaches. She has a history of oestrogen receptor positive breast cancer treated with mastectomy and adjuvant radiotherapy 8 years ago. There is no focal neurology, and she is not pigmented. Her only medication is anastrozole. JVP is +2cm, and blood pressure is 110/60 mm Hg, without a postural drop. Serum sodium is 125 mmol/L, potassium 3.8 mmol/L, serum osmolality 260 mosmol/L, and urine osmolality 520 mosmol/L. Serum cortisol is 40 mmol/L, and increases to 600 mmol/L 60 min following 250 mcg synacthen i.m. TSH is 3 mIU/L [reference range 0.27-4.20], and free T4 8 pmol/L [reference range 12.0-22.0] In addition to strict fluid balance monitoring, what is the best initial approach to treat her hyponatraemia? A. Commence 1L of 0.9% saline over 24 hours and restrict free fluids to 1 liter a day. B. Commence 3% saline at 20 mls/hour and recheck serum sodium in 2 hours. C. Commence hydrocortisone. D. Commence thyroxine.

6. ANS: C Recognize secondary adrenal failure as a cause of hypotonic hyponatraemia

DM PT H6 6. A 45-year-old female presents with acute abdominal pain with imaging on CT scan consistent with acute appendicitis. She has a history of excessive alcohol intake although this has reduced in the last 3 years. She currently has 2-3 standard drinks per night. A pre-operative coagulation test is performed. Platelets 190 x 109 /L (150-400) INR 1.0 (0.9-1.2) APTT 52 sec (28 - 38) APTT 50:50 mix 48 sec Fibrinogen 4.5 g/L (2.0 - 4.0) The surgical team consults you in regard to her prolonged APTT. What is the most appropriate advice? A. Proceed to surgery with administration of perioperative fresh frozen plasma B. Factor VII levels C. Lupus anticoagulant testing D. Von Willebrand Disease screen

6. ANS: C • A non-correcting APTT is suggestive of an inhibitor • Most common inhibitor the lupus anticoagulant. • This is a procoagulant condition where DVT prophylaxis should be instituted postoperatively. • Factor VII is assessed in the extrinsic pathway (PT), not the intrinsic pathway (APTT) • An intrinsic factor deficiency (eg. haemophilia) which FFP would also treat should lead to a correcting APTT by mixing with normal serum. • Similarly, VWD (Type C) that stabilises factor VIII can lead to a prolonged APTT though correction is also expected. If doesn't correct/partial correction = lupus anticoag, FXII-I, HMWK AB, FVIII-I, FIX I, FXI inhibitor

DM PT ID6 6. In recent years there has been increased interest in improving patient outcomes through using extended infusions of beta-lactam antibiotics. Which statement most appropriately describes the rationale for this practice? A. Extended/continuous infusions achieve higher peak concentrations of the antibiotics B. Extended/continuous infusions achieve a higher area under the concentration time curve at 24 hours, thus increasing efficacy C. Extended/continuous infusions require less access of the line, thus decreasing the risk of nosocomial infection D. Extended/continuous infusions achieve a greater proportion of time above the organism's minimum inhibitory concentration (MIC)

6. ANS: D • Different antibiotic classes achieve efficacy and bacterial killing according to different parameters and this help understand their dosing regimens. • For beta-lactam agents the pharmacokinetic-pharmacodynamic parameter is time above the organism's MIC and extended/continuous infusions are more likely to achieve this. • The clinical data supporting this hypothesis has been mixed but there is a role for extended or continuous infusions in critically ill or immunocompromised patients. Time dependent = beta lactams, mycins (incl. van + clinda), cyclines + linezolid = T > MIC Conc dependent = aminoglycosides, daptomycin, metro, quinolones = higher AUC

DM PT Neuro6 6. Which cell type gives rise to peripheral nerve myelin? A. Astrocytes B. Oligodendroglia C. Purkinje cells D. Schwann cells

6. ANS: D • Peripheral nerve myelin is antigenically distinct from central nervous system myelin, manufactured by a different cell type. • Auto-immune diseases often affect peripheral OR central nervous components, rather than both.

DM Mock 60. A couple undergo preconception carrier screening for cystic fibrosis (CF). They have no family history of CF. The carrier screening tests for the 80 most common CF-causing mutations, which detects approximately 90% of all CF-causing mutations. One of the parents tests positive for the deltaF508 mutation and the other parent tests negative. What is the couple's residual risk of having a child with CF? A. No risk B. 1/250 C. 1/500 D. 1/1000

60. ANS: D Test detects 90% of all CF-causing mutations, so a negative result still leaves a 10% residual risk of being a carrier. Population risk of being a carrier is 1 in 25, so residual risk for parent testing negative is 1/25 x 1/10 = 1 in 250. Risk for couple is thus 1 (the carrier parent) x 1/250 (the parent testing negative) x ¼ (each parent has a 1 in 2 risk of passing on a CF mutation if they are a carrier) = 1 in 1000.

DM Mock 61. A 93-year-old man from a nursing home is admitted to hospital with a reduced level of consciousness. He has end-stage Parkinson's Disease and is usually bed-bound and fully dependent, with all care. There is a large open pressure area on his sacrum. Clinical pathology shows acute on chronic kidney injury and an albumin of 16. He has previously expressed a wish for conservative management but there is no formal Advanced Care Directive in place. His wife would like to honour his wishes but his son, who is a lawyer, has requested active treatment. What is the most appropriate next course of action? A. Allow him to eat and drink a supervised diet B. Keep him nil by mouth (NBM) whilst awaiting a surgical review C. Insert an NG tube for feeds and medications D. Total parenteral nutrition (TPN) to remedy his malnutrition

61. ANS: A Evidence of multiorgan failure in a very frail patient. A desire for conservative treatment, supported by his Person Responsible indicates a palliative approach.

DM Mock 63. You are following up a 38-year-old male in the outpatient clinic for ongoing HIV care. He has been receiving elvitegravir/cobicistat/emtricitabine/tenofovir alafenamide for the last one year, after being diagnosed on routine STI screening. Other medications include pantoprazole 40mg daily for gastro-oesophageal reflux disease and amlodipine 10mg daily for hypertension. He has been working as a junior accountant and is adherent to his ART with a most recent CD4+ T cell count at 800 cells/uL and an undetectable viral load. He occasionally drinks alcohol, and admits to injecting methamphetamine occasionally; and, on previous review, was noted to have a newly positive hepatitis C antibody. He has never received hepatitis C treatment before. His hepatitis C viral load has now returned at 108,000 copies/mL; and has been identified as genotype 1a. A recent liver ultrasound did not reveal liver cirrhosis or hepatocellular carcinoma. What is the next best step in management? A. Refer to drug rehabilitation before starting hepatitis C treatment B. Start sofosbuvir + velpatasavir for 12 weeks C. Perform a fibroscan to further investigate for liver cirrhosis D. Check drug-drug interactions before prescribing hepatitis C treatment

63. ANS: D As per TG - all patients with HIV should have drug interactions checked before prescribing hepatitis C treatment (option B and C). Option C is not warranted before treatment given normal liver USS and should not delay commencement of treatment (could be performed later if indicated). Option A is a distractor - whilst this would be ideal, it is not paramount and again should not delay commencement of treatment and a harm minimization approach (population health).

DM Mock 65. A 32-year-old woman with recurrent focal seizures with retained awareness, and a single secondarily generalised tonic clonic seizure10 months ago, presents for advice regarding contraception. She is taking carbamazepine 300mg twice daily, which has been effective in controlling seizures completely. She had had an incomplete response on 200mg twice daily. She has tolerated this well, without side effects. She takes the combined oral contraceptive pill COCP, 30 micrograms of ethinyloestradiol. Which advice is most appropriate to offer? A. Avoid progestin-releasing intrauterine contraceptive devices (eg Mirena) B. Change to the progestin only contraceptive pill C. Continue 30 microgram ethinyloestradiol COCP, as this has been effective and without side effects D. Increase COCP to ethinyloestradiol 60 microgram

65. ANS: D REF: Up to date: Management of epilepsy during preconception, pregnancy, and the postpartum period -hepatic enzyme-inducing anti-epileptic drugs (AEDs) increase the metabolism of both progesterone and oestrogen. As the concentrations of these hormones may be lowered by 50% or more, adjustments are required in contraceptive regimes to ensure that pregnancy is prevented - Ideal contraceptive is mirena, and this would be the best advice (if this were one of the options: frustrating question!). - the progestogen-only pill is not recommended as reliable contraception in women and girls taking enzyme-inducing AEDs - Continuing 30ug ethinyloestradiol COCP is known to have an unacceptable failure rate - Increasing to ethinyloestradiol COCP content at at least 50ug is often recommended in practice, as well as combining with barrier methods: D is the best of the available options. Unsatisfactory - yes - but that's contraception for you!!

DM Mock 66. A 68-year-old woman who is ECOG 0 has completed adjuvant chemotherapy for stage II breast cancer. The cancer is ER+ PR+ and HER2 -. She has had a mastectomy as her initial therapy. What would be the next step in reducing her risk of breast cancer recurrence? A. Tamoxifen for 5 years B. Tamoxifen for 10 years C. Aromatase inhibitor for 5-10 years D. Aromatase inhibitor for 2 years, then tamoxifen for 3 years

66. ANS: C The recommended adjuvant hormone therapy for post-menopausal woman is an aromatase inhibitor for 5-10 years. Tamoxifen is recommended as the first- line endocrine therapy for premenopausal breast cancer

DM Mock 67. A 30-year-old female presents with a 6-week history of arthralgias, dyspnoea, cough and small volume haemoptysis. There is no history of weight loss or travel. She had completed a 2-week course of amoxycillin with clavulanic acid with her GP, with no improvement. Her background is significant for polycystic ovarian syndrome and sinusitis. Her sputum sample has not cultured any organism. PR3-ANCA is positive. Her vital signs and physical examination are normal. CT chest is given below: Cavitating lesion What is the next step in your investigation and management? A. Repeat ANCA testing and intravenous ceftriaxone B. CT guided lung biopsy C. M. tuberculosis interferon gamma release testing D. Nasal examination and biopsy

67. ANS: D The case is consistent with a clinical presentation of granulomatosis with polyangiitis (GPA). This can present with constitutional symptoms, haemoptysis, cavitary lung nodules or masses, sinusitis, haematuria, palpable purpura and mononeuritis multiplex. The initial differential diagnosis included microbial lung abscess however there had been little response to antibiotic therapy. Also, the patient has confirmed PR3-ANCA seroconversion which is associated with GPA (answer A is incorrect). CT guided lung biopsy is not recommended in pulmonary abscess unless malignancy a more likely diagnosis. The patient's history of sinusitis suggests that nasal examination and biopsy may lead to tissue diagnosis of GPA (answer D is correct).

DM Mock 68. Degenerative cervical spondylosis is a chronic, progressive deterioration of osseocartilaginous components of the cervical spine, and is a common part of the aging process. Which of the following statements is true about this condition? A. 50% of people have disk degeneration on MRI by the age of 50 B. More common in women than men C. Peak incidence between the ages of 40 to 60 D. Treatment is generally surgical

68. ANS: C Reference: Theodore N. Degenerative cervical spondylosis. New England Journal of Medicine 2020; 383: 159 - 168. Rationale: Degenerative cervical spondylosis is more common in men, than women, has a peak incidence between 40 - 60 and is usually managed conservatively without the need for surgery. Interestingly 80-90% of people have disk degeneration on MRI by the age of 50!

DM PT Onc7 For each of the following scenarios (Q7 and Q8), choose the most appropriate answer from the following options. A. 99% B. 90% C. 70% D. 50% E. 30% F. 10% G. 5% H. 1% 7. A 31-year-old man is diagnosed with metastatic testicular seminoma with lung metastases and normal AFP, normal hCG and normal LDH. He has undergone an orchiectomy and received BEP chemotherapy. What is the chance (approximate percentages) that he will remain disease-free at 5 years?

7. ANS: A Patients with pure seminomas have excellent prognosis. This patient has pure seminoma with good prognosis. Pure seminomas are divided into good-prognosis and intermediate-prognosis (note seminomas are NEVER poor-prognosis). Good prognosis is defined as: no non-pulmonary viscera; normal AFP, any hCG, any LDH, any primary site. Intermediate prognosis is defined by any of the following: non-pulmonary visceral metastases, any hCG, any LDH, normal AFP, any primary site. Patients with intermediate prognosis seminoma have a 5-year progression-free survival of around 70%.

DM PT Resp7 7. A 50-year-old man with a history of Type 2 diabetes mellitus presents with fever > 38 degrees and productive cough. His oxygen saturation is 94% on room air and his respiratory rate is 24 breaths per minute. He has been commenced on amoxycillin and doxycycline. His chest X-ray is shown (patchy R UL consolidation) What is the next step in your investigation and management? A. Send three early morning sputum samples for smear microscopy and mycobacterial culture B. Bronchoscopy with bronchial washings for smear microscopy and mycobacterial culture C. Mantoux test D. Interferon gamma release assay (e.g. QuantiFERON Gold)

7. ANS: A TB Discussion • The right upper lobe consolidation with cavity on chest X-ray in a patient with diabetes is suggestive of a diagnosis of tuberculosis infection. • Initial investigation in a patient who is productive of sputum should be with the least invasive test i.e. sputum samples • If this sample is negative, follow-up with induced sputum or bronchoscopy is recommended for sampling. • Mantoux test or IFN-g release assay do not confirm a TB diagnosis (answers c and d incorrect).

DM PT E7. A 74-year-old woman presents with cramps and paraesthesia. Serum calcium is 1.9 mmol/L (normal range 2.1-2.6), and serum PTH of 95 ng/L (10-65). eGFR is 35 ml/min. Which of her recently started medications is the most likely cause for her presentation? A. Denosumab B. Ranitidine C. Prednisolone D. Indapamide

7. ANS: A • Benefits on BMD increase and fracture reduction rapidly lost • Dramatic case reports of overshoot bone remodeling and multiple vertebral fractures • Increased incidence of multiple vertebral fractures in patients discontinuing DMAB (compared to placebo) in post hoc analyses of FREEDOM trial • Clinical implications: - Adhere to strict 6-months injection schedule during treatment - Do not initiate drug holiday: Post-cessation course of bisphosphonate likely to become standard of care

DM PT R7 7. Which of the following is an absolute contraindication to renal donation? A. History of metastatic malignancy B. Hypertension, controlled with 1 agent C. Microscopic haematuria D. Age > 65

7. ANS: A • Metastatic malignancy absolute contraindication • Immunosuppression poor prognosis with active malignancy cf to general population • All other relative contraindications Calculators to assess risk to live donors H/T and number of agents Diabetes Age / dependant on age of recipient Albuminuria Relationship o recipient GFR < 80 ml/min • Deceased donation - ? Dependant on recipient

DM PT H7 7. A 28-year-old primigravid female presents for her first 8-week antenatal visit. Routine blood tests detect thrombocytopenia. Further questioning reveals a history of easy bruising and heavy periods. A dental extraction performed at age 18 was uncomplicated. There is no family history of bleeding disorders. She is blood group O RhD+. Her other background is significant only for anxiety and depression for which she previously took desvenlafaxine that was ceased prior to her pregnancy. Examination does not reveal any signs of bruising or bleeding. Haemoglobin 119 g/L (115-135) MCV 81 fL (80-100) White cell count 9.2 x 109 /L (4-11) Platelet count 55 x 109 /L (150 - 400) Mean Platelet Volume 13 fL (8-11) Ferritin 37 ug/L (30-400) Her blood film shows no platelet aggregates, clumping or satellitism, with mild platelet anisocytosis though generally small and well granulated. What is the most likely cause for these findings? A. Idiopathic Thrombocytopenic Purpura B. Gestational thrombocytopenia C. HELLP syndrome D. Type 2B VWD

7. ANS: A • The first trimester presentation makes gestational thrombocytopenia unlikely, along with the degree of thrombocytopenia <80. • HELLP is similarly unlikely in the first trimester and patient is otherwise well. • The lack of bleeding history with dental extraction, and rarity of Type 2B von Willebrand's, makes this less likely. • Common causes of early pregnancy thrombocytopenia include chronic ITP and hereditary thrombocytopenia.

DM PT Geris 7 7. Amitriptyline may be prescribed for various conditions, including neuropathic pain and insomnia. Each of the following are recognised side effects except... A. QT prolongation B. Diarrhoea C. Urinary retention D. Dry skin

7. ANS: B

DM PT EBP2 7. A new point-of-care device has been developed which tests blood for presence of the HLAB*15:02 allele. This genetic profile is associated with a high risk of carbamazepineassociated severe skin reactions and is far more common among Chinese people than Caucasian people. In a validity study of the new device among Chinese people, which of the following parameters will be increased compared to a validity study of the same device among Caucasian people? A. Sensitivity B. Specificity C. Positive predictive value D. Negative predictive value

7. ANS: D Negative predictive value plug into equations

Book R7 and 8 A. Abatacept B. adalimumab C. anakira D. etanercept E. infliximab F. leflunomide G. rituximab H. Tociluzumab 7. Which drug inhibits activity of IL6? 8. Which drug neutralises activity of IL1?

7. H Tocilizumab inhibits IL-6. Involved in RA and sJIA. Toc and MTX > toc monotherapy. 8. C Anakira. recombinant ILR antag. RA and ?familial mediterranean fever Abatacept - CTLA4-Ig adalimumab TNFa etanercept/infliximab: TNFa leflunomide: pyrmidine synthesis I(inhibiting DHODH) rituximab: antiCD20/b cell depletor

DM PT Geris8 8. With regards to falls in the older person, which of the following statements is FALSE? A. High dose vitamin D supplementation can reduce the risk of falls. B. Bifocal glasses can increase the risk of outdoor falls C. Gait and balance training can reduce the risk of falls D. Falls occur more often in nursing home residents than in community dwellers.

8. ANS: A Study by Sanders et al, JAMA 2010 - yearly megadose vitamin D a/w increased risk of fractures. Prevention of falls Multimodality exercise Home modification Multi-component intervention Footwear assessment Medication assessment Regular toileting

DM PT Onc8 For each of the following scenarios (Q7 and Q8), choose the most appropriate answer from the following options. A. 99% B. 90% C. 70% D. 50% E. 30% F. 10% G. 5% H. 1% 8. A 52-year-old peri-menopausal women with a distant smoking history is diagnosed with stage 1 breast cancer. She undergoes wide local excision with clear margins and sentinel lymph node biopsy which did not reveal any lymph node metastasis. She received adjuvant radiotherapy. Given the tumour was strongly ER and PR+, and HER2-, she proceeded to receive adjuvant tamoxifen. Adjuvant chemotherapy was not recommended at the time. What is the likelihood that she would be alive at 5-years?

8. ANS: A Patients with breast localized breast cancer have excellent survival. The patient did not receive adjuvant chemotherapy suggesting that the tumour was low risk. 5-year survival of patient with stage I breast cancer ~99%, stage 2 > 90%, stage 3 ~60%; stage 4 >15% (based on UK data between 2002-2006 from Cancer Research UK so 5-year survival have since improved)

DM PT EBP3 8. A 10-year cohort study was undertaken to examine the association between smoking and glioma. The unadjusted relative risk for smoking versus non-smoking was 1.21 (95% confidence interval [CI] 1.04-1.38), and the relative risk adjusted for age was 1.07 (95% CI 0.90-1.25). Which of the following interpretations of these findings is CORRECT? A. Smoking is an independent predictor of glioma. B. Age is a confounder in the association between smoking and glioma. C. The adjusted relative risk was statistically significant at the 5% level of significance. D. After adjustment, smoking was associated with an increase in incidence of glioma by 7% per person-year.

8. ANS: B Age is a confounder in the association between smoking and glioma

DM PT E8 8. A 58-year-old man presents with bothersome erectile dysfunction. Libido is preserved. Comorbidities include T2DM diagnosed 8 years prior, and a previous non-STEMI with a coronary stent 3 years ago. He stopped smoking 5 years ago, with a 40-pack year history. Recent stress echo was normal. Medication include metformin, empagliflozin, rosuvastatin and perindopril. HbA1c is 7.9%, fasting total testosterone is 10.5 nmol/L, and repeat 10.9 nmol/L. LH is 5 mIU/L. On exam, BMI is 29 kg/m2 , and testes are 18 ml bilaterally, Rectal exam reveals a normal prostate. No plans for paternity. What is the best initial approach? A. Commence trial of testosterone gel to increase serum testosterone into the mid-normal range (~15 nmol/L) for 3-6 months; continue testosterone only if improvement in ED. B. Commence a trial of a PRN phosphodiesterase-5 inhibitor C. Refer for sexual counseling D. Hold off treatment, perform a pituitary-directed MRI first.

8. ANS: B In addition to lifestyle measures (stop smoking, weight loss) phosphodiesterase-5 inhibitor treatment is 1st line treatment for erectile dysfunction. C might be reasonable if phosphodiesterase-5 inhibitor treatment does not work. The benefits of T treatment are not established, especially if T concentrations are only modestly reduced. Pituitary MRI is not indicated unless T is severely low (e.g. < 5.0 nmol/L) or there is clinical suspicion (e.g. mass effect -headaches, visual field defect)

DM PT Resp8 8. A 70-year-old ex-heavy smoker with COPD attends the outpatient clinic with wheeze and dyspnoea. They have required one course of oral prednisolone and amoxicillin for a chest infection in the last 12 months but have not presented to hospital. Their oxygen saturation is 93% on room air. Current medications include tiotropium 18mcg mane via handihaler and salbutamol MDI 100mcg 2 puffs when required. They are up to date with the yearly flu vaccination and inhaler technique is adequate. Lung function is provided below: Which of the following statements is the most correct regarding their management? A. Combination therapy with long acting muscarinic antagonist, long-acting beta agonist and inhaled corticosteroid is recommended B. Combination therapy with long acting muscarinic antagonist and long acting beta agonist is recommended C. Combination therapy with long acting beta agonist and inhaled corticosteroid is recommended D. LAMA inhaler should be changed to a Respimat device

8. ANS: B • The patient in this scenario has symptomatic COPD which is moderately severe according to spirometry. • They may benefit from combined LAMA/LABA for symptomatic relief and to reduce exacerbations (answer b is correct). • They do not appear to be a frequent exacerbator and the spirometry is not suggestive of an alternative diagnosis of asthma (answers a and c incorrect). • Their technique is adequate and therefore a device change as the sole change in management is not recommended (answer d is incorrect).

DM PT H8 8. A 26-year-old female refugee of African descent is referred by a GP for management of sickle cell disease. She has a history of frequent hospital admissions for sickle pain crises. She is non-compliant with hydroxyurea and penicillin prophylaxis. Which of the following statement is NOT true regarding hydroxyurea in sickle cell anaemia? A. Hydroxurea raises HbF levels B. Hydroxyurea reduces admissions for pain crisis C. Hydroxyurea is contraindicated in severe anaemia D. Hydroxyurea reduces granulocyte counts which leads to vasodilation

8. ANS: C All statements are current except C • Hydroxyurea is indicated in severe anaemia as the anaemia is caused by sickling. • Hydroxyurea increases HbF levels (fetal haemoglobin) which is resistant to sickling and therefore decreases blood transfusion requirements and sickling episodes.

DM Mock 8. A 35-year-old journalist returns from a working holiday in Nigeria. Three days later, he presents with fevers, headache and diarrhea. Whilst he was over there, he ran out of antimalarial prophylaxis. On examination, he appears drowsy and jaundiced. His blood film is as shown below He is diagnosed with Plasmodium falciparum malaria and the parasite count is measured at 4%. He is immediately commenced of IV artesunate 2.4mg/kg, and two sets of blood cultures are collected. What other adjunctive management should the patient receive? A. IV quinine B. Prednisolone 1mg/kg C. IV ceftriaxone and paracetamol D. Intravenous Immunoglobulin

8. ANS: C This patient has severe malaria as evidenced by the parasite count > 2% and clinical features of impaired consciousness and jaundice. He requires immediate treatment with IV artesunate. The recommendation from Therapeutic Guidelines in patients with severe malaria is to give adjunctive therapy with IV ceftriaxone due to high rates of concomitant bacteraemia, and paracetamol (IV or oral) to protect for haemolytic acute kidney injury (based on recent evidence of benefit in the paediatric population - Plewes K et al. Acetaminophen as a Renoprotective Adjunctive Treatment in Patients with Severe and Moderately Severe Falciparum Malaria: A Randomized, Controlled, Open-Label Trial. Clin Infect Dis. 2018 Sep 14;67(7):991-999.) IV quinine dihydrochloride (option A) is also recommended in patients who have travelled from certain regions where artesunate resistance is increasing (Greater Mekong Subregion - Thailand, Vietnam, Cambodia, Laos and Myanmar) - this does not include Africa. The other options are not recommended.

Deltamed 5/8. Which of the following is NOT thought to be a contributing factor in age-related sarcopenia? A. Reduced activity/nutrition B. Hormonal changes C. Reduced inflammation D. Reduced motor neurons

8. ANS: C • Contributors to age-related sarcopenia are : - Reduced activity Reduced nutrition - Reduced sex-hormones Meta-inflammation - Less motor neurone innervation

DM PT R8 8. A 29-year female non-smoker presents with difficult to control hypertension. Ultrasound of her renal arteries shows right sided renal artery stenosis, with subsequent CT angiography showing a characteristic "string of beads" appearance. Which of the following is not indicated? A. Ultrasound Doppler of intracerebral and extracerebral vessels B. Aggressive medical management of hypertension C. Referral for genetic testing D. Referral for renal angioplasty

8. ANS: C • Fibromuscular dysplasia Vasculopathy medium sized vessels Female predominance, childbearing age Secondary cause H/T Renal involvement renal (60-75%), cerebrovascular (25-30%), visceral (9%), limbs (5%) Genetics ~ 10% cases?? Hormonal basis Diagnosis FMD in any vascular bed - CTA/MRA of all vessels cerebral to abdomen/ pelvis Aspirin Smoking cessation

Delta med 4/8. In men with rheumatic diseases, which DMARD is most likely to affect fertility? A. Adalimumab B. Methotrexate C. Prednisolone D. Sulfasalazine

8. ANS: D SULFASALAZINE • Not teratogenic • Associated with • Oligo-/azoospermia • Reduced sperm mobility • Reduced sperm quality

DM PT ID 8 A 67-year-old immunocompetent man presents with progressive memory loss over three months. A non-contrast CT brain does not show any abnormalities. A lumbar puncture is performed with the following results: Opening pressure 25 cm H20 White cell count 176x106 /L, with 70% lymphocytes Glucose 3mmol/l and protein 2g/L Cryptococcal antigen test: Positive What is the most appropriate initial therapy? A. Liposomal amphotericin B 3 mg/kg daily B. Fluconazole 800 mg daily C. Liposomal amphotericin B 5 mg/kg daily and Fluconazole 800 mg daily D. Liposomal amphotericin B 3 mg/kg daily and Flucytosine 25 mg/kg 6-hourly

8. ANS: D • This clinical stem is consistent with a diagnosis of cryptococcal meningitis. • There are two main species: Cryptococcus neoformans and Cryptococcus gatii. • Cryptococcus neoformans is the more common organism and is typically associated with disease in immunocompromised patients such as patients living with HIV or transplant recipients. • Cryptococcus gatii conversely occurs predominantly in immunocompetent patients and is largely limited to Australia and the north-western United States/Canada. This clinical presentation is more consistent with C. gatii. • Fortunately, the treatment is the same - for induction therapy, Amphotericin B and Flucytosine in combination remain the cornerstone of therapy. • A prior trial showed that this combination was superior to Fluconazole alone and that combination Amphotericin B/Fluconazole was not superior (Day et al. N Engl J Med 2013; 368:1291-1302). • Monitor renal function and potassium levels.

DM PT G9 9. What is the mechanism of action of tofacitinib? A. Janus Kinase inhibitor, preferentially JAK1 and JAK3 B. Anti-TNF C. IL-13 and IL-23 D. Inhibits DNA synthesis

9. ANS: A Tofacitinib for UC • Oral, small-molecule JAK inhibitor • inhibits all JAKs but preferentially inhibits JAK1 and JAK3 • More effective than placebo for induction and maintenance therapy in patients with moderately to severely active UC in large phase 3 RCT • AEs included infection (especially herpes zoster) and cardiovascular events

DM PT H9 9. A 34-year-old female is admitted with a 2-day history of shortness of breath and chest pain. She has had severe menorrhagia since the birth of her second son. She has not received a blood transfusion before. Her haemoglobin is 65g/L (135-160) on admission, with a MCV of 62 fL (80-100). Two units of packed red blood cells are requested from Blood bank. 10 minutes into the first unit she reports severe pain at the infusion site, back pain, dark urine, increased shortness of breath and temperature rises from 36.5 to 39o C. What statement is true regarding acute haemolytic transfusion reactions? A. The most common cause is 'wrong blood in tube'. B. The DAT is negative C. The most common groups implicated are Kell, Duffy and Rhesus D. Anti-A and Anti-B antibodies require a previous blood transfusion to develop

9. ANS: A • Acute haemolytic reactions are caused by the transfusion of ABO incompatible red blood cells. • The anti-A and anti-B antibodies are formed early in life from natural exposure it does not need blood transfusions to develop • Immediately haemolysis on exposure to as little as 15-20ml of incompatible blood. • In modern times it is most likely caused by procedural error such as mislabelling or laboratory error (UK SHOT report)

DM PT Geris 9 9. A 90-year-old community-dwelling man with Vascular dementia is admitted to hospital with urosepsis. He is sleeping a lot and only intermittently lucid. His wife reports he is more confused than usual, though not combative or aggressive. Which of the following statements about his condition is FALSE? A. His mortality risk is 40% B. The lack of agitation is a positive prognostic sign C. He is at risk of malnutrition and pressure areas D. There is a greater risk he will be placed in residential care

9. ANS: B • Hypoactive delirium is a negative prognostic sign • Delirium puts sufferers at risk of malnutrition, pressure areas, constipation and other iatrogenic effects • The mortality rate is up to 40% • The rates of cognitive dysfunction at 12 months post delirium are high, necessitating admission to a nursing home

DM PT EBP4 9. A placebo-controlled clinical trial was undertaken to assess the efficacy of low-dose aspirin in preventing recurrent strokes. Over the two-year trial duration 6% of participants in placebo arm experienced recurrent stroke, compared to 4% in the aspirin arm. What was the number needed to treat to prevent stroke over one year? A. 2 B. 20 C. 50 D. 100

9. ANS: C NNT = 1/(absolute risk difference) = 1/(6%-4%) = 1/2% = 50 Number of people needed to be treated to prevent the outcome in one

Deltamed PT9 9. Which cardiac chamber develops the majority of volume overload in ventricular septal defect? A. Left atrium B. Right atrium C. Left ventricle D. Right ventricle

9. ANS: C Left ventricular volume overload is the classic complication of VSD. Blood is shunted L->R according to pressure gradient. Over time, this causes volume overload in the LV to compensate for reduced cardiac output.

DM Mock 9. A 22-year-old woman presents two days following removal of her right lower wisdom tooth with unilateral tongue swelling on that side. This is on the background of a prior episode of hand swelling after an appendicectomy for suspected appendicitis a year prior. As part of her work-up, C4 was noted to be low at 0.06 g/L. What would be the next best test to order? A. Mast cell tryptase B. Specific IgE to local anaesthetic C. Skin prick and intradermal testing to local anaesthetic D. C1-inhibitor function

9. ANS: D History of swelling in setting of recent procedure as well as low C4 suggestive of hereditary angioedema. Thus C1-inh is next best test to order. History is not suggestive of an IgE mediated process, although tryptase is another test worth doing to exclude mastocytosis (it is however not the 'best' next test)

DM PT E9 9. In chronic kidney disease due to type 2 diabetes, which of the following medications has LEAST evidence for improved renal outcomes? A. Irbesartan. B. Empagliflozin C. Dulaglutide D. Glargine.

9. ANS: D Recognize recent evidence regarding renal and cardioprotective benefits of anti-diabetic agents See answers

DM PT Resp 9 9. A 60-year-old patient with a history a pulmonary embolism diagnosed 3 months ago presents with persistent dyspnoea on exertion. Which of the following is the most correct regarding their investigation and management? A. A negative CTPA excludes CTEPH (chronic thromboembolic pulmonary disease) B. A normal echocardiogram should prompt investigation with VQ scan C. Mean pulmonary artery pressure above 25mmHg at diagnosis is associated with increased risk of progressive pulmonary hypertension D. Mean pulmonary artery pressure above 50mmH at diagnosis is associated with increased mortality at 2 years

9. ANS: D • Mean pulmonary artery pressure above 50mmHg has been associated with increased mortality (answer d is correct) and that above 30mmHg has been associated with increased risk of progressive pulmonary hypertension (answer c is incorrect). • A negative CTPA does not rule out CTEPH (answer a is incorrect). • In a patient who presents with persistent breathlessness 3-6 months post-acute pulmonary embolism, an echocardiogram should be performed in the first instance and if there is evidence of pulmonary hypertension or right heart strain followed with a VQ scan to exclude chronic thromboembolic disease (answer b is incorrect).

DM PT Onc9 For each of the following scenarios (Q9 and Q10), choose the most appropriate answer from the following options. A. Loperamide B. Metronidazole C. Ciprofloxacin D. Ganciclovir E. Atropine F. Infliximab G. Mycophenolate H. Budesonide 9. A 58-year-old man has been receiving ipilimumab and nivolumab for metastatic melanoma His tumour have been significantly responding to treatment. After cycle three, he presents with severe diarrhea, with 10 or more stools per day, and is found to have significant intravascular volume depletion. The patient is hospitalized and rehydrated with IV fluids. Extensive evaluation excluded infectious causes. The patient is started on high-dose IV steroids as well as nonspecific low motility agents. After 3 days of steroids, the diarrhea continues with eight or more watery stools. Bowel perforation was excluded on imaging. What is the next step in his management?

9. ANS: F The patient has immune-mediated colitis. Most patients respond to steroids. The few who do not require escalation of therapy. Infliximab is an anti-TNF-alpha Ab and is helpful in the management of steroid-refractory/resistant immune-mediated colitis

Dunedin gastro Approximately 10% of patients have a reduced TPMT activity. This patients' level was measured at 8.7 units/ml RBC (normal 9.3 - 17.6 unit/ml RBC). What are your concerns in view of this result? A. The patient might develop an allergic reaction to azathioprine B. This patient is at risk of myelosuppression C. This patient can develop lymphoma D. No risk since the level is just below normal E. Measurement of the TPMT is not used in clinical practice

??B

deltamed renal An otherwise stable dialysis patient has the following biochem: Ca 2.50 mmol/l (N 2.1 - 2.6) Phos 2.1 mmol/l (N 0.8 - 1.5) Albumin 44 gm/l (N 35 - 42) iPTH 35 pmol/l (N < 7) The patient currently is prescribed CaCO3 600 mg i tds with meals. The most appropriate next step in management is: A.Elective parathyroidectomy B.Increasing CaCO3 to 2 tabs three times a day C.Adding Al(OH)3 1 tab three times per day D.Adopting a strict low phosphate diet E.Adding calcitriol 0.25 ug daily

??B or C

Dunedin gastro A 46 year old man presents acutely with a large bowel obstruction. At surgery an obstructing tumour is found and a left hemicolectomy is performed. In the specimen adenocarcinoma is confirmed and three small tubular adenomas are also found in the specimen. A. the presence of microsatellite instability in tumour tissue is diagnostic of HNPCC B. the combination of tumour and polyps is suggestive of attenuated FAP (familial adenomatous polyposis) C. first degree relatives of the patient should be recommended to have screening colonoscopy D. a normal fundoscopy precludes a diagnosis of FAP E. this is a common presentation of sporadic colorectal cancer

?E

CLS Genetics Which one of the following is the most appropriate way to undertake DNA genetic diagnosis of Huntington's disease? A. RFLP (restriction fragment length polymorphism) analysis. B. PCR (polymerase chain reaction) analysis. C. Size estimation of a triplet repeat. D. Southern hybridisation. E. Identification of gene-specific mutations.

?b

72 man with a history of chronic obstructive airways disease and smoking presents with an apparent viral upper respiratory tract infection. On clinical examination he is plethoric and has respiratory distress. His laboratory studies indicate the following: Haemoglobin 199 g/dL 128-175 Haematocrit 59% 36-50 Total WCC 19.5 x 10^9/L 4-8 Neutrophils 10.1 x 10^9/L 2-4 Lymphocytes 0.9 x 10^9/L 2-4 Eosinophils 0.5 x 10^9/L 0.0-0.5 Basophils 0.1 x 10^9/L 0-0.1 Platelets 405 x 10^12/L 150-450 Serum Erythropoietin 34.0 units 4.8-21.9 The most likely explanation is: A. Carcinoma of lung B. High affinity haemoglobin. C. Dehydration. D. COPD. E. Polycythaemia vera

A

A 32-year-old woman with a 2-year history of RA is taking methotrexate 20 mg weekly, leflunomide 20 mg once daily, and folic acid 5mg/week + celecoxib 200mg bd. She is interested in getting pregnant. Which of the following steps is/are necessary prior to conception? A) Initiate cholestyramine 8 g tds daily for 11 days B) Decrease MTX to 5 mg/ wk, inc folic acid to 5mg 3 x/wk C) Stop methotrexate, safe to conceive in one month D) Change celecoxib to naproxen E) Stop all cDMARDs and NSAIDs and start rituximab

A

Dun cardio Differentiation between cardiogenic and noncardiogenic pulmonary oedema is most reliably performed by measurement of: A. left ventricular end diastolic pressure. B. left ventricular peak systolic pressure. C. pulmonary artery peak systolic pressure. D. pulmonary artery diastolic pressure. E. pulmonary artery mean pressure.

A

CLS GI Quiz Which one of the following statements about the management of coeliac is INCORRECT? Select one: a. Tranglutaminase-IgA antibody levels are an accurate reflection of adherence to the gluten free diet and mucosal healing b. A major aim of treating refractory coeliac disease type 2 is to destroy the aberrant clonal intra-epithelial lymphocyte c. Small intestinal mucosal healing can be slower if coeliac disease was diagnosed later in life or in the presence of certain medications d. Pancreatic insufficiency and microscopic colitis are associated with coeliac disease and may cause non-responsive disease e. The commonest cause of non-responsive coeliac disease is ongoing gluten exposure

A

CLS Haem 28 man fractured ankle. Pre-op the following investigations are performed: Haemoglobin 105 g/dL 128-175 RCC 5.6 x 10^12/L 4.5-6.5 MCV 62fL 80-96 Total WCC 12.5 x 10^9/L 4-8 Platelets 390 x 10^12/L 150-450 Ferritin 95ng/ml 20-300 HbA2 (HPLC) 5.2% 1.8-3.5% HbF 1.2% 0-2.0% HPLC No abnormal bands detected Cause? A. beta-thalassaemia minor B. chronic blood loss. C. anaemia of chronic disease. D. congenital sideroblastosis. E. Sickle cell trait.

A

CLS Haem 47 man with severe Haemophilia A and high titre FVIII inhibitor needs hip surgery. Peri-op replacement coagulation product of choice is: A. Recombinant activated FVII. B. Recombinant FVIII. C. Recombinant FIX. D. FFP. E. Cryoprecipitate.

A

CLS Haem 50 woman, which of following laboratory abnormalities would be associated with highest level blood viscosity? A. Serum IgM 40.0 (0.5-3.0). B. Peripheral blood leucocytosis 25.0 (3.9-12.7). C. Haematocrit 0.46 ( 0.32-0.42). D. Serum IgG 40.0 (7.5-15.6). E. Peripheral blood thrombocytosis 500 (150-396).

A

CLS Haem A previously well 37 man p/w lethargy and easy bruising over several weeks. Haemoglobin 125 g/dL 128-175 Total WCC 8.8 x 10^9/L 4-8 Platelets 1060 x 10^12/L 150-450 Blood film Numerous platelets with many large forms Most Likely diagnosis is: A. Essential thrombocythaemia. B. Occult haemorrhage. C. Primary myelofibrosis. D. Occult carcinoma. E. Chronic myeloid leukaemia.

A

CLS Haem A young woman is developing a non febrile transfusion reaction after her current blood transfusions. What is the reason to use a filter? A. Decrease donor white cells B. Decrease bacterial contamination C. Decrease donor cytokines D. Decrease CMV transmission E. Decrease ABO antigens

A

CLS ID A 22 year old renal transplant patient is relisted for transplantation after graft failure. They are highly sensitised and eculizumab (a complement inhibitor) is being considered. Which of the following is one of the most important to consider in the pretransplant work-up? A. Vaccination for Hib, meningococcal and pneumococcal infections B. Prophylaxis post transplant with voriconazole C. Update of live vaccines D. Remind her she she should not be in contact with any pets E. Review travel history

A

CLS ID A 30 year old man presents with a 4 day history of fever and a sore throat. Over the last 48 hours he has developed an intensely itchy vesicular rash, a dry cough and shortness of breath. His chest radiograph showed diffuse nodular opacities. The appropriate specific treatment is: A. Intravenous acyclovir B. Intravenous ceftriaxone C. Oral acyclovir D. Intravenous gancyclovir E. Intravenous liposomal amphoteracin

A

CLS ID A 69 year old male presents with malaise, headache and intermittent confusion for last 12 hours. On examination there is mild photophobia. CSF: protein 0.6, glucose 4.2 and lymphocyte count 8. CT brain is normal. What is the most appropriate management? A. Admit and IV acyclovir B. Admit and IV ceftriaxone C. Admit for observation and analgesia D. Discharge with analgesia E. Discharge without treatment

A

CLS ID What is the main role of the quantiferon gold test? A. detect exposure to MTB B. determine extent of MTB C. response to Rx D. predict active E. rule out MTB

A

CLS Rheum Which of the following activities is most likely to cause pain in osteoarthritis of the patello‐femoral joint? A. Walking up stairs B. Jogging C. Walking on flat ground D. Getting out of a chair E. Standing

A

DM ID 37yo cattle farmer presents with fevers, aches, drenching sweats, nausea, LOA and dry cough for 10 days. Did not respond to Augmentin from LMO. Has systolic murmur heard but otherwise normal exam. Keen to get back to farm as it is calving season. Which of the following is least true? • A - He has man 䇺flu and needs to toughen up • B - Should receive doxycycline • C - Should get a TTE • D - Check serology for Q fever, leptospirosis, brucellosis • E - If this is Q fever, he should not get vaccinated for it after this

A

DM ID 76 yo female with persistent dry cough over 8 months. 2kg loss of weight. Exertional dyspnoea. Malaise. Normal to examination. Imaging as shown. Bronchoscopy performed and BAL cultures AFB, as does the post- bronch sputum. Rapid PCR result = not TB. • Which of the following statements about pulmonary MAC infection is true? • A. Three times a week therapy gives equivalent outcomes to daily therapy • B. This is likely to represent colonisation • C. The patient should be isolated • D. If she cannot tolerate therapy with clarithromycin, rifabutin and ethambutol, the clarithromycin should be stopped • E. Treatment should be delayed until susceptibility results are available

A

Dun Neuro A 23-year-old right-handed man presents with a history of recurrent episodes characterised by a very brief feel of warmth and a floating sensation, then impairment of consciousness. Observers report that his head consistently turns to the left, and he fiddles with his hands, holding the left hand stiffly compared to the dextrous right hand. The episodes last about 60 seconds and he is often amnestic for the event. His inter EEG is normal. His cranial magnetic resonance (MR) scan is shown. The most likely diagnosis is: A. right temporal lobe epilepsy related to mesial temporal sclerosis B. left temporal lobe epilepsy related to an amygdaloid cyst C. right temporal lobe epilepsy related to a tumour. D. lefttemporal lobe epilepsy related to a neuronal migration defect. E. right cortical epilepsy related to a presumed perinatal event

A

Dun Neuro An 18-year-old gives a history of recurrent morning myoclonus and absences after sleep deprivation. The most likely epilepsy syndrome is: A. juvenile myoclonic epilepsy. B. benign myoclonic epilepsy. C. petit mal epilepsy. D. benign Rolandic epilepsy. E. absence epilepsy.

A

Dun Neuro The most common identifiable cause of epilepsy in the elderly is: A. cerebrovascular disease B. traumatic injury C. Alzheimer's disease D. tumours E. alcohol

A

DM ID A 23-year-old male is involved in a motor vehicle accident and sustains multiple fractures, some of which are compound, as well as significant head and thoracic injuries. He is being managed in the intensive care unit and has been ventilated for 6 days. On day 7, he is noted to be increasingly hypoxic, with need for increasing fraction of inspired oxygen and increased pressure support. Increasingly purulent tracheal aspirates are being obtained by suctioning by his nurse and are sent off for Gram stain and culture. His white cell count has risen from 15 the day before to 27 today. CRP has increased from 45 three days earlier to 170 today. Chest X-ray is crappy. This has occurred in the context of him being on intravenous piperacillin-tazobactam since his last operation three days ago. • The diagnosis of ventilator-associated pneumonia is made, a bronchoscopy is performed to obtain specimen for broncho-alveolar lavage (BAL), and his antibiotics are changed to meropenem and vancomycin. Subsequently, his BAL specimen is reported as culturing a heavy growth of Candida albicans. All other cultures of blood and urine are negative. • Which of the following is correct? • A. This is very unlikely to represent a true pathogen and anti-fungal therapy is not required at this time. • B. Therapy with fluconazole should be commenced. • C. Therapy with caspofungin should be commenced. • D. Therapy with amphotericin should be started but could be changed to fluconazole later, provided that the isolate is susceptible. • E. Surgical resection of a fungal ball may be required.

A

DM PT Im3 3. Which of the following is NOT a surface receptor on dendritic cells? A. NLRP3 (NOD-, LRR- and pyrin domain-containing protein 3) B. Fc-gamma receptor C. Complement C3d receptor D. DC-SIGN (C-type lectin receptor)

A

DM PT Rh2 2. In a young 24-year old patient with rheumatoid arthritis which of the following medications is the safest to take during pregnancy: A. Certolizumab B. Etanercept C. Golimumab D. Infliximab

A

DM PT Rh3 3. A 62-year-old woman with seropositive rheumatoid arthritis presented for routine ocular review. She was found to have visual acuity of 20/20 in both eyes. The retinal examination showed a bull's-eye pattern of hypopigmentation bilaterally and visual field testing demonstrated ring scotomas, and retinal imaging on optical coherence tomography showed corresponding loss of photoreceptors and retinal pigment epithelium. Which of her regular medications is most likely to be causing these changes? A. Hydroxychloroquine B. Methotrexate C. Prednisolone D. Sulfasalazine

A

DM PT Rh6 6. In patients presenting with scleroderma, which organ is least likely to be affected by the disease? A. Brain B. Gastrointestinal tract C. Heart D. Kidney

A

DM PT Rh8 8. A 20-year-old woman presents with bilateral conductive deafness, palpable purpura on the legs, and haemoptysis. A radiograph of the chest shows a thin-walled cavity in the left lower zone. Investigations reveal red cell casts in the urine and an elevated serum creatinine level. What is the most likely diagnosis? A. Granulomatosis with polyangiitis B. Henoch-Schonlein purpura C. Polyarteritis nodosa D. Takayasu's arteritis

A

Deltamed Cardio 54 y/o man sudden onset AF. Attempt made of pharmacological cardioversion. Prescribed flecainide + metoprolol. What is the purpose of metoprolol? A. prevent 1:1 conduction if A.Flutter occurs B. reduce myocardiac ischaemia if CAD present C. reduce sympathomimetic complications of fleicanide D. Reduce risk of fleicanide induced V arrhythmias E. synergistically increase chance of reversion

A

Deltamed ID 56M morbidly obese with T2DM on metformin and empagliflozin. Presents with urinary retention, extremely painful groin and features of sepsis. • Immediate treatment should consist of all of the following except: • A. Hyperbaric oxygen therapy • B. Surgical debridement • C. Broad spectrum ABx (e.g., Taz) • D. Inotropes to maintain BP • E. Cessation of empagliflozin

A

Deltamed Immuno Activation of B lymphocytes by thymus‐ dependent antigens requires all of the following EXCEPT A. T cells expressing CD8 surface marker B. T cells expressing CD3 surface marker C. Processing of antigen and presentation of antigenic peptides bound to MHC class II antigen D. T cells expressing T cell receptors E. Costimulation through CD40/CD40L interactions

A

Deltamed stats The risk of human immunodeficiency virus (HIV) transmission by blood transfusion, which of the following is the most important property of a blood screening test? A. Sensitivity B. Specificity C. Positive predictive value D. Positive likelihood ratio E. Accuracy.

A

Dun Cardio 4. What is the most sensitive index of cardiac function? A. Cardiac output. B. Stroke volume. C. Ratio of stroke volume to end diastolic volume. D. Ratio of stroke volume to cardiac output. E. Ventricular end diastolic volume.

A

Dun ID A 50 yo male undergoing chemotherapy for AML. Neutropenia ongoing for 12 days, he has daily fevers. He has been on acyclovir and posaconazole prophylaxis. The lab calls, a blood culture obtained 3 days prior shows a yeast - identified as a Candida species. • Best therapy to start: 1.) Start caspofungin 70 mg IV load, then 50 mg IV daily 2.) Start high-dose fluconazole 1200 mg daily IV 3.) Start liposomal amphotericin at 5 mg/kg daily 4.) Start voriconazole 6 mg/kg IV q 12 hours x 2 dose, then 3 mg/kg q12 hr

A

Dun Neuro A 20-year-old man presents following a generalised tonic-clonic seizure precipitated by alcohol excess and sleep deprivation. He has an eight-year history of episodes of a brief loss of time without other symptoms. Observers report that these episodes last 10 to 15 seconds during which he stares blankly and is unresponsive to command. His left hand may stiffen and he may fiddle with his buttons using his right hand. He will then look surprised and resume his conversation. Cranial magnetic resonance imaging(MRI) (T1 weighted) is shown below. The most likely diagnosis is: A. mesial temporal sclerosis. B. generalised epilepsy with absences. C. hippocampal tumour. D. callosal agenesis. E. temporal lobe neuronal migration abnormality.

A

Dun Rheum A 25yr patient with SLE wishes to start a family: • All of the following regarding SLE and pregnancy are true EXCEPT: A. Hydroxychloroquine is not recommended in pregnancy B. Miscarriage is more common in SLE C. Pre-eclampsia is more common in patients with lupus nephritis D. Pregnancy can increase lupus flares E. SLE patients have similar rates of fertility to women without SLE

A

Dun stroke A 66‐year‐old alcoholic man collapses and is brought to the Emergency Department. He is found to have fixed dilated pupils, and is somnolent with no upgaze. Reflexes are brisk with bilaterally upgoing plantar responses. A cranial computed tomography (CT) scan performed after one hour is normal. An electroencephalography (EEG) reveals intermittent generalized anterior predominant slowing, with frontal intermittent rhythic delta activity (FIRDA) without focal of epileptiform features. Neurological findings remain unchanged over a period of 24 hours. The most likely diagnosis is: A. Ponto‐mesencephalic ("top of the basilar") stroke B. Middle cerebral artery stroke with cerebral herniation ("coning"). C. Bilateral middle cerebral artery stroke. D. Post‐ictal encephalopathy. E. Wernicke's encephalopathy.

A

Dun stroke A patient presents with weakness of the right hand and face, but sparing the leg. The most likely anatomic location of the stroke is: A. Cerebral cortex B. External capsule. C. Internal capsule. D.Pons. E. Medulla.

A

Dunedin Resp A 22 year old man presents to ED with 2 weeks of cough and sputum. He tells you he had a lung transplant for CF 2 years ago . He thinks his last "lung tests" were "fine" he feels more short of breath and has had fevers and sweats. A CXR shows a patchy LLL infiltrate • Which statement is most correct? A. He has a bacterial infection from his native single lung into his transplanted lung and should be treated for known bacterial pathogens in the first instance but may need a bronchoscopy B. He has the RAS variant of CLAD and needs to be referred to the transplant unit to discuss re transplant C. He has acute rejection and needs treatment for this D. He has CMV infection and needs antiviral therapy E. He has bacterial infection and should be treated for known bacterial pathogens in the first instance but may need a bronchoscopy

A

Dunedin Resp A 67 year man with chronic obstructive pulmonary disease (COPD) has frequent admissions with exacerbations. He is overweight (BMI 36) and is on treatment for hypertension and type II diabetes mellitus. He stopped smoking 3 years ago. FEV1 0.54L (18% predicted), FEV1/FVC 19%. Total lung capacity 7.96L (121% pred), residual volume 5.09L (245% pred). DLco 6.1 (19% pred). Echocardiogram shows severe right ventricular dysfunction. Arterial blood gases on room air show pH 7.39, PaO2 57 mmHg, PaCO2 45 mmHg. Which option will improve his survival? A. Pulmonary rehabilitation program B. Supplemental oxygen therapy C. Annual influenza vaccination D. Non-invasive ventilatory support E. Lung volume reduction surgery F. Inhaled corticosteroid (ICS) G. Combined long acting beta agonist plus long acting muscarinic antagonist (LABA-LAMA) inhaler H. Maintenance oral steroid

A

Dunedin Resp A 69 year old woman with established COPD is referred to your outpatient clinic with increasing breathlessness on a background of one previous hospitalisation for an infective exacerbation. She has a 20 pack-year smoking history but stopped 3 years ago. She denies any past history of asthma. FEV1 1.14L (47% predicted), FEV1/FVC 40%. CAT (COPD assessment test) score is 18. Oxygen saturation is 92% whilst breathing room air. Chest x-ray shows hyperinflated lungs only. Which option will improve her CAT score most? A. Pulmonary rehabilitation program B. Supplemental oxygen therapy C. Annual influenza vaccination D. Non-invasive ventilatory support E. Lung volume reduction surgery F. Inhaled corticosteroid (ICS) G. Combined long acting beta agonist plus long acting muscarinic antagonist (LABA-LAMA) inhaler H. Maintenance oral steroid

A

Dunedin Resp Which one of the following concerning allergic bronchopulmonary aspergillosis is false? A. A history of asthma is required to make the diagnosis. B. Serum IgE is usually elevated. C. Precipitating serum antibodies to A. fumigatus may be helpful in making the diagnosis. D. It can result in fleeting infiltrates on CXR and bronchiectasis on HRCT. E. Treatment is with oral corticosteroid and itraconazole

A

Dunedin endo An Adrenal incidentaloma A. Is defined as an adrenal lesion found incidentally that is greater than 1cm B. Will be non functioning in <70% C. If it is a functioning lesion is most likely to be aldosterone secreting D. Hounsfield units >10 represent a benign lesion E. If it is found to be benign and non functioning, then annual CT scans should be undertaken.

A

Dunedin endo Question 7: A 55 year old woman with postmenopausal osteoporosis has been treated with risedronate for the past 2 years. Her diet provides sufficient elemental calcium and she take calciferol 1.25mg per month.A follow-up DEXA scan on the same machine shows a decline in BMD in the spine (6%) and the hip (5%) compared to values 2 years ago. Which of the following is the most likely explanation? A. Non adherence to treatment B. Malabsorption C. This is the expected decline of BMD with bisphosphonate therapy D. Positioning error by the DEXA technician E. Lack of potency of this particular medication

A

Dunedin endo WITH REGARD TO SUBCLINICAL HYPERTHYROIDISM WHICH IS UN-TRUE? A. it is benign B. it is associated with sudden death in the elderly C. it is usually transient D. it is associated with reduced bone mineral density E. can be used to aid weight loss

A

Dunedin endo Which of the following statements is most correct regarding the pathogenesis of Type 1 Diabetes? A: It is a T cell mediated disorder B: B cell autoantibody production is essential C: Disease progression is halted by B cell depletion D: Presence of autoantibodies indicates insulin therapy is required E: The most common trigger for Type 1 diabetes is exposure to cows' milk

A

Dunedin gastro 39 year female, has a long history of substernal burning pain following meals. Upper GI endoscopy reveals 1-3 cm tongues of erythematous mucosa extending from OG junction at Z-line up into oesophagus. Biopsies from erythematous mucosa reveal areas of gastric cardiac-type mucosa. In patients with this condition, all of the following statements are true except?: a. Would benefit from H. pylori eradication b. Are predisposed to oesophageal adenocarcinoma cell carcinoma c. Would benefit from high dose PPI and aspirin therapy d. Have a risk of malignancy which increases with the length of the abnormal mucosa e. Have a prevalence in western world of ~2% with males > female

A

Dunedin gastro 60 yr old male with Dukes B colonic carcinoma resected 6 mths previously. Presents with RUQ pain, CT abdo shows hypodense lesion of 2cm diameter in right lobe liver. The next best investigation would be (one answer) A. FNA biopsy of the lesion B. Ultrasound of the liver C. angiography D. MRI E. ERCP

A

Dunedin gastro A 69-year-old man has a long history of dyspepsia and a previous peptic ulcer. After a recent severe flare of epigastric pain, he is referred for endoscopy which reveals an active duodenal ulcer. Antral biopsies show H pylori infection. He receives 14 days treatment with omeprazole, clarithromycin and metronidazole. The result of a 14C-urea breath test conducted 8 weeks after completion of therapy is positive. The most likely explanation for the positive H pylori test in this patient is: a. Treatment failure due to clarithromycin resistance. b. Treatment failure due to metronidazole resistance. c. Treatment failure due to the short duration of therapy. d. False positive test due to low sensitivity and specificity of the urea breath test following eradication therapy. e. False positive test due to an insufficient period between the end of the eradication therapy and the urea breath test.

A

Dunedin gastro Regarding eosinophilic oesophagitis, which of the following statements is incorrect? a. Symptomatic response accurately reflects histological improvement in >90% of cases b. Allergy testing (skin-prick or skin-patch) only predicts which food is the culprit allergen in ~50% cases c. Milk and wheat/gluten account for ~50% cases of adult eosinophilic oesophagitis d. Following failure of response to PPI's, oral budesonide has been shown to be effective e. Dilatation is a recognised management tool to treat dysphagia

A

Dunedin gastro What therapeutic move has been shown to reduce the incidence of hepatorenal syndrome in patients admitted with SBP? a) IV albumen on admission and again on day 3 b) Large volume paracentesis c) Terlipressin d) None of the above

A

Dunedin gastro Which of the following is the most effective in the prevention of NSAID induced gastric ulceration? a. Misoprostol b. Ranitidine c. Sucralfate d. Omeprazole e. Bismuth

A

Dunedin gastro You are the weekend registrar, you are phoned 10am Saturday morning regarding a 65 yr man who has presented to the ED with 24hrs of recurrent melaena following 1 week of dyspepsia. He has a history of a recent NSTEMI, with 3 x DES insertion 4 weeks ago. On DAPT - aspirin 100mg and clopidogrel 75mg daily. Other drugs cilazapril and statin. Clinically Hb 83 (was 145 during cardiology admission), pulse 75, BP 130/70. On admission aspirin was continued but clopidogrel was withheld. At endoscopy bleeding ulcer clipped. Post endoscopy, the most appropriate management would be? a. Restart clopidogrel as soon as endoscopist indicates good bleeding control obtained. b. Withhold clopidogrel until relook endoscopy in 4 weeks c. Withhold both agents until relook endoscopy in 4 weeks d. Stop aspirin permanently and restart monotherapy with clopidogrel 7 days post endoscopy e. Withhold both agents until discharge or 7 days then restart both agents

A

Dunedin Resp Which one of the following is true? A. mMRC grade 4 = too breathless to leave the house or breathless when dressing or undressing. B. mMRC grade 3 = walks slower than other people of the same age on the level. C. CAT stands for chronic airflow-obstruction trend test. D. FEV1 50-80% of predicted describes mild airflow obstruction according to GOLD criteria. E. FEV1 is part of the assessment of symptoms/risk of exacerbations in the refined GOLD ABCD assessment tool.

A 0: SOB with strenous exercise 1: SOB hurrying or up slight hill 2. walk slower than others of same age, or SOB level 3: SOB 100m/few minutes 4. too SOB to leave home. Dressing etc

DM PT Rh9 9. A 74-year-old woman presents with a severe headache and jaw claudication. She proceeds to a temporal artery biopsy. What are the most likely findings from the biopsy? A. Large vessel vasculitis with interrupted internal elastic lamina B. Medium vessel vasculitis with fibrinoid necrosis C. Mixed small and large vessel vasculitis with granuloma D. Small vessel vasculitis with predominant eosinophilic infiltrate

A A positive histologic diagnosis demonstrates inflammation of the arterial wall with fragmentation and disruption of the internal elastic lamina. Multinucleated giant cells are found in fewer than 50% of cases and are not specific for the disease.

Deltamed Onc 49 y/o. Met melanoma. Commenced on ipilimumab. Dry cough, R pleural pain at basline. after 4 doses some of lung mets larger by 20%. Sx stable incl. biochem + haem. Most appr next step in Mx? A. transient worsening before regression. Given stable then closely monitor until next scan B. transient worsening before regression. Proceed with further doses of ipi as stable sx C. worsening of disease = diease progression. refer for nivolumab D. worsening of diease = progression. Consider chemo E. disease progression. Further ipilimumab in combo with chemo

A B - has completed course of ipi

Deltamed renal 67 y.o. diabetic has significant renal impairment with the following results: Creatinine 180ȝmol/L (eGFR 34ml/min) Hb 105 gm/L (115 - 135 gm/L) Proteinuria 0.38 gm/day (<0.12 gm/day) Blood pressure 150/90 mmHg and HbA1c 7.5% Which of these is likely to have the greatest impact on slowing the progression of CKD? A. Blood pressure reduction. B. Tight glycaemic control. C. Erythropoietin therapy. D. Control of serum phosphate. E. Low protein diet.

A BP > tight BSLs

Dun ID A 67 year old tourist suffers a haemorrhagic stroke while on holiday in India. He is evacuated back to your hospital's ICU. He has a central line, catheter and tracheostomy in situ. He remains febrile and and a carbapenem-resistant E. coli is identified in blood cultures (producing NDM-1). This treatment regimen is most likely to include which of the following antibiotics? a. Colistin b. Tigecycline c. Ertapenem d. Linezolid e. Vancomycin

A Colistin in combination with 1 or 2 other potentially active agents e. g colistin + high-dose meropenem + high-dose tigecycyline Despite being "resistant" to meropenem, there may be some benefit if MIC <8 Don't forget to replace/remove central line and IDUC! Ceftazidime-avibactam monotherapy? Mainly useful for KPC producing organisms, not NDM. Possibly 1) Ceftazidime-avibactam + aztreonam 2) aztreonam-avibactam in future?

Book R11. 23 y/o F. Recurrent episodes of fever + abdo pain. 1 q2 months. Last 1-2 days. Associated diarrhoeas. Raised WCCs and CRP. Between episodes = well. Sometimes associated with menses, were less freq when taking OCP. Normal colo + CT 3months ago. Which of the following is the most appropriate rx for this pt? A. colchicine B. etanercept C. high dose estrogen OCP D. IFN-alpha E. prednisolone

A Familial mediterranean fever. AR. Increased serum amyloid Colchicine prevents acute attacks and improves prognosis If myalgia/arthritis = add NSAIDs + roids

Dun Pharm Warfarin is predominantly cleared by hepatic metabolism, with a low hepatic extraction ratio and therefore a low hepatic first pass effect and high bioavailability after oral administration. Which one of the following best describes the most likely effect on warfarin pharmacokinetics of concomitant administration of an inhibitor of warfarin metabolism, such as erythromycin? Key to symbols: ↔ = little or no change ↑ = significant increase ↓ = significant decrease Effect on Bioavailability Effect on Clearance A. ↔ ↓ B. ↔ ↔ C. ↑ ↓ D. ↑ ↔ E. ↓ ↔

A High bioav so won't have much effect

R 29 Which 1 of the following is a manifestation of SLE? A. haemolytic anaemia B. leucocytosis C. lymphocytosis D. eosinophilia E. thrombocytosis

A Normochromic normocytic anaemia Warm type AIHA Lymphopenia, mild thrombocytopenia.

Haemodyanamics CLS lecture 32 year old male with a dilated cardiomyopathy is referred for hemodynamic assessment to determine if he is a candidate for cardiac transplantation. Findings: Pulmonary artery pressure 70/30 (mean 45) mmHg Mean pulmonary capillary wedge pressure 30 mmHg Cardiac output is 5.0 L/min. Which of the following is the next step in management? A. Based on the PVR, he can be listed for cardiac transplantation. B. He should undergo further hemodynamic evaluations during the infusion of nitroprusside. C. Based on the pulmonary vascular resistance, he is not a candidate for cardiac transplantation. D. He should be considered for combination heart-lung transplantation. E. More information is required to determine the pulmonary vascular resistance.

A PVR = (mPA - mLA) / Qp PVR = (45-30) / 5 = 3 WU Elevated pulmonary pressures relate to high left atrial pressure (reflects LV systolic dysfunction) rather than to pulmonary vascular disease

DM PT Gen6 6. Angelina is BRCA1 mutation positive and Brad is BRCA2 mutation positive. What is the risk to each of their children of not inheriting a BRCA mutation? A. 25% B. 50% C. 75% D. 100%

A Punnet square. See handout

CLS Resp A 60yo man has features of restless leg syndrome. What is he most likely to be deficient in? A. Iron B. Magnesium C. B12 D. Folate E. Potassium

A Reduced intracellular iron (particularly CNS iron) appears strongly associated Multiple studies have demonstrated an association between RLS and low serum ferritin - iron replacement is recommended if Ferritin is less than 75mcg/L Family history of RLS is common (40-60%) but no genes have been identified Medications can provoke or precipitate - antihistamines, dopamine antagonists, antidepressants DDx - uraemia, neuropathy (diabetes), spinal cord injury, pregnancy, MS, Parkinson disease

Deltamed onc 75 y/o man. Met prostate ca Back pain, uro incontinence, altered sens in legs. Reduced senation perianally and in legs. Reduced reflexes, dowgoing plantars A. cauda equina b. cerebral mets c. SCC D. paraneoplastic PN

A SCC = hyperreflexic, upgoin

Dun ID A 71 year-old man is evaluated for fever, intense generalised headache and myalgias, which started 3 days after he returned from Laos, where he was teaching agriculture in rural areas for several months. On physical examination, temperature is 38.6°C. Other vitals are normal. He is noted to have an eschar with a black crust on the right thigh. Laboratory studies show thrombocytopaenia. Which of the following is the most likely diagnosis? A. Scrub typhus B. Cutaneous anthrax C. Lyme disease D. Cellulitis

A Scrub typhus: a mite-borne infectious disease most commonly occurs in rural areas of Southeast or East Asia and northern regions in Australia. • Orientia tsutsugamushi: A gram(-) coccobacillus transmitted by chigger mites. • Symptoms usually begin within 10 days of being bitten: Fever, headaches, diffuse myalgias, metal status changes, lymphadenopathy, rash, pneumonia, MOF in severe, untreated cases. • Nonpruritic macular or maculopapular rash (one-half), characteristic eschar • Treatment: Doxycycline

Dunedin Resp You are on call and admit a 48 year old man with rapidly progressive SOB. He tells you his private specialist diagnosed an interstitial lung disease 3 weeks ago. He is still smoking 5 cigarettes a day but tells you he will give up if you can refer him for a lung transplant. Even on high flow oxygen he is hypoxic. You do basic blood tests and he is CMV and EBV positive • Which is most correct? A. He has ILD and would be suitable for a SLT B. He is too young to have coronary artery disease and won't need an angiogram C. Because he is young he will have a good outcome from lung transplant D. Because he is CMV positive he is not at risk of CMV disease after transplant E. Because he is young he doesn't need and echo

A Smoker 0 needs Angio ECHO everyone

Deltamed Onc What features on immunohistochemistry are most likely to raise suspicion for a small cell carcinoma? A. Chromogranin positive, synaptophysin positive B. TTF1 positive C. CK7 positive, CK20 negative D. p40 positive

A TTF = adeno CK7 = endothelial p40 = Sq

Dun ID A 23 year-old female presents with sepsis secondary to pyelonephritis. She is started on IV cefuroxime. The next day she remains febrile, and the lab reports a pure culture of Proteus vulgaris from her urine with susceptibility pending. The best treatment option is: a. Meropenem b. Sulfamethoxazole-trimethoprim c. Piperacillin-tazobactam d. Amoxicillin-clavulanic acid (Augmentin) e. Ceftriaxone

A This organism may appear sensitive to cephalosporins on initial testing. However it belongs to a group of organisms ("ESCHAPPM" ) that produce an AmpC beta-lactamase. • If septic - treatment with meropenem • Options may include non-beta lactam drugs e.g. gentamicin, ciprofloxacin, co-trimoxazole

Book D2 A pt enquires about sunprotection. Which of the following is correct? A. reg use of sunscreen reduces development of pre-cancerous actinic keratosis B. sunburn is caused by UVA C. SPF is based on how effectively it blocks UVA rays D. sunscreens only block UV A radiation E. UV B rays can penetrate below the dermis

A UVA = ageing/IS. Can penetrate below dermis UVB = burns. mostly absorbed by epidermis SPF based on UVB SPF 15 = 150mins to burn comapred to 10mins without that sunscreen Reg sunscreen = prevent actinic keratoses, skin cancers but not BCCs

D6 Which 1 of the following is most impt in determing the prognosis for a pt dx with malignant melanoma? A. depth of invasion B. hx of pigmented naevus C. absence of pigmentation D. number of mitotic figures in microscopic specimen E. ulceration

A Ulceration = worse prognosis than nonulcerated Determine prognosis for advanced stage melanoma: site of mets, number of met sites + serum LDH

DM PT Gen7 7. A couple undergo preconception carrier screening for cystic fibrosis (CF). They have no family history of CF. The carrier screening tests for the 80 most common CFcausing mutations, which detects approximately 90% of all CF-causing mutations. One of the parents tests positive for the deltaF508 mutation and the other parent tests negative. What is the couple's residual risk of having a child with CF? A. 1/1000 B. 1/500 C. 1/250 D. No risk

A Work out the carrier risk for each parent: • One parent is a carrier (deltaF508 mutation). • Other parent negative screening test. 10% chance of still being a carrier, so residual risk is 1/25 (general population risk) x 1/10 = 1/250 • Each parent has a ½ chance of passing on the mutant allele if they are carriers (so ¼ combined) • 1 x 1/250 x ¼ = 1/1000 (Answer A)

Dunedin gastro A 38 year man has had upper abdominal pain for several months. For the past week he has had nausea. A stool sample is tested for occult blood and is positive. An upper GI endoscopy reveals no oesophageal lesions, but there is a solitary 1 cm diameter shallow, sharply demarcated ulceration of the stomach. Which of these statements regarding this lesion is the most appropriate: a. It is probably located on the lesser curvature of the stomach b. It is probably malignant c. It is probably associated with increased gastric acid production d. Because of it's small size, a biopsy is not necessary e. A gastrinoma of the pancreas is probably present

A benign only single so not gastrinoma

Dunedin gastro 32 year man. 1 year history progressive dysphagia. Originally solids but now liquids as well. Weight loss. Endoscopy findings demonstrated. Which of the following tests is likely to confirm the diagnosis? a. Barium swallow b. CT chest c. Connective tissue screen d. Oesophageal pH study e. Oesophageal biopsy

A cats bottom on endoscopy

Dunedin Resp An active 66 year old man, ex-smoker with 30 packyear history, is referred following an episode of haemoptysis. Chest x-ray shows an 18 mm opacity in the right upper zone. A chest x-ray 2 years earlier showed hyperinflated but clear lungs. Spirometry shows FEV1 2.10L (63% predicted). CT chest shows an 18 mm diameter spiculated nodule in the right upper lobe posteriorly. PET scan shows an FDG-avid nodule in the right upper lobe but elsewhere there was no significant FDG uptake. The next most appropriate step is: A. Thoracic surgery for right upper lobectomy. B. CT-guided FNA of the nodule. C. Sputum cytology. D. Fibreoptic bronchoscopy for cytology.

A cruddy lungs, central lesion. difficult to get to if low FEV1 meaning not operative candidate = SABR

Dun cardio A 70-year-old male with type 2 (non-insulin-dependent) diabetes mellitus and hypertension experiences severe retrosternal pain while playing soccer. He arrives at the hospital emergency department thirty minutes later. His blood pressure is 105/70 mmHg. His ECG shows extensive anterior ST elevation. He is given aspirin and tenecteplase followed by a heparin infusion, with rapid resolution of the ST elevation. His pain settles rapidly with morphine and metoclopramide. Soon after the heparin infusion is commenced, he begins vomiting. His heart rate is 50/minute and blood pressure 185/100 mmHg. He is drowsy, but has no focal neurological signs. The most likely cause of his deterioration is: A. intracranial haemorrhage. B. painless myocardial ischaemia. C. allergy to tenecteplase. D. hypertensive encephalopathy.

A cushings reflex. brady, hypertensive

Dunedin Resp An 82 year old man is being assessed for compensation from asbestos exposure. He had previously worked as a boilermaker. He admits to breathlessness when climbing slopes but has no other symptoms. He has never smoked. Chest x-ray is shown (pl plaques, nasty). Lung function tests show obstr, restriction. Low DLCO, high KCO: The most likely diagnosis is: A. Asbestos pleural plaques. B. Asbestos-induced pulmonary fibrosis. C. Emphysema. D. Mesothelioma.

A if fibrotic = low KCO too

Dunedin pharm Oral drugs A + B have the following properties A: 100% abs, 2% bioav, VD 350L, clearance 2L, hepatic clearance 10L, protein binding 75% B: 100%, 90%, 50L, 1 litre, 2 litres, 25% Comapred to healthy individuals pts with cirrhosis have a much higher conc of drug A than B. Why? A. bioavail B. Vd C. renal clearance D. hepatic clearance E. protein binding

A liver failure = decreased FPM = higher bioavailability

Dunedin gastro 78 yr lady. History HPT, NSTEMI 2012 (no stent), CVA 2006, subsequent recurrent TIAs. Admission medications: aspirin, ibuprofen, metoprolol, cilazapril, citalopram. Presents with several days abdominal pain, and now 24hrs hematemesis and melaena. Haemodynamically stable, Hb 75, urea 24.3 Cr 100 (e-GFR 67) PR melaena. Which statement is most accurate about endoscopic finding in peptic ulcer disease? a. Most gastric ulcers occur at junction of fundus and antrum along the lesser curvature b. Most duodenal ulcers are found in 2nd or 3rd part duodenum c. Malignant ulcers have a smooth regular rounded edge with flat smooth base and the surrounding mucosa shows radiating folds d. Most common finding in individuals undergoing endoscopy for dyspepsia is peptic ulcer

A occur in 1st part of duodenum. Most common endoscopy finding for dyspepsia = nothing

DM PT R10 10. A 74-year-old woman on peritoneal dialysis presents with cloudy effluent. Culture of the fluid shows S. epidermidis, and she is commenced on appropriate antibiotic therapy. She has had 2 similar episodes within the past 12 months, with different coagulase-negative staphylococcus infections. She is very keen to continue peritoneal dialysis and does not have haemodialysis access. Which of the following is the least appropriate course of action? A. Peritoneal dialysis catheter exchange, with ongoing peritoneal dialysis B. Peritoneal dialysis catheter removal, and placement of temporary access for haemodialysis C. Referral for re-education of septic procedures D. Referral for elective formation of an arteriovenous fistula

A Coagulase negative Staph normally derived form skin • Target rate peritonitis < 0.33 episodes per year • Recurrent Coagulase negative Staph peritonitis suspicious of technique failure Most common org: culture neg > CONS

Dun Neuro A 38-year-old man, born in Thailand and living in Australia for the past two years, presents to the emergency department with his first generalized seizure. A head CT scan is performed as shown below and shows multiple calcified lesions. Management should focus on which of the following? A. Seizure prevention B. Parasite eradication C. Transmission reduction D. Surgical removal E. Bowel decontamination

A Neurocysticercosis is caused by pork tapeworm larvae. Rx seizures If viable/degenerating = antiparasitic rx, but not if untreated hydrocephalus, encephalitis or calcified lesions only. Albendazole, praziquantel. Adjunct roids

Deltamed Gastro lecture 1 The manometric feature most characteristic of achalasia is A. lack of peristalsis in oesophageal body B. high resting tone of LES C. failure of LES relaxation with swallowing D. poor propagation of esophageal contraction waves E. diffuse spasm in oesophageal body

C

Dunedin Endo Which of the following people have the greatest risk of developing Type 1 diabetes? A: Personal history of Coeliac disease B: Sibling of a patient with Type 1 diabetes positive for multiple islet associated antibodies C: Father with Type 1 diabetes D: Dizygotic twin with Type 1 diabetes E: HLA DR3/DR4 haplotype

B

Deltamed endocrine Which CT imaging characteristic is most suggsetive of a benign lesion? A. Size 3cm B. Hounsfield units of -10 C. 10min washout 30% D. heterogenous appearance

B <10 units = 100% benign <4cm, smooth, homogenous, >50% washout

CLS ID Control of MTB replication is dependent on a number of critical factors including TNF alpha production and IFN gamma response. What other immune effector plays maj role in controlling replication of MTB? A. B Cell B. complement C. NK cell D. neutrophil E. T cell

E

Deltamed Onc 40 year old male with metastatic melanoma on PDͲL1 inhibitor Pembroluzimab presents with new onset fatigue, generalized weakness, diarrhoea, anorexia & weight loss • Supine BP 135/80 Standing BP 70/50 • Na 124 K 5.9 Urea 15.0 Cr 89 Glucose 3.6 • What is the most likely cause for his acute presentation • A. Adrenalitis • B Adrenal metastasis • C. SIADH • D. Sellar metastasis

A. adrenalitis

Dunedin endo Question 2: A 65-year-old man presents with sudden onset of severe headache. He appears generally unwell and has a blood pressure of 90/60 mmHg. There are no focal neurological signs and visual field testing is normal. An non contrast T1 weighted MRI scan is performed. Which of the following is the most appropriate first step in management? A. Glucocorticoid administration B. Emergency transphenoidal surgery C. Pituitary radiotherapy D. Dopamine agonist therapy E. Somatostatin analogue therapy

A. apoplexy. ideally send cortisol 1st

Deltamed immuno With respect to the major histocompatibility complex (MHC) in man, which of the following is false? A the relevant genetic loci are on chromosome 15 B all molecules coded by MHC genes are expressed on the great majority of blood cells C the Class I antigens are very polymorphic D the human complement components C2 and C4 are coded by MHC genes E Class II MHC antigens are coded by at least 3 genetic loci

A. chrome 6

Deltamed renal 67 y.o. diabetic on PD presents with abdominal pain, low grade fever and cloudy dialysate. She has mild generalised abdominal tenderness and guarding but does not appear particularly unwell. She is started on intra-peritoneal cephalexin. The next day, her dialysate culture grows: Enterococci, E-coli and Klebsiella species. The most appropriate management step is: A. exploratory laparotomy. B. add intraperitoneal Ampicillin. C. removal of the Tenckhoff catheter. D. intraperitoneal gentamicin. E. change to intravenous antibiotics.

A. exploratory laparotomy Fungal = remove catheter

Deltamed renal 65-y.o. on HD for 9/12, is on EPO 4,000u 2x/week. Previously Hb had been stable (105 - 115 gm/L). She now presents with a Hb of 89 gm/L. There have been no changes to her EPO dose or other medications. MCV 84 fL [78-98] Serum iron 13 ȝmol/l [13-35] Iron saturation 18% [15-46] Serum ferritin 630 ȝg/l [20-300] Retic count 30 x 109/L [12-100] Which is the most likely cause for her EPO resistance? A. Inflammation. B. Pure red cell aplasia. C. Vitamin B12 deficiency. D. Iron deficiency. E. Hyperparathyroidism.

A. inflammation If pure red cell would have 0 retics

CLS Rheum Where do patients with osteoarthritis of the hip most commonly report pain? A. Groin B. Medial thigh C. Lateral thigh D. Knee E. Buttock

A groin

Deltamed Immuno What is a predominant function of the complement protein C3b? A. Regulation of the classical complement pathway B. Regulation of the alternative complement pathway C. Initiates the membrane attack complex D. Causes mast cell degranulation E. Opsonisation of pathogens

E

Deltamed Immuno Which one of the following biologically active components of the complement cascade is most directly responsible for the formation of transmembrane channels leading to cell lysis? A. C3a B. C3b C. C4a D. C5a E. C5‐9

E

Deltamed rheum ɵ All of the following are appropriate treatments for axial symptoms of ankylosing spondylitis except: a) Methotrexate b) Physiotherapy c) Adalimumab d) Naproxen e) Secukinumab

A. No role for DMARDs in axial

Deltamed PT4 4. A 40-year-old lady presents to Emergency with increasing dyspnoea. She has a known background of idiopathic pulmonary hypertension, on dual therapy with ambrisentan and tadalafil. Her heart rate is 86bpm, SpO2 96% on room air, BP 115/70mmHg, RR 14/min and she is afebrile. A bedside echocardiogram demonstrates an enlarged right ventricle with severe pulmonary hypertension (right ventricular systolic pressure 74mmHg), and a large pericardial effusion measuring up to 3cm maximally. What is your next step? A. Continue current medications and admit to hospital B. Pericardiocentesis C. Phone cardiac surgeons to request pericardial window D. Thrombolysis

A. Pericardial effusion in severe pulmonary hypertension is often an end-stage and ominous sign. It usually reflects a compensatory attempt by the heart to help maintain forward right heart pressure in the setting of severe pulmonary hypertension. The large size of this effusion supports that it has developed chronically. From the vignette, the patient is dyspnoeic (unsurprising) but haemodynamically very stable. Attempts to drain the effusion that may be maintaining right heart haemodynamics could cause her to arrest on the end of the needle. She should be admitted for symptomatic investigation and optimization of her dyspnoea

Deltamed neuro A 50 year old lady presents with a 10 week history of gradually progressive symmetrical weakness in both proximal and distal muscles. Examination further reveals impaired distal sensation, absent or diminished tendon reflexes. Investigations show elevated cerebrospinal fluid protein level without pleocytosis, and demyelinating nerve-conduction studies. Associated findings are most likely to include: A. Steroid responsiveness B. IgM paraproteinemia C. Anti ganglioside antibodies. D. Elevated HbA1c E. Optic neuritis

A. CIDP

DM PT G5 5. Which of the following is a complication of portal hypertension? A. Increased cardiac output B. Decreased cardiac output C. Splanchnic arteriolar vasoconstriction D. Decreased portal collateral blood flow

A. PH is the formation of portosystemic collaterals. Concomitant or even preceding development of collaterals, splanchnic vasodilatation occurs, leading to increased flow into the gut and into the portal venous system. Vasodilation leads to activation of neurohumoral and vasoconstrictive systems, sodium and water retention, increased blood volume, and increased cardiac output; that is, a hyperdynamic circulatory state that further increases portal venous inflow and PP. Additionally, activated vasoconstrictive systems to further contribute to intrahepatic vasoconstriction. Treatment of etiology, by ameliorating fibrosis/inflammation, target the mechanical component of the increased intrahepatic resistance. Vasodilators (like the a-adrenergic blocking effect of carvedilol) target its functional component (this is the site of action of statins). NSBBs (b2-adrenergic blocking effect), SMT, and VP act by causing splanchnic vasoconstriction, thereby reducing portal venous inflow. NSBBs also act by decreasing cardiac output (b1-adrenergic blocking effect). The TIPS connects the hypertensive portal vein with a normotensive hepatic vein, thereby b

Deltamed obs A 32 year old primigravida presents at 34 weeks with Hb 9.0 g/L, platelets 52 with fragments on blood film and LDH 720. LFTS AST 154, ALT 180, bilirubin 35, INR 1.3.Creatinine 85 mmol/L and proteinuria spot pr/creat ration 100 mg/mmol. BP readings 160/80. The most likely diagnosis is a. HELLP b. Cholestasis of pregnancy c. Pregnancy triggered atypical HUS d. Acute fatty liver of pregnancy e. Acute hepatitis

A. Most likely diagnosis, pͲaHUS a possibility but less common suspect if not improving

Deltamed stats A new D‐dimer assay has a sensitivity for deep vein thrombosis (DVT) of 95% and a specificity of 50%. It is proposed to use it to screen a group of passengers after long distance travel. Previous data have suggested this group has a pre‐test probability of DVT of 1%. What is the best estimate of the post‐test probability of DVT in an individual with a positive D‐dimer result? A. 2% B. 5% C. 10% D. 50% E. 95%

A. 2% since rare, pre-test prob = pre test odds = 1% LR = sens / 1 - spec = 0.95 / 0.5 (approx 1 / 0.5) = 2 Post test odds = LR x pre test odds = 2 x 1% Post-test odds = post-test prob as rare

Book D1 Which 1 of the following is freq altered in melanoma resulting in dysreg of MAPK pathway? A. B-raf B. VEGF C. Kras D. EGFR E. p53

A. Braf V600E mutation Missense mutation. VEGF - RCC, CRC Kras - EGFR - CRC p53 - tumour suppressor

Deltamed renal Dialysis patients have a high mortality (average survival 40-50% at five years). For a new patient commencing dialysis, the most likely cause of death will be: A. infection. B. malignancy. C. electrolyte abnormality. D. cardiovascular event. E. withdrawal from dialysis.

A. D.

Deltamed Neuro Which feature on nerve conduction studies is most sensitive in detection of early diabetic polyneuropathy? A. F wave latency B. Motor amplitude C. Motor conduction velocity D. Sensory amplitude E. Sensory conduction velocity

A. F wavws in lower limbs Early small fibre neuropathy = aggressive BSL control not useful. Later painless = hyperglycaemia more impt

Dunedin gastro A 45 year old man with Crohn's disease presents with fever, rigors and right lower quadrant pain. His medications include prednisone - 10mg per day and sulphsalazine. On examination, he is febrile with temp of 37.6 degrees and has exquisite tenderness in the right lower quadrant but no generalised peritonitis. Investigations: CT abdo reveals a right sided mass lesion 5-6cm in diameter, containing gas and fluid. What is the best initial management for this man? A. IV antibiotics B. IV hydrocortisone C. Laparotomy D. IV hydrocortisone alone E. Drainage

A. initial

DM Resp Which of the following treatmentshas been demonstrated in RCT's to assist with steroid weaning in ABPA? A. Itraconazole B. Lebrikizumab C. Mepolizumab D. Pirfenidone E. Rituximab

A. itraconazole

Deltamed renal Cardiovascular disease is the major cause of death in dialysis patients. Elevation of which of the following clinical parameters carries the highest mortality risk? A. Serum phosphate. B. Serum cholesterol. C. Serum homocysteine. D. Serum parathyroid hormone. E. Blood pressure.

A. serum phosphate Reverse epi in dialysis. Low chol, BP = higher risk. Flat r,ship with PTH

Deltamed Resp A 65yo woman with known atopic asthma presents with cough, intermittent sputum production and fevers. CT chest demonstrates bronchiectasis and CT sinuses demonstrates mucosal congestion.Bloods demonstrate a raised IgE (>1000), moderate eosinophilia.What is the most likely diagnosis? • Eosinophilic granulomatosis with polyangiits • Granulomatosis with polyangiitis • HyperͲIgE syndrome (Job's Syndrome) • Allergic bronchopulmonary aspergillosis • Polyarteritis nodosa

ABPA Bronchiectasis, raised IgE

Dun Renal 76 yr male with a 2 week history "flu‐ like' illness, low grade fever, myalgias and arthralgias. No past history, no history of rash, haemoptysis, sinusitis or nasal discharge. Examination: 38.1oC, BP 158/72, JVP 4 cm, mild oedema. Lab data ‐ creatinine 0.43 mmol/l, urea 32 mmol/l, urine 2+ protein, 4+ haematuria, elevated LFTs, GGT 224, ALP 205 and ESR 115, CRP 96. 1. ANCA titres will be negative in > 55% of cases. 2. A linear pattern of IgA staining on IF is expected. 3. Prednisone alone is the preferred therapy. 4. Anti‐GBM crescentic GN can be associated with ANCA titres in up to 30% of cases. 5. Cholesterol emboli may present with this form of renal lesion.

ANCA neg in >80% AntiGM cross react with ANCA Histology shows a focal necrotising GN with crescents in over 50% of glomeruli. Crescent formation initiated by rupture of glomerular capillaries by inflammatory processes. Inflammatory cytokines, chemotatic and growth factors stimulate macrophage influx & activation with epithelial proliferation. Cellular crescents evolve into fibrocellular & fibrotic crescents as cells are replaced by collagen 3 major categories for crescentic GN. 1.Anti ‐GMB GN 2.Immune complex GN 3.Pauci ‐immune GN (ANCA)

Deltamed PT3 3. Which of the following has a strong association with coarctation of the aorta? A. Intracranial aneurysm B. Renal artery stenosis C. Raynaud's phenomenon D. Peripheral vascular disease

ANS: A Intracranial aneurysms are consistently associated with coarctation of the aorta. In MRbased screening studies, the prevalence of intracranial aneurysms has been documented as up to 10%. They are thought to be more prevalent when the coarct is detected late and when the patient has established hypertension

DM PT ID7. A 32-year-old lung transplant recipient develops worsening dyspnoea. She received a bilateral lung transplant for cystic fibrosis 2 years prior and has a past history of Pseudomonas and Aspergillus infection. Bronchoalveolar lavage (BAL) is performed and Gram negative rods, but no fungal elements, are visualized on the Gram stain. Aspergillus galactomannan on the BAL specimen is positive. What is the most appropriate initial therapy? A. Piperacillin-tazobactam and voriconazole B. Cefepime and liposomal amphotericin B C. Piperacillin-tazobactam D. Cefepime

ANS: A • Piperacillin-tazobactam or cefepime would both be appropriate empiric choices for treating the Gram negatives visualized on Gram stain as they both have antipseudomonal activity. • The stem is not categorical, but the past history of Aspergillus infection and positive Aspergillus galactomannan are suggestive of possible pulmonary aspergillosis, even if fungal elements were not visualised. • Aspergillus galactomannan is a biomarker that forms part of the mycological definition of invasive aspergillosis (see DePauw et al. Clin Infect Dis. 2008 Jun 15;46(12):1813-21) and can be performed on plasma, serum, BAL fluid or CSF. • Both voriconazole and amphotericin B have activity against Aspergillus spp. but voriconazole is typically used as front-line therapy due to superior results when it was compared with amphotericin B for primary therapy of invasive aspergillosis (Herbrect et al. New Eng J Med 2002; 347:408-415). • Amphotericin B would be used if the patient had been on azole prophylaxis and had breakthrough infection, or if coverage for additional organisms such as Mucor spp. was necessary.

Deltamed PT8 8. In which group of people is spontaneous coronary artery dissection most likely to occur? A. People with peripheral vascular disease B. Pregnant women C. People with hypertension D. People using recreational stimulant drugs (i.e. methamphetamines)

ANS: B Pregnancy-associated spontaneous coronary artery dissection (also known as P-SCAD) is a well-documented subtype of SCAD, defined as SCAD occurring in pregnancy or within 3 months post-partum. The increased rate is hypothesised to relate to the hormonal changes of pregnancy with generalised joint and endothelial relaxation. P-SCAD is almost universally the cause of a myocardial infarction in a pregnant woman (although atherosclerosis is starting to appear in this group with the rising prevalence of maternal hypertension, diabetes and obesity, and increased maternal age). Outcomes are improving but it remains a life-threatening condition with potential for dual mortality

Deltamed PT2 2. You are doing a medical admissions job in Emergency, and a 60-year-old Asian lady has presented with shortness of breath. Vital signs are stable, and chest X-ray) is unremarkable. An ECG is performed: What is the diagnosis? A. Pulmonary embolus B. Apical hypertrophic cardiomyopathy C. Arrhythmogenic right ventricular cardiomyopathy D. STEMI

ANS: B This ECG shows the typical 'giant negative praecordial T waves' classic for apical HCM. This is a variant form of HCM, more common in East Asian populations, and sometimes also referred to as Yamagutchi cardiomyopathy. As the hypertrophy is localized to the apex, there is no outflow tract obstruction, however a mid-cavity obstruction or gradient may develop. Patients may develop significant angina and dyspnoea in the setting of impaired diastolic function.

DM PT Gen2 2. What term best describes when multiple, often seemingly unrelated, physical effects are caused by a mutation in a single gene? A. Heteroplasmy B. Pleiotropy C. Allelic heterogeneity D. Locus heterogeneity

ANS: B • Pleiotropy: multiple phenotypic expressions or disorders caused by a single gene. E.g. Marfan syndrome causing aortic root weakening, skeletal stretching and lens dislocation. Heteroplasmy: the presence of a mixture of more than one type of mitochondrial DNA within a cell or individual. It is a factor for the severity of mitochondrial diseases

DM PT ID1 1. Some antimicrobials have oral bioavailability which is nearly equivalent to their parenteral availability. Which of the following antimicrobials has poor oral bioavailability? A. Linezolid B. Amoxicillin C. Ciprofloxacin D. Bactrim

ANS: B • There are several classes of antimicrobials that have high oral bioavailability. • Given this and their lower cost, patients can be easily switched to these as long as there are no other GI issues affecting absorption. • In addition to the agents listed above, azoles (e.g. fluconazole), lincosamides (e.g. clindamycin), macrolides (e.g. azithromycin), metronidazole and doxycycline also fit into this category. • Unfortunately, this is not the case for beta-lactam agents so some caution must be exercised, especially when treating high burden infections.

DM PT E3 3. A 70-year-old woman discharged after 4 days of ICU to the ward for ongoing treatment of severe pneumonia. No iodine contrast was given. She has new onset of atrial fibrillation and has been commenced on beta-blockade and anticoagulation. On hospital day 7, TSH is 0.15 mIU/L [reference range 0.27-4.20], free T4 13.5 pmol/L [reference range 12.0-22.0], free T3 3.5 pmol/L [reference range 3.1-6.8]. On examination, the thyroid is not enlarged, and there are no palpable thyroid nodules. Heart rate is 70 and irregular. What is the next best step? A. Perform thyroid uptake scan. B. Request thyroid antibodies. C. Repeat TFTs in 2 weeks. D. Start carbimazole.

ANS: C Non-thyroidal illness syndrome TSH normal to low (0.04-0.3 mIU/L), but not <0.01 mIU/L, TSH increases with recovery (but remains <20 mIU/L) and TSH increase precedes fT4/3 recovery fT3 low/normal T4 normal to low Reverse T3 increased Drug effects: Glucocorticoids dopamine, dobutamine decrease TSH; Glucocorticoids, betablockers decrease fT4 to fT3 conversion Hormone concentrations inversely correlated with severity

Deltmed 5/10 10. Which of the following drugs has the least efficacy for the axial symptoms of Ankylosing spondylitis? A. Adalimumab B. Celecoxib C. Methotrexate D. Naprosyn

ANS: C • NSAIDs - first line therapy in AS • Disease retarding! • MTX can be helpful in peripheral joint arthritis but has no role in management of axial disease • Adalimumab (Humira) market leading anti-TNF agent!

DM PT G6 6. A 60-year-old man has been diagnosed with Diffuse Large B Cell Lymphoma and is about to start treatment. His screening reveals the following hepatitis B serology: core Ab negative surface Ag negative surface Ab positive What is the next most appropriate step in management? A. Commence entecavir B. Monitor liver function and hepatitis B viral load 3 monthly C. Order a FibroScan D. No further management required

ANS: D HBV and immunosuppressive therapy or chemotherapy • All patients undergoing chemotherapy or significant immunosuppressive therapy should be tested for HBV (sAg, cAb, sAb) prior to immunosuppression Antiviral therapy recommended for: All HBsAg-positive patients HBsAg-negative/anti-HBc-positive patients at high risk of reactivation (antiCD20 monoclonal Ab or conditioning for stem cell transplantation)

DM PT G8 8. A 60-year-old man presented to ED with a 5-month history of increasing shortness of breath on exertion. He has no cough, chest pain, sputum production or wheeze. He smokes 20 cigarettes per day and has consumed 1-2 bottles of wine per day for many years. On examination he has central cyanosis, finger clubbing, scleral icterus and spider naevi. His oxygen saturation (on room air) was 91% supine and 85% standing. His HR is 75 beats per minute and blood pressure is 150/90. His JVP is not elevated. Abdominal examination revealed moderate ascites and there was mild ankle oedema. What is the most likely cause for his dyspnoea? A. Right heart failure B. Hepatic hydrothorax C. Portopulmonary hypertension D. Hepatopulmonary syndrome

ANS: D Hepatopulmonary syndrome Disorder in pulmonary oxygenation, caused by intrapulmonary vasodilatation leading to hypoxaemia via ventilation-perfusion mismatch and oxygen diffusion limitation Less commonly, by pleural and pulmonary arteriovenous communications Commonly in presence of portal hypertension, and in particular: Hepatic portal hypertension with underlying cirrhosis • Pre-hepatic or hepatic portal hypertension in patients without underlying cirrhosis Less commonly in presence of: • Acute liver failure, chronic hepatitis • Can coexist with other pulmonary disease eg. PPH or COPD • Should exclude other diagnoses • Clinical manifestations of HPS in patients with chronic liver disease primarily involve dyspnoea, platypnoea and orthodeoxia • Also features of CLD, especially spider naevi

Deltamed PT5 5. Jack is an 86-year-old patient admitted under your team with a urinary tract infection. In auscultating his heart, you appreciate a harsh ejection systolic murmur, which is confirmed on the echocardiogram you order to be severe aortic stenosis. You return to Jack to take a more detailed history regarding cardiac symptoms. What would be the most concerning symptom Jack might report? A. Palpitations B. Syncope C. Angina D. Fluid on the ankles and difficulty breathing

ANS: D The natural history of aortic stenosis is one of the classic survival graphs in cardiology. Essentially, each symptom portends a different prognosis. From development of angina, survival is typically around 5 years, from development of syncope it is 3 years, and once a patient is having episodes of heart failure it is only 2 years. Unspecified palpitations do not carry a defined prognosis (obviously documented ventricular arrythmias would be concerning).

Deltamed PT7 7. Margaret, 72, is admitted to hospital with shortness of breath and a heart rate of 110bpm and blood pressure of 100/60mHg. She has a history of hypertension and systemic lupus erythematosus. Chest X-ray in Emergency showed an increased cardiothoracic ratio, and you have requested an echocardiogram. Echocardiography shows a large pericardial effusion measuring up to 30mm, with echogenic material within the fluid suggestive of fibrin strands. Her left atrium is severely dilated and there is right ventricular diastolic collapse. Which feature of the echocardiogram report would worry you that Margaret has tamponade physiology? A. Size of effusion B. Presence of fibrin strands C. Severe left atrial dilation D. Right ventricular diastolic collapse

ANS: D Ultimately tamponade is a clinical diagnosis, and Margaret's tachycardia and borderline blood pressure are concerning. However, classic echocardiographic features suggesting tamponade physiology in pericardial effusion are: diastolic collapse of the right atrium or right ventricle (right atrium more sensitive sign, right ventricle more specific) vrespiratory variation in flow across the mitral and tricuspid valves vand a dilated and noncollapsing inferior vena cava. The large size of this effusion and the presence of fibrin strands are suggestive that this is likely to be a subacute / chronic process. The left atrial dilation is likely age and hypertension related.

DM PT ID9 9. Certain patient populations have a higher burden of colonisation with multi-drug resistant Gram-negative organisms. Which of the following is NOT a risk factor for colonisation with these organisms? A. Residence in a high-level care facility B. Recent travel to South East Asia C. Admission to hospital for >2 weeks D. History of recurrent skin infections with abscess formation

ANS: D • The importance of recognising colonization with multi-drug resistant organisms (MDROs) is that it can significantly impact empirical therapy. • Many regimens used for empirical sepsis treatment may not provide coverage for MDROs and have to be adjusted accordingly depending on the presence of risk factors. • Residence in long-term care facilities has long been associated with several categories of MDROs and is also thought to be at the root of transmission of MDROs between different hospital networks. • Australia's regional neighbours, particularly in Asia, are hotspots of Gram negative resistance and many travellers become colonised. • Prolonged hospitalisation also serves as a risk factor because of the risks of inhospital transmission and antibiotic exposure during hospitalisation. • Recurrent skin infections with boils are a marker of MDRO colonisation (MRSA) but not Gram-negative MDRO colonisation as specified. • Also consider Candida auris in patients with risk factors for MDR Gram negatives.

DM PT Onc1 1. You are reviewing a 62-year-old man is up to Cycle 5 Day 11 of his 7th cycle of FOLFOX chemotherapy for the management of T3N2M0 colorectal cancer. The patient is overweight and has a history of type 2 diabetes (diet controlled). His vital signs are: Temperature 38.3 o C. Blood pressure is 150/70, HR is 100 and RR is 20. His neutrophil count is 0.4 x 10 9 cells/L He reports feeling hot and flushed, has an intermittent dry cough, constipation, intermittent tingling in his fingers. He denies dysuria, hematuria, headache, rash, or any other gastrointestinal symptoms. He has portacath in situ. The patient has a history of rash due to penicillins. What is the next step in your management? A. Take 2 sets of blood cultures, one from the periphery and one from the portacath, urine culture, organise chest X-ray, and start antibiotics depending on the results of the investigations. B. Take 1 set of blood cultures, one from the portacath, urine culture, organise chest X-ray, and start tazocin 4.5 g TDS C. Take 2 sets of blood cultures, one from the periphery and one from the portacath, urine culture, organise chest X-ray, and start aztreonam 1g TDS + vancomycin 1.5g IV BD. D. Take 2 sets of blood cultures, one from the periphery and one from the portacath, urine culture, organise chest X-ray, and start ceftazidime 2g TDS.

ANS: D • The patient has febrile neutropenia defined as temperature of 38.0°C or greater and neutrophil count of less than 0.5 x 109 cells/L, or less than 1.0 x 109 cells/L and predicted to fall to lower than 0.5 x 109 cells/L. The empiric management consists of appropriate microbiologic evaluation consisting of blood culture x 2 sets (one set consists of 2 bottles). Options include: One peripheral + one catheter (preferred) or Both peripheral or Both catheter (if unable to obtain a peripheral blood); urine culture if symptomatic or abnormal urinalysis or if the patient has a catheter, site specific investigations (in this case, chest X-ray as the patient has a cough). • The tingling in his fingers and constipation are side effects of FOLFOX chemotherapy. • Given the patient is hemodynamically stable, the antibiotic of choice is tazocin. However, he has a non-life-threatening penicillin allergy, in which case ceftazidime is recommended. If the patient was hemodynamically unstable, C is correct. B is incorrect because 2 sets of cultures should be taken, including from the portacath.

Dunedin pharm A bioavailability study has been performed to compare a proposed new generic capsule, in a strength of 50 mg, with a marketed tablet preparation in a strength of 100 mg. The active ingredient is a drug with linear kinetics. A cross‐over design was used, with each subject receiving, in random order and with an adequate wash‐out period, a single oral dose of one tablet on one occasion and one capsule on the other. The mean results for the area under the plasma concentration versus time curve (AUC) are 300 mgh/L for the capsule, and 500 mgh/L for the tablet. The best estimate of the relative bioavailability of the capsule with respect to the tablet is: A. 0.3. B. 0.6. C. 1.2. D. 1.7. E. 3.3

AUC (new) / AUC (reference) need to adjust for dose 300/50 // 500/100 6 / 5 C

DM PT Gen10 10. A 23-year-old female reports a family history of Fragile X syndrome. She undergoes genetic testing and is shown to carry a CGG repeat size of 180 (within the pre-mutation range). What are the risks to her of transmitting Fragile X syndrome to her offspring? A. She has a 50% chance of passing on the pre-mutation allele. If she has a son it will expand into the full mutation, whereas if she has a daughter it will remain within the pre-mutation range. B. She has a 50% chance of passing on the pre-mutation allele, which could expand into the full mutation as she is a female transmitting parent. C. She has a 25% chance of passing on the pre-mutation allele, which could expand into the full mutation as she is a female transmitting parent. D. She has a 25% chance of passing on the pre-mutation allele, but it won't expand into the full mutation as she is a female transmitting parent.

Answer B: 50% chance of passing on the pre-mutation allele, which could expand into the full mutation as a female transmitting parent Size of expansion influenced by sex of transmitting parent • Anticipation

Deltamed Haem 1 A 24‐year‐old African woman had a routine full blood count and the following results were noted. She is on sertraline for depression. She had no other medical illness, in particular no recent infections. There was no lymphadenopathy or splenomegaly on examination. Haemoglobin: 140g/L (115 - 165) MCV: 85fL (80 - 99) White cell count: 3.8x109/L (4 - 11) Neutrophils: 1.7x109/L (2 - 8) Lymphocytes: 2.0x109/L (1 - 4) Monocytes: 0.1x109/L (<1.1) Platelets: 300x109/L (150 - 450) Blood film: Normochromic normocytic red cells. Platelets are plentiful and normal in morphology. Mild neutropenia with no dysplastic features. What is the next most appropriate management? A. Repeat FBE in 1 week. B. Perform a bone marrow biopsy C. Cease sertraline D. Test double stranded DNA (dsDNA)

Answer: A Rationale: The patient has mild neutropenia with no other worrying features on blood film. A bone marrow biopsy is not indicated as there are no other concerning features on FBE or blood film. Sertraline is not a common cause of drug induced neutropenia. There are no features to suggest ds‐DNA will help in determining the diagnosis. She likely has benign ethnic neutropenia. Confirmation with repeat FBE is the best management plan.

Deltamed Haem 1 A 74‐year‐old woman underwent an elective left total knee joint replacement. Pre‐operation,she was on irbesartan for hypertension and warfarin for paroxysmal atrial fibrillation. She had unfractionated heparin for prophylaxis against deep vein thrombosis. On day 5 post‐operation,she was recommenced on warfarin. Day 8 post‐operation,she noticed swelling of her right calf and an ultrasound confirmed a deep vein thrombosis. Blood tests on the day showed: Haemoglobin: 100g/L (135 - 165) INR: 1.3 MCV: 85fL (80 - 99) APTT: 40s (27 - 35) White cell count: 17x109/L (4 - 11) Fibrinogen: 6g/L (1.5 - 4.0) Neutrophils: 13x109/L (2 - 8) Lymphocytes: 3.5.0x109/L (1 - 4) Monocytes: 0.5x109/L (<1.1) Platelets: 45x109/L (150 - 450) What is the next most appropriate management? A. Send for platelet serotonin release assay. B. Cease heparin and warfarin. Commence fondaparinux. C. Platelet transfusion given risk of bleeding with thrombocytopenia post‐operation. D. Send for heparin induced thrombocytopenia screen (PF4 ELISA).

Answer: B Rationale: The patient has a high pre‐test probability on 4T score for heparin induced thrombocytopenia. The PF4 ELISA is a screening test and the serotonin release assay is the gold standard test for confirmation of HIT. However, the best next management is to cease all forms of heparin (including in central lines as "hep‐lock") and warfarin; and commence a non‐heparin based anticoagulant. HIT is a prothrombotic condition and platelet transfusions are not recommended. Warfarin should only be commenced when the platelet count has recovered.

CLS Renal A 49 y.o. healthy lady, with good kidney function, normal BP, normal BMI, normal glucose, and no proteinuria or haematuria, would like to donate one of her kidneys to her 52 y.o. husband with near end stage kidney disease (eGFR 12) secondary to autosomal dominant polycystic kidney disease. There are no other potential living donors. Which of the following is incorrect? A. Her husband will likely have better transplant outcome compared with starting dialysis and waiting for a deceased donor kidney B. She is blood group A+ and he is blood group O+ (blood group incompatible), and therefore her husband will have to wait for a deceased donor kidney C. After donation, she is at no higher risk of developing end stage kidney disease compared with the general population D. After donation, she is likely at higher risk of developing ESKD compared with other healthy non-donors, but the absolute risk remains low E. Her recent history of melanoma may preclude her from kidney donation

Answer: B • A) Superior patient & graft survival • Living vs deceased donors • Pre-emptive vs non-pre-emptive (after starting dialysis) living donor Tx • C) & D) Donors are generally from a highly selected population => "Lower risk" of ESKD • Higher risk than healthy non-donors • Low absolute risk • Difficult to find a matched cohort of healthy non-donors E) Donor transmission of cancers a concern • Melanomas exacerbated by immunosuppression • If transmitted (even though probably low risk), most likely deadly • General advice: contraindication for donation even for completely excised early stage melanoma B) ABOi - no longer an absolute contraindication in living donor kidney Tx • Generally requires antibody removal pre-Tx • Options: • Antibody removal for low antibody titres • Kidney exchange program for high titres • Similar outcomes to ABOc Tx in the contemporary era

DM PT Im5 5. Which of the following therapies is most specific for B-cell mediated autoimmune disease? A. Cyclophosphamide B. Plasma exchange C. Mycophenolate D. Cyclosporin

B

Dun stroke A 78‐year‐old man presents with sudden onset left sided unsteadiness, vertigo and vomiting. Examination reveals left sided Horner's syndrome and a right‐ sided loss of pain sensation. Which of the following vessels is the least likely cause of these clinical features: A. Posterior inferior cerebellar artery B. Superior lateral medullary artery C. Basilar artery D. Middle lateral medullary artery E. Vertebral artery

Answer: C This patient has lateral medullary syndrome (Wallenberg's syndrome) that is due to infarction of a part of the lateral medulla and inferior cerebellum. It can be caused by all the above expect the basilar artery in addition inferior lateral medullary artery can cause the above syndrome.

Deltamed Haem 1 A 40‐year‐old man with infective endocarditis underwent a mitral valve replacement. Day 3 post‐ operation, he was on benzylpenicillin and gentamicin. His FBE showed: Haemoglobin: 100g/L (135 - 165) MCV: 85fL (80 - 99) White cell count: 14x109/L (4 - 11) Neutrophils: 10x109/L (2 - 8) Lymphocytes: 3.5.0x109/L (1 - 4) Monocytes: 0.5x109/L (<1.1) Platelets: 88x109/L (150 - 450) Blood film: Normochromic normocytic red cells. Neutrophils show toxic granulation. Platelets are reduced in number with normal morphology. What is the least likely reason for the thrombocytopenia? A. Sepsis related thrombocytopenia B. Antibioticsinduced thrombocytopenia C. Post‐cardiac bypassthrombocytopenia D. Heparin induced thrombocytopenia

Answer: D Rationale: Thrombocytopenia post cardiac bypass surgery is common. In addition to mechanical consumption of thrombocytes during cardiac bypass, thrombocytopenia is often due to concurrent sepsis and antibiotics use. Unless the patient had previous HIT, the timing of thrombocytopenia is not in keeping with heparin‐induced thrombocytopenia.

CLS Renal 57 y.o. male presents with nephrotic syndrome. He has oedema to mid shins, lethargy and loss of appetite. Renal biopsy shows membranous nephropathy with significant vascular changes. His circulating anti-PLA2R (phospholipase A2 receptor) antibody was positive. He is negative for all causes of secondary membranous on testing. After 6 months of maximal ACE inhibition, his proteinuria remains heavy at 8 g/day despite satisfactory BP control (125/70 mmHg) and has a slowly rising serum creatinine of 145 µmol/L (from 110 µmol/L). Serum albumin remains low at 20. He continues to be oedematous despite diuretics. He is also on warfarin and statin therapy. What treatment below will most likely induce and maintain remission long-term and reduce the risk of significant worsening of kidney function? A. Cyclosporin only for 12 months B. Tacrolimus only for 12 months C. High dose prednisolone alone D. Methotrexate and prednisolone alternating monthly for 6 months E. Cyclophosphamide and prednisolone alternating monthly for 6 months

Answer: E • Firstly, major advance in the understanding of pathogenesis of idiopathic membranous nephropathy => PLA2R Ab (phospholipase A2 receptor Ab) - Beck et al NEJM 2009 • Present in 70% of idiopathic MN and negative in secondary MN => non-invasive diagnostic test • Treatment of membranous nephropathy • No definite right or wrong answer! • First question => to treat or not with immunosuppression? • Second question => what to treat with?

CLS Haem 38 year old woman presents with a one week history of SOBOE. No previous medical history. No meds. Physical examination is unremarkable except for mild pallor and a palpable spleen tip Results as shown in table: Haemoglobin 73 g/dL 128-175 MCV 98fL 80-96 Total WCC 11.8 x 10^9/L 4-8 Platelets 370 x 10^12/L 150-450 Bilirubin 42umol/L <20 LDH 880u/L 20-200 Direct Coombs IgG+++ C3B+++ Blood Film Sphercytosis, retulocytosis Which of the following conditions is the most likely cause of these findings: A. Hereditary spherocytosis B. SLE C. Infectious mononucleosis D. Mycoplasma infection E. Cold agglutinin disease

B

CLS Haem What is the most common reason for major blood ABO incompatibility? A. Antibodies to donor blood not detected in pre-transfusion sample B. failure to obtain pre-transfusion sample from the correct patient C. failure to identify correct patient pre-transfusion D. lab error E. unexpected ABO antibodies in donor blood

B

CLS Haem Which of the following is the most appropriate management of a patient with mild hemophilia A (FVIII level 5%), planned for pilonidal sinus surgery? A. Crypoprecipitate for severe bleeding B. Factor VIII for perioperative period C. Factor VIII from 7 days post op D. DDAVP 24 hours perioperatively E. Factor VIII only for severe bleeding

B

CLS Resp Regarding COPD exacerbation, which statement is true? A. Severe dyspnoea with clinical signs suggestive of respiratory muscle fatigue is not an indication for non-invasive mechanical ventilation. B. When added to usual care (in patients fitting criteria), non-invasive ventilation approximately halves the mortality rate. C. A venous blood gas is an accepted alternative for monitoring respiratory acidosis treated with bi-level positive airways pressure (BiPAP). D. Typical initial BiPAP settings are 20cmH2O of IPAP and 10cmH2O of EPAP. E. Intravenous methylxanthines are a first-line therapy.

B

CLS Rheum A 44 year old female has a 20 year history of systemic lupus erythematosus. She was previously on cyclophosphamide but is now on prednisolone and hydroxychloroquine for skin manifestations. Past history includes type II diabetes. She now presents with ventricular fibrillation and dies. What is the most likely underlying reason for this presentation? A. Pericarditis and tamponade B. Coronary artery disease C. Libman sacks endocarditis D. Viral myocarditis E. Coronary vasculitis F. Antiphospholipid syndrome with large pulmonary embolus

B

CLS Rheum A diabetic labourer sustains an injury to his foot when a brick falls on it. Several weeks later he develops increasing burning pain severely limiting mobility, increasing sweating and sensitivity to touch or cold. The most appropriate diagnostic investigation is: A. Doppler ultrasound of lower limb vasculature B. Bone scintigraphy C. Bone biopsy D. X-ray

B

DM ID 30yo Sudanese man with persistent cough & haemoptysis. 3 month old baby at home. CXR shows upper zone involvement with a cavity. Sputum has AFBs. Management should include all of these except? • A - Admit and put in negative pressure isolation room • B - Quantiferon Gold Assay • C - HIV, HBV & HCV tests • D - Start TB therapy (e.g., HREZ) once appropriate samples taken for micro & cultures • E - Screening of family members by Health Department

B

DM ID Which of the following is most true regarding culture- negative endocarditis? • A. Fastidious organisms (e.g., C. burnetii, L. pneumophila or T. whipplei) will be identified if the blood cultures are kept for two weeks • B. Combination of vancomycin and ceftriaxone will treat most aetiologies • C. 16S PCR techniques on explanted valve tissue will identify nearly all microbial causes • D. Blind sub-cultures from the blood culture bottle will help to identify HACEK organisms • E. The most likely cause is non-infectious (e.g., Libman-Sacks endocarditis)

B

DM ID Which one of the following statements concerning Creutzfeldt-Jakob Disease (CJD) is true? A. Variant CJD (vCJD) can be differentiated from sporadic CJD (sCJD) on cerebral MRI scanning B. The absence of a CSF pleocytosis supports the diagnosis of vCJD C. Amyloid plaques are not usually present in the brains of vCJD cases D. The 14-3-3 protein can be detected by PCR of CSF in most patients with sCJD E. Neurosurgical instruments used on a patient with suspected CJD should be soaked in 10% gluteraldehyde for 1 hour prior to autoclaving

B

DUnedin gastro Pt with UC Following the colonoscopy, she becomes very ill with loose bowel motions up to 10 times per day. With the above findings you make a diagnosis and start treatment. A. 5-aminosalicylates (e.g. Asacol 800mg tds) B. Corticosteroids on a tapering regimen (e.g. Prednisone 40mg od) C. Immunomodulators (e.g. azathioprine 2.5mg/kg bodyweight) D. Loperamide to stop the diarrhoea E. Ibuprofen to modify the inflammatory component of this disease and reduce the abdominal pain

B

Deltamed Immuno A T lymphocyte encountering its specific antigen in the absence of a costimulatory signal via CD28 is most likely to A. undergo apoptosis (programmed cell death) B. become anergic (unresponsive to appropriate stimulation) C. proliferate D. produce low levels of helper T cell type 1 (Th1) cytokines E. produce low levels of helper T cell type 2 (Th2) cytokines

B

Deltamed Onc 55 y/o F. Vague abdo pain. Bx of lesion in abdo shows spindle shaped cells. Most likely dx? A. Adeno esophagus B. GIST C. limitis plastica D. adrenal tumour E RCC (hypernephroma)

B

Deltamed Onc Which paraneoplastic syndrome is most likely to occur in this patient with SCLC? A. Hypercalcaemia B. Syndrome of Inappropriate antidiuretic hormone secretion C. Myasthenia gravis D. Lambert Eaton Syndrome

B

Deltamed immuno A woman typed as Group O HLA‐A1, HLA‐B 7/8 has a child typed as Group A, HLA‐ A 1/2, HLA‐B 5/8. Which of the following of her partners could have been the father? A Group O, HLA‐1 1/2, HLA‐B 5/8 B Group A, HLA‐A‐ 2/3, HLA‐B 5/12 C Group AB, HLA‐A 1/9, HLA‐ B 5/7 D Group A, HLA‐A 1/2 , HLA‐B 8/12 E Group B, HLA‐A 2/9, HLA‐B 5/8

B

Deltamed neuro Which of the following disorders is most likely to produce muscle pain? A. Becker's muscular dystrophy B. Atorvastatin myopathy C. mitochondrial myopathy D. polymyositis

B

Deltamed onc The patient undergoes a CT guided lung biopsy. Which EGFR mutation is most likely to be found in the setting of resistance to first-line EGFR targeted tyrosine kinase inhibitors? A. Exon 21 L858R B. T790M C. Exon 20 ins D. Exon 19 del

B

Deltamed stats A test is used to screen for a target disorder. The best definition of the specificity of the test is "the proportion of individuals: A.with the target disorder who have a positive test result B.without the target disorder who have a negative test result C.with a positive test result who have the target disorder D.with a negative test result who do not have the target disorder E. with or without the target disorder who have a correct result

B

Dun Cardio. Inf STEMI presenting to rural centre. SBP 80 Next step in Mx? A. temp pacemaker B. IV isoprenaline C. IV atropine D. Transfer for angio E. IV nitroglycerin

B

Dun Neuro A 24-year-old man with a severe closed head injury is receiving phenytoin after having three seizures. He has received a loading dose of 1000 mg and then 300 mg/day via nasoenteric tube. His phenytoin concentration one week later is 8 µmol/L [therapeutic range 40-80]. A number of his other treatments are listed below. Which one is most likely to have contributed to the sub therapeutic phenytoin concentration? A. Metronidazole. B. Nasoenteric feeds. C. Omeprazole. D. Hyperventilation. E. Erythromycin

B

Dun Neuro In patients dying of Sudden Unexpected Death in Epilepsy (SUDEP), the most likely mechanism of death is: A. aneurysmal rupture B. central hypoventilation C. ventricular fibrillation D. neurogenic pulmonary oedema E. laryngospasm

B

Dun Pharm 52 y/o with bipolar. on lithium 750mg/day. Well for 4yrs. Recently prescribed 40mg/day frusemide. Li toxic at 1.9. Ataxia, tremor, diarrhoea Most appropriate Mx is to cease A. Li and repeat level in 1w B. Li and Fr. Repeat level 24hrs C. Fr. repeat level 24hrs D Li and Fr. Repeat in 1w E. Fr. repeat in 1w

B

Dun Rheum This picture shows the hands of a 38 yr old lady (Raynaud's). Which rheumatic disease has the greatest association with this finding? A. SLE B. Scleroderma C. MCTD D. Dermatomyositis E. Sjogrens

B

Dun Rheum What is a transition to avoid in osteoporosis treatment? a. teriparatide to denosumab b. denosumab to teriparatide c. teriparatide to zoledronic acid d. alendronate to denosumab e. alendronate to zoledronic acid

B

Dun cardio A doctor makes a diagnosis of hypertrophic cardiomyopathy in a 34-year-old woman. She responds well to medication and returns for regular follow-up appointments. Which one of the following is the most appropriate action to extend care to the woman's family? A. Advise the woman of the genetic basis of hypertrophic cardiomyopathy. B. Advise the woman that her immediate relatives should seek medical advice regarding their risk of hypertrophic cardiomyopathy. C. Contact the relatives regarding the risk of them having hypertrophic cardiomyopathy. D. Contact the relatives' doctors and recommend that the relatives be assessed regarding the possibility of hypertrophic cardiomyopathy. E. Confirm the familial basis of the diagnosis of hypertrophic cardiomyopathy by DNA testing.

B

Dun cardio The predominant cause of stroke after cardiac surgery is: A. air embolism. B. aortic atherosclerotic embolism. C. intra-cranial haemorrhage. D. hypotension. E. carotid stenosis.

B

Dun stroke A patient presents 12 hours after a thunderclap headache. A contrast cranial computed tomography (CT) scan is normal. To determine if there is aneurysmal bleeding, the most appropriate next investigation is: A. Cranial magnetic resonance imaging (MRI) with diffusion weighted imaging. B. Cerebrospinal fluid (CSF) spectroscopy for bilirubin. C. CSF spectroscopy for crenated red blood cells. D.Four vessel cerebral angiogram. E. Cranial magnetic resonance imaging/angiography (MRI/MRA).

B

Dun stroke A right‐handed 70‐year‐old lady has a CT scan of her brain, which reveals a lesion at the left angular and supramarginal gyrus. On examination one would expect all of the following signs EXCEPT: A. Dysphasia B. Inappropriate affect C. Finger agnosia D. Constructional apraxia E. Difficulty with left - right discrimination

B

Dun stroke Compared to stenting, endarterectomy is most likely to increase the risk of which of the following outcomes? A. death B. myocardial infarction C. renal failure D. sepsis E. stroke

B

Dunedin endo Which of the following people with diabetes would be at highest risk of a severe hypoglycaemic episode in the next 24 hours? A: A Type 1 Diabetic patient being changed form twice daily NPH to Glargine B: A Type 1 Diabetic patient who has just been treated for a severe hypoglycaemic episode C: A Type 1 Diabetic patient with nocturnal hypoglycaemic episodes twice per week D: A person with post pancreatectomy diabetes E: A Type 1 diabetic with co-existent Addisons disease

B

Dunedin gastro 42 year lady. Complaining of chronic dyspepsia. Solitary 2cm smooth mass with central small overlying ulcer seen in fundus. Biopsied. Immunohistochemistry +ve for CD117(c-KIT). Which is most appropriate course of management? a. Colonoscopy b. CT scan and EUS c. HP eradication and repeat endoscopy in 18/12 d. Referral for genetic counselling e. Repeat UGI in 5 years

B

Dunedin gastro A 17 year old girl has had coeliac disease since she was 3 years old. She presents with 4 weeks of foul-smelling diarrhoea, flatulence, crampy mid abdominal pain and bloating. Her blood test are normal. Her upper GI endoscopy is macroscopically normal, this is her small bowel biopsy. You should: a. Perform a barium small bowel series b. Question diet compliance c. CT abdomen d. Start treatment with metronidazole e. Start fibre supplement

B

Dunedin gastro A 46 year old man has history of ulcerative colitis since the age 21 years. He has been taking Pentasa which he does not always remember to take. Biopsy results demonstrate chronic active colitis and moderate dysplasia. You would recommend which of the following treatments? A. Increase Pentasa B. Pentasa and prednisone C. Total colectomy D. Sigmoid colectomy

B

Dunedin gastro A 50 year man has had persistent nausea for many years with occasional vomiting. An upper GI endoscopy performed 5 years previously showed a pattern of gastritis and microscopically there was chronic inflammation with the presence of Helicobacter pylori. He now undergoes a second endoscopy, and a small area of gastric mucosa is noted in the fundus with loss of rugal folds. A biopsy reveals well-differentiated adenocarcinoma confined to the mucosa. This lesion is best characterised by: a. Metastases limited to the regional lymph nodes b. A 5 year survival following resection of >90% c. A signet ring pattern by light microscopy d. The gross appearance of linitis plastica e. A high incidence of occurrence in the United states

B

Dunedin gastro A 69-year-old man has a long history of dyspepsia and a previous peptic ulcer. After a recent severe flare of epigastric pain, he is referred for endoscopy which reveals an active duodenal ulcer. Antral biopsies show H pylori infection. He receives 14 days treatment with omeprazole, clarithromycin and metronidazole. The result of a 14C-urea breath test conducted 8 weeks after completion of therapy is positive. Which one of the following is the best interpretation of the test result? a. Re-infection with H. pylori. b. Failure of triple therapy to eradicate the infection. c. Test performed too soon after therapy d. Test is inaccurate in patients older than 65 years. e. False positive due to oral infection.

B

Dunedin gastro A patient with persistent diarrhoea and foulsmelling stools undergoes an endoscopy. Biopsy of the duodenum suggest coeliac disease. In addition to increased intra-epithelial lymphocytes, which description bests describes the histological features seen a. Crypt hypoplasia + villous hypertrophy b. Crypt hyperplasia + villous atrophy c. Breakdown of basement membrane d. Intestinal lacteals congested with macrophages e. Paneth cell metaplasia + crypt abscesses

B

Dunedin gastro A young male presents with anaemia, RIF pain and a palpable mass. He has a long history of mild watery diarrhoea and 5kg weight loss. A small bowel series shows a stricture and oedema in the small bowel RIF with inflammatory mass. The next step in management A. oral prednisone B. colonoscopy C. CT abdomen D. oral metronidazole E. laparotomy

B

Dunedin gastro Difficulty swallowing has been noted for over 6 months by a 45 year old woman. A barium swallow reveals an area of stricture in the lower oesophagus just above the gastrooesophageal junction. She has an upper GI endoscopy performed and biopsies of the lower oesophagus are taken which show no acute inflammation or ulceration, only sub-mucosal atrophy along with fibrosis of smooth muscle. These findings strongly suggest: a. Mallory-Weiss syndrome b. Scleroderma c. Iron deficiency d. Portal hypertension e. Barrett's oesophagus

B

Dunedin gastro Regarding the demonstrated pathology (birds beak) all of the following are true except: a. Squamous cell carcinoma is more common b. Oesophageal dilatation is contraindicated c. Oral nitrates can be helpful d. Weight loss is a late sign e. Manometry is required in the diagnostic workup

B

Dunedin gastro You are the weekend registrar, you are phoned 10am Saturday morning regarding a 65 yr man who has presented to the ED with 24hrs of recurrent melaena following 1 week of dyspepsia. He has a history of a recent NSTEMI, with 3 x DES insertion 4 weeks ago. On DAPT - aspirin 100mg and clopidogrel 75mg daily. Other drugs cilazapril and statin. Clinically Hb 83 (was 145 during cardiology admission), pulse 75, BP 130/70. In this patient which of these is the most appropriate pre-endoscopy management? a. Withhold aspirin and clopidogrel b. Withhold clopidogrel continue aspirin c. Withhold clopidogrel, continue aspirin, IV tranexamic acid d. Continue aspirin and clopidogrel e. Withhold aspirin and clopidogrel, PO tranexamic acid

B

DM PT ID4 The epidemiology of culture-negative endocarditis has changed over time. Please choose the most appropriate causative pathogen for the clinical scenarios described below. A. Tropheryma whipplei B. Coxiella burnetii C. Bartonella quintana D. Cardiobacterium valvarum E. Streptococcus mutans F. Haemophilus aphrophilus G. Mycobacterium haemophilum H. Staphylococcus aureus 4. A 67-year-old sheep farmer presents with fevers and increasing dyspnoea. He has a past history of aortic stenosis. A chest X-ray shows left lower lobe consolidation. No microbiological diagnosis is made, and he is treated with amoxicillin and doxycycline for 7 days with improvement. Two months later, he develops left-sided weakness and is noted to have suffered a right middle cerebral artery stroke. Carotid ultrasound is unremarkable, but a transthoracic echocardiogram shows a 1.5 cm vegetation on the aortic valve.

B 4. ANS: B • Coxiella burnetii is the causative agent of Q fever and is spread through transmission from infected animals, typically sheep or goats. • It causes several clinical manifestations including pneumonia, hepatitis, meningitis and endocarditis. • Endocarditis typically occurs as a subacute or chronic complication and is particularly common in those with underlying valvular disease. • It should be routinely tested for in culture-negative endocarditis and should also be a consideration in cases of atypical pneumonia or unexplained hepatitis.

DM Mock 1.What is the most common site for thrombus to form in a patient with chronic atrial fibrillation? A. Left ventricle B. Left atrial appendage C. Pulmonary veins D. Aortic valve

B >90% of AF patients with a cardiac thrombus have it in the LAA; if you find thrombus elsewhere, it is almost always coexistent with an LAA thrombus. Hence the performance of a TOE to exclude LAA thrombus in non-anticoagulated patients requiring cardioversion. The LAA is not well visualized on TTE and hence need for a TOE.

Book R5 A 43 y/o F with longstaning scleroderma comes to you with worsening constipation for several months. Recently associated with abdo pain + very hard stools. Occasional vomiting. Which 1 might explain her sx? A. Achalasia B. Autonomic nerve dysfunction C. Bacterial overgrowth D. GAVE E. Prolonged gallbladder emptying time

B Autonomic n dysfunction = smooth mucle atrophy + muscle fibrosis. Hypomotility of oesophagus (strictures), stomach, SI, LI = gastroparesis, pseudo-obstruction, wide mouthed diverituclar.

Dunedin Resp 16. A 46 year old woman with hypertension and epilepsy is referred to outpatients for management. She is a current smoker, with 30 pack-year history. What is the most appropriate intervention to achieve smoking cessation for this patient? A. Cognitive behavioural therapy. B. Nicotine replacement therapy. C. Bupropion. D. E-cigarette substitution.

B Buproprion CI in epilepsy

Deltamed immuno Which one of the following cell surface molecules is least important in the regulation of natural killer cell function? A. Major histocompatibility complex (MHC) class I on the target cell B. MHC II on the target cell C. Killer‐activating receptors on the NK cell D. Killer‐inhibiting receptors on the NK cell E. Receptor for the Fc portion of the immunoglobulin G (IgG) on the NK cell

B CD8 = MHCI

Deltamed Onc 70 y/o man. Metastatic CRC. multiple liver mets. CEA at dx is 150 Chemo with 5FU, oxaliplatin, bevacizumab. after 6 cycles CEA 100, 30% reduction in tumour size. Increasing pins and needles in fingers + toes. Difficulty buttoning shirt Most appropriate Mx? A. cease beva + continue others B. cease oxali, contine others C. reduce bevac D. reduce oxali by 50% E. cease oxali + bevacizu

B Cease as functional effects. If just parasthesias could dose reduce

D7 58 y/o. Longstanding T2DM, HTN. Bad HbA1c. Shin rash. What is the most likely dx in this pt? A. acanthosis nigricans B. necrobiosis lipoidica C. erythasma D. contact dermatitis E. plaque type psoriasis

B Chronic granulomatous dermatitis of unknown cause. 25% it develops before diabetes. Yellow, brown, telangectatics plaques Shins or dorsum of feet. 30% have ulcers. Assoc with retinopathy/nephropathy Erythrasma - fungal infection of 5th intertriginous space

Dun Rheum With respect to positive ANA testing, all of the following are correct except; A. An ANA in conjunction with positive dsDNA is highly specific for SLE B. An ANA with a DFS 70 staining pattern is specific for systemic sclerosis C. Specificity is low, irrespective of titre D. Is more likely in immune mediated myositis than rheumatoid arthritis E. In conjunction with anti Sm antibodies is highly specific for SLE

B DFS = less likely SLE

R 31 50 y/o man. Horner syndrome. partial ICA dissection. Stretch skin, hypermobile joints. increasing scarring. Nil FHX. Most likely dx A. Marfan B. Ehlers-Danlos C - tuberous sclerosis D - noonan E. klinefelters

B Ehlers-Danlose. 1 type can lead to aneurysms, valve prolaspe, intestinal perf + spontaneous pneumothorax. Marfan: long bones, aortic root disease TS: seizures, neurofibromas, subependymal nodules, renal angiomyolipomas, lymphangioleiomyomatosis

D8 27 y/o. Reddish brown macule on face after taking cotrim. Heals over next few weeks with hyperpig. Similar lesion 3yrs prior with cotrim. Most likely dx? A. Discoid lupus B. fixed drug reaction C. pemphigus vulgaris D. atopic dermatitis E. pityriasis rosea

B Fixed drug reactions occur at same site and there's often residual hyperpigmentation. Common, immune mediated, cutaneous. topical roids + avoid offending drug Discoid lupus - discrete, slightly infiltrated, extends into hair foccile. Face, neck, scalp. Heal with depressed central scars. Pemphigus vulgaris - flaccid bullae begin in orophyarnx.

Dunedin Haem The most reliable test for beta thalassaemia is: A. HbF level B. HbA2 level C. HbH bodies D. Hb stability test E. Immunochromatographic strip for gamma tetramers

B HbF usually up but not diagnostic

CLS Rheum A year old man is on a statin (80mg simvastatin), thiazide diuretic and other medications. He presents with muscle aches and pains. His blood pressure is poorly controlled, so he is started on diltiazem 180mg controlled release formulation. Four weeks later he develops proximal weakness and muscle aches, and has noted "brownish coloured urine". The following investigations are performed: Ix: Na 135, K 2.9, Creatinine and Ur Normal AST 150, ALT 258, CK 30 000 What is the most likely diagnosis? A. Hypothyroid myopathy B. Statin myopathy C. Polymyositis D. Diltiazem myopathy E. Hypokalaemic myopathy

B Myalgia more common than myositis, no↑ in CK • 1% to 5% of patients muscle pain and weakness • Patients experiencing muscle pain, weakness or cramps should have their CK level measured. If ↑ (eg >5x upper limit of normal) or if myopathy is suspected, treatment should be stopped • Rhabdomyolysis characterised by plasma CK > 20,000 IU/L, myoglobinuria and extreme muscular pain occurs in about 1 in 250,000 patients • More common in females, elderly, hypothyroidism and with concomitant therapy with cytochrome P450 3A4 metabolised drugs e.g. cyclosporin, erythromycin, and fibrates (particularly gemfibrozil)

Deltamed What completion staging investigations should this patient with SCLC have have? A. CT brain B. CT brain + PET C. PET D. MRI

B PET doesn't see brain well

DM PT Rh4 4. Febuxostat is a non-purine xanthine oxidase inhibitor that is an alternative to allopurinol. Which of the following is true regarding this medication in comparison to Allopurinol? A. Better cardiovascular safety profile B. Lower risk of hypersensitity syndromes C. More effective at lowering uric acid D. No need for colchicine prophylaxis with initiation

B Patients with gout and cardiovascular disease were assigned to receive febuxostat or allopurinol. At 32 months, there was no significant between-group difference in a composite cardiovascular end point, but all-cause and cardiovascular mortality were higher with febuxostat.

DM PT Rh7 7. Allopurinol hypersensitivity reactions are a serious and potentially life-threatening complication of Allopurinol therapy. There is no definite treatment, and care is supportive. Which of the following is not a risk factor for this complication? A. High starting dose B. HLA-B*5801 positive C. Long time on therapy D. Renal impairment

C

Dunedin gastro You admit a 43 y/o female with alcoholic hepatitis. You calculate the discriminant function to be 38. The patient becomes acutely psychotic with steroids so these are discontinued. What other options should you consider? a) Colchicine b) Pentoxifylline c) Liver transplant d) Supportive care only e) Give steroids anyway

B Prednisone 40 mg daily for 28 days, followed by a 16 day taper Lille score can be used to determine response to tx Pentoxifylline 400 mg three times per day as an alternative Inhibitor of TNF (Controversial) Discontinue nonselective beta blockers

R19 32 y/o. Dry eyes + mouth. +ve ANA, anti-Ro, anti-La. Which 1 of the following is she predisposed to developing? A. asthma B. lymphoma C. pleural effusion D. primary salivary gland neoplasms E. SCLC

B Primary Sjogren = lymphoma. (NHL) Risk of lymphoma = marginal zone B cell lymphoma, severe involvement of exocrine glands, vasculitis.

CLS Rheum A patient presents with back pain. Which feature is most suggestive of spinal canal stenosis? A. Pain on prolonged weight-bearing B. Paraesthesia on weight-bearing C. Osteoarthritic changes on X ray D. Loss of deep tendon reflexes E. Loss of vibration sense in feet

B Pseudoclaudication • Most common symptom = discomfort in buttock, thigh or leg on standing or walking that is relieved by rest and not produced by vascular insufficiency • Generally relieved by lying down or sitting • Adopting a posture of flexion at the waist, leaning over a shopping trolley/church pew/bending forward when walking (simian stance)

DM PT Gen9 9. Which of the following mutations would most likely result in a single amino acid change? A. Single nucleotide deletion B. Substitution of the 2nd nucleotide in a codon C. Substitution of the 3rd nucleotide in a codon D. 6 nucleotide deletion

B Redundancy in the system. For substitutions, changing the 1st or 2nd nucleotide will result in a single AA change, but changing the 3rd nucleotide may not result in an AA change. The Questions was: Which of the following mutations would most likely result in a single amino acid change? A. Single nucleotide deletion à disrupts reading frame B. Substitution of the 2nd nucleotide in a codon à will change 1 amino acid C. Substitution of the 3rd nucleotide in a codon à may not change the amino acid D. 6 nucleotide deletion à will result in loss of 2 amino acids

Dun cardio A patient with aortic regurgitation has the following haemodynamic measurements: cardiac output (CO) 7.5 L/minute heart rate (HR) 75/minute left ventricular end-diastolic volume (LVEDV) 200 mL left ventricular end-systolic volume (LVESV) 50 mL The regurgitant fraction is defined as the ratio of the regurgitant volume to the total volume flowing through the valve with each beat. The regurgitant fraction in this patient is: A. 25%. B. 33%. C. 50%. D. 67%. E. 75%. 200 - 50/

B SV = CO / HR = 7500 / 75 (as need in mls) SV = 100mls/beat volume thru aortic valve/beat = SV = LVEDV - LVESV = 200 - 50mL = 150mL (so 50mLs being regurged) 50mL / 150mL = 1/3

Deltamed Onc 59 y/o truck driver. Met GIST with peritoneal mets. Initial sx respond to imatinib 400mg OD. Scans at 6m and 12m show partial response. at 18m = increasing sx. Restaging reveals 30% progress in pertioneal disease. Most appropriate next step? A. Represents imantinib resistance. Switch to another TKI sorafenib B. represents tumour progression, increase imantinib dose C. Represents tumour refractoriness to TKI. Stop rx and palliative D. Represents isolated tumour prorgress in pertio. Refer to surg for debulking

B WIth this med, when you get resistance/progression you can increase dose. AEs: puffy eyes, cardiotoxicity, ankle oedema. peritoneal disease hard to operate on

Dunedin gastro 35 year female. Known systemic sclerosis. Complains of worsening acid reflux and vomiting. She has previously been started on PPI for this. On examination the abdomen is soft and non-tender. Hydrogen breath test is negative Which of the following is most likely to benefit her long term symptom management? a. Double PPI b. Metoclopramide c. Lifestyle modification d. Refer for fundoplication e. Ranitidine

B dysmotility

Dun Rheum A 44 yr old man has a 10 yr history of well controlled chronic plaque psoriasis and psoriatic arthritis. He presents with a severe flare of his skin psoriasis and joint disease, far worse than any previous episode. Multiple joints are involved. He has skin involvement over 90% of his Body surface area. He has no other PMHx. He takes sulphasalazine. O/E he has 8 swollen joints, extensive severe psoriasis a temp of 37.8 ̊C. Cervical lymphadenopathy is present as well epitrochlear and post auricular nodes. Together with the rash below, Which of the following is the most appropriate diagnostic test? A. CCP antibodies B. HIV C. HLA B27 D. Hepatitis B E. Streptococcal antibodies

B erythrodermic psoriasis. Life threatening form of psoriasis. HIV worsens DDx. high dose roids Strep causes guttate psoriasis

Deltamed Onc 60 y/o. advanced melanoma with liver + lung mets. V600E BRAF negative. Ipilimumab 3 weekly. 6weeks in = mod severe diarrhoea. Autoimmune colitis strongly suspected. Mx? A. antidiarrhoeals. IVF. Metro B. anridiarrhoeals. IVF, oral roids C. antidiarrhoeals. IV hydration, metro, roids D. IV hydration, metro, roids E. IV hydration, inflix

B mod colitis (up to 6 times) so oral roids. If severe = IV. If still ognoing = infliximab. Not consistent with infection

Deltamed onc This patient is ECOG performance status 2. What does this mean? A. Symptomatic, ambulatory B. Symptomatic, in bed <50% of the day C. Asymptomatic D. Symptomatic, in bed >50% of the day

B 0 = asymp 1 = symp ambulatory 2 = symp, bed <50% 3 = symp, bed >50% 4 = symp, unable to do self care 5 = dead

CLS Resp What is the underlying mechanism of male infertility in cystic fibrosis? A. Gonadal dysgenesis B. Cilial dysmotility C. Obstructive azoospermia D. Chronic inflammation E. CFTR mutation affecting production of LH and FSH

B 97-98% of males with CF will be infertile due to obstruction or congenital absence of the vas deferens

Book R1 1. Which 1 of the fllowing is true regarding MTX? A. it activates DHFR B. it inhibits the productive of the active form of THF from inactive DHF C. it increases folic acid excretion D. it induces DNA cross linking E. it is not teratogenic

B. MTX binds + inhibits DHFR. inhibits nucleotide and DNA synthesis. Competitively inhibits conversion of DHF to active THF. Folinic acid given to reduce myelosuppression but it doesn't antagonise anti-inflamm effects.

Deltamed endocrine Which of the following is most suggestive of Cushing's syndrome? A. 4mm pituitary adenoma on MRI B. dorsocervical fat pad C. purple, 2cm axillary striae D. 1cm adrenal adenoma, 10 HU

B. Most discriminating Easy bruising Facial plethora Proximal myopathy Striae (espec if red/purple, >1cm wide) weight gain in kids with decreasing growth velocity

Dun stroke In patients with atrial fibrillation, anticoagulation is often required. Whilst Warfarin was commonly used previously, the newer anticoagulants are becoming increasingly popular. When comparing warfarin to dabigatran in patients with atrial fibrillation with a creatinine clearance of > 30mL/min and normal hepatic function requiring anticoagulation. Which of the following statements is false? a. Dabigatran 110mg twice daily has a lower major bleeding rate than warfarin b. Dabigatran 110mg twice daily has a lower ischemic stroke rate than warfarin c. Dabigatran 150mg twice daily has similar rates of major haemorrhage compared to warfarin d. Dabigatran 150mg twice daily has a higher rate of gastrointestinal bleeding than warfarin e. The rates of haemorrhagic stroke are lower with dabigatran 110mg AND 150mg twice daily compared to warfarin

B B ‐ The most important trial comparing warfarin and dabigatran was the RE‐LY trial published in the NEJM in 2009. The main outcomes from this were that at a dose of 110mg BD, dabigatran was non inferior to that of warfarin in reducing ischemic stroke, but had a lower major bleeding rate, where as dabigatran at 150mg was superior to warfarin in reducing ischemic stroke but had equal rates of major haemorrhage. It had reduced rates of haemorrhagic stroke, but higher rates of gastrointestinal bleeding.

CLS Rheum 27 year old male presents with an 18 month history of back pain, heel pain and a swollen knee. Has restriction in spinal flexion, is HLA-B27 positive and CRP is 25. In addition to physiotherapy, what is the most appropriate initial pharmacologic management? A. Prednisone B. Naproxen C. Methotrexate D. Etanercept E. Salazopyrin

B First step is NSAID plus exercise

CLS rheum Patient with well-managed rheumatoid arthritis (on methotrexate and statin), presents for review, describing increasing lethargy. Nil evidence of active joint inflammation - oral ulcers on examination. FBE: Hb 102, WCC 2.5, Neut 1.2, Platelets 90 What is the drug of choice to manage these findings? A. Folic Acid B. Folinic Acid C. Cholestyramine D. G-CSF

B Folinic acid: • A 5-formyl derivative of tetrahydrofolic acid • Readily converted to other reduced folic acid derivatives (e.g. tetrahydrofolate) → so has vitamin activity equivalent to folic acid as it does not need the action of dihydrofolate reductase for its conversion • its function as a vitamin is not affected by inhibition of this enzyme by drugs such as methotrexate → so useful in treatment of MTX overdose

CLS Rheum What is the difference between inclusion body myositis and polymyositis? A. Bilateral ptosis B. Weak long finger flexors C. Weak deltoids D. Weak quadriceps E. Truncal weakness

B Inflammatory muscle disease characterized by progressive muscle weakness and wasting • Often DDx for cases of polymyositis unresponsive to therapy • Onset of muscle weakness in IBM is generally gradual (over months or years) • Falling and tripping are usually the first noticeable symptoms • For some patients the disorder begins with weakness in the hands causing difficulty with gripping, pinching, and buttoning Occurs more frequently in men than women • Affects both proximal and distal muscles • There may be weakness of the wrist and finger muscles & atrophy of the quadricep muscles • Atrophy or shrinking of the forearms is characteristic • Dysphagia occurs in approximately 50% of IBM cases • Symptoms of the disease usually begin after the age of 50, although the disease can occur earlier

Dun Pharm All of the following drugs have a sign variation in response to genetic polymorphims. Which is most related to pharmacodynamic (not kinetic) principles? A warfarin B. carvedilol C. azathioprine D. phenytoin E. isoniazid

B PD = what DRUG does to body PK = what body does to drug

Dun ID 45 yo female with HIV, CD4 =50 cells/ul, Viral load = 500,000 copies/mL. Presents with SOB, ABG: PO2 68 mm Hg. CXR diffuse BL infiltrates. BAL = + CMV by PCR and + PJP by PCR. • The patients should be treated: A. ganciclovir + TMP-SMX B. TMP-SMX + prednisone C. ganciclovir + TMP SMX + prednisone D. TMP-SMX alone

B CMV in HIV - 50% BAL will show CMV need tissue bx • PaO2 <70 mm Hg - addition of prednisone • Improved mortality TMP SMX - best • (HIV+ pt 100-fold more likely to get SJS) Primaquine + Clindamycin Parenteral Pentamidine • (pancreatic toxin - necrosis islet cells hyperglycemia- even after stop drug) • Dapsone - mild disease (sulfone), methemoglobinemia • i.e. pH 7.4; PO2 96, PCO2 39, O2 sat'n 79% • If PaO2 < 70 - steroids given • Expect improvement 4-8 days

Dunedin Resp Which of the following is not a risk factor for early death from acute pulmonary embolism? A. Age. B. Previous thromboembolism. C. History of cancer. D. Altered mental status.

B PESI score +60 for altered mental status +30 SBP <100, cancer +20 arterial sat <90, temp <36, HR >110, RR >30 +10 Male, CHF, chronic lung disease Lysis Accelerated clot lysis short term physiologic benefit • No improvement in mortality • No reduction in recurrent PE • Improved right ventricular function • Most commonly accepted indication is persistent hypotension

Deltamed Neuro In an acute radiculopathy (first 7 days), which abnormal finding is most likely on needle EMG of an affected muscle? A. Abundant fibrillations and positive sharp waves B. Decreased motor unit recruitment C. Early motor unit recruitment D. Increased motor unit amplitude E. Increased motor unit duration

B.

Deltamed neuro Which feature is best in differentiating motor neuron disease from multifocal motor neuropathy at diagnosis? A. Bulbar deficits B. Nerve conduction studies C. Hyperreflexia D. Sensory deficits E. Anti GM1 antibodies

B.

DM PT E1 1. A 56-year-old homeless man presents with a fall. On examination, BMI is 18 kg/m2 . Gait is noted to be ataxic. He appears euvolaemic. Serum sodium is 119 mmol/L, serum osmolality, 251 mosm/L, urine osmolality 100 mosm/L, urine sodium <20 mmol/L. What is the most likely cause of his hyponatraemia? A. Cortisol deficiency B. Low solute intake C. SIADH D. Volume depletion

B. 1. ANS: B Approach to Hyponatraemia 1. Hypotonic? Yes, serum OSM low If OSM normal/high: a) Pseudohyponatremia (increased solid components lipid, protein) =isotonic a. Lab dilution artifact, not present with blood gas machine b) Trans-locational (glucose, other active osmolyte) =iso- or hypertonic 2. Urine osmolality >100? (if yes, ADH activity is present) NO, uOSM <100 = ADH switched off, consider: a) Primary polydipsia b) Low solute intake/beer potomania Urine amount limited by low solute intake* c) Transient ADH stimulus (e.g. stress, thiazides) no longer present *Solute= NaCl and protein (urea); if NaCl intake is low, urine Na is low

Deltamed rheum In addition to glucocorticoids, monoclonal antibody treatment is now available for Giant Cell Arteritis that blocks which of the following cytokines? a) Interleukin-1 b) Interleukin-6 c) TNF-alpha d) Interleukin-17 e) Interleukin-23

B. IL6

Deltamed stats A randomised trial compared aspirin plus clopidogrel with aspirin alone given to over 15,000 patients with either established vascular disease or multiple risk factors. The two treatments were similar overall (event rate 6.8% vs 7.3%, p=0.22). The investigators performed 20 pre‐specified analyses in subgroups defined by differing baseline variables. The uncorrected p‐value for the comparison of the treatment effect between symptomatic and asymptomatic patients was 0.046. Which of the following most accurately summarises the subgroup finding? A. The subgroup effect is significant because the comparison was pre‐specified B. With 20 subgroup analyses, the chance of a p‐value under 0.05 is over 50% C. The subgroup effect is almost significant after correction for 20 subgroup analyses D. The subgroup effect suggests a benefit in symptomatic patients which should be tested in a further trial E. The subgroup effect would have been significant if the trial had been larger.

B. 5/100 studies you would expect a p value <0.05

DM PT G7 7. Significant aortic valve stenosis can be complicated by gastrointestinal bleeding from gastrointestinal angiodysplasia. Why does this occur? A. Use of antiplatelet agents B. Acquired type 2A von willebrand syndrome C. Increased expression of angiogenic factors D. Cardiac congestion causing venous obstruction of submucosal veins in the GI tract

B. AS and GI angiodysplasias (Heyde's syndrome) • Associated with acquired type 2A von Willebrand syndrome • Proposed due to proteolysis of VWF as it passes through the stenotic valve due to high shear forces • VWF abnormalities are common in severe AS (67-92%), correlate with severity of AS and can be corrected by valve replacement • Bleeding episodes reported in 21%, most commonly skin or mucosal • Angiodysplasias at high risk of bleeding since haemostasis in these lesions depends on VWF (probably depends on presence of susceptible lesions) • Chronic venous obstruction with dilated submucosal veins and increased expression of angiogenic factors implicated in pathogenesis of angiodysplasias

Deltamed renal A 45 year old, Chinese born man with liver failure due to HCV induced cirrhosis had a liver transplant in 2000. His eGFR is now 15ml/min. Which of the following statements is least likely to be true: a. Acute renal failure following his liver transplant indicated an increased risk of chronic renal failure (CRF). b. Asian recipients are more likely to develop CRF following liver transplantation c. Cyclosporin use may be a contributor to his CRF d. Renal transplantation should be considered and may improve his life expectancy e. HCV positivity placed him at increased risk of CRF

B. Asian + Male are protective for developing renal failure post-non-renal tx

Book R10 Which 1 of the following is a CI to use of TNFa inhibitor? A. previous rx TB B. septic arthritis 4m ago C. fully resected BCC 5yrs ago D. Breast ca 15 yrs ago E. presence of pyoderma gangrenosum

B. CI: prev untreated TB, recurrent chest infections, septic A within 12m, infected prosthesis, IDC, MS, malignancy within 10 yrs (except fully resected BCC 5yrs+), CHF, chronic cutaneous ulceration

CLS Haem Previously well 32 man p/w progressive lethargy. O/E: spleen 6 cm below costal margin. No jaundice or lymphadenopathy FBE was taken and is as shown: What is the most likely cytogenetic abnormality: A. hyperdiploidy. B. t(9;22). C. t(15;17) D. 5q minus E. Trisomy 21. Haemoglobin 125 g/dL 128-175 Total WCC 85.5x 10^9/L 4-8 Platelets 525 x 10^12/L 150-450 Blood film Increased white cell count with a left shift

B. CML. L shift, high platelets

Deltamed Onc Which of the following statements with regards to stereotactic radiosurgery to the brain is true? A. Treatment of multiple lesions in one session is not possible B. Treatment of surgically inaccessible (e.g. brain stem) lesions is possible C. Treatment of very large metastases (e.g. > 7 cm) is possible D. Treatment with high dose to target lesion does not allow sparing of normal tissue E. A and B

B. Can treat up to 10 lesions (surgery 3 or less)

R16 60 y/o man. Diagnosed with RA after several months. Sx prorgression, multuple Js. Rheumatoid nodules. RF + Concerned about his prognosis. Which of the following is assoc with a favourable course for pt with RA? A. insidious onset B. acute onset in few large Js C. age >40 D. positive RF E. constitutional sx

B. Favourable = acute onset restricted to few large joints of <1 yr, negative RF. Worse = <20 or >60yrs, >20 Joints involved, high RF titre, antiCCP, radiological erosion,

CLS ID Which antifungal is first-line therapy for an intensive care patient with Candida glabrata candidaemia? A. 5-Flucytosine. B. Caspofungin. C. Fluconazole. D. Griseofulvin. E. Itraconazole. F. Liposomal amphotericin. G. Terbinafine. H. Voriconazole

B. Indications - treatment of candidaemia in neutropaenic or unstable patients, resistant Candida

DM PT Neuro4 4. A 29-year-old woman presents to emergency with vertical diplopia. On examination, her left eye does not look downwards in adduction. Which left-sided extraocular muscle is most likely involved? A. Inferior oblique B. Inferior rectus C. Superior oblique D. Superior rectus

C

Deltamed obs A 32 year old lady with history of lupus nephritis presents 6 weeks pregnant in her first pregnancy.She has inactive disease with creatinine 85 mmol/L and 1gm protein/24 hours. Baseline meds hydroxychloroquine 400 mg daily and prednisolone 5 mg. Which of the following is incorrect? a. There is a higher risk of PreͲEclampsia, early delivery and growth restriction b. Hydroxychoroquine should bereduced or discontinued and prednisolone increased c. She has renal impairment d. 150 mg Aspirin should be started in the 1st trimester e. Ro ab and APL screen and baselines activity profile should be sent in early pregnancy

B. a Increase in lupus population, addition renal impairment and proteinuria b - Incorrect - benefits of continuing hydroxychloroquine decrease flares c - Creatinine abnormal for pregnancy d- To prevent PreͲEclampsia e- Baseline

Book R3 Which 1 of the following is assoc with increased freq of HLA-DR1 allele? A. RA - for RF B. RA + for RF C. AS D. Narcolepsy E. Porphyria

B. Assoc with RF or antiCCP +ve Alleles contain a common AA motif in HLA-DRB1 shared epitope

Deltamed onc 60 y/o man. 3d post- IV gemcitabine for pancreatic cancer. SOB, cough. What is most likely? CT showing vague opacities a. lymhangitis carcinomatosis b. chemo pneumonitis c. neutropaenic sepsis d. APO e. mets

B. gemcitabine = pneumonitis too quick for the others/not right pattern carcinomatosis most common for adenos (cervix, colon, stomach, breast, prostate, thyroid, larynx, lung) (Certain Cancers Spread By Plugging Lymphatics)

DM PT Im4 4. A 53-year old lady presents for the first time with two nocturnal episodes of angioedema of the tongue and lips. She is on no regular medications except hormone replacement therapy and can identify no clear food triggers. Skin testing shows positive reactions to grass pollens, but staple foods were negative. Blood tests show normal C3, but markedly low C4. C1 Esterase level was 0.13 (normal >0.18). Gene testing for common mutations in C1 Esterase inhibitor was normal, and her three children had normal C3 and C4 levels. What is her condition most compatible with? A. Chronic spontaneous angioedema B. Acquired C1 Esterase inhibitor deficiency C. Oestrogen related angioedemea (Factor XII gain-of-function mutation) D. Grass pollen-related angioedema

B. Hereditary angio can be type I (low C1-inh) or II (dysfunctional protein, normal or increased C1-inh) Acquired C1-I deficiency in B cell lymphoprolif, AID or both. C1q levels reduced Chronic spontaenous angioema: not driven by allergen. May have autoimmune component. Rx with H1 blocker

Deltamed PT6 Janice is brought in as a stroke code, with new onset of left sided weakness. She appears to also have an expressive dysphasia but with her right arm is making a fist in front of her chest. You obtain an ECG which shows inferior ST elevation. What is your next step? A. CT brain + CT perfusion, then to cath lab B. CT brain + CT perfusion + aortogram C. CT brain + CT perfusion then thrombolyse D. CT brain + CT perfusion then dual antiplatelets and heparin.

B. The combination of 'chest pain and neurology' is concerning for a type A aortic dissection. This certainly needs to be excluded prior to putting any interventional catheters in the aorta, either in the context of cath lab or interventional radiology for clot retrieval. When coronaries dissect as part of a Type A aortic dissection, the right coronary artery is classically the artery involved, resulting here in the pattern of an inferior STEMI.

Deltamed renal Which of the following carries the highest risk for mortality in dialysis patients: A. Serum cholesterol 6.0 mmol/l B. Blood pressure 110/75 C. Serum homocysteine 18 mmol/l (N < 13) D. Body Mass Index (BMI) 36 E. Serum triglyceride 3.0 mmol/l

B. (U shaped curve)

CLS Resp Bronch lecture • What is true about the epidemiology of lung cancer in Australia? - The incidence in males is rising whilst the incidence in females has stablised It is the fourth most common cause of cancer but the leading cause of cancer related deat It accounts for the highest health expenditure in non- Caucasian Australians - Large cell cancer is more common than small cell cancer - Squamous cancer is the commonest histopathological subtype of lung cancer

B. 4th most common cause cancer, leading cause of cancer death

CLS Haem 68 man for routine surgery noted to have enlarged non-tender cervical lymph nodes. An Full blood examination was performed and the results are shown. With respect to immunophenotyping which of the following antigens is most-likely co-expressed with CD19 in this patient? A. CD3 B. CD5. C. CD8. D. CD10. E. CD34 Haemoglobin 125 g/dL 128-175 Total WCC 19.5 x 10^9/L 4-8 Neutrophils 2.5 x 10^9/L 2-4 Lymphocytes 16.2 x 10^9/L 2-4 Platelets 213 x 10^12/L 150-450 Blood film Increased lymphocytes with smear cells

B. CD5

Deltamed gastro A 44yo male with ulcerative colitis is currently treated with 5-ASA and 8 weekly infliximab therapy. They present to the hospital with worsening bloody diarrhoea up to 8x daily with mild abdominal and associated low grade fever. What is the most important diagnosis to exclude on endoscopy with histopathological analysis? A) Clostridium difficile colitis B) CMV colitis C) Crohn's Disease D) Ischaemic colitis

B. CMV colitis

Dunedin endo Question 3: A 71 year old woman with metastatic melanoma being treated with chemotherapy is admitted to hospital with weight loss, lethargy and orthostatic hypotension. She has taken thyroxine for Hashimotos thyroiditis for many years. MRI scan of the head shows homogeneous enlargement of the pituitary and pituitary stalk. Which of the following medications is the most likely cause of the pituitary abnormalities? A: Bevacizumab B: Ipilimumab C: Temozolamide D: Sunitinib E: Dacarbazine

B. CTLA4 > PD1 > PDL1

Deltamed renal 48. A 53-year-old woman with advanced CKD is found to have worsening anaemia. She has co-morbidities of gastro-oesophageal reflux, hypothyroidism and intermittent constipation. Her current medications include lisinopril 5 mg daily, atenolol 50 mg daily, pantoprazole 40 mg daily and thyroxine 100 mcg daily. Laboratory tests prior to her outpatient appointment are as follows: Normal values Haemoglobin 91 g/L [115-160] White cell count 5.1 × 109/L [4.0-11.0] Platelets 301 × 109/L [140-400] Iron 9 ȝmol/L [9-30] Transferrin 2.0 g/L [2.0-3.6] Transferrin saturation 9% [15-45] Ferritin 47 ȝg/L [10-200] C-reactive protein 3.1 mg/L [< 5.0] What is the best initial treatment for her anaemia? A. Erythropoietin. B. Intravenous iron. C. Oral iron. D. Packed red cell transfusion. E. Pentoxifylline.

B. IV iron

Deltamed renal Which glomerulopathy is the leading cause of the need for renal replacement therapy in Australia? A. Thin Membrane Disease B. IgA Nephropathy C. Minimal Change Disease D. Primary FSGS E. Polycystic Kidney Disease

B. IgA > presumed

Deltamed Immuno Which statement about immunoglobulins below is incorrect? A. IgA is the predominant immunoglobulin secreted into breast milk • B. IgM has low avidity but high affinity C. IgE does not bind complement effectively D. IgG provides passive immunity to the fetus by crossing the placenta at 30 weeks gestation E. IgE binds to mast cells to cause degranulation

B. IgM = high avidity, low affinity (1st one), then more effective ones come along

CLS Rheum A 56yo man presents with right knee swelling after a long walk. He has a history of obesity and psoriasis of 12 years. Right knee xray shows loss of cartilage in the medial compartment with chondrocalcinosis. Serum urate 0.55 (0.24 - 0.44) Synovial analysis Viscosity: normal, Clarity: normal WCC: 900mm3 (<200), Differential: 90% mononuclear Crystals : occasional extracellular urate crystals What is the cause of this man's right knee swelling? A. Pseudogout B. OA C. Gout D. Psoriatic arthritis E. Medial meniscal tear

B. OA

R 23 46 yo M. Painful, swollen L knee. Aspiration = CPP crystals. Xray of L nee is shown. Which additional test should be performed? A skeletal survey B. PTH C. LDH D. serum protein electrophoresis E. serum light chain

B. PTH CPPD Knees and wrists. Stress response to illness or surgery HH, Gitelman, hyperparathyroid, hypothyroid, low mag, low P, X-linked hypoP rickets, familial hypocalciuric hypercalcaemia

Dun Neuro 25 y/o man dislocates R shoulder playing footy. Reduced in ED. after sling removed = can't abduct arm. Weak abduction, sensory loss lat upper arm. Nerve involved is A. accessory B. axillary C. musculocutaneous nerve D. radial nerve E. suprascapular nerve

B. axillary nerve

Deltamed Onc After three months of erlotinib the patient has reduction in size of lung and brain metastases, with sclerosis of bony metastases. Erlotinib is continued for 18 further months, at which time CT scans show increasing pulmonary metastases. What is your next recommended step in management of this patient? A. Platinum-pemetrexed chemotherapy B. CT guided biopsy lung metastasis C. Osimertinib D. Pembrolizumab

B. bx to prove T790M mutation If +ve = osimert If -ve = chemo Pembro = less effective if driver mutation

Deltamed renal Which of the following is most likely to increase serum phosphate in chronic renal failure? A. Calcium trisilicate. B. Calcitriol. C. Cinacalcet. D. Calcitonin. E. Alendronate.

B. calcitriol

Dunedin pharm The plasma conc of a drug at steady state is mostly determined by the drugs A. Vd B Cl C. elimination rate constant D. extent of protein binding E. elim half life

B. clearance

CLS Rheum Diabetic neuropathic arthropathy is most likely to affect which of the following areas? A. Knee B. Mid-foot C. 1st metatarsophalangeal joint D. Elbow E. Ankle

B. midfoot May occur with or without neuropathic ulcers - typically in 5th and 6th decades - often relatively 'normal vascular exam' - neuropathic ulcerations are common Precipitating factors: - peripheral neuropathy - unrecognized injury - continued repetitive stress on injured structures - increased local blood flow Pathoanatomy: - charcot joint can result in fragmentation of periarticular areas & subluxations - usually affects midtarsal joints can lead to prolapse of the arch or valgus deviation of the forefoot

Deltamed Onc 50 y/o M with chronic liver disease sec to HepB. Prev endoscopy has revealed large oesophagela varices. alb 32, PT 14 (high), plt 70. AFP 300. CT 1 liver lesion. Optimal rx? A. Chemoembo B. liver tx C. cryo D. local resection E. RFA

B. optimal can't do cryo or resect as bad liver

Dunedin endo Question 1: A 32 year old woman presents 2/12 postpartum with severe headache and bitemporal hemianopia. A pituitary mass lesion is seen on MRI scan. Endocrine testing reveals secondary hypoadrenalism but other pituitary function is normal. She has a past history of Hashimotos thyroiditis. The most likely diagnosis is: A. Pituitary macroadenoma B. Lymphocytic hypophysitis C. Pituitary apoplexy D. Metastasis to the pituitary E. Pituitary infarction

B. postpartum. affects ACTH predominantly

Dunedin gastro A 75 year man complains of some years of acid reflux symptoms, bloating and weight loss. He had surgery for a perforated duodenal ulcer years ago. Known hypertension on cilazapril. Examination is unremarkable besides a laparotomy scar. Blood test show a macrocytosis. Which investigation is most likely to determine the underlying cause for these symptoms? a. Oesophageal manometry and pH studies b. Hydrogen breath test c. Upper GI endoscopy d. CT abdomen e. Colonoscopy

B. small bowel overgrowth

Deltamed neuro A 47 yo woman with Hx of alcoholism. Acute illness with a nadir of 4 weeks - Leg > arm numbness, tingling, burning - Leg > arm weakness - Gait instability and falls One month prior: - Nausea, vomiting and diarrhoea - Last 2 -3 weeks severe with minimal po intake other than fluids - Marked weight loss (15 kg from 84 to 69 kg). Examination: - Requires a walker. - distal symmetric weakness in hands and feet - areflexia in the lower extremities - absent pinprick toes to knees - absent vibration and proprioception at toes, ankles and fingers - marked sensory limb and gait ataxia. What is the most likely diagnosis? Axonal variant GBS: Acute motor sensory axonal neuropathy (AMSAN) B. Beriberi C. Porphyria D. Sjogren's syndrome

Beriberi (B1) Sensorimotor most common Nutritional hx, axonal physiology, typically normal CSF, low B1

CLS Haem 59 year old mother of three has no personal history of venous thrombosis or PE but is investigated for hypercoagulability because • one of her children developed a suspected PE at the age of 39 years after a fall from his bicycle • another had a spontaneous PE at age of 35 • Third child is well, 33 years. • Her results are shown. What is the best advice for this woman? Antithrombin III 102% 80-120% Protein C 122% 70-140% Free Protein S 27% 60-124% A. Life-long anticoagulation with direct anti-thrombin inhibitor B. Reassurance that she is not at increased risk C. Venous thrombosis prophylaxis restricted to periods of "high risk" D. Life long aspirin treatment E. Life long warfarin treatment with the target range of INR 1.5-2.0

C

CLS Haem A 25 year old man presents with painful swelling in his right leg. The following investigation results are obtained: The results indicate which one of the following? APTT 45sec 26-35 sec PT 14sec 12-14 sec Thrombin clotting time 18sec 16-20 sec Fibrinogen 2.5g/L 1.8-4.0 g/L APTT post mix 40sec 26-35 sec A. Activated protein C resistance B. Factor XII deficiency C. Lupus anticoagulant D. Von Willebrands disease E. Protein C deficiency

C

CLS ID A 23 year old presents to hospital 3 weeks after a one month holiday in Kenya and Tanzania. He has a 5 day history of fever, with headaches and diarrhoea. On examination his Glasgow coma score is 15/15, he has a temperature of 39.0 C, his pulse rate is 110 beats per minute, his blood pressure is 108/68mmHg, and his respiratory rate is 22 breaths/minute. Examination is otherwise non-contributory. Blood tests reveal a haemoglobin of 94g/L, and platelets of 88 x109 /ml. A thin film identifies parasites consistent with Plasmodium falciparum with a parasite density of 15%. The most appropriate initial treatment is: A. Chloroquine (oral) B. Mefloquine (oral) C. Artesunate (intravenous) D. Quinine + doxycycline (oral) E. Artemether-lumafamtrine (PO)

C

CLS ID The most common cause of rx failure in pulmonary TB is A. coexisting cOPD B. cavitations C. poor adherence D. HIV coinfection E. isoniazid resistance

C

CLS Rheum A 75 year old lady presents with unilateral temporal headache. Which one of the following is the most specific for temporal arteritis? A. Tenderness over the temporal area on palpation B. Hip and shoulder tenderness C. Jaw (masseter) claudication D. Fever E. Blurring vision F. Elevated ESR G. Proximal weakness

C

CLS Rheum Arthritis in which of the following joints is most suggestive of haemachromatosis? A. Knee B. Ankle C. Distal interphalangeal D. Metacarpophalangeal E. Metatarsophalangeal

C

CLS Tumour marker Which of the following is the most appropriate tumour marker for F/U of a person with medullary thyroid cancer? A. Thyroglobulin B. CEA C. Calcitonin D. Thyroid binding globulin E. Chromogranin

C

DM ID A 38 year old male presents to ED 3 days after returning from a trip to the Haj. He has fevers, chills, myalgias, cough, dyspnoea and diarrhoea. Which of the following is correct? • A. Unless he's had direct contact with camels, it is unnecessary to test for MERS-CoV • B. Given the very limited secondary human - human spread of MERS, isolation is not required • C. Viral cultures are the appropriate test for MERS • D. If he becomes severely ill, treatment with interferon- alpha-2b plus ribavirin could be considered • E. As MERS is the likely diagnosis, empiric therapy with oseltamivir is not indicated

C

DM ID Q1. A strain of Clostridium difficile with increased virulence is being reported overseas and now in Australia. This is called ribotype 027. • Which of the following is correct regarding this new strain? • A. The most likely risk factor is recent use of clindamycin • B. This strain remains susceptible to fluoroquinolone antibiotics • C. This strain produces substantially larger quantities of toxins A and B in vitro than other C. difficile strains • D. It has deletion of the usual binary toxin gene • E. Diarrhoea is usually self-limiting in younger, healthier patients

C

DM ID Regarding oseltamivir, which is the most correct? • A - It is only active against influenza A viruses • B - It has been shown to reduce the average duration of symptoms of influenza illness by 3 - 4 days • C - Used prophylactically, it has been shown to reduce the number of cases amongst close contacts of people with influenza • D - During last year䇻s influenza pandemic, oseltamivir was the most important factor in reducing severity • E - It cannot be used safely in pregnancy or during breast feeding

C

DM PT Im10 10. A 25-year-old woman presents with an unprovoked DVT. She is treated with low molecular weight heparin, but on day two of admission develops chest pain and dyspnoea. CTPA confirms pulmonary embolus. Procoagulant testing showed a strongly positive anticardiolipin IgG, negative anticardiolipin IgM, positive beta-2glycoprotein-I IgG, and positive lupus anticoagulant. Assuming these antibodies persist, what is the optimal therapeutic approach long-term? A. warfarin for six months, then no treatment B. warfarin for six months, then aspirin C. warfarin lifelong D. apixaban lifelong

C

DM PT Im2 2. A 77-year-old right-handed woman presents with a 2-year history of slowly progressive, painful swelling of her fingertips. What is the autoantibody associated with this condition? A. Sm B. CCP C. Centromere D. SSA

C

DM PT Im9 9. A tri-specific monoclonal antibody is engineered to treat myeloma, by binding CD38 on myeloma cells, CD3 on T cells, and CD28. Why is the CD28 component targeted? A. to strengthen the binding between the myeloma cells and the T-cell B. to directly target cytotoxic T cells C. to co-stimulate effector T cells D. to inhibit regulatory T cells

C

Deltamed ID 22 yo F on a student visa from Nepal presented three months ago with several months' history of cervical lymphadenopathy, fevers and weight loss. Node biopsy showed necrotising granulomata and cultured fully susceptible M. tuberculosis. She was started on standard 4 drug therapy with HREZ and then after 2 months continued on HR. During that time, she felt better and nodes got smaller. She now presents with acutely painful, swollen & fluctuant nodes, plus fevers and anorexia. Best course of action? • A - Add in moxifloxacin for drug-resistant resistant TB • B - Stop all therapy and arrange for node biopsy • C - Continue the therapy and perform I+D • D - Give infliximab • E - Go back to HREZ and make it DOT

C

Deltamed ID Significant risks of reactivation of latent TB include all the following except: • A. HIV infection • B. Cancer chemotherapy • C. Alcohol abuse • D. Corticosteroid use • E. Anti-TNF therapy • F. Diabetes mellitus • G. End stage renal failure • H. Cigarette smoking

C

Deltamed Onc Which of the below treatments carries a risk of neutropenia? A. Fulvestrant B. Letrozole C. Palbociclib D. Tamoxifen

C

Deltamed Onc Staging investigations reveal this patient has locally advanced disease (SCLC lung), without distant metastases. What treatment will you recommend for this patient? A. Platinum-etoposide B. Platinum-etoposide + radiotherapy C. Platinum-etoposide + radiotherapy + prophylactic cranial irradiation D. Best supportive care

C

Deltamed endocrine 65 y.o with paroxysmal atrial tachycardia. treated with amiodarone for 5yrs presents with worsening palps. High T4, high-ish T3, low TSH. Commenced on carbimazole and 3w later tests are worsened. What is the most likely cause? A. Graves B. type I (iodine induced) amiodarone induced thyrotoxicosis C. type II (inflammatory) D. subacute viral thyroiditis E. toxic multinodular goitre

C

Deltamed endocrine Which of the following is NOT a screening test for Cushings syndrome? A 24hr urinary free cortisol B. 1mg overnight DST C. ACTH D. midnight salivary cortisol

C

Deltamed onc At initial dx which factor is most predictive of survival in met RCC A. age <60 B. chromophobe hx C. ECOG 3 D. lung mets

C

Deltamed onc What is the most important prognostic factor in early breast cancer A Tumour Grade B. Tumour Size C. Axillary node involvement D. Oestrogen receptor (ER) status E. Her2 status

C

Deltamed rheum 68 y/o man w/ severe RA (on adalimumab, MTX, prednisone) presents with 3d fevers + rigors. Most likely organism is A. pneumocystis B. aspergillus C. pneumococcus D. strongyloides E. pseudomonas

C

Deltamed stats A trial comparing two antiplatelet treatments for the prevention of vascular events randomises 11,000 patients into two equal groups, follow‐up for up to six years. Event rates are 8.6% for drug A and 8.3% for drug B. The p‐value for the comparison is 0.051. Which statement most accurately describes the interpretation of the p‐value? A. The result is clinically and statistically significant B. There is a trend to a positive result C. In a trial of this size, such a p‐value is not significant D. The result is statistically significant but not clinically significant E. The result is clinically but not statistically significant

C

Deltamed stats In clinical research which of the following best describes the p‐value? The probability A. that the hypothesis is incorrect B. that the hypothesis is correct C. of observing a result at least this magnitude if the null hypothesis is correct D. of observing a result at least this magnitude due to chance E. that the hypothesis is due to chance

C

Dun cardio A 56-year-old man is referred for assessment of resistant hypertension. Blood pressure readings at his general practitioner's surgery have been between 145 and 172 mmHg systolic and 86 and 98 mmHg diastolic despite various combinations of up to two anti-hypertensive agents at a time. He has no other vascular risk factors. He has no family history of hypertension. Examination does not reveal any clinical findings suggesting a secondary cause of hypertension. He has normal creatinine and electrolytes. An ECG and trans thoracic echocardiogram are normal What is the most appropriate next investigation? A. Aldosterone : renin ratio. B. Magnetic resonance angiography of renal arteries. C. Ambulatory blood pressure recording. D. Renal ultrasound. E. Urine testing for drug compliance.

C

Dun cardio In heart failure with reduced systolic function the largest reduction in mortality is seen with: A. Angiotensin receptor blocker B. ACE inhibitor C. Beta blocker D. Mineralocorticoid receptor antagonist E. Alpha blocker

C

Dun cardio Which of the following maternal conditions is associated with the greatest risk of cardiac complications during pregnancy? A. Atrial septal defect. B. Ventricular septal defect. C. Eisenmenger syndrome. D. Patent ductus arteriosus. E. Mitral valve prolapse.

C

Dun stroke A 30‐year‐old woman is seen in the hospital for worsening headaches and visual changes over the past week. She also notes nausea and transient visual loss when she bears down. She gave birth to a daughter 3 weeks ago following a full‐term pregnancy. On examination, she appears mildly uncomfortable. She has a left superior quadrantanopsia and slight bilateral papilledema. MRI is shown. Which of the following is the most appropriate next step in management? A. Acetazolamide B. Decompressive hemicraniectomy C. IV heparin D. Thrombectomy E. Optic nerve sheath fenestration

C

Dun stroke A 35 year old otherwise healthy woman presents with the "worst headache" of her life. Neurological examination is within normal limits. Blood pressure is 120/70 mm Hg. Head CT is performed in the emergency room and shown. Laboratory studies include a normal CBC and comprehensive metabolic profile and a urine toxicology panel positive for cocaine. Lumbar puncture is performed and shows a mildly elevated WBC at 10, and a normal protein, no red blood cells or xanthrochromia is noted. Imaging is shown. Which of the following is the most likely etiology for the patient's presentation? A. Hemolysis, elevated liver enzymes, and low platelets (HELLP) B. Posterior reversible leukoencephalopathy syndrome (PRES) C. Reversible cerebral vasoconstriction syndrome (RCVS) D. Aneurysmal subarachnoid hemorrhage (aSAH) E. Cerebral amyloid angiopathy (CAA)

C

Dun stroke Using the CHADS‐VASc scoring system for stroke risk with Atrial Fibrillation, which of the following patients would have the HIGHEST estimated annual stroke risk? In answering this question, assume that there are no other co‐morbidities except those listed. a. 64 year old lady with hypertension and diabetes mellitus b. 82 year old man with hypertension c. 76 year old lady with diabetes mellitus d. 68 year old man with a previous stroke e. 67 year old lady with congestive heart failure

C

Dunedin Resp A 33 year old man, HIV positive with CD4 count 450 cells/µl (N 650-2000), presents with a 4 day history of fever, cough with brown sputum production and increasing breathlessness. He also reports seeing flecks of blood in the sputum. Chest x-ray is shown (R lung opacification). Which of the following should be undertaken next? A. Fibreoptic bronchoscopy. B. Echocardiogram. C. Commence cephalosporin and macrolide antibiotics. D. Commence anti-tuberculous medications.

C

Dunedin Resp An 82 year old man with rheumatoid arthritis is admitted with a 7 day history of fever, malaise and breathlessness. He also reports cough with purulent sputum and occasional haemoptysis. Clinical examination is consistent with a large left sided pleural effusion, confirmed on chest x-ray. Which of the following is the strongest indication for intercostal tube drainage? A. CT scan of chest showing visceral pleural enhancement. B. Pleural fluid protein 48 g/l. C. Pleural fluid pH 7.16. D. Persistent fever despite high dose intravenous cefotaxime.

C

Dunedin Resp In the assessment of an incidental solitary pulmonary nodule detected on CT chest which of the following is least predictive of malignancy? A. Spiculation. B. History of extra-pulmonary cancer. C. Absence of calcification. D. Upper lobe location.

C

Dunedin Resp Which one of the following statements regarding chronic cough is false? A. Gastro-oesophageal reflux disease is a common cause of chronic cough. B. Upper airway cough syndrome is related to inflammation and secretions in the nose and paranasal sinuses. C. Chronic cough is a rare feature of non-asthmatic eosinophilic bronchitis. D. Cough secondary to ACE inhibitor usually resolves within four weeks of ceasing treatment. E. In asthma, cough is the most refractory symptom to treatment with inhaled corticosteroid.

C

Dunedin Resp Dr Wong session Quesion 12 A 46 year old Maori woman presents to the emergency department with persistent non-productive cough for several weeks associated with breathlessness and lethargy. She is a current smoker with 40 pack-year history. Chest x-ray is shown (L veil sign indicating collapse). Which one of the following statements is your next step? A. Commence intravenous cephalosporin plus macrolide. B. Isolate and commence empiric anti-tuberculous medications. C. Request CT scan of chest. D. Commence high dose oral steroid.

C

Dunedin endo A 57 year old woman with acromegaly still has elevations of GH and IGF-1 after transphenoidal surgery. As a complication of her acromegaly she has difficult to control diabetes. Which of the following adjunctive therapies is most likely to worsen her diabetes? A: Octreotide LAR B: Lanreotide Depot C: Pegvisomant D: Cabergoline E: Pasireotide

C

Dunedin endo A patient with Type 1 diabetes is being treated with the following therapy: Glargine 20 units at 6pm Novorapid 10 units with each meal You are asked to calculate his insulin sensitivity factor. Which of the following figures is the best estimate? A: 1 B: 1.5 C: 2 D: 2.5 E: 3

C

Dunedin endo Monogenic diabetes (MODY) a) Is another term for type 2 diabetes b) Is usually a manifestation of insulin resistance c) Diagnosis requires: a family member under 25 yrs with DM, non insulin dependence demonstrated (C peptide measurable >5 yrs after diagnosis) and at least 2 consecutive generations of diabetes d) Genetic testing is available for all types of MODY e) Is not associated with normal insulin sensitivity

C

Dunedin endo Prevention of Type 2 diabetes: A. Is impossible. B. Cannot be achieved with lifestyle advice only. C. can be achieved with intensive, sustained lifestyle changes in 50% of patients. D. Is best achieved with Metformin alone in 50 % of patients E. Can be achieved for all patients with intensive sustained lifestyle changes.

C

Dunedin endo Question 3: A 40 year old woman is found to have a serum calcium that is mildly but consistently elevated between 2.6-2.7 mmol/L. Her PTH ranges from 67-78 pg/ml (10-65). Her vitamin D level is 82 nmol/L. A urine collection reveals a calcium creatinine ratio of 0.01. She feels well and has no history of renal calculi or bone pain. Which of the following is the next best step in the management of this patient? A. Referral to a surgeon for neck exploration B. DEXA scanning C. Genetic testing for a CASR mutation D. Sestamibi scan E. Continued annual monitoring

C

Dunedin endo Question 4: A 68 year old woman has had several calcium measurements in the range of 2.7-2.8 mmol/L. She feels well. Laboratory results: Albumin normal Estimated GFR = 40 ml/min Serum PTH 110pg/ml (10-65) Urinary calcium 4.7 mmol/day (2.5-7.5) 25 hydroxyvitamin D 74 nmol/L(62-199) Which of the following is the next best step? A. Measure 1,25 dihydroxyvitamin D B. Parathyroidectomy with identification of all 4 glands C. Sestamibi scan D. Renal ultrasonagraphy E. Repeated calcium measurement in 6 months

C

Dunedin endo Question 8: A 31-year-old woman presents with an absence of menstrual periods since stopping the oral contraceptive pill five months ago. She has the following investigations: Prolactin 2566 mIU/L [60 - 550 mIU/L] LH <1.0 IU/L [3.0 - 12.0 IU/L] FSH <1.0 IU/L [2.0 - 10.0 IU/L] Oestrogen 35 821 pmol/L [100 - 2400 pmol/L] Testosterone 3.1 nmol/L [0.5 - 2.5 nmol/L] Which of the following is the most likely explanation? A. Prolactinoma B. Polycystic ovarian syndrome (PCOS) C. Pregnancy D. Congenital adrenal hyperplasia E. Hypothalamic amenorrhoea

C

Dunedin endo WHICH OF THE FOLLOWING IS LEAST TRUE OF GRAVES EYE DISEASE? A. It is characterized by swelling of the extraocular muscle and adipose tissue B. Its activity runs in parallel to the thyroid state of the patient C. Its severity is not related to the titre of the TSI D. It is worsened by smoking E. It can be affected by radioactive iodine

C

Dunedin endo the HBA1c A. Ensures all cases of IGT are identified B. Is a good measure of Impaired fasting glucose levels C. Is a useful screening test to identify patients with Diabetes D. Diabetes is diagnosed with an HBA1c>7% (53mmol/mol) E. Is ideal when monitering diabetes control, especially if there is concern regarding hypoglycaemia.

C

Dunedin gastro 78 yr lady. History HPT, NSTEMI 2012 (no stent), CVA 2006, recurrent TIAs. Medications: aspirin, ibuprofen, metoprolol, cilazapril, citalopram. Presents with several days abdominal pain, and now 24hrs hematemesis and melaena. Haemodynamically stable, Hb 75, urea 24.3 Cr 100 (e-GFR 67) PR melaena. Which one of these choices represents the most appropriate initial management? a. Continue NSAID, stop aspirin, IV PPI, next available endoscopy list. b. Change ibuprofen to Cox-II, stop aspirin, IV PPI, next available list c. Stop NSAID, and aspirin, PO PPI, next available endoscopy list d. Stop NSAID, continue aspirin, PO PPI, if stable discharge for OPD endoscopy e. Stop NSAID and aspirin, PPI infusion for 72 hours as there is a mortality advantage, next available list

C

Dunedin gastro A 45 year old man has had vague abdominal pain and nausea for several years unrelieved by antacid medications. He has had no difficulty swallowing and no heart burn following meals . Upper GI endoscopy reveals antral mucosal erythema, but no ulcerations or masses. Several biopsies are taken, and microscopically there is a chronic non-specific gastritis. Which of the following conditions is most likely to be present: a. Gastrinoma (Zollinger-Ellison syndrome) b. Pernicious anaemia c. Helicobacter pylori infection d. Adenocarcinoma e. Crohn's disease

C

Dunedin gastro With respect to coeliac disease the best non invasive test is: a. Anti-endomysial IgA b. Anti-DGP (deamidated gliadin peptide) IgG c. Anti-TTG (tissue transglutaminase) IgA d. Anti-parietal IgA e. HLA DQ2/DQ8

C

deltamed gastro A 30 year old female with Crohn's colitis develops her first perianal abscess which is 3 cm in size. The best treatment strategy is - A) Surgical drainage only B) Surgical drainage then medical therapy with azathioprine C) Surgical drainage then medical therapy with combination infliximab / azathioprine therapy and antibiotics D) Medical therapy only with combination infliximab / azathioprine therapy and antibiotics

C

Deltamed stats Which of the following statements is true? "The standard error of the mean of a sample A.measures the variability of the observations B.is the accuracy with which each observation is measured C.is a measure of how far the sample mean is likely to be from the population mean D.increases with increasing sample size E. is greater than the estimated standard deviation of the population

C (SD = variability)

Dun Neuro A 44-year-old man presents with a tonic-clonic seizure. He gives a six-month history of episodes of confusion beginning with a sensation of déjà vu, followed by loss of awareness. His electroencephalogram (EEG) is shown below (temp spikes).The most likely diagnosis is: A. late-onset primary generalised epilepsy B. frontal lobe epilepsy C. temporal lobe epilepsy secondary to a cavemoma of the temporal lobe. D. temporal lobe epilepsy secondary to an acute abscess of the temporal lobe. E. non-epileptic attacks.

C (too long hx for abscess)

Deltamed endocrine 40 y/o with low libido, central wt gain, fatigue. Low tesosterone, normal FSH and LH What is the LEAST useful Ix A. Iron studies B. prolactin C. karyotype D. MRI pituitary

C 2* cause cos inappropriately low FSH/LH (should be high). Ie. tumour, iron overload, Kallmann, high PRL, opioids, obesity, excess exercise Klinefelters = very high cos testes failure

Deltamed Onc 42 y/o banker concerned about FHx of bowel ca. 1 pat uncle had bowel ca 48yrs. Father aged 79. Nil bowel sx Which of the following recommendations is most appropriate for this man? A. Asymp, ave risk. FOBT as screening B. Slightly above ave risk. Colonoscopy age 50 C. Slightly above ave, colo now D. mod increasd risk. FOBT + flexi sig E. mod. CEA

C 5 yrs before relative. FOBT = nil role in individual

Dun Neuro Which of the following anticonvulsant drugs is most likely to cause weight gain in young women? A. Topiramate B. Lamotrigine C. Sodium valproate D. Carbamazepine E. Phenytoin

C A = wt loss

Dunedin pharm A loading dose at the start of a constant rate infusion: A. increases the steady state drug concentration B. gets to steady state more quickly C. starts closer to the steady state drug concentration D. allows a lower infusion rate E. always achieves a steady state immediately

C A+ D depend on clearance B depends on half lives

R 24 25 y/o F. Admitted to ICU for ARDS and multiple cerebral microinfarcts. Hx of recurrent miscarriges. Nil VTE. Creat 250. Blood +++ on UA. Which 1 of the following Ix should be undertaken in this pt? A. protein C and ATIII level B. protein S level + ATIII level C. anti-cardiolipin D. antiGBM E. complement levels

C APLS. Lupus anticoag more specific, anticardiolipin (n = more sensitive) Heart valve disease, livedo reticularis, low plt, nephropathy, neuro mangifestations, Catastrophic APLS - 3 diff organ systems over period of days/weeks. Normally smaller vessels (kidney, lungs, CNS)

Book D4 50 y/o construction worker. Several rough irregular lesions on face, scalp, backs of hands. Hyperkeratotic with small rim of surrounding erythema, Not painful or itchy. Appeared over years. Which is true? A. Pts risk of developing cutaneous malignancy in relation to lesions <1% B. lesions are precursors to melanoma and should be removed C. Rx by cryo or cutterage has been found to be effective in prev progression to cancer D. themost impt RF in development of these lesions is FHx E. small SqCC in sunexposed areas are more likely to metastasize

C Actinic keratosis. Precursor for SqCC (other precursor = Bowens) 1-10% over 10 yrs (more in >5 lesions) Small SqCCs arising from AK are less likely to metastasize than those that are poorly diff or appear in non-exposed areas. General population: BCC > SqCC Transplant = opposites

CLS Rheum 38 year old woman presents with a six-week history of increasing shortness of breath while playing tennis and a two-week history of increasing muscle weakness. She reports mild symmetrical polyarthralgia of the hands, wrists & knees for six months and Raynaud's phenomenon during the recent winter. Examination: proximal muscle weakness, a hyperkeratotic rash with fissuring on the palmar and lateral surfaces of the fingers on both hands. Investigations: serum creatine kinase level of 890 (16-139), an interstitial pulmonary infiltrate on HRCT of the chest. Autoantibody tests show the following results: ANA titre 1:2560, speckled; anti-ENA Jo-1 positive, anti-DNA Ab negative. The most likely diagnosis is: A. Progressive systemic sclerosis B. Dermatomyositis C. Polymyositis (anti-synthetase syndrome) D. SLE E. Sjogren's syndrome

C Antisynthetase syndrome I • Rare, chronic autoimmune disease of unknown etiology • Syndrome considered as a subgroup of the idiopathic inflammatory muscle diseases • Hallmark is presence of serum autoantibodies (recognize the aminoacyl-tRNA synthetases, a family of intracytoplasmic enzymes which play a vital role in protein synthesis= anti-ENA Jo-1).

Dun cardio A 65-year-old male has chest pain when he walks up hills. His pattern of angina has been unchanged over the last five years. He is taking optimal medical therapy and follows a healthy lifestyle. He asks about his prognosis with angioplasty and stenting. Compared with optimal medical therapy, stenting would be most likely to reduce: A. his risk of myocardial infarction. B. his risk of cardiac death. C. the frequency of angina. D. the need for subsequent coronary artery bypass grafting. E. the risk of arrhythmia.

C As an initial management strategy in patients with stable coronary artery disease, PCI did not reduce the risk of death, myocardial infarction, or other major cardiovascular events when added to optimal medical therapy

Dun ID A 28 year-old physio therapist is going to spend three months in Uganda with her partner for a volunteer work at a local NGO. Her only medical problem is epilepsy, for which she takes levetiracetam. She is not on contraception and wants to conceive. Which of the following is the most appropriate option for her malaria chemoprophylaxis? A. Mefloquine B. Chloroquine C. Atovaquone/Proguanil D. Doxycycline

C Atovaquone + proguanil: Compatible with pregnancy, begin 1-2 days before and continue 1 wk after, can be used for treatment. Mefloquine: Compatible with pregnancy, begin 2-3 wks before and continue 4 wks after, can be used for treatment, contraindicated for seizure and psychiatric disorders. • Chloroquine: Only works in limited areas.

Dunedin Resp Regarding COPD exacerbation, which statement is true? A. A dose of prednisone 40mg for 7-14 days is recommended. B. Eosinophilia is associated with a worse prognosis. C. Atrial fibrillation is associated with a worse prognosis. D. Antibiotics should be given routinely for all acute exacerbations of COPD. E. Nebulised short-acting bronchodilators should be given in preference to short-acting bronchodilators via metered-dose inhaler with spacer.

C DECAF score

DM PT Rh10 10. A patient presents to the rheumatology clinic with the rash shown below and joint pain. Which of the following conditions is not associated with this rash? A. Behcet's Disease B. Inflammatory bowel disease C. Rheumatoid Arthritis D. Sarcoidosis

C Idiopathic > infection > sarcoid > drugs > pregn > enteropathies

Dunedin Haem Which of the following statements is true? A. The pathological target of antiphospholipid antibodies is cardiolipin B. 80% correction in a 1:1 coag mixing study is suggestive of an antiphospholipid antibody C. Correction of an APTT following addition of excess phospholipid is suggestive of a lupus anticoagulant D. Lupus anticoagulants are associated with bleeding E. Thrombocytopenia is one of the diagnostic criteria for APS

C If add phospholipid = swamp out AB = corrects

Deltamed Haem2 A 77‐year‐old female is brought in by ambulance with decreased conscious state. Her background includes atrial fibrillation for many years on beta blockade for rate control. Her stroke prevention anticoagulation was initially warfarin, though she switched to dabigatran (a direct thrombin inhibitor) 7 years ago, without previous complications. Initial investigations are below‐ H aemoglobin 114g/L (115‐150) Platelet 178x109/L (150‐400) INR 1.6 (0.9‐1.2) APTT 50sec (24‐35) Fibrinogen 3.2g/L (2‐4) CT Head confirms a large intracerebral bleed with mass effect. What is the most appropriate next management? A. Human prothrombin complex concentrate (Prothrombinex‐VF) B. Dialysis C. Idarucizumab D. FFP

C If on Xa could consider A

CLS Resp Which of the following treatment options are potentially harmful in a patient diagnosed with idiopathic pulmonary fibrosis? A. Colchicine B. Pirfenidone C. Ambrisentan D. Oxygen therapy E. Pulmonary rehabilitation

C In a RCT, Ambrisentan was shown to increase the risk of hospitalization and disease progression. There was also a trend towards increased mortality. Pirfenidone (rash, GI) and Nintendanib (diarrhea) are both PBS listed • Both are not curative but do appear to slow disease progression • No other therapies have been shown to alter the natural history of IPF Diagnosis must be confirmed by MDT - tissue diagnosis is not required if radiological features are definite or probable IPF (=UIP pattern) + combined with appropriate clinical features (or lack of CTD, occupational or other exposures) Worth remembering the UIP pattern features on HRCT: • Subpleural and basal predominance • Reticular abnormality • Honeycombing (with or without traction bronchiectasis) • Absence of features listed as inconsistent with UIP (mid or upper zone, peribronchovascular changes, extensive ground glass opacity, profuse micronodules, cysts, mosaic attenuation, consolidation)

Re: Lambert Eaton myasthenic syndrome: A. Decrement in compound muscle action potentials amplitude is seen with 50Hz repetitive stimulation B. The majority of cases are not paraneoplastic C. Antibodies to presynaptic ion channels are present in most cases - D. Single fibre EMG can distinguish this syndrome from myasthenia gravis

C Increased jitter in both LEMS and MG Small initial CMAP

DM PT Gen8 A 28-year-old female is found to have the following karyotype: Which of the following statements best describes the likely reason for having ordered the karyotype in this woman? A. Family history of cancer predisposition. B. Intellectual impairment and seizures. C. Multiple miscarriages. D. Short stature and amenorrhoea

C Karyotype showing translocation This is a balanced reciprocal translocation between chromosomes 5 and 13 = t(5;13). No missing chromosomal material, so unlikely to have any health impact on the woman herself. Will be able to conceive, but increased risk of conception with unbalanced chromosomes à higher risk of miscarriage. Chromosome Translocations • Frequency 1 in 500 (not uncommon) • 'Balanced' or 'apparently balanced' if no genetic material is lost ('carrier) • Consequences - "difficulty conceiving" - azo/oligospermia - miscarriages - liveborn with unbalanced amount of chromosome information • NB: balanced translocations are not detected by chromosome microarray

Deltamed Onc Which of the following is not a risk factor for chemotherapy induced emesis A. Female gender B. History of pregnancy induced nausea C. History of high alcohol intake D. Age < 50 E. Previous chemotherapy exposure

C Low risk; dexamethasone &/or dopamine antagonist (eg metoclopramide) Moderate risk; dex + 5HT-3 antagonist (eg ondansetron, palonosetron) • High risk; dex + 5HT-3 + aprepitant (NK-1 inhibitor) + olanzapine • Randomised placebo controlled trial shows reduced acute and delayed nausea with addition of olanzapine 10mg daily for 4 days

DM PT Im8 8. A 25-year-old woman presents to casualty in anaphylaxis following a bee-sting. Tryptase level taken at presentation was 38 (normal less than 11.4). Four weeks later, on review in the immunology clinic, tryptase was documented to be 25. What is the next appropriate investigation? A. 24-hour urinary HIAAs B. 24-hour urinary histamine C. Bone marrow biopsy D. Serum histamine level

C Mastocytosis Multiple and various presentations with flushing, hives, urticardia,a ngioedema, SOB, diarrhoea. Idiopathic anaphylaxis or after insects Raised serum tryptase >20 Increased mast cells on BM bx

Dun ID A 27 year old female presents 15 days after returning from a trip to the Philippines. She was involved with rice planting and sustained multiple mosquito bites. The illness began abruptly with a non-productive cough, coryza, and conjunctival suffusion, and a temperature of 40 degrees, followed three days later by the development of diffuse macular rash that started on the face and spread to the rest of the body. She did not receive any pre-travel vaccinations and is unsure of her childhood vaccination history. The most likely diagnosis is: A. Dengue B. Acute HIV C. Measles D. Leptospirosis

C Measles: Classic symptoms. Incubation period typically 10 - 14 d (max 21 days) Dengue fever: Incubation period typically around 7 days Acute HIV: Possible Leptospirosis: Early petechial rash may occur, but is transient, typically only lasts 24 hours, and not a prominent feature

R 28 Amyloidosis is characterised by A. intracellular depostion of proteins which are remnants of nuclear break down B. intracellular depoisiton of Ig light chains C. extracell tissue deposition of fibrils composed of LMW subunits derived from variety of proteins, many which circulate as plasma constituents D extracell deposition of glycoproteins E. extra deposition of Ag/Ab complexes and complement components

C Misfolding of proteins = fibrils. Insoluble tau proteins deposited in tissues in bundles of beta sheet. Most common = AL (Ig light chains) and AA (chronic diseases) All deposits contain serum amyloid P a glycoprotein. A - SLE B - light chain amyloid C D - E

Dun Renal A 40 year old presented with peripheral oedema. Investigations demonstrate normal renal function and urinalysis showed proteinuria of 4 gm I 24 hours. A renal biopsy was performed (subepithelial). A. Electron microscopy will show subendothelial electron dense deposits. B. This lesion is commonly seen with NSAID usage C. There is a 50% likelihood of progression to ESRF at 5 years. D. Over 85 % of cases respond to steroids. E. This lesion is seen in 65% of adults presenting with the nephrotic syndrome.

C NSAIDs -> MCD Membranous GN • No influx of inflammatory cells. • No expansion of mesangium. • Expansion of basement membrane

CLS Rheum Nail fold capillaroscopy of a 30 year old female is shown. Which connective tissue disease is most likely? A. SLE B. Rheumatoid arthritis C. Systemic sclerosis D. Polymyositis E. CREST

C Systemic sclerosis is characterized by widespread: collagen deposition and fibrosis microvascular abnormalities Nail fold capillaroscopy: noninvasive method to evaluate vascular dysfunction. Majority of patients with SSc have capillary dilatation associated with avascular areas and loss of normal capillary organization.

Dunedin gastro 78 yr lady. HPT, NSTEMI 2012 (no stent), CVA 2006, recurrent TIAs. Medications: aspirin, ibuprofen, metoprolol, cilazapril, citalopram. Presents with several days abdominal pain & 24hrs hematemesis and melaena. Haemodynamically stable, Hb 75, urea 24.3 Cr 100 (e-GFR 67) PR melaena. Started on PO PPI. Aspirin and NSAID withheld. At endoscopy the gastric lesion is injected and clipped. Post procedure placed on IV omeprazole infusion. With regards to H. pylori in this patient: You would a. Not test as the result is irrelevant b. Do a rapid-urease test on a stomach biopsy c. Send a biopsy for histological examination to look for H pylori organisms d. Arrange H. pylori faecal antigen test e. Arrange a urease breath test

C On PPI so B won't work

Dun ID A 26 year old man underwent alloBMT for relapsed Hodgkin's; complicated by GVHD. He is on prednisone and tacrolimus. Admitted with fever, headache, and ataxia MRI shows cerebellar mass and brain biopsy shows septate hyphae with branching at acute angles What is your diagnosis? A. Mucor sp. B. Cryptococcus gatii C. Aspergillus fumigatus D. Candida sp

C Only 2 are moulds - Aspergillus & Mucor • Mucor non-septate • Aspergillus - septate

CLS Resp Obstructive sleep apnoea is uncommon in pre-menopausal women. Which hormone stimulates respiratory drive? A. LH B. FSH C. Progesterone D. Oestradiol E. Prolactin

C Progesterone - steroid responsive area within the medulla has been demonstrated in cats (did not respond to oestradiol, testosterone or cortisol) Progesterone - can improve but not eliminate daytime hypercapnia in people with OHS and doesn't treat the underlying pathology Acetazolamide - increases renal excretion of bicarbonate and elevates PaCO2 in the brain (both of which should increase ventilatory drive) BUT it impairs conversion of bicarbonate to CO2 in the lung Will only lead to a reduction in PaCO2 if the person can increase their minute ventilation -> can actually worsen respiratory acidosis Theophylline - increases the hypoxic ventilatory response and prevents the fall in response that normally occurs after 15min but not studied in OHS/OSA

Deltamed endocrine Regarding bone biology: a) RANK is expressed on osteoblasts b) PTH binds to a receptor on osteoclasts c) OPG (osteoprotogerin) binds to RANK ligand as a decoy receptor d) An activating mutation in the LRP5 gene is associated with osteoporosis

C RANK on clasts PTH binds to blasts Activating mutation increases BMD

Dunedin Resp A 68 year old man with motor neurone disease is under regular monitoring in the respiratory clinic. Which of the following tests is least helpful in determining progression of disease? A. PaCO2. B. Maximum inspiratory and expiratory pressures. C. Forced expired volume in 1 second (FEV1). D. Vital capacity (VC).

C Restrictive defect with resp muscle issues leading to hypoventilation

R20 84 y/o F. 1w of malaise, severe R temp headache, visual disturbance. Hx of PMR but weaned off pred 6m ago. Clinical dx of GCA. CRP 65. Commenced pred 60. 1d later = normal temporal A bx. Which is the MOST LIKELY explanation for bx finding? A. bx was done 1 day after taking pred B. elevated CRP excludes dx of GCA C. arterial lesion is segmental D. the pt has had PMR prev E. the pred administed was too high

C Roids will reduce chance but unlikely with just 1 day of rx. Can be segmental. Need decent length.

Dun ID A 32 year old female is s/p heart transplant for viral myocarditis. Which of the following is the greatest risk factor for CMV infection posttransplant? A. Donor CMV +/Recipient CMV + B. Donor CMV -/Recipient CMV - C. Donor CMV +/Recipient CMV - D. Donor CMV -/Recipient CMV +

C SOT. HSCT opposite

Dun cardio In preventing the recurrence of rheumatic fever in young patients with rheumatic heart disease, which is the most effective strategy? A. Early treatment of throat infection. B. Vaccination against common types of streptococcus. C. Secondary prophylaxis with benzathine penicillin G. D. Secondary prophylaxis with amoxycillin. E. Topical treatment of impetigo in family members.

C Stops sore throat, prevents recurrences of ARF and aids in regression of RHD All persons with ARF with no or mild carditis MINIMUM 10 years after most recent episode or age 21 All persons with ARF and moderate carditis MINIMUM 10 years after most recent episode or age 35 All persons with ARF and severe carditis MINIMUM 10 years after most recent episode or age 35 and then specialist review for need to continue. Post surgical cases definitely lifelong.

Book R4 Which of the following correctly describes the synvoial immunological process in RA? A. expression of IL1 is suppressed B. JAK are suppressed C. Macrophages in synovial membrane release cytokines such as TNFa and IL1 D. neutrophils and mast cells in synovial fluid don't play a major role E. TNFa inhibits angiogenesis and activates regulatory T cells

C Synovial membrane = macrophages, mast, NK (MMN) Synovial fluid = neutrophils. Macrophages are major effectors of synovitis + act thru releae of TNFa, IL1, ROS. IL1 and IL6 (promote angiogenesis and suppress T regs) TNFa (activation of WCCCs, fibroblats) in RA

Deltamed Neuro The following treatments of immune mediated neuropathies are NOT proven in controlled studies: A. Plasmapheresis in Guillain Barre syndrome (GBS) B. IVIg in chronic inflammatory demyelinating peripheral neuropathy (CIDP) C. IVIg in multifocal neuropathy with conduction block D. Prednisolone in CIDP E. Prednisolone in GBS

E

Dun ID A 60 year old lady is travelling to Angola as part of a missionary group. She has a diagnosis of polymyalgia rheumatica and is on Prednisone 10mg daily. She had a Yellow-fever vaccination 15 years ago and still has the vaccination certificate. Which of the statements is correct with regards to further Yellow Fever vaccination for this patient: A. Yellow fever vaccination is required as she is traveling to a high-risk country and it is greater than 10 years since her last vaccination B. Yellow fever vaccination is contraindicated due to concurrent prednisone use C. Yellow fever vaccination as a single dose is protective D. Yellow fever vaccination is not required as it has been eradicated from Africa

C The 17D Yellow Fever live-virus vaccine produces protective immunity in 90% of individuals within 10 days (and nearly 100% within 3 - 4 weeks) • The WHO Strategic Advisory Group of Experts in Immunization concluded in 2013 that a single primary dose of YF vaccine is sufficient for lifelong protection • World Health Assembly adopted recommendation to remove the 10-year booster dose requirement from the International Health Regulations by June 2016 • Main contra-indications: age <6 months; egg allergy; CD4 count<200; thymus disorder; other severe immunodeficiency; concurrent immunosuppressives use (including Prednisone >20mg)

CLS ID 28 year old male with a history of rectal gonorrhoea 1 month ago, presents 33 hours after unprotected receptive anal intercourse with ejaculation with a casual partner. The source is known HIV positive who is reliably adherent to treatment with an undetectable viral load 2 months ago. The best treatment option for this patient is: a) Offer sexual health screening and 2 drug PEP with Tenofovir/Emtricitabine for 28 days b) Offer sexual health screening, 3 drug PEP with Tenofovir/Emtricitabine and Dolutegravir for 28 days, and consider PrEP after completion of course c) Offer sexual health screening, give reassurance of low risk, and consider PrEP d) Offer sexual health screening, 2 drug PEP with Dolutegravir and Ritonavir for 28 days, and consider PrEP after completion of course e) Offer sexual health screening, give reassurance of low risk and discharge.

C Undetectable = untransmissable

Dunedin gastro A 22 y/o healthy nurse gets splashed in the face with liquid stool from a patient with hepatitis A. She has never been vaccinated for HAV. What should you advise? a) Immediate immune globulin injection b) Vaccination and immune globulin injection c) Vaccination only d) Quit working in the GI lab

C Vaccinate high risk groups • Vaccinate up to the time of departure in the young (<40). Two weeks prior in older, immunocompromised, or those with liver disease. • Postexposure prophylaxis with single dose of vaccine within two weeks in healthy persons 1-40 years Vaccine and IG in <1, >40, immunocompromised, or liver disease

Dun cardio A 43-year-old woman presents with a six month history of hypertension which has been difficult to control. She is now taking three agents for her blood pressure control. She has a serum potassium of 2.7 mmol/L [3.6 - 5.2 mmol/L] and you are concerned about primary hyperaldosteronism (Conn's syndrome). Which of the following medications is most likely to interfere with the interpretation of an aldosterone:renin ratio result? A. Amlodipine. B. Enalapril. C. Spironolactone. D. Metoprolol. E. Alpha-methyl-dopa.

C Withdraw medications, which significantly affect the ARR [at least 2 weeks before testing for (a) and (b); at least 4 weeks for (c)] 1. Beta-blockers, clonidine, methyldopa, nonsteroidal anti-inflammatory drugs 2. Angiotensin converting enzyme inhibitors, angiotensin receptor blockers, dihydropyridine calcium blockers 3. Diuretics (including spironolactone, eplerenone, and amiloride) 4. Commence, where necessary to maintain hypertension control, other antihypertensive medications, which have lesser effects on the ARR (verapamil slow-release±hydralazine and prazosin or doxazosin) 5. Estrogen-containing oral contraceptive agents may lower active renin concentration and cause false positive ARR when concentration of active renin (rather than PRA) is measured. Do not withdraw unless confident of alternative, effective contraception

Dun cardio You are asked to review an 80 year old patient listed for elective hysterectomy. She is asymptomatic with clinical findings of an ejection systolic murmur. Echocardiography reveals aortic stenosis with a peak gradient of 75mmHg, mean gradient, 45mmHg, AVA 0.9cm2 . LV systolic function is normal. The correct management recommendation is: A. Aortic valvuloplasty before surgery B. Exercise stress testing to assess haemodynamic response to stress before surgery C. Proceed to surgery D. TAVI before surgery E. Coronary angiography before consideration of mode of valve replacement pre surgery

C asymptomatic = good function if MET <4 = stress test

Dunedin gastro You admit a 45 y/o male with a history of alcoholic liver disease and recurrent hepatic encephalopathy. He improves with lactulose and supportive care. He is presently having three soft bm's per day. When his diet is advanced, he becomes encephalopathic. What is the best course of action? a. Restrict the protein in his diet to 0.8gm/kg/day b. Increase the lactulose c. Consult with dietetics about substituting vegetable protein for animal protein d. Neomycin 500 mg three times per day

C bowels already soft dangerous to restrict diet

Dunedin gastro A 28-year old woman was referred for investigation of diarrhoea and intermittent abdominal pain of at least 6 months duration. She smokes 10 cigarettes per day, drinks no alcohol and recently finished a trial of NSAID for a sprained ankle. Stool cultures were unremarkable apart from some leukocytes. A trial of loperamide was unhelpful. Her mother had life-long bowel troubles but no definite diagnosis. On examination she is pale but well built. Abdomen non-tender, no masses. PR unremarkable. Lab results show slightly increased WBC, ESR and platelets. Low grade iron deficiency anaemia, hypoalbuminaemia. Which of the following interventions will provide the most helpful information? A. Plain abdominal x-ray B. Small bowel follow-through C. Colonoscopy D. CT abdomen E. Faecal calprotectin

C can already see there's WCCs in stool so calprotectin less useful

Dun Rheum A 40yr old woman presents with a 6 month history of pain and swelling in her left thumb, left 5th finger and left foot. She has morning stiffness lasting 2 hours. She has had back pain of 4 yrs duration, worse with bending and lifting, better with rest. Naproxen is mildly helpful. O/E she has patches of erythema and scaling behind her right ear. Fusiform swelling of the left thumb and 5th finger and 3rd right MTPJ. Spinal alignment and range of movement is preserved. Nail findings are shown (pitting). Which of the following is the most likely diagnosis? A. Ankylosing spondylitis B. Inflammatory bowel disease associated arthritis C. Psoriatic arthritis D. Reactive arthritis E. Osteoarthritis with dandruff

C correlates more with psoriatic arthritis than skin changes

Deltamed Onc 56 y/o Queensland builder. Stage IV melanoma. HTN. ECOG1 Most likely mutation that is actionable? A EML4ALK B ER C. EGFR D. VEGF E. BRAF F. PDGFRA G. KIT H. KRAS

E

Dunedin gastro 45 y/o male with ESLD with ascites and esophageal varices is admitted with worsening ascites and confusion. Meds include: Spironolactone 100 mg daily Propranolol 60 mg daily Furosemide 40 mg daily Multivitamin daily Ascites fluid with 840 WBC/mm³ (85% PMN's), high gradient. His Cr is 90 mg/L. You start him on cefotaxime 2 gm every 8 hours. What is the most manipulation of his existing medications should you make? a) Increase the spironolactone b) Increase both the spironolactone and furosemide c) Stop the propranolol d) Hold all the diuretics e) Add terlipressin

C discontinue NSBB (higher mort + hepatorenal

Dunedin gastro A 75 year lady complains of heartburn and diarrhoea. Endoscopy shows ulcerative oesophagitis and she is treated with omeprazole 20mg/day. The heart burn resolves but the diarrhoea persists. Serum gastrin on omeprazole is 400pg/ml (<100). The most appropriate next test is: a. Secretin stimulation test with serial gastrins b. Endoscopic ultrasound c. Suspend omeprazole and repeat serum gastrin estimation d. Gastrin secretion studies e. Abdominal CT

C gastrinoma often >1000

Dun Neuro In a woman receiving long-term phenytoin therapy for epilepsy, pregnancy is likely to result in a reduction in the total concentration of phenytoin measured in plasma. The pharmacokinetic alteration most important for the interpretation of plasma concentrations during pregnancy is that there is: A. reduced absorption of phenytoin from the gastrointestinal tract B. a dilutional effect resulting from an increase in plasma volume. C. reduced protein-binding resulting in an increased free fraction of phenytoin. D. increased hepatic clearance of phenytoin. E. increased renal clearance of the metabolites of phenytoin.

C increased free

Deltamed Onc Which result is most likely to indicate a Lynch syndrome associated cancer? A. MSI-H - with 3 unstable microsatellite markers B. IHC - MLH1 positive, PMS2 positive, MSH2 positive, MSH6 positive C. IHC - MLH1 positive, PMS2 positive, MSH2 negative, MSH6 negative D. MSS - no unstable microsatellite markers E. IHC - MLH-1 negative, PMS2 negative, MSH2 positive, MSH6 positive, BRAF positive

C negative = mutation E would be right but BRAF indicates acquired

Dunedin endo A. Occurs more often in men than women B. Peak incidence is between 20 and 40 years of age C. Concordance rate among monozygotic twins is 35% D. Family history of thyroid disease is unlikely E. Graves opthalmopathy is clinically apparent in 10% of individuals

C peak 40-60 Ophthalmopathy >10%

Dunedin gastro A patient with cirrhosis is admitted following a fall at home with an intracranial bleed. His INR is 2.8 and his serum fibrinogen is measured as very low. Platelets are 55,000. What is the best intervention to reduce the bleeding? a) A six pack of platelets b) 3 units of fresh frozen plasma c) Cryoprecipitate d) Prothrombin complex concentrate e) Vitamin K

C plts already >50 low fibrinogen

DM PT Geri1 1. Which of the following features is NOT suggestive of a diagnosis of Dementia with Lewy Bodies (DLB)? A. Recurrent falls B. Visual hallucinations C. Poor sensitivity to risperidone D. Autonomic dysfunction

C • Fluctuating cognition with pronounced variations in attention and alertness • Recurrent visual hallucinations that are typically well formed and detailed • REM sleep behavior disorder, which may precede cognitive decline • One or more spontaneous cardinal features of parkinsonism (bradykinesia, rest tremor, rigidity) Supportive clinical features • Severe sensitivity to antipsychotic agents • Postural instability • Repeated falls • Syncope or other transient episodes of unresponsiveness • Severe autonomic dysfunction (eg, constipation, orthostatic hypotension, urinary incontinence) • Hypersomnia • Hyposmia • Hallucinations in other modalities • Systematized delusions • Apathy, anxiety, and depression

Deltamed stats In a study of 88 births to women with a certain medical condition, the same condition was recorded in 20% of babies (95% CI 13% to 30%): Which of the following statements is true? A. Another sample of the same size will show a rate for the condition between 13% and 30% B. 95% of such women have a probability of between 13% and 30% of having a baby with this condition C. It is likely that between 13% and 30% of births to such women would show the same condition D. If the sample were increased to 880 births, the 95% confidence interval would wider E. It would be impossible to get these data if the true rate (for births to women who have this condition) was 10%.

C ALL women true prob - 95% confident somewhere between 13% and 30% Always be wary of impossible

DM PT Neuro5 5. A 23-year-old male Australian rules footballer suffers a direct blow to the face which knocks him to the ground. There is no loss of consciousness. After several minutes, though, he seems dazed and is taken from the ground. 30 minutes after the injury, he remains confused with GCS 14. He cannot recall events in the play in the minutes before the injury or details of current events. He repeatedly asks as to what stage the game is at. This is his third such episode in 2 years, having made seeming full recovery from the first two episodes within a week of each injury. If a neuropathological examination was to be performed at this stage, where would perivascular phospho-tau neurofibrillary tangles most likely be found? A. In brainstem nuclei such as locus coeruleus B. In medial temporal lobe C. In the depths of frontal cortical sulci D. In widespread white matter

C Associations of Chronic Traumatic Encephalopathy (CTE): • Increased risk early age & recurrent headstrike • Neurodegenerative due to progressive Tau clumping Stage 1 Cortical sulci loss of attention and concentration, headache, Stage 2 + Basal ganglia depression, mood swings, explosivity Stage 3 + atrophy, ventricular dilatation memory loss prominent Perivascular phospho-tau neurofibrillary tangles medial temporal lobe, cortex, brainstem, spinal cord Stage 4 + atrophy, hippocampal sclerosis, dementia, executive dysfunction, aggression, paranoia, gliosis of cortex depression, Parkinsonism

Dunedin endo Question 4: One week after a 47-year-old male has transphenoidal pituitary surgery for a histologically confirmed non-functioning pituitary tumour, the following endocrine results are obtained: TSH <0.05 mIU/L [0.30-5.00] Free thyroxine (FT4) 16 pmol/L [10-23] Synacthen Test: 0800 plasma cortisol 90 nmol/L [100-700] 0900 plasma cortisol 680 nmol/L [>550] Luteinising hormone (LH) <1 IU/L [2-8] Follicle-stimulating hormone (FSH) <1 IU/L [2-8] Testosterone 8 nmol/L [10-30] Growth hormone (GH) <0.03 ng/ml [ <10] What is the next most appropriate management step at this point in time? A. Observe and repeat testing in six months. B. Commence thyroxine. C. Commence hydrocortisone. D. Commence testosterone. E. Commence growth hormone.

C Can't rely on synacthen test this early after surgery as will be normal

DM PT Neuro7 7. A 73-year-old farmer presents with progressive gait disturbance and unsteadiness over several months. On examination, cranial nerve function is normal. His gait has a scissoring quality. He has - apparently unnoticed - wasting of the right first dorsal interosseous and abductor digiti minimi, and to a lesser extent right thenar and forearm extensor muscles. Fasciculations are seen in the right FDI. Left upper limb examination is normal. Lower limb muscle bulk is normal. Reflexes are normal bilaterally at biceps and brachioradialis and reduced at bilateral triceps. Lower limb reflexes are increased at knees and ankles symmetrically bilaterally, and plantar responses are upgoing. Hypertonia is detected in lower limbs bilaterally. There is no apparent sensory deficit. Which of the following most likely characterizes the lower limb hypertonia? A. Near-fixed equinovarus deformity at bilateral ankles, precluding passive dorsiflexion B. Variability of tone, at times normal, at times increased, throughout the range of movement C. Velocity dependence, with increased tone present on rapid passive movement D. Velocity independence, with increased tone present throughout the range of passive movement

C Cervical Spondylotic Myelopathy. • Upper motor neuron signs are apparent in the lower limbs. • UMN signs may be accompanied by LMN signs at the level of cord or root compression • Increased tone with UMN lesions (spasticitiy) is a velocity dependent increase in tone, present on rapid passive movement, often relatively normal with slower movements. • The angle at which the 'spastic catch' appears can be used to grade spasticity, the earlier in the arc of movement, the more severe. • A contractures from chronic UMN lesions, unlikely in this setting • B Gegenhalten, variable hypertonia, classically in frontal lobe syndromes • D Leadpipe rigidity, an extrapyramidal pattern. With superimposed tremor, this is 'cogwheel rigidity'.

CLS CLL Lecture A 58 year old woman with a diagnosis of CLL completed chemotherapy 3 months ago. She now presents with shortness of breath and lethargy. Blood results are as follows: Na 135, K 3.4, Cr 120, Ur 14, Bili 43, ALP 50, GGT 38, ALT 40, AST 50, Hb 65, MCV 105, WCC 6, lymphocytes 2.1, Plt 148, haptoglobin <0.1, LDH 580, ferritin 30, transferrin saturation 14% What is the most appropriate next step: A. Perform B12 and folate assays B. Measure soluble transferrin receptors C. Perform direct antiglobulin test D. Refer for gastroscopy and colonoscopy E. Commence treatment with intravenous immunoglobulin

C Pred would be 1st line rx

Book D3 Which of the following features can be found in someone with SJS? A. cutaneous lesions are usually found on limbs B. eosinophilia is seen on FBC C. full thickness loss of epidermis D. minimal inflammtory cells in skin biopsy E. mucosal involvement is infrequent

C T cell mediated. Type IV. Trunk lesions predominate. MM nearly always involved. Epidermal necrosis on bx with full thickness loss of epidermis. Dense superficial dermal lymphocytic inflammation.

CLS Resp A 56yo male presents complaining of dyspnea which is worse when standing. His oxygen saturation is 85% standing and 97% when lying flat The most likely underlying cause for his dyspnea is: A. Ascites B. Pneumonia C. Pulmonary shunt D. Pulmonary embolism E. Asthma

C Uncommon DDx • Interatrial shunt (PFO, ASD) (+ additional factor that creates right -> left blood flow) • Intrapulmonary shunt (hepatopulmonary syndrome, pulmonary avms, emphysema) - basal>apical shunts theorized to cause this Orthodeoxia and platypnoea

Dunedin endo Question 1: In which form is calcium predominantly found in the blood? A. Bound to albumin B. Bound to globulin C. Ionised D. Complexed with bicarbonate E. Complexed with Phosphate

C ionised

Deltamed renal 56 y.o. man is referred for resistant HT. BP's at his GP between 145-172 systolic and 86 - 98 diastolic despite various combinations of up to two anti-HTs. He has no other vascular risk factors. He has no FHx of HT. Examination does not suggest a secondary cause of HT. Normal renal function, ECG and TTE What is the most appropriate next investigation? A. Aldosterone : renin ratio. B. Magnetic resonance angiography of renal arteries. C. Ambulatory blood pressure recording. D. Renal ultrasound. E. Urine testing for drug compliance

C.

Deltamed renal A 65 y.o. man who weighs 74 kg has a serum creatinine of 125umol/l and the lab provides an eGFR of 54.7ml/min. What is the most correct interpretation of these results? A. the GFR result is an underestimate because he is malnourished B. the estimated GFR calculations are inaccurate in this range C. he has significant renal impairment D. the result should be confirmed with a 24 hour urine collection E. he should be referred for renal biopsy

C.

deltamed renal Which of the following measures is most effective in preventing contrast nephropathy in at-risk patients undergoing imaging procedures involving iodine based contrast agents? A. The use of a low osmolar contrast agent. B. The use of a non-ionic contrast agent. C. Hydration with normal saline prior to the procedure. D. Pre-treatment with N-acetyl cysteine. E. Pre-treatment with sodium bicarbonate.

C.

DM PT G3 3. A 30-year-old man presents with worsening dysphagia. It occurs primarily with solid foods and has been happening on and off for the past few years. He has not had any loss of weight, does not smoke and his only medication is loratidine as required for hayfever. What is the most likely diagnosis? A. Oesophageal candidiasis B. Achalasia C. Eosinophilic oesophagitis D. Oesophageal cancer

C. Characterised by dysphagia, usually solids only and often intermittent Strong association with allergic disorders, asthma and atopy Most common in men in 20s-30s • Histologically confirmed eosinophil predominant oesophageal inflammation +/- stricture formation ≥15 Eosinophils/HPF (60 Eos/mm2 ) Management: • First-line therapy: • Medical • PPI: 30-50% of cases are PPI responsive • Topical Steroids: budesonide, fluticasone • Dietary • Elimination Diets (6-food elimination diet has best evidence) • Allergy testing directed diets • Endoscopic dilation is efficacious for patient with persistent oesophageal strictures not-responsive to First-line therapy • Anti-histamines have no role in the treatment of EoE

Dun Neuro 39 y/o F 3m of numbness pains and needles in R hand involving all fingers. Loses sleep at night NCS show marked slowing of R median nerve distal latency, absent median finger sensory potentials, reduced amplitude of ascending action potential. Next best step? A. NCS of LL B. Xray of R wrist + elbow C. R carpal tunnel decompression D. surgical exploration of R prox median nerve E. use of splint + diuretics

C. Distal latency = demyelination from compression absent sensory + reduced amp = axonal axonal loss = surgery sensory changes only = E

Deltamed practice test 1 You are relaxing in Economy on your way to a medical conference when a call is made asking if there is a doctor on board. After enthusiastically leaping up and identifying yourself, you are taken to see a 60-year-old man who feels short of breath. He tells you he has no significant comorbidities and is a non-smoker. He does not look particularly unwell. His heart rate is 72 bpm, BP 110/60mmHg, RR 14 and you measure his SpO2 at 92%. What is your summary which you relay to the ground based medical system who are trying to determine if a plane diversion might be required? A. Likely previously unrecognized lung disease B Likely cardiac event C. Normal examination thus far D. Likely dehydration

C. For commercial passenger flights, cabins are typically pressurised to an altitude of 5-8,000 feet. Due to Boyle's Law which states that pressure varies inversely with volume, there is less oxygen (fewer oxygen particles) with increasing altitude, thus a reduced PaO2 available at an alveolar level. SpO2 of around 90-92% are therefore quite physiological in-air and usually well tolerated.

Deltamed renal Most patients with Conn's Syndrome (primary hyperaldosteronism) present with? A. Hypertension and hyperkalaemia B. Hypertension and hypokalaemia C. Hypertension and normokalaemia D. Other irrelevant answer A E. Other irrelevant answer B

C. Only 40% have low K

deltamed endocrine Adrenal incidentaloma, which of the following is LEAST consistent with dx of subclinical Cushings A. serum ACTH 2 (low) B. serum cortisol 100 after 1mg DST C. Serum DHEAS 11.5 (high) D. 24hr UFC 100 (normal)

C. Would get low DHEAS

DM PT Gen4 4. Emily has an uncle (mother's brother) and a brother affected with Haemophilia A. Emily's daughter is pregnant. What is the risk to Emily's grandchild of developing Haemophilia A? A. 1 in 4 B. 1 in 8 C. 1 in 16 D. 1 in 32

C. X-Linked Recessive inheritance Draw the pedigree and fill in what you know. Emily's mother is an OBLIGATE carrier (affected brother and son with Haemophilia A). Emily at 1 in 2 risk of being a carrier. Emily's daughter at ½ x ½ = 1 in 4 risk of being a carrier. Emily's daughter's pregnancy at ¼ x ½ = 1 in 8 risk of inheriting the Haemophilia mutation. Only male pregnancies will develop Haemophilia, so the chance the pregnancy is male and inherits the Haemophilia mutation is ½ x 1/8 = 1/16 (Answer C)

Dunedin Resp A 32 year old woman is referred with 6 months history of persistent cough, productive of copious green sputum with occasional flecks of blood, requiring monthly courses of antibiotics. She is a current smoker (5 pack-years) and is on no regular medications. On examination she is obese (BMI 42) and hypertensive (BP 164/96). Sputum specimens grow Haemophilus influenzae. Spirometry is normal. High resolution CT chest scan shows bronchiectatic changes in both lower lobes. Which one of the following is most likely to improve her symptoms? A. Regular exercise program with weight loss. B. Intravenous immunoglobulin infusions. C. Long term oral macrolide. D. Nebulised DNase.

C. main sx = exacerbations azithro 3x/week. need to make sure doesn't have non-TB myco

R15 30 y/o man. Oligoarthritis, urethritis, conjunctivitis. CRP 78. RF- Which of the following is also assoc with the suspected dx of Reiter syndrome/reactive arthritis? A. AIN B. AS C. keratoderma blenorrhagica D. pericarditis E. pulmonary fibrosis

C. Urethritis, conjunctivitis, seroneg arthritis 1-4w after infection Chalmydia, enterics. Mono/oligo arthritis particularly in legs. Remains chronic in 15% Conjunctivitis, ant uveitis, keratoderma blann (hyperkeratotic brown rash on palms soles) mouth ulcers, plantar fasciitis, Achilles, circinate balanitis and aortic incompetence

Book R6 Which 1 of the following is observed in SLE? A. Anti-DNA ABs cross react with NMDA in kidney B. Complement component C1q accumulation in brain resulting in neurocogn defects C. activation of complement by ICs + complement consumption D. anti-Ro ABs that interfere w/ coag system E. antiphospholipid ABs that alter the function of the cardiac conduction system

C. ANA binds to abundant nuclear material that isn't cleared properly due to deficient Fc. Active sLE = complement activated + levels of C3 and C4 depressed. Some anti-DNA ABs cross react with NMDA in brain

Dunedin gastro You admit a 65 y/o male with known cirrhosis/ALD and recurrent ascites for encephalopathy (stage 2). Diagnostic paracentesis is done. WBC 300 (90 % neutrophils, 10 % lymphs) Albumen 10 gm/l (serum albumen 25 gm/l) ∆=15 LDH 430 (ULN for serum 333 IU/L) Glucose 2.5 mmole/L Total protein 12 gm/L What is the best treatment option? a. Cefotaxime 2 gm every eight hours b. Cefotaxime as above with repeat diagnostic paracentesis in 48 hours c. Abdominal CT d. Repeat the paracentesis e. Order a gram stain on the ascites fluid f. Gentamicin and Ampicillin/sulbactam

C. Consider perf if 2/3 total protein >10 glucose <2.8 LDH > UL serum

Dun Neuro 66 y/o man. numbness R thumb and index, 1st dorsal webspace + dorsoradial forearm. Nil neck pain. Normal power. Normal biceps + triceps reflexes. Reduced brachiorad reflex on R A. median neuropathy B C6 radiculo C Radial neuropathy D. brachial plexopathy E cortical stroke

C. Mainly sensory. Not stroke nil neck pain so less likely C6 plexopathy = weakness

Deltamed genetics Melanie has 2 brothers affected with Duchenne muscular dystrophy. Melanie's daughter is pregnant. What is the risk to Melanie's grandchild of developing DMD? A. 1 in 2 B. 1 in 4 C. 1 in 16 D. 1 in 32 E. 1 in 64

C. 1 in 16 Melanie = 1/2 risk of being carrier Daughter at 1/2 x 1/2 = 1/4 risk of being a carrier Melanie's daughters pregnancy = 1/4 x 1/2 = 1 in 8 but only M pregnancies will develop = 1/8 x 1/2 = 1/16

Deltamed Genetics A single nucleotide polymorphism (SNP) with 2 alleles, A and B, has been shown to be associated with an increased risk of celiac disease in homozygotes of allele B. In the general population, 36% of the population is homozygous for allele A. In the same population, what proportion of the population is at increased risk of celiac disease as a result of homozygosity for allele B? A. 4% B. 6% C. 16% D. 36% E. 64%

C. 16% A2 = 0.36 so A = squareroot of 0.36 = 0.6 A+B = 1 so B = 0.4 B2 = 0.16 (16%)

Dun Neuro Which APOE forms increase risk of AD? A. E2 B. E3 C. E4

C. E4 (late onset) E2 protective

Deltamed renal Apart from ESRF, which of the following occurs most frequently in patients with adult polycystic kidney disease? A. Mitral valve prolapse. B. Renal infection. C. Ischaemic heart disease. D. Pancreatic cysts. E. Cerebral aneurysm rupture.

C. IHD ppl with heart disease get kidney disease

Deltamed Onc 45 year old male with metastatic melanoma presents for 18th cycle of Pembrolizumab with a 5 day history of headache and fatigue • TSH < 0.01 T3 6 T4 • Cortisol 6 • What is the most appropriate next investigation • A. Testosterone • B. ACTH • C. MRI pituitary • D. Thyroid Scan

C. MRI ACTH will take days

R21 65 y/o with recent TURP. Acute lower back pain, tender L4/L5. L sided foot drop and fever. Staph aureus in blood. Haemodyn stable Which is the MOST appropriate INITIAL 1st step of Ix? A CT of L4/5 B. PSA C. MRI spine D. 3 phase technetium bone scan E. indium 111 WCC scintigraphy

C. MRI Neuro impairment and features of vertebral osteomyelitis. Need to R/O nerve compression from disc or epidural abscess. Bone scans +ve few days after onset but non-specific findings. Indium: more specific for epidural abscess but low sens for vertebral OM Staph > E.coli. If recent spinal surgery = CONS

Dun pharm Which analgesic most likely to lead to serious drug interaction if combined with a MAO-I A morphine B indomethacin C tramadol D celecoxib E fentanyl

C. SSRI

Deltamed endocrine 24 year-old woman referred for serum calcium 2.95 mmol/L z Lassitude, nocturia z Amenorrhoeic x 6 months z PTH 150 ng/L z Father had "neck surgery for calcium problems" z What is not indicated? A. Parathyroid sestamibi B. Urinary calcium C. Serum calcitonin D. Pregnancy test E. Screening for MEN1

C. calcitonin In MENII you would get XXX 1st

Deltamed gastro A 25 Y.O. male with ileocolonic Crohn's disease becomes steroid-dependent. All the of the following are effective maintenance therapies except - A) Azathioprine B) Infliximab C) Mesalazine D) Ustekinumab E) Vedolizumab

C. mesalazine not in CD

Deltamed Onc A 52 year old lady presents with a 5cm left breast mass, and undergoes a wide local excision and sentinel lymph node biopsy. Histology shows a 35mm, high grade invasive carcinoma, ER 8 PR 5 HER2 negative, margins >10mm, LVI negative, 2/3 sentinel nodes involved. Which pathologic feature is not predictive in this patient? A. ER status B. PR status C. Nodal status D. HER2 status

C. nodal status prognostic not predictive ?difference predictive = gives info about effect of potential therapeutic prognosis = info about cancer outcome regardless of rx

Deltamed Immuno Which one of the following events is least important in the role played by Langerhans (dendritic) cells in the afferent arm of the immune response to infections in the skin? A. Activation following the binding of microbial polysaccharides to the Langerhans cell. B. Migration of the Langerhans cell to the draining lymph node C. Up‐regulatution of the major histocompatibility complex (MHC) class I molecules on the Langerhans cell D. Up‐regulation of the MHC II molecules on the Langerhans cell E. Up‐regulation of the co‐stimulatory molecules on the Langerhans cell

C. viruses

Dunedin gastro A 45 year man has had malabsorption for the past year associated with a low volume non-bloody diarrhoea. He has had a polyarthritis and complains of occasional visual hallucinations. An abdominal CT reveals no masses , only generalised lymphadenopathy. On upper GI endoscopy there are no oesophageal or gastric lesions but in duodenum there are broad flattened villi. Biopsies show numerous PAS-positive macrophages in the sub-mucosa. How to rx? a. Gluten free diet b. Corticosteroids c. Antibiotics d. Segmental duodenal resection e. Antacids

C. whipples

Deltamed ID Regarding syphilis serology, which of the following is true? • A. By the time a chancre is present, the serology will be positive • B. Positive RPR in pregnancy usually requires treatment to prevent congenital syphilis • C. FTA-Abs will remain positive after curative therapy • D. CSF VDRL will always be positive in neurosyphilis • E. Following successful treatment for late latent syphilis, the RPR level will fall to undetectable with 6 months

CE

Deltamed rheum Which of the following statements about anti-TNF Rx is true? Ideally recommended for all pts with RA CI if moderate bronchiectasis CI with class II NYHA CI with active TB CI with healing osteomyelitis CI with ON

CI if moderate bronchiectasis - true CI with class II NYHA - false III or IV CI with active TB - false, rx 2m prior CI with healing osteomyelitis - T CI with ON - T

Dunedin pharm In a study of the new drug Yetanotherib the measured renal clearance is 150 mL/min. GFR in the study population was 150 mL/min. Protein binding of Yetanotherib is 80%. Which of the following is correct? A. There is net secretion of drug B. Excretion is filtration only C. Drug is metabolised in the kidney D. There is net re‐uptake of drug E. Excretion is primarily hepatic

CLR = CLS + fu x GFR CLS + Fu x GFR = CLR CLS = CLR - FU x GFR = 150 - 0.2 x 150 = 150 - 30 = 120 = net secretion A

Deltamed resp A 60yo pigeon breeder presents for a routine health assessment.He is an exͲ smoker of 30 pack years.He is asymptomatic with good exercise tolerance.His lung fields are clear on auscultation, his oxygen saturations are normal on room air and his RFTs demonstrate some mild fixed airflow obstruction with a normal DLCO. His HRCT is reported as normal.Serum precipitins are positive for IgG to pigeons. The most likely diagnosis is: • Hypersensitivity pneumonitis (pigeon fanciers lung) • Asthma • COPD • Sarcoidosis • Idiopathic pulmonary fibrosis

COPD asympt bird handler but nil other features to suggest H.P

R22 60 y/o F. 3m malaise, anorexia, wt loss Epistaxis in past 3 w. Crackles bibasally. rash LLs (purpuric) Urine ++ protein. Many RBCs + casts. Creat 210. CRP 80 Name most likely dx paired with its supportive serological test A. SLE, ANA B. poststrep GN, ASOT C. Goodpasture, anti-GM D. Wegener granulomatosis, ANCA E. cryo vasculitis, cryoglobulin

D GPA Pulmonary + renal involvement. Pulmonary haemorrhage, haemoptysis, epistaxis, infiltrates on CXR, cavitations. Rapidly progressive segmental necrotising GN. cANCA, PR3

D10 56 y/o. Kidney tx 5yrs ago. 4d of painful lesions on ant legs. Firm, tender. Panniculitis. Nil vasculitis. Most likely dx A. SLE B. fungal C. Pyoderma gangrenosum D. erythema nodosum E. pretibial myxoedema

D. Most common cause: strep pharyngitis

Deltamed Onc This patient commences erlotinib. What is the most common side effect of erlotinib? A. Diarrhoea B. Nausea C. Fatigue D. Rash

D. rash

CLS Haem Which of the following is most characteristic of a patient with haemophilia A with 8% baseline factor VIII activity? A. Spontaneous cutaneous purpura. B. Spontaneous gastrointestinal haemorrhage. C. Spontaneous deep muscle bleeds. D. Spontaneous haemarthroses. E. No spontaneous bleeding.

E

CLS ID Use of ABx (especially vancomycin and broad spectrum ABx) increases incidence of VRE. What other factor contributes to VRE colonisation? A. antifungal B. chronic IS C. C.diff D. Diabetes E. residential care

E

Deltamed Stats Shown below on the same graph are two ROC curves. These are representations of scores of two questionnaires designed to test for early dementia compared to an actual diagnosis determined within the following 5 years. Which is the best way to compare accuracy of the two questionnaires? A.Sensitivity at A2 and sensitivity at B2 B.Specificity at A1 and specificity at B1 C.Likelihood ratios at A1 and likelihood ratios at B1 D.Sensitivity and specificity at A2 and B1 E.AUC A vs AUC B (area under the curve)

E

Dun Rheum What is the most common presenting feature of scleroderma? A. Diffuse hand swelling B. Dysphagia C. Dyspnoea D. Pruritus E. Raynaud's Phenomenon

E

Dun stroke A 72‐year‐old man presents to the emergency department after experiencing a 10‐ minute episode of slurred speech and left facial droop. He has a history of diabetes, hypertension, gout, hyperlipidemia, and alcoholism. He is an active cigarette smoker with a 25 pack‐year history. His current blood pressure is 186/99 mm Hg. Neurologic examination is normal. His low‐density lipoprotein cholesterol level is 3.0 mmol/l. Controlling which of the following risk factors will give the patient the largest benefit in risk reduction? A. alcoholism B. diabetes C. hyperlipidemia D. hypertension E. smoking

D

Deltamed endo A 30yo woman presents with 25kg weight gain over the last 2 years. She reports no recent medication use and on examination has a 'moon' face and prominent dorsocervical fat pad, prominent abdominal striae and difficulty standing from a chair. BP is 160/100. Biochemistry ordered by her GP shows serum cortisol of 16 [>100] and ACTH of 0.5 [2-10]. Which of the diagnoses below is consistent with these findings? a. Cushing's disease b. Adrenal adenoma c. Adrenal carcinoma d. Surreptitious glucocorticoid use e. Ectopic ACTH syndrome

D Only dex would do this Others X-react so would be detected

DM ID A nurse in a neonatal unit develops a persistent cough, with paroxysms starting in the second week. PCR of nasopharyngeal aspirate confirms that she has whooping cough. Which of the following is most correct? • A. She can continue to work if the mothers of the babies in the neonatal unit are immunised • B. As she is now in the second week of illness, she is no longer contagious • C. All staff and patients in the neonatal unit should be immediately vaccinated • D. As this is spread via aerosolised droplets, she can pass this on to people even without direct contact and the morbidity and mortality in neonates is substantially higher • E. It is now too late to give her antibiotic therapy such as azithromycin as she is no longer contagious

D

Dun renal Which is least correct: A. Statins reduce cardiovascular morbidity but not mortality in the setting of CKD. B. Correction of anaemia results in improvement in quality of life but no other benefits. C. Correction of anaemia to normal levels with erythropoietin results in harm. D. Pure red cell aplasia only occurs with iv EPO alfa. E. Iron resistance plays a key role in the anaemia of CKD

D

CLS Tumour markers Tumour markers are often used indiscriminately to monitor rx. For which of the following tumour markers + associated tumours does normalisation with therapy best predict overall survival? A. Ca 153 breast cancer B. CEA in bowel C. LDH in lymphomas D. AFP in testicular cancer E. Ca199 in pancreatic cancer

D

Dunedin gastro 55 year man with 4cm length of Barrett's. Surveillance gastroscopy demonstrates a single small nodule within the Barrett's, with biopsies showing high grade dysplasia (HGD). Repeat chromoendoscopy confirms the finding and multiple biopsies show no evidence of dysplasia in the rest of the Barrett's. Regarding this finding which of the following statements is the most correct? Question 4 a. Careful 3 monthly surveillance of the lesion and refer to surgery if progresses to early intramural adenocarcinoma b. Radiofrequency ablation (RFA) of the Barrett's including the nodule and then 3-6 monthly surveillance c. Endoscopic mucosal resection (EMR) removal of lesion and then repeat surveillance gastroscopy in 3 months d. Endoscopic mucosal resection (EMR) of the nodule and then RFA of the rest of the Barrett's e. Start aspirin and high dose PPI and continue usual surveillance programme

D

Dunedin gastro 71 year man, hyperlipidaemia, Type II DM, hypertension, BMI 35 Referred for ongoing reflux symptoms. At endoscopy 5cm length of Barrett's noted. No focal nodules or lesions found. Serial biopsies taken. Histology report notes changes consistent with Barrett's oesophagus with several areas of low grade dysplasia. A repeat gastroscopy in 6 months shows no macroscopic or histological deterioration. Currently recommended management is: a. Continue surveillance with serial biopsies at 5 year intervals b. Refer for oesophagectomy c. Refer for argon plasma coagulation therapy d. Refer for radio-frequency ablation e. In view of his age and only low grade changes, no further surveillance endoscopy required

D

Dunedin gastro 78 yr lady. HPT, NSTEMI 2012 (no stent), CVA 2006, recurrent TIA. Medications: aspirin, ibuprofen, metoprolol, cilazapril, citalopram. Presents with abdominal pain, and 24hrs hematemesis and melaena. Haemodynamically stable, Hb 75, urea 24.3 Cr 100 (e-GFR 67) PR melaena. Started on PO PPI. Aspirin and NSAID withheld. The gastric lesion was injected and clipped. Post procedure placed on IV omeprazole infusion. H. pylori testing was positive. Patient is well after 72 hours IV omeprazole and is switched to oral PPI prior to discharge. Regarding her anti-platelet treatment you would: a. Restart aspirin in 4 weeks b. Stop aspirin, start clopidogrel in 3 months. c. Re-scope 4 weeks, then decide re: platelet therapy d. Restart aspirin prior to discharge e. Instruct GP that no antiplatelet agent to be given for 4 months

D

Dunedin gastro A 36 year old man presents with a 1½ years history of lethargy particularly affecting his rather physical job in the afternoon. He reports two soft bowel motions per day which has not changed for many years. On rare occasions he experiences some discomfort in the mid-abdomen. No significant family Hx. The patient does not take any medication. The clinical examination is unremarkable. The patient's abnormal lab tests are shown below: Hb: 136 g/l (130 -180) MCH: 24 pg (26.5 - 33.5) Ferritin: 12 µ/l (14 - 233) Transf. Sat. 20 % (30 - 50%) A. In this age group colonoscopy is indicated first B. Abdominal U/S is highly sensitive in the diagnosis of gastrointestinal mass lesions C. Assessment and consequently correction of this man's diet will resolve his problem D. Upper GI endoscopy has the highest diagnostic yield in this clinical context E. Oral iron substitution and reassessment in 3 months is justified

D

Dunedin gastro Biopsies are taken from oesophagus (furrowed webs) Which is incorrect a. Dietary manipulation is likely to be helpful b. There is a strong association with atopy c. An oesophageal stricture is a recognised cause for dysphagia in these patients d. Resolution of symptoms following 8 weeks of high dose PPI therapy excludes this diagnosis e. This is a recognised cause of refractory GORD (unresponsive to PPI treatment) In this case which statement is incorrect?

D

Dunedin gastro H. pylori testing was positive. Regarding her +ve H. pylori test, which one of these following statements is correct: a. Treatment with 7 days of triple therapy (PPI, amoxicillin, clarithromycin) is best first choice b. Post eradication testing is unnecessary in the absence of recurrent bleeding/symptoms c. Checking H. pylori serology is the most practical method of post-eradication testing as this allows PPI treatment to continue d. In the absence of knowledge regarding local H. pylori antibiotic resistance, quadruple therapy (PPI, clarithromycin, amoxicillin, metronidazole) x 14 days is indicated

D

Deltamed neuro A 63 year old man with polymyositis comes to EMG. His motor units in proximal muscles are likely to show: A. Increased duration, increased amplitude, polyphasic. B. Normal duration, normal amplitude, reduced recruitment, no fibrillation potentials. C. Normal duration, normal amplitude, prominent fibrillation potentials. D. Reduced amplitude, reduced duration, polyphasic potentials, early recruitment, full interference pattern.

D A - chronic denervation B. demyelination C - acute denervation

Deltamed Rheum Which statement is true concerning Xrays in RA? periarticular erosions are seen at the DIP A Periarticular erosions are seen at DIP B Large punched out erosions are seen at PIP C Erosion and new bone formation are seen at MCP D Periarticular erosions are seen in carpal joints E Subchondral sclerosis is seen at the MCP

D A - not DIP in RA B - gout C. PsA E. OA

Dun Neuro Which one of the following EEG abnormalities best supports the diagnosis of herpes simplex encephalitis? A. Generalised 3-4 Hz delta activity B. Posterior, reactive 10 Hz alpha activity with occasional right temporal spikes C. Posterior, reactive 10 Hz alpha activity with bursts of generalised spike-wave activity. D. Pseudoperiodic, sharp-slow wave complexes in the right anterior temporal region, with diffuse background slow wave activity, maximal bitemporally E. Burst-suppression pattern of background activity

D A = sleep or encephalopathic E = coma, GA

DM PT Im7 7. Antibodies to omega 5 gliadin of IgE isotype are most closely associated with... A. Coeliac disease B. Wheat allergy C. Crohn's disease D. Wheat-dependent exercise induced anaphylaxis

D Anaphylaxis triggered by wheat ingestion followed by exericse, NSAIDs, alcohol or infections. All increased GI allergen permeability

Dunedin Resp A 21 year old Thai international student is referred to hospital with an abnormal chest x-ray. He gives a history of persistent dry cough for 2 months associated with fever and sweats in the past one week. He has poor appetite and admits to 15kg weight loss. Chest x-ray is shown. Under ultrasound guidance 400mls of slightly bloodstained pleural fluid is aspirated. Pleural fluid analysis shows: pH 7.24, protein 60, LDH 409, glucose 3.5; Gram stain shows no organisms, differential shows polymorphonuclear cells 5%, lymphocytes 95%. Which of the following will confirm the diagnosis? A. Pleural fluid culture. B. Induced sputum. C. TB interferon-gamma release assay (IGRA). D. Pleural biopsy

D C will only show if every had TB infection Aim to diagnose by isolation of organism from secretions or tissue AFB stain Culture important for drug sensitivity testing Nucleic acid amplification (Xpert MTB/RIF) provides rapid diagnosis 15-20% diagnosis is clinical Pleural fluid characteristics High protein (exudate), high LDH, lymphocytosis Organisms in fluid scanty Pleural fluid culture +ve 42% Pleural biopsy culture +ve 64%

Deltamed Onc Which other cancer is not associated with the following breast cancer causing mutations A. CDH1 and gastric cancer B. TP53 and soft tissue sarcoma C. PTEN and thyroid cancer D. BRCA1 and renal cancer

D CDH1 = lobular TP53 = Liu Fraumeni

Dunedin Resp A 60 year old man with severe COPD asks you in clinic if he could be referred for a lung transplant assessment. He has recently been admitted with type II respiratory failure. He quit smoking 1 year ago and has and FEV1 of 19%. He has no other comorbidities and is breathless on undertaking ADL's. He has never done pulmonary rehabilitation but still works in an office job when he can. He has a 6MW of 350m and his BMI is 21 • Which is most correct? A. He doesn't need to do pulmonary rehabilitation because he is still working B. Because he has COPD he will need a bilateral lung transplant C. He does not need a coronary angiogram because he has no history of IHD D. He needs to attend pulmonary rehabilitation before he can be referred for a lung transplant assessment E. Because he has had type II respiratory failure he is too sick for a lung transplant assessment

D COPD - single Smoker - needs Angio

DM PT Im6 6. A 40-year-old man has a 5-year history of PR3-positive Granulomatosis with Polyangiitis involving the buccal mucosa. He was treated with steroids and methotrexate with clinical remission but remained PR3 seropositive. He now presents with left facial pain, and imaging shows bony erosion in the left ethmoidal and sphenoidal regions. What is the LEAST appropriate immediate therapeutic approach now? A. Continuation of methotrexate at higher doses B. Rituximab C. Intravenous cyclophosphamide D. Plasma exchange

D Controversial role. Early studies showed benefit for AAV if creat >500 but not indicated for limited GPA

CLS Renal Risk factor for IgAN progression - IgAN is the most common cause of GN leading to ESKD in ANZ 40 y.o. with Ig A nephropathy, what is the strongest risk factor for later developments of renal failure? A. Male gender B. Low complement C. Heavy haematuria rbc > 100 (alternate recall - RBC > 10 cells) D. Proteinuria > 1g (alternate option >2g) E. BP > 130/70

D D • Degree of proteinuria (esp. not responding Rx) generally the strongest predictor for renal decline in glomerular diseases • Interaction between BP & proteinuria • Proteinuria probably most important - reflection of glomerular injury • BP control still essential, generally more treatable • Proteinuria/albuminuria is also a predictor of CV events & all-cause mortality in CKD • Male and low complements - not major risk factors • Haematuria/ number of red cells - not a predictor, but most common

Dun Rad In Paget's Disease the bone shows • A. Decreased density • B. Decrease in size • C. Narrowed cortex • D. Coarse trabecular pattern • E. Decrease bone scan activity

D Dense Coarse trabecula Wide cortex In long bone Goes to one end Scapula, pelvis, vertebra Whole bone Larger Softer - bends Cortical fractures High Alk.Phos

Deltamed Onc A 60 year old attends for discussion around adjuvant chemotherapy after resection of a stage IIIB descending colon adenocarcinoma. They are four weeks post surgery, their only regular medication is omeprazole 20 mg OD for long standing gastro- oesophageal reflux disease. They accept a planned course of four cycles (3 months) of capecitabine and oxaliplatin chemotherapy. Five days into the first cycle the present with severe stomatitis, pancytopaenia and diarrhoea. The most likely cause for the severe toxicity is: A. Omeprazole interacted with capecitabine prolonging its metabolism B. They have a UGT1A1 variant decreasing the metabolism of oxaliplatin C. They inadvertently took the incorrect dose of capecitabine D. They have a dihydropyrimidine dehydrogenase deficiency decreasing the metabolism of capectabine E. They have an ERCC1 polymorphism decreasing the metabolism of capecitabine

D ERCC1 = cisplatin

Deltamed Neuro A 35 year old man presents with foot drop after prolonged leg crossing on an airline flight. You suspect a peroneal neuropathy. At what time can the peroneal motor nerve conduction studies provide definitive information regarding whether the foot drop is due to severe axonal loss or neurapraxia? At what time can the peroneal motor nerve conduction studies provide definitive information regarding whether the foot drop is due to severe axonal loss or neurapraxia? A. Within 24 hours B. 1 to 3 days C. 4 to 7 days D. 10-14 days E 21 days

D 10-14d Prior to d7 = motor conduction distal to axonal lesion may be preserved. NCS will not distinguish between demyelinating and axona

DM PT Neuro3 3. A 57-year-old woman presents in Emergency department with acute onset 80 minutes ago of left arm and leg weakness with mild sensory loss. On examination, she has NIH stroke score 10. She is alert and oriented. She has not had previous similar symptoms, migraine, or recent surgery. Her CT brain is shown. What early sign of ischaemic cerebral infarction is seen on her CT? A. Cortical sulcal effacement B. Hyperattenuation of large vessel: "hyperdense MCA sign" C. Loss of gray-white matter differentiation in the basal ganglia D. Loss of the insular ribbon

D Early signs of infarction include: Obscuration of the lentiform nucleus Cortical sulcal effacement Focal parenchymal hypoattenuation Loss of insular ribbon or obscuration of Sylvian fissure Hyperattenuation of large vessel (eg, "hyperdense MCA sign") Loss of gray-white matter differentiation in the basal ganglia

CLS Resp A 45yo male presents with dyspnoea. He has a 20 pack-year smoking history but quit 10 years ago. His father had a pneumothorax at age 80 on a background of COPD. He is haemodynamically stable with SpO2 97% and respiratory rate of 15 on room air. He has crackles at the left base. Chest x-ray reveals a right-sided pneumothorax that is 1.5cm Which of the following options is the most compelling reason he should have further investigation and management in hospital? A. Family history of pneumothorax B. Size of the pneumothorax C. Presence of dyspnoea D. Crackles at the left lung base E. History of smoking

D Family history suggests he is susceptible to cigarette smoke but wouldn't necessarily compel further inpatient investigation Crackles in the 'good lung' aren't explained - suggests a secondary pneumothorax and warrants further evaluation 1.5cm suggests small pneumothorax so this is not compelling BTS guidelines suggest small is <2cm when measured at the level of hilum Admit symptomatic or large Lower threshold for admission/intervention in secondary pneumothoraces (which are more likely to be symptomatic due to presence of underlying lung disease)

DM PT Gen1 1. In a certain homogeneous population, the carrier frequency for an autosomal recessive disorder is 1 in 10. What is the risk of a child born in this population being homozygous-affected? A. 1 in 40 B. 1 in 100 C. 1 in 200 D. 1 in 400

D For autosomal recessive conditions where both parents are carriers Before a child is born: There are 4 possibilities Chance of having an child who is homozygous affected is 1 out of 4 Risk of father being a carrier x risk of mother being a carrier x ¼ Carrier frequency = 1 in 10 1/10 x 1/10 x ¼ = 1/400 (answer D)

DM PT Im1 1. Which of the following statements is INCORRECT with regard to IgG in comparison to IgM? A. IgG has higher affinity for the target antigen B. IgG shows more V-region somatic mutations C. IgG engages more immunological effector functions D. IgG is better at activating complement

D IgM: rapid response, neutralises target, activates complement IgG: isotype switching, hypermutation, activates complement

Dun ID A 35 yo female with AML day 15 after induction therapy. She C/O fever, chills, diffuse erythematous rash. Exam -100/62, HR 120, grade 2 oral mucositis - diffuse, blanching, erythematous rash across chest/abdomen. CXR- bilateral diffuse infiltrates. As an outpatient she was on ciprofloxacin and acyclovir. Blood culture shows +GPC in chains This is most consistent with infection with which of the following organisms? A. Streptococcus pneumoniae B. Coagulase-negative Staphylococcus C. Enterococcus faecalis D. Streptococcus mitis

D Key points: neutropenia, mucositis, high-dose cytosine - use of Cipro - Remember Cipro barely gets above MIC to be effective Gram Positive abx • Classic presentation - fevers, rash and stomatitis (think where it lives) • Assoc'd with VGS SHOCK SYNDROME - about 24-48 hr in 1/3 of cases

Dun cardio Which of the following findings most favours a diagnosis of constrictive pericarditis over cardiac tamponade? A. Pulsus paradoxus. B. Third heart sound. C. Kussmaul's sign. D. Prominent y-descent. E. Low voltage on ECG.

D Kussmauls in Constrictive + restrictive PP also seen in asthma, resp distress low voltage = tamponade

CLS Resp A 30yo female with severe asthma and recurrent exacerbations presents for review. Which of the following features may suggest she will benefit from the use of mepolizumab? A. Elevated ratio of neutrophils on bronchoalveolar lavage B. Elevated serum procalcitonin levels C. Positive mannitol provocation testing D. Elevated serum eosinophil count E. Large response to short acting beta agonist (>20%) on spirometry

D Mepolizumab is an anti-IL5 monoclonal Ab which targets eosinophilic inflammation Eosinophilic asthma Mepolizumab - humanized monoclonal Ab to IL-5 • IL-5 mediates eosinophil growth, differentiation, recruitment and activation - elevated eosinophils indicate poorly controlled Th2 mediated asthma • Efficacy when peripheral eosinophils >150/microL but ideally >300/microL or if sputum eosinophils are >3% SC injection every 4 weeks equally effective (Other anti-IL-5 agents: Reslizumab and (anti-IL-5R) Benralizumab)

DM PT Resp4 4. A 24-year-old man presents with a 4-year history of excessive daytime sleepiness. He describes a short sleep latency, sleep fragmentation and rarely has sleep paralysis. His Epworth Sleepiness Scale score is elevated at 16/24. Which of the following is diagnostic of narcolepsy type 2? A. A history of cataplexy B. High levels of orexin-A in cerebrospinal fluid C. Low levels of orexin-A in cerebrospinal fluid D. A mean sleep latency of 7 minutes and 2 sleep onset REM episodes

D Narcolepsy Type 1 Narcolepsy type 1 is a disorder primarily characterized by excessive daytime sleepiness and signs of REM-sleep dissociation, the most specific of which is cataplexy. • Narcolepsy type 1 is caused by a deficiency of hypothalamic hypocretin (orexin) signalling. • Patients with sleepiness and low or absent CSF hypocretin-1 levels are classified as having narcolepsy type 1, even if they do not manifest cataplexy. Narcolepsy type 2 is characterized by excessive daytime sleepiness and abnormal manifestations of REM sleep on polysomnography and MSLT ≤ 8 minutes. • Cataplexy is absent, although some atypical sensations of weakness triggered by unusual emotions such as stress and anger may be reported.

Dunedin Resp LL 5. A 49 year old man, with no significant past medical history, is brought to hospital because of sudden onset of breathlessness whilst sitting in a meeting. When assessed in the emergency department he appears comfortable. Examination is unremarkable. Arterial blood gas taken on 40% supplementary oxygen shows: pH 7.40, PaO2 225 mmHg, PaCO2 38 mmHg, HCO3 23 mmol/l, saturation 100%. Which of the following statements is correct? A. An air bubble has contaminated the sample. B. Request echocardiogram. C. Request CT-pulmonary angiogram. D. He should be reassured and discharged home.

D Normal alveolar gas equation

DM Mock 14. A 36-year-old man with seropositive rheumatoid arthritis comes to see you in the outpatient clinic to discuss starting a family. He is currently well controlled on Hydroxychloroquine 200mg mane, Methotrexate 20mg weekly, Folic Acid 5mg daily except the day of Methotrexate, and Tofacitinib 5mg BD. Which of the following medications (if any) should be ceased prior to conception? A. Hydroxychloroquine B. Methotrexate C. None of these medications D. Tofacitinib

D Reference: Sammaritano LS, Bermas BL, Chakravarty EE. et. al. 2020 American College of Rheumatology guideline for the management of reproductive health in rheumatic and musculoskeletal disease. Arthritis & Rheumatology 2020; 72(4): 529 - 556. Rationale: Several trials have shown that Methotrexate taken by men at the time of conception, or during pregnancy, do not increase the risk of birth defects, or of other adverse pregnancy outcomes. There is insufficient data about Tofacitinib to say if this is safe, and therefore this should be ceased. Hydroxychloroquine is safe for men or women to take before, during or after pregnancy

Dunedin Resp With regard to the radiological manifestations of sarcoidosis which of the following statements is most correct? A. Radiological stage IV disease is characterised by diffuse reticulo-nodular lung infiltrates. B. Pleural effusions are a common finding in sarcoidosis. C. Normal chest x-ray excludes the diagnosis of pulmonary sarcoidosis. D. Lung parenchymal changes typically are more pronounced in the upper and mid-zones on chest x-ray.

D Stage I - hilar & mediastinal lymphadenopathy (50%) • Stage II -lymphadenopathy + lung parenchymal infiltrate (25%) +/- roids • Stage III - lung parenchymal infiltrate (15%) roids • Stage IV- lung fibrosis (5-10%)

R18 60 y/o F with 16yr hx of RA. On MTX and pred. Evaluated pre-op for hysterectomy Previous synovectomy of both wrists + silicone implants in 7 MCPKs Increasing morning stiffness, fatigue. Mild alopecia. Shooting pains in forearms and hands with progressive weak hand grip + mouth ulcers. Weakness of hand grip, hypereflexic, hypertonic Mutliple subcut nodules. High RF and CRP. Which of the following is the most appropriate action at this time? A. Commence TNFa B. test for cryo C. obtain XRs of both hands D. obtain MRI of cervical neck E. test for ANA, antiDNA and complement levels

D Suggest atlantoaxial sublux due to stretching/erosion of transverse lig that holds odontoid process in place

Deltamed Immuno Secretion of which of the following cytokines is most characteristic of T helper cells from patients with cell‐mediated immune disorders such as multiple sclerosis? A) Interleukin‐4 (IL‐4) B) Interleukin‐10 (IL‐10) C) Interleukin‐12 (IL‐12) D) Interferon‐ gamma (IFN‐gamma) E) Interferon‐ beta (IFN‐beta)

D TH1 cell = IFN gamma, TNF alpha

Dun ID A septic patient on the surgical ward has a positive blood culture for Enterococcus faecium. They are currently on ceftriaxone and metronidazole. Pending sensitivities, this is best treated by: a. amoxicillin + gentamicin + metronidazole b. Oral nitrofurantoin c. Linezolid d. Vancomycin e. Daptomycin

D The majority of Enterococcus faecium are resistant to amoxicillin & penicillin. Enterococci are naturally resistant to cephalosporins. A glycopeptide e.g. vancomycin or teicoplannin should be added.

Dun Rad Which of these inhalations has basal predominance on CXR? • A. Allergen • B. Toxic gas • C. Silicosis • D. Asbestosis • E. Histoplasmosis

D allergic = diffuse silcosis, gas = upper histo

Deltamed Onc Which of the following tumour markers is most useful in diagnosing the associated cancer A. Seminoma and AFP B. Melanoma and LDH C. Breast cancer and Ca 19-9 D. Germ cell cancer and HCG E. Bowel cancer and CEA

D melanoma = LDH for prognosis not dx pure seminoma = not AFP breast ca not CA199 Bowel cancer CEA for prog not dx

Dun ID A 34 year old woman with renal transplant develops a brain abscess with concomitant pulmonary lesions - the biopsy shows this organism on modified acid fast smear. She is maintained on prednisone and tacrolimus. • Previously when given trimethoprimsulfamethoxazole (TMP-SMX), she developed a non-desquamating rash (no oral mucosal involvement), fevers, LFT abnormalities, myalgias. She is on linezolid + meropenem but has progression of the lung lesions. • Of the following agents, which would be appropriate now • A. meropenem plus amikacin • B. intravenous azithromycin • C. do nothing, it is a commensal organism • D. TMP-SMX after desensitization

D nocardia. severe. only rash so can desensitise AFB = nocardia

Deltamed neuro 54-year-old man presents with weakness in the left biceps and deltoid muscles. He initially had severe pain in the shoulder and upper arm radiating to the thumb. The pain has abated, but he is now developing progressive weakness and has lost the biceps tendon reflex. Except for some dysaesthesiae at the shoulder there are no sensory findings. What is the most likely diagnosis? A. C5 disc herniation B. C6 disc herniation C. neoplastic infiltration of the upper trunk of the brachial plexus D. acute brachial plexopathy

D painful Usually unilateral. C5-C7 = brachial neuritis, radiation, trauma C8-T1 = malignant infiltration, cervical rib

Deltamed Onc All of the following have been observed as mechanisms resulting in the activation of an oncogene except: A. A point mutation altering the function of the protein product B. A chromosomal translocation fusing the oncogene with another gene C. Amplification as small sub-chromosomal fragments (double minutes) D. Inactivation by telomerase activity E. Capture of the oncogene sequence by a retrovirus

D telomerase shortening = mutations not activation by telomerase

Deltamed Onc Which of the following is not a common side effect of BEP chemotherapy (Bleomycin, Etoposide, Cisplatin) A. Tinnitus B. Pulmonary toxicity C. Alopecia D. Diarrhoea E. Peripheral Neuropathy

D tinnitus (high tones), alopecia, peripheral neuro with platin pulm tox with bleomycin alopecia yes

Dunedin pharm Regarding the therapeutic monitoring of digoxin, which of the following is most correct? A. blood samples for drug levels should be taken within 6 hours of a dose B. steady state is reached within 2 days C. the therapeutic concentration range is 0.5‐2mg/L D. Concentrations above 2microgram/L invariably indicate toxicity E. Blood samples for drug levels taken at times greater than 6 hours after a dose are satisfactory

E

DM PT Neuro8 8. A 37-year-old male professional cyclist presents concerned about painless wasting of his right hand first web space. He first noticed this 2 weeks ago, though is not sure how quickly it may have developed. He is most concerned, as his paternal uncle died from motor neuron disease in his early 50's. He has not noticed wasting elsewhere. There is no fasciculation. There are no sensory symptoms. On examination, there is marked wasting of the right first dorsal interosseous muscle. Hypothenar and thenar muscle bulk and power is preserved. Upper limb reflexes are present and normal at biceps, brachioradialis and triceps. Finger jerks, pectoral jerks and Hoffman's sign are not present. Sensory examination is normal. What is the most likely cause of this presentation? A. C8 radiculopathy. B. Motor neuron disease, beginning with 'split hand' sign. C. Ulnar neuropathy at the elbow, retrocondylar D. Ulnar neuropathy in the distal portion of Guyon's canal.

D • Ulnar neuropathies at Guyon's canal can variably involve motor branches to ADM, FDI and ulnar sensory distribution. • Pure motor involvement of the deep branch to FDI can occur in the distal Guyon's canal, and is classically seen in cyclists with persisting deep palmar pressure. • Split hand syndrome - a relatively specific sign of MND - would be expected as well to involve thenar muscles. • More proximal ulnar or C8 involvement would have more diffuse distal motor and sensory involvement than this isolated picture.

DM ID 23M brought in to ED with severe dyspnoea, hypoxia and fevers. Rapidly requires intubation and transfer to ICU for mechanical ventilation and vasopressors. See CXR. Needle marks in left cubital fossa. Soft ESM. Appropriate antibiotic therapy? • A. Benzylpenicillin and doxycycline • B. Ceftriaxone and azithromycin • C. Benzylpenicillin, flucloxacillin and gentamicin • D. Vancomycin, ceftriaxone and azithromcyin • E. Linezolid and meropenem

D Covering severe pneumonia and MRSA as IVDU

CLS Resp A 60yo man with hypertension and previous coronary revascularization for IHD presents with moderate daytime sleepiness, low mood and erectile dysfunction. After undergoing polysomnography you diagnose him with obstructive sleep apnoea. Which of the following options are the least likely to occur with CPAP therapy? A. Improved blood pressure control B. Reduced daytime sleepiness C. Improved erectile dysfunction D. Reduced risk of acute myocardial infarction E. Improved depressive symptoms

D EDS - Remains the primary reason to treat patients with CPAP SAVE study and others have shown improvements in depressive symptoms with CPAP therapy Hypertension should improve with CPAP therapy but not by more than medications Erectile dysfunction studies are a bit more obscure but an effect appears to be present SAVE study (McEvoy et al, NEJM 2016) RCT comparing CPAP with usual care in asymptomatic with moderate-severe OSA and known CV disease (secondary prevention) Demonstrated no effect of CPAP on cardiovascular events (composite endpoint and individualized events) Improvements with CPAP in somnolence and depression symptoms despite excluding the severely sleepy (ESS >15) Adherence to therapy was 3.3hrs per night Consistent with previous meta-analysis (CPAP had no effect on CV endpoints) (post-hoc there may have been a signal for stroke)

Dun Neuro 25 y/o. soft tissue injury to R shoulder in footy tackle. Wekaness of shoulder abd 4/5 and IR 3/5 after game. Normal reflexes. Patchy loss over lateral forearm. Most likely injury? A. axillary B. suprascap C. C5 nerve root D. upper trunk of brachial plexus E. musculocut nerve

D Not C5 as normal reflexes Sensory loss too low for suprascap + axillary musculocut would decrease biceps reflex IR = subscap/posterior cord

CLS Resp REM sleep behavior disorder could be the initial presentation of which neurological disorder? A. Alzheimer's dementia B. Creutzfeldt-Jakob disease C. Fronto-temporal dementia D. Parkinson disease E. Dementia with Lewy bodies

D PD is the most strongly associated Motor behaviours during REM that correlate with dreams (enacting dreams) Can be violent, vocalisations, purposeful, can include leaving the bed Key features: wake quickly at termination of event without amnesia or confusion, often recall the dream, usually second half of the night (when REM is more frequent) Polysomnography - Actual events are rarely captured but key diagnostic feature is REM sleep without atonia (limb and chin EMG) Risk of PD or DLB - 20% at 5 years, 40% at 10 years, 50% at 12 years

R 25 43 y/o man. bilateral hand pain and swelling intermittently for 6m. 2nd and 3rd DIP on R hand and 4th on L are worst. Swollen fingers. Non-tender, normal ROM. Scaling on scalp and elbows. Which of the following changes on XRay is most likely? A. periarticular osteoporosis B. chondrocalcinosis in wrist C. subchondral cysts D periostitis at distal phalanges E. subchondral bony sclerosis

D Psoriasis precedes PsA in 70% PsA. Pencil in cup. Bony erosions, effusions A - RA (periart osteoporosi, marginal erosions, soft tissue swelling) B - CPPD 0 chondrocalcinosis C - OA (loss of J space, phytes, subchondral systs, subchondral sclerosis) D E - OA

Dunedin gastro A 58 year man has had increasing difficulty with swallowing for the past 6 months. He has lost 10kg in the past 2 months. Upper GI endoscopy reveals a nearly circumferential mass with overlying ulceration in the mid-oesophageal region. Biopsy of the mass reveals pink polygonal cells with marked hyperchromatism and pleomorphism. Risk factors for this lesion in the oesophagus include all the following except a. Zinc deficiency b. Food contaminated with Aspergillus c. Chronic alcoholism d. Diet rich in fruit e. Food containing nitrosamines

D SqCC

Dun cardio A 58-year-old male presented with an acute inferior ST elevation myocardial infarction to a rural centre and received thrombolysis. There was no resolution of ST elevation but he became pain free after 18 hours of chest pain. The patient had no further chest pain. A week later coronary angiography was performed. This showed a proximal occlusion of the right coronary artery with no significant disease elsewhere. Cardiac systolic function was only mildly impaired. In addition to optimal medical therapy and risk factor modification, which of the following is the most appropriate next step? A. Coronary artery bypass grafting of the occluded vessel. B. Percutaneous angioplasty and stenting of the occluded vessel. C. Percutaneous angioplasty and stenting of the occluded vessel with a drugeluting stent. D. No additional intervention. E. Implantation of a cardiac defibrillator.

D missed event

DM PT E10 10. A 60-year-old man presents with a 2-month history of increasing difficulty walking especially climbing stairs. There is no significant past medical history. Examination is notable for proximal myopathy in the lower limbs (3/5), blood pressure of 150/90 mmHg. Neurological exam is otherwise normal. There is no facial plethora, evidence of easy skin bruising, thin skin, or striae. Serum potassium is 2.8 mmol/L, random glucose is 12 mmol/L, and random serum cortisol 900 mom/L. What is the most likely diagnosis? A. Cortisol-producing adrenal adenoma B. Androgen-producing adrenal adenoma C. Cushing's disease D. Ectopic ACTH syndrome

D ?why

Deltamed uro + GI cancer A 58F presents with painless jaundice. In association she has noticed clay coloured stool, but she denies fevers, chills or nightsweats. She has a past history of alcohol misuse but has been abstinent for the past 10 years. On examination she is afebrile and has icteric sclerae. Bowels sounds are normal, and the abdominal examination is soft non tender. The liver span is 12cm to percussion and is palpable at the right costal margin. LFTs show an ALT 122, ALT 168, ALP 483, GGT 735 with a bilirubin of 221. On abdominal US, the GB cannot be visualised and there is dilatation of intrahepatic bile ducts but not the common bile duct. What is the diagnosis? A) Cholangiocarcinoma B) Cholecystitis C) Gallbladder cancer D) HCC E) Pancreatic cancer

D HCC given alcohol, GGT, palpale liver

Dun Rheum A 30yr old woman with severe rheumatoid arthritis is planning to commence tofacitinib ( (JAK) janus kinase inhibitor). • The following are true EXCEPT: A. Tofacitinib should not be used with other biologic DMARDs B. Dose reduction is required in renal impairment C. It is safe to receive the annual influenza vaccine D. It is safe to receive MMR vaccine while taking tofacitinib E. Pneumovax is safe and recommended for patients starting tofacitinib

D can't have live vaccines tofa = monotherapy

R14 40 y/o F. Fatigue weakness for 2m. Difficulty getting out of chair. Blanching of hands, stiffness lasting several hrs of hands/wrists/feet. Finger swelling. Prox muscle weknaess, synovitis of MCP, PIP, wrists, MTP. ANA 1:640. Which auto-Ab to ENA most likely to be positive? A. dsDNA B. Scl70 C. Smith D. Jo1 D. SSA

D polymyositis. Arthralgia and arthritis. Wrists, knees, small joints of hands. Non-erosive. AntiJ01. synthetase syndrome: fibrosis, Raynaud, polyarthritis, and sometimes Mechanics hands. Anti-Sm and dsDNA = SLE Scl70 = scleroderma AntiSSa SLE or Sjogrens

Deltamed endocrine 36 yo woman presents with pain in both feet. Her serum phosphate is noted to be 0.49 mmol/L. There is no clinical evidence of myopathy. Her serum calcium is 2.35 mmol/L, PTH is 4.5 pmol/L and 25OHD 51 nmol/L. She has a sister with the same disorder. z Next best test: a) Bone biopsy b) Bone scan c) IFE EPG d) Urine phosphate corrected for GFR (TmP/GFR) e) A genetic test

D). PHEX Vitamin D-Resistant Rickets/hypophosphataemic: most common is X-linked hypophos rickets due to PHEX mutation = increased FGF23 (phosphaturic)

Deltamed renal A small island nation appears to have a high rate of ESRF. You wish to screen the entire adult population of 18,000 inhabitants for evidence of renal disease. Which of the following would provide the best screening tool: A. Serum creatinine cut-off of 130 umol/l [normal <120 umol/l] B. Urinary protein excretion of 0.2 gm/l [normal < 0.12 gm/l] C. Urinary albumin excretion rate of 200 ug/min [normal <20] D. Urinary albumin/creatinine ratio of 3.5 mg/mmol [normal < 3.4] E. Calculated GFR of 70 ml/min [normal 90-150].

D,

Deltamed Liver A 25 year old women is currently 28 weeks pregnant. Blood test indicate she is: HBsAg +ve, eAg ‐ve and sAb -ve Viral load is 5000 IU/ml Which of the following is appropriate advice? A. She should be advised to have a caesarean section to avoid transmission of hepatitis B. She should commenceTenofovir 300md daily C. She should not breast feed D. The baby should receive HBIG and vaccination after delivery E. All of the above

D, Low viral load so doesn't need tenofivir Can b/f as long as nipples not cracked/bleeding

Deltamed onc Photo showing Hand Foot syndrome Which of the following rx is most likely cause for problem? a. bevacizumab b. oxaliplatin c. erlotinib D. capecitabine E. 5FU

D.

Deltamed renal A 45 year old woman with ESRD due to mesangiocapillary GN Type II (Dense Deposit Disease) presents 8 years Following renal transplantation with a slowly rising serum Creatinine (80 to 160umol/L over past 12 months). Rx - cyclosporin 75mg bd, azathioprine, prednisolone BP 150/90, examination otherwise normal. Dipstick urine 2+ protein only. MSU no cells seen. 24h urine - creat. clearance 35ml/min, protein 1.0gm/day The most likely diagnosis is: a. Recurrent glomerulonephritis b. De novo glomerulonephritis c. Acute rejection d. Chronic allograft nephropathy e. Transplant renal artery stenosis

D.

DM PT G10 10. A 24-year-old woman was diagnosed with ulcerative colitis 3 months ago and commenced on azathioprine. The dose has been uptitrated to 100mg daily, for the last month. Her weight is 50kg. Her other medication includes sulfasalazine. Her thiopurine metabolites at the follow-up appointment are as follows. 6MMP 8000 (250-5700) 6TGN 100 (235-450) What does the thiopurine metabolite testing indicate and what is the next best management option to optimise her medical management? A. Noncompliance. Counsel regarding medication compliance B. Subtherapeutic dose. Increase azathioprine dose. C. Genetic intolerance to azathioprine. Check TPMT D. Shunting. Reduce azathioprine dose and add allopurinol

D. Allopurinol = XO inhibition = shifts metabolism towards 6TGN production (hence, reduce dose by 25% too or else leukopenia etc)

Book R2 2. A pt with suscepted vasculitis undergoes Ix for ANCAs. While the pANCAs are weakly positive on direct IF, they are reported negative on ELISA. What could be the cause of this? A. lab error B. less specificty of ELISA C. interpreter error D. pANCA +ve on indirect IF ass. with anti-cathepsin ABs E. X-reactivity of cANCA with pANCA

D. May be assoc with concomitant positivity against neutrophil Ags ie. BPI, cathepsin G, lactoferrin. 2 types of ANCA are ELISA (more specific) and indirect IF (more sensitive) cANCA -> PR3 on indirect IF. 90% GPA pANCA -> MPO. 70% MPA but also BPI, cathepsin, lactoferrin cause pANCA pattern Screen with cheaper IF (more sensitive) then confirm +ve results with ELISA

Deltamed obs A 33 year old kidney transplant recipient presents 12 months after successful renal transplantation with normal kidney function, no proteinuria or hypertension for preͲ pregnancy counselling. Her current medications include mycophenolate 500 mg b.d, prednisolone 5 mg and Tacrolimus 2 mg b.d. She has no history of rejection episodes. General advice to both her and her partner include: a. Contraception till planning to conceive with condoms tominimise medication interactions b. Avoid mycophenolate in pregnancy and change over to azathioprine as soon as pregnancy detected c. Minimise antiͲrejection medications to avoid side effects during pregnancy d. Start aspirin 150 mg in 1st trimester e. Vaccination for rubella prior to pregnancy if not immune.

D. a-condoms not effective form contraception b-At least 6 weeks before conception c-Risk Rejection d-Prevention PET e-Rubella live vaccine contraindicated in pregnancy and Immunosuppression

DM Resp 17 y/o girl with wrinkling of hands after brief exposure to cold water The most likely diagnosis is: A. Allergic bronchopulmonary aspergillosis B. BͲcell lymphoma C. Common variable immunodeficiency D. Cystic fibrosis E. Limited scleroderma

D. Aquagenic wrinkling

Deltamed endocrine 23 year-old Indian woman presents with bone pain and radiographic abnormalities as shown. Her serum calcium is 2.70 mmol/L, PTH 45 pmol/L, ALP 780 U/L, and 25OHD 37 ng/mL. Next best test: a) Mammogram b) Bone biopsy c) Bone scan d) Parathyroid sestaMIBI e) CT chest/abdomen/pelvis

D. brown tumours

Dun pharm Which of the AED is most likely produce a clinically signif SE with lamotrigine? A. clonazepam B gabapentin C vigabatrin D. carbamaz E vapro

E valpro. competes with lamotrigine = inhibits = doubles lamo

Deltamed Genetics Bill and Cindy are planning to have a child and are concerned about having a child with phenylketonuria (PKU). Bill had a son with a previous partner that died from PKU. Cindy has no family history of PKU. The couple are non-consanguineous. The carrier frequency of PKU in the general population is 1 in 50. What is the risk of the couple having a child affected with PKU? A. 1⁄4 B. 1/50 C.1/100 D.1/200 E. 1/300

D. 1/200 Bill is carrier Cinday has 1/50 chance of being carrier If both carriers = 1/4 chance 1/50 x 1 x 1/4 = 1/200

CLS Haem In females under 20 years of age treated successfully with radiotherapy for Hodgkin Lymphoma which one of the following is the most frequent second tumor occurring after 10 years? A. Acute Leukaemia B. Thyroid cancer C. Non-hodgkin's lymphoma D. Breast cancer E. Ovarian cancer

D. Breast cancer

Deltamed endocrine 40 y/o F presents with morning serum cortisol of 10 (repeat 12). ACTH is 20. Asymptomatic and well. Normal electrolytes Inactivitng mutation in which gene is most likely? A. ACTH B. 11 BHD1 C. 11 BHD2 D. CBG

D. CBG Test measure free Plasma CBG concentration increased by pregnany, oestrogen, hyperthyroid Decreased by CBG gene mutation, protein deficiency, cirrhosis, hypothyroidism

CLS Rheum Best indicator of future erosive joint disease in RA A. Swollen joint count B. Tender joint count C. Weight loss D. CRP

D. CRP CRP correlates closely with changes in inflammation/disease activity, radiological damage, progression, functional disability • CRP levels useful for monitoring the effect of therapy on halting disease progression • Monitoring CRP levels at baseline and after treatment should become a routine part of clinical practice

DM PT G4 4. A 28-year-old woman with known colonic Crohn's disease presents with a fever and perianal pain. Her Crohn's was previously well controlled on azathioprine, and a colonoscopy 4 months ago was normal. Her bowels are opening 3-4 times per day which is painful and associated with some urgency, but the stools are small, soft and formed. There is no associated abdominal pain, nausea or vomiting. On examination, her abdomen is soft, non-tender and there are no palpable abdominal masses. Full digital rectal examination is unable to be performed due to pain but a small opening is seen adjacent to the anus with a purulent discharge. What is the most appropriate next step in management? A. Mesalazine suppositories B. Change azathioprine to methotrexate C. Infliximab D. Metronidazole

D. Crohn's disease with perianal fistula Presentation suggestive of perianal abscess/perianal fistulizing disease 1st step is to eradicate infection • Oral antibiotics (metronidazole or ciprofloxacin) • Reduce fistula drainage, improve symptoms and fistula closure • May be needed for weeks to monthe • Often need surgical exploration and local drainage (EUA+seton placement or fistulotomy) Biologic therapy (TNF inhibitor) may be indicated, especially for complex fistula with abscess, failed antibiotics, recurrent fistulae or severe underlying Crohn's • But after control of infection (not next most appropriate step) • Refractory disease may respond to addition of immunomodulator eg. azathioprine (patient already on this) • Methotrexate best avoided in young female of child-bearing age (teratogenic/category D) • Limited role for 5-ASAs in Crohn's • Only mild disease with limited involvement • No role in this setting

Deltmed uro + GI ca Q 63M has been diagnosed with metastatic CRC with liver and lung metastases. He has a performance status (ECOG) of 1. Mutation analysis of his cancer shows that he is wild type for KRAS but has a mutation in NRAS. He is subsequently commenced on bevacizumab/FOLFIRI What is FALSE regarding this man's case? A) Bevacizumab should be used because his tumour's NRAS mutation makes him unsuitable for EGFR monoclonal antibody inhibitors B) He is NOT a candidate for curative resection because he has multiple sites of metastatic disease C) Bevacizumab when added to his regimen may extend survival by 23 months D) Bevacizumab side effects include hypotension and a predisposition to excessive bleeding E) Venous thromboembolism and increased gastrointestinal fistulas are side effects of his treatment

D. HYPERtension

Deltamed gastro A 25 Y.O. male with steroid-dependent Crohn's disease requires a thiopurine immunomodulator (azathioprine/mercaptopurine). In discussing the risks and benefits of these agents in IBD all of the following are correct EXCEPT A) Thiopurines are associated with an increased risk of non-melanoma skin cancer (NMSC) B) Thiopurines are associated with an increased risk of lymphoma C) Thiopurines are effective as steroid-sparing and maintenance agents in Crohn's disease D) Thiopurines are associated with an increased risk of colorectal cancer

D. If anything decrease

R17 Which of the following Js is most commonly affected in acute attacks of pseudogout? A. ankle B. Metacarpal C. MTP joint D. knee E. wrist

D. Knee in 50% Associated: familial chondrocalcinosis (AD), joint trauma, HH, hyperparathyroidism, low Mag, low P Joint aspiration, intra-art roids.

Deltamed Onc Which of the following known genetic mutations has no targeted therapy? • A. EGFR • B. BRAF • C. ALK • D. NRAS

D. NRAS

Dun neuro 24 y/o wakes with R shoulder/trap discomfort. 2 weeks of pain then weakness of R shoulder movements. Nil c spine pain0/5 power deltoid and infraspinatus. Other power normal, normal reflexes. Normal sensation. Normal MRI and USS A. herpes zoster radiculopathy B C5 radic C. rotator cuff D inflammatory brachial plex E suprascap

D. Pain then weakness Not painful so not zoster Neck OK and MRI OK = not C5 USS OK = not rotator cuff Suprascap would affect sensation +

DM Resp Worldwide, the leading cause of pulmonary arterial hypertension is: A. HIV infection B. Scleroderma C. Cocaine D. Schistosomiasis E. BMPR2 mutations

D. Schistosomiasis

Deltamed ID 35yo male in ICU post multi-trauma develops fever and high WCC. Blood cultures MRSA. Vancomycin given. Trough level 19 mg/L. Ongoing fever, & positive cultures at day 6. Vanc MIC = 2mg/L. Getting worse. Appropriate course of action? • A - Increase the vanc dose, aiming for a higher trough • B - Change to colistin • C - Add rifampicin and fusidic acid • D - Change to linezolid • E - Change to daptomycin

D. change to linezolid colistin not for gram positive

Deltamed Which of the following statements regarding infections and the development of malignancy is not true? A. Schistosoma haematobium infection has been shown to be associated with development of squamous cell carcinoma of the bladder B. Helicobacter pylori gastritis may be associated with gastric adenocarcinoma or gastric lymphoma C. An association between Opisthorchis viverrini infection and cholangiocarcinoma has been demonstrated D. EBV-associated Burkitt's lymphoma has been associated with chromosome 12 translocation and de-regulation of the c-MYC oncogene

D. chrom 8

Deltamed Onc For which of the following ca is screening most strongly assoc with improved survival? A. LUng B. prostate C. ovarian D. colon E melanoma

D. colon - can remove adenomas melanoma checks picks up superficial/early Lung - only in smokers so not true screening

Deltamed Renal The leading cause of ESRF in Australia is? A. Hypertensive Nephropathy B. Glomerulonephritis C. Reflux Nephropathy D. Diabetic Nephropathy E. Polycystic Kidney Disease

D. diabetes DM > GN

Dun Neuro A 16-year-old girl presents with a three-month history of blank spells. The 16 lead electroencephalography (EEG) shows normal background rhythm and right temporal sharp wave activity. The drug of choice in the treatment of this condition is: A. carbamazepine B. ethosuximide C. phenytoin D. lamotrigine E. vigabatrin

D. focal = lamo > carba. leve gen = valpro > lamo

Deltamed renal A 30-y.o woman presents with sustained HT (170/110 mmHg) on amlodipine monotherapy. Also on combined OCP. There is no radiofem delay or abdo bruit. Investigation reveals the following: Plasma renin activity <40 fmol/L/s [130-2350] Plasma aldosterone 320 pmol/L [80-1040] Serum potassium 3.9 mmol/L [3.5-5.5] Urinary metanephrines 1.3 ȝmol/24h [<1.5] Which of the following secondary causes of HT is most likely? A. ACTH-producing tumour. B. OCP C. Phaeochromocytoma D. Primary hyperaldosteronism E. Renal artery stenosis

D. primary hyperaldo calculate ratio

Deltamed renal In a patient with IgA nephropathy who has a serum creatinine in the normal range, which of the following is the best predictor of developing end-stage renal failure? A. Urine red cell count. B. Serum IgA level. C. Creatinine clearance. D. Level of proteinuria. E. Blood pressure.

D. proteinuria then HTN

DM PT G1 1. A 65-year-old man presents with haematemesis. He has a history of ischaemic heart disease on aspirin and clopidogrel but is due to see his cardiologist due to symptoms suggestive of unstable angina for the past few weeks. There is a significant history of heavy alcohol use, drinking up to 12 cans of beer per day for over 10 years. His blood pressure is 100/80 and HR 90 beats per minute. Blood tests show Hb is 75, platelets 90, urea 10 and creatinine 75. Which of the following would not be recommended in his management? A. Gastroscopy B. Blood transfusion C. Proton pump inhibitor D. Terlipressin

D. terlipressin Vasoactive therapy for AVH Should be initiated ASAP/before endoscopy • Decreases active bleeding, facilitates endoscopic therapy, improves control of bleeding and potentially survival Terlipressin • 1-2mg IV 4-6 hourly Can cause myocardial ischaemia so best avoided with unstable angina Somatostatin/Octreotide • IV bolus then continuous infusion Other management Restrictive transfusion approach usually preferred (target Hb 70-90) but higher end reasonable given cardiac history • PPI therapy not effective for AVH but reasonable considering patient on aspirin and cirrhosis/varices not yet proven • Short course of PPI after band ligation may reduce post-banding ulceration

Dun cardio A 25-year-old female indigenous patient has a history of rheumatic fever. She presents with increasing shortness of breath. She describes paroxysmal nocturnal dyspnoea and pedal oedema. On examination her blood pressure is 95/60 mmHg. The jugular venous pressure is elevated with a prominent a wave. The S1 is loud, S2 is normal. There is an early diastolic sound after S2 with a diastolic rumble at the apex. There is also an early diastolic murmur which is accentuated with expiration. There is hepatomegaly and pitting oedema to the mid shin. What is the most likely diagnosis? A. Mitral stenosis. B. Mitral stenosis and tricuspid regurgitation. C. Mitral stenosis and tricuspid stenosis. D. Mitral stenosis and aortic regurgitation. E. Mitral stenosis and pulmonary stenosis

D??? why

CLS ID A 20-year old man presents with a one-day history of fever, headache and neck stiffness. CSF shows 4,000 white cells/μL (100% PMNs), with gram-negative diplococci on Gram stain. Which of the following is true? a) This organism remains universally susceptible to penicillin b) Antibiotic prophylaxis is indicated for all involved healthcare workers c) Dexamethasone reduces mortality, hearing loss and neurologic sequelae in this condition d) He is likely to have a non-blanching petechial rash e) He should be managed in droplet precautions until he has received 24 hours of effective antibiotics

E

CLS Haem 67 man, presents with new onset new angina. Clinical examination reveals him to be plethoric and hypertensive His laboratory studies indicate the following: Haemoglobin 205 g/dL 128-175 Haematocrit 57% 36-50 Total WCC 11.6 x 10^9/L 4-8 Neutrophils 9.5 x 10^9/L 2-4 Lymphocytes 0.6 x 10^9/L 2-4 Eosinophils 0.8 x 10^9/L 0.0-0.5 Basophils 0.4 x 10^9/L 0-0.1 Platelets 499 x 10^12/L 150-450 Serum Erythropoietin 3.9 units 4.8-21.9 A. Carcinoma of lung B. High affinity haemoglobin. C. Dehydration. D. COPD. E. Polycythaemia vera.

E

CLS Haem Adult Ph+ CML (chronic phase), treatment with which of the following is associated with highest rate of Complete Molecular Remission? A. Hydroxyurea. B. Radioactive Phosphorous. C. Interferon. D. Cytosine arabinoside ( Ara-C). E. Tyrosine Kinase Inhibitor

E

CLS Resp A 50yo male, current smoker presents with 2 years of worsening dyspnoea. On examination there is widespread wheeze and CXR demonstrates hyperinflation. Respiratory function tests are below: FEV1 1.2 (40% predicted) TLCO 45% predicted RV/TLC 0.55 (upper limit 0.46) FVC 1.9 (60% predicted) KCO 50% predicted FER 0.63 (lower limit 0.69) TLC 115% predicted Which of the following strategies are most likely to slow progression of lung disease in this person? A. Flu vaccination B. Long acting beta agonist (LABA) C. Long acting muscarinic antagonist (LAMA) D. Inhaled corticosteroid/LABA (ICS/LABA) E. Smoking cessation

E

CLS Resp Pulmonary embolism is a life threatening condition that doesn't always present in the same manner. What factor on the Wells Criteria contributes the highest risk? A. Haemoptysis B. Immobilisation C. Tachycardia D. Malignancy E. PE as likely or more likely than other DDx

E

CLS Rheum A 52 year old female complains of 3 years of progressive dry eyes. Investigations: positive ANA speckled, 1:640, ENA + Ro and La, ESR 98. She is otherwise well with normal examination apart from Schirmer's test 1mm, and positive Rose-Bengal staining. The best initial treatment is: A. Prednisolone B. Hydroxyurea C. Methotrexate D. Prednisolone eye drops E. Hypomellose tear substitute

E

DM ID 28yo presents with red, cellulitic leg with very severe pain in the thigh. Temp=39.5, HR=130, BP=95/55. WCC, CRP, creatinine and CK all elevated. Best management? • A - Surgical exploration and debridement of devitalised tissue • B - IV benzylpenicillin & clindamycin • C - IV meropenem • D - IVIG • E - All of the above

E

DM ID 34 year old male recently returned from a business trip to Bangkok. He has had several days of worsening asymmetric oligoarthritis. No fever. Most correct answer? • A. A high CRP or high joint aspirate cell count would differentiate septic arthritis from gout • B. Disseminated gonococcal infection is unlikely in the absence of a rash/fever • C. Send off that new whiz-bang alpha-defensin test to sort out whether this is infection or not • D. If he's been up to no good, empiric ciprofloxacin is indicated to treat for DGI • E. As per D but empiric ceftriaxone rather than ciprofloxacin

E

DM ID 52 yo female with RA, originally from Laos. Rheumatologist keen to start infliximab. BCG scar is present. Screening Quantiferon is positive. Which of the following is true? • A - Infliximab is contraindicated • B - Infliximab can be given as she has had BCG • C - Infliximab can only be given after 9 months of isoniazid • D - Infliximab could be started after one month of isoniazid • E - The QFN result is likely false +ve from the BCG

E

DM ID 53 yo M has 2 days of red, hot & swollen knee. LMO gave cefalexin yesterday. Temp 37.4O and WCC 11.3. • Aspirate has 130,000 cells/mm3 with 95% neutrophils. Gram stain is negative and no crystals. • Most appropriate initial management? • A - IA steroids • B - IV fluclox • C - Oral cephalosporin • D - Oral colchicine • E - Surgical drainage of the knee

E

DM ID 76 yo M with mechanical MVR needs gastroscopy & biopsy. What endocarditis prophylaxis is appropriate? • A - Amoxicillin • B - Ceftriaxone • C - Gentamicin • D - Metronidazole • E - No prophylaxis

E

DM ID A 60-year-old male returns from trip to Hong Kong. 3 months later has a TRUS biopsy. 2 days later, fevers and rigors. E. coli cultured from urine & blood. Ceftriaxone started. Call from lab: 䇾It looks like an ESBL䇿. Patient no better. Appropriate management: • A - Change to fosfomycin • B - Change to ciprofloxacin • C - Add gentamicin • D - Change to tigecycline • E - Change to meropenem

E

DM ID Which of the following best explains the mechanism of resistance to vancomycin in enterococci? • A. The Gram-positive cell wall is thickened leading to an increased number of D-Ala-D-Ala molecules that trap the vancomycin • B. Porins excrete the vancomycin that diffuses into the enterococci • C. It is due to alterations in the glycopeptide binding proteins • D. There is intracellular methylation of vancomycin • E. The D-Ala-D-Ala in the cell wall is changed to D-Ala- D-Lac which binds vancomycin with lower affinity

E

DM ID Which of the following is most helpful in differentiating smallpox from chickenpox? • A - Lesions in the same body part are at the same stage in varicella • B - Smallpox patients have no or mild prodrome • C - Smallpox lesions progress more rapidly than varicella lesions • D - Patients with smallpox are usually afebrile • E - Varicella tends to give a greater concentration of lesions on the trunk compared to the limbs

E

DM ID • Which other the following is the strongest contributor to the risk of colonisation with VRE? • A - Fluconazole • B - Long term corticosteroid use • C - Recent C. diff infection • D - Renal failure • E - Residential care

E

DM Resp Which of the following best describes the mechanism of action of riociguat? A. Calcium channel blockade B. Endothelin A receptor antagonist C. Endothelin A&B receptor antagonist D. Phosphodiesterase type 5 inhibition E. Soluble guanylate cyclase stimulation

E

Deltamed Gastro A 27 Y.O. male is diagnosed with Crohn's colitis with an IBD-arthropathy and requires steroids at diagnosis. Six months later he develops his first perianal fistula. Which clinical feature is associated with the most aggressive phenotype of Crohn's disease? A) Colonic disease location B) Young age of onset C) IBD arthropathy D) Need for corticosteroids at diagnosis E) Perianal disease

E

Deltamed ID Q. Which of the following is correct regarding the New Delhi Metallo-beta-lactamase (NDM-1) enzyme? • A. This encodes for resistance to all extended spectrum penicillins and cephalosporins but is susceptible to carbapenems and aztreonam • B. This is mostly seen in Salmonella and Shigella species • C. This enzyme can only be transferred between bacteria of the same genus and species • D. Bacteria carrying this enzyme have only been found in India and Pakistan • E. Treatment options are limited but may include colistin, cefiderocol or tigecycline

E

Deltamed Immuno Dendritic cells are distinguished by an exceptional ability to carry out which one of the following immune functions? A. Phagocytosis B. Cytokine secretion C. Chemokine secretion D. Immunoglobulin secretion E. Antigen presentation

E

Deltamed Immuno The large granular lymphocytes observed in the early cellular response to virus‐infected cells are most likely to be A. B lymphocytes B. Naïve T lymphocytes C. Memory CD4+ lymphocytes D. Memory CD8+ lymphocytes E. Natural Killer cells

E

Dun cardio A 60-year-old man who previously had a rash following flucloxacillin therapy has aortic valve endocarditis due to flucloxacillin sensitive Staphylococcus aureus. He has been treated for one week with cefazolin 2 g, 8 hourly intravenously. He develops pulmonary oedema and has a new early diastolic murmur in the aortic area. What is the most appropriate management? A. Addition of gentamicin to cefazolin. B. Change to flucloxacillin after rapid desensitisation. C. Change to vancomycin and rifampicin. D. Transfer to ICU for aortic balloon pump insertion. E. Urgent aortic valve replacement

E

Dun cardio A 65-year-old man presents to the emergency department with ischaemic chest pain. No abnormalities are noted on examination apart from mild hypotension. His full blood examination shows: haemoglobin 125 g/L [128-175] white cell count 10.2 x 109 /L [3.9-12.7] (normal white cell differential) platelets 180 x 109 /L [150-396] Coagulation investigations show: activated partial thromboplastin time (APTT) 100 seconds [26-38] APTT 1:1 mix (patient: normal plasma) 52 seconds prothrombin time-international normalised ratio (PT-INR) 1.2 [1.0-1.3] thrombin clotting time (TCT) 200 seconds [<24] fibrinogen 4.5 g/L [2.0-4.0] reptilase time 18 seconds [<24] The most likely cause of the coagulation test abnormalities is: A. disseminated intravascular coagulation. B. administration of low-molecular weight heparin. C. administration of thrombolytic agent. D. lupus inhibitor. E. administration of unfractionated heparin

E

Dun cardio A 68-year-old diabetic has chest pain and T wave inversion in the precordial chest leads. In this patient, elevation of which biomarker is most predictive of an adverse outcome? A. B natriuretic peptide. B. Myeloperoxidase. C. Blood glucose. D. High-sensitity C-reactive protein. E. Troponin.

E

Dun cardio A 70-year-old male presents with an irregular heart beat and the ECG confirms atrial fibrillation. In terms of stroke prevention, which of the following is the strongest indication for recommending long term oral anticoagulation in this patient? A. The patient's age. B. Hypertension. C. Diabetes. D. Dyslipidaemia. E. Mitral stenosis.

E

Dun cardio A 74-year-old woman with severe heart failure (NYHA Class III) due to longstanding hypertension is now stable on frusemide, metoprolol and perindopril. Blood pressure is 140/75 mmHg; ECG shows sinus rhythm with left bundle branch block (QRS duration 130 mSec). Which of the following would provide the greatest improvement in her survival probability? A. Add digoxin. B. Change metoprolol to carvedilol. C. Add candesartan. D. Implant a biventricular pacemaker. E. Add spironolactone.

E

Dun cardio Which of the following factors is most strongly associated with risk of sudden death in the first six months after myocardial infarction? A. Persistent occlusion of the infarct-related artery. B. Cigarette smoking. C. 3-vessel coronary disease at angiography. D. Low left ventricular ejection fraction. E. High LDL (low density lipoprotein) cholesterol.

E

Dun stroke In patients presenting with subacute lower extremity symptoms, the presence of pyramidal lower extremity weakness, a mid‐thoracic sensory level to pin‐prick but relatively preserved posterior columns is most consistent with: A. Subacute combined degeneration of the cord. B. Brown‐Séquard syndrome. C. Syringomyelia. D.A central cord syndrome. E. An anterior spinal artery syndrome

E

Dunedin Haem Which of the following is true about VWF? A. VWD normally circulates as a multimer of 10 subunits B. VWD can be diagnosed if the VWF levels is repeatedly less than the 50 IU/dL (ref int 50-150) C. VWF is exclusively stored in subendothelial connective tissue D. Increased secretion of VWF is the cause of TTP E. People with Type O blood group have lower VWF levels

E

Dunedin endo A 19 year old presents with a BP 210/110, there is a family history of hypertension affecting her mother and her maternal grandmother.Her K+ is 3.5mmol/L(3.5-5.2). How should you proceed? A. Treat immediately with ACE and labetalol B. Treat after investigation with ACE and labetalol C. Perform a 24 hour BP recording before investigation D. Perform genetic testing and treat with a thiazide E. Perform renin/aldosterone levels and then treat with an ACE or calcium channel blocker.

E

Dunedin endo FEATURES SPECIFIC TO GRAVES DISEASE INCLUDE? A. Tachycardia B. Lid lag C. Reduced bone density D. Amenorrhea E. Pre-tibial myxodema

E

Dunedin endo Question 5: Which of the following statements regarding hypoparathyroidism is most correct? A. The commonest cause is idiopathic hypoparathyroidism B. rhPTH (1-84) therapy normalises 24 hour urinary calcium excretion C. Calcium carbonate is preferred in patients receiving a PPI D. Bone density is usually reduced at all sites E. Maintaining a serum calcium x phosphate product of < 4.4 mmol2 /L2 is recommended

E

Dunedin gastro 29 year man. No significant medical problems besides asthma. Presents with two year history of intermittent dysphagia for solids. Minor heartburn. No weight loss. Shown are endoscopy findings. Test most likely to give diagnosis: a. Barium swallow b. CT chest c. Connective tissue screen d. Oesophageal manometry e. Oesophageal biopsy

E

Dunedin gastro 65 yr lady. Hypertension, Type II Diabetes, previous TIA and NV AF Drugs: Cilazapril, metformin, insulin, rivaroxaban Recent first episode gout treated by patient with NSAIDs Presents with a pre-syncopal event associated with passage of several melaena stools. BP 90/65, pulse 115, Hb 83. Patient transfused 2 units, BP 120/70, pulse 95, repeat Hb 100, further passage of one melaena stool. Besides stopping NSAID and withholding rivaroxaban, which of the following would be most sensible approach: a. Vitamin K IV + FFP and scope within 6 hours b. Give platelet transfusion, and wait 24 hours then scope c. Recombinant activated factor VII + Prothrombin complex concentrate (PCC) and scope urgently d. Idarucizumab IV and scope within 24 hours e. Continue resuscitation and if stable wait 24 hours then scope

E

Dunedin gastro Patient proceeds to manometry, result is shown (type I achalasia). Based on manometry finding your next step would be: a. Prescribe oral budesonide b. Fundoplication surgery c. PET CT d. High dose PPI + tricyclics e. Surgical referral for lower oesophageal sphincter myotomy

E

Dunedin gastro With respect to coeliac disease, which of these is incorrect? a. There is an increased incidence in 2nd degree relatives b. Gluten free diet will improve dermatitis herpetiformis c. Is associated with increased incidence of pancreatitis d. Is strongly associated with microscopic colitis e. Associated with recurrent miscarriage and impaired fertility

E

Dun Renal A 20 year old male presents with haematuria 36 hours after an URTI. On examination, his blood pressure is normal, urinalysis demonstrates 2+ protein, 2+blood. Renal function is normal. The renal biopsy is shown. What is the most likely diagnosis? a. Goodpasture's syndrome. b. Post infectious GN. c. Wegener's granulomatosis. d. Minimal change GN. e. IgA nephropathy.

E (synpharyngitic cf: PIGN)

Dun Neuro A 22-year-old woman has been on a stable dose of anticonvulsant for three years. She has recently experienced difficulties in seeing at night. A formal ophthalmological assessment demonstrates visual field constriction . For which one of the following anticonvulsants is it important to monitor visual fields? A. Carbamazepine B. Gabapentin C. Lamotrigine. D. Valproate E. Vigabatrin

E (vigabatrin for viusal. also topiramate)

Dun Neuro Which of the following is least likely to improve after plasmapheresis? A. MG B. CDIP C. TTP D. mesangiocapillary GN secondary to cryoglob E. MS

E (would help NMO)

CLS ID A 50 year old woman with coeliac disease and autoimmune thyroiditis is being investigated for anaemia; her blood film demonstrates occasional Howell Jolly bodies. Current prevention of infection guidance would suggest she needs: A. Conjugate pneumococcal vaccine (PCV13) every 5 years B. 23 valent polysaccharide vaccine (PPV23) at least once C. Lifelong oral amoxicillin daily D. Polysaccharide ACWY meningococcal vaccine E. PCV13 one dose followed by PPV23

E 1. PCV13 then PPV23 5 yrly (x3) 2. 4MenCV 2 doses then 5 yrly 3. MenBV 2 doses 2 mths apart 4. Hib 5. Influenza annually

Dunedin pharm The term linear pharmacokinetics means: A. the plot of a drug concentration versus time is linear B. half‐life increases proportionally with dose C. a constant amount of drug is eliminated per unit time D. clearance is proportional to dose E. steady state concentration is proportional to dose

E 1st order

DM PT ID 5 The epidemiology of culture-negative endocarditis has changed over time. Please choose the most appropriate causative pathogen for the clinical scenarios described below. A. Tropheryma whipplei B. Coxiella burnetii C. Bartonella quintana D. Cardiobacterium valvarum E. Streptococcus mutans F. Haemophilus aphrophilus G. Mycobacterium haemophilum H. Staphylococcus aureus 5. A 24-year-old university student presents with one month of intermittent fevers, and visual change in his right eye over the last 5 days. He underwent a dental procedure two months ago. The patient's general practitioner prescribed oral amoxicillin two weeks ago for a presumed upper respiratory tract infection. On examination, fundoscopy shows Roth spots. Multiple blood cultures are drawn but remain negative. A transthoracic echocardiogram shows a 1.2 cm mitral vegetation.

E 5. ANS: E • While culture-negative endocarditis usually prompts work-up for unusual pathogens, a significant proportion of culture-negative cases are due to more common pathogens such as viridans streptococci (including S. mutans). • These bacteria can be very sensitive to penicillins; hence, any antibiotic administration may affect the ability to culture them from the blood. The stem is highly characteristic with a presentation of subacute bacterial endocarditis on a background of recent dental procedure. • 16S rRNA PCR performed on valve tissue has become an increasingly important diagnostic modality in culture-negative endocarditis and can help to diagnose all of the above listed pathogens

Dun Pharm Which of the following drug interactions is based on inhibition of a transporter? A. St John's Wort and ciclosporin B. Erythromycin and warfarin C. Phenytoin and carbamazepine D. Valproate and lamotrigine E. Quinidine and digoxin

E A - CYP3A4 = reduces ciclos B - inhibits 3A4 = increases warfarin D - vapro competes with lamotrigine for glucose (ie. inhibits) so Va doubles lamo E quinidine blocks p glyco = inhibits secretion of dig = increases level

CLS Resp A 50yo woman with an unprovoked symptomatic pulmonary embolism presents to your rooms for review after 3 months of anticoagulation. She wants to know how she can reduce her risk of recurrence. Which of the following statements are most correct? A. Extending anticoagulation for another 3 months will reduce her lifetime risk of recurrent PE B. A normal D-dimer level while on anticoagulation predicts a lower risk of recurrence C. Low dose Aspirin therapy has no influence on the risk of recurrent VTE D. Only those with an identifiable thrombophilia benefit from prolonged anticoagulation E. Prolonged anticoagulation may be appropriate if bleeding risk is not high

E A negative D-dimer at 1-3 months off anticoagulation may identify a low-intermediate risk of recurrence (although positive results are non-specific) but testing while on anticoagulation doesn't have predictive value Variable recommendations in guidelines but a shift in approach appears to be happening In 1st unprovoked PE Point 1 3 months of anticoagulation is the duration of choice Point 2 If low to moderate bleeding risk prefer extended therapy (indefinite) over 3 months (Grade 2B recommendation = weak "this is our suggestion but you may want to think about it") Apixaban 2.5mg bd (low dose Apixaban) now on PBS • Prevention of recurrent venous thromboembolism • Treatment Phase: Continuing treatment Clinical criteria: • Patient must have a history of venous thromboembolism

CLS ID A 78 year old man presented yesterday with fever, cough and an exacerbation of COPD. His fever and other symptoms started 5 days ago and his chest X-ray has not changed since his last admission 4 months ago. He was treated with an IV cephalosporin and admitted to a 4-bedded room with other patients. This morning his nasopharyngeal swab is positive for influenza A. What is the most useful next course of action? A. Treat the man with a neuraminidase inhibitor B. Change his IV antibiotic to oral amoxycillin C. Vaccinate the man against seasonal influenza D. Vaccinate the other people in the room against seasonal influenza E. Treat the other patients in the room with oseltamivir

E A. Treat the man with a neuraminidase inhibitor after 4 days of illness is of little benefit B. Change his IV antibiotic therapy to oral amoxycillin he doesn't have pneumonia (CXR unchanged) - stop antibiotics C. Vaccinate the man against seasonal influenza potentially useful, but will not change the current illness D. Vaccinate the other people in the room against seasonal influenza potentially useful, but the incubation period for influenza is very short (2 days) so this will not prevent them becoming unwell after exposure to this man E. Treat any other non-vaccinated patients in the room with oseltamivir has been shown to prevent influenza illness, perhaps is the best role for oseltamivir

Book R9 77 y/o man presents with severe R knee pain for 8hrs. Nil trauma or consti sx. CHG, CD, HTN. 2m ago = admitted for UGIB Very uncomfortable. Marked swelling + redness of R knee with effusion. Which 1 should be undertaken? A. NSAIDs B. COX2-Is C. High dose colchicine for 2 weeks D. allopurinol E. arthrocentesis followed by administration of intra-art roids

E Acute gout: colchicine, NSAIDs, roids. E. - can evaluate fluid, relieve pain of large effusion

Dun ID The most common mechanism of vancomycin resistance found in Enterococcus faecium is: a. Porin loss in the outer lipid membrane b. Production of a beta-lactamase c. Efflux pumps d. Acquisition of the the Van C gene cluster e. Amino-acid substitution in the pentapeptide side chain

E Alteration in the penta-peptide sidechain with substitution of the terminal alanine molecule with a lactate. Porins are found in Gram negative bacteria. Enterococcus is a Gram positive bacteria Beta-lactamases destroy beta-lactam drugs. Vancomycin is not a beta-lactam! Van C gene cluster is a naturally occurring set of genes found on chromosomes of specific Enterococci species (E. gallinarum & E. casseliflavus).

Deltamed Onc 78 y/o man. new dx prostate ca with bony mets. Lost 5kg. Independent, walks unadided. Recommended to start ADT with goseralin + bicalutamide. Rationale behind 2 rxs? A. GnRH-I and antiandrogen = better PSA response B. GnRH ag + antiandrogen = better long term tox profile C. Combining GnRH-I and antiandrogen = improved survival D. GnRH inhi + antiandrogen prevent disease flare due to transient rise in testost E. combining GnRH ag + antiandrogen = prevent diease flare etc

E GnRH agonist = initial increase in testost then decrease

D9 36 y/o redness and itching with tiny vesicles, scaling, fissuring. Simple emolients haven't helped. Which of the following is NOT used for the rx of severe hand eczema? A. topical roids B. topical tacrolimus C. oral retinoids D. oral ciclosporin E. hydroxychloroquine

E Hand eczema

CLS Resp A 50yo male, current smoker presents with an acute increase in cough, sputum and dyspnea on a background of 2 years of gradually worsening dyspnoea. On examination there is widespread wheeze and CXR demonstrates hyperinflation. Respiratory function tests are below: FEV1 1.2 (40% predicted) TLCO 45% predicted RV/TLC 0.55 (upper limit 0.46) FVC 1.9 (60% predicted) KCO 50% predicted FER 0.63 (lower limit 0.69) TLC 115% predicted Which of the following treatments are most likely to reduce exacerbations over the next 12 months A. Simvastatin B. Long acting beta agonist (LABA) C. Long acting muscarinic antagonist (LAMA) D. Inhaled corticosteroid/LABA (ICS/LABA) E. LABA/LAMA

E Least effective - LABA monotherapy Next - LAMA monotherapy or ICS/LABA Best - LABA/LAMA

Dun Rheum Which of the following is the most likely reason for poor response to methotrexate in a patient with Rheumatoid arthritis? A. Obesity B. Renal failure C. Smoking D. Increased age E. Poor compliance

E Look up MTX SEs (how common) and when you need to renally adjust

Deltamed endo Which of the following scenarios would best explain the following TFT results? TSH 3.2 [0.4-4.0] fT4 37 [9-19] fT3 8.2 [3.5-6.5] a. Untreated Graves' disease b. Toxic thyroid nodule c. Thyroiditis d. Recombinant TSH administration for thyroid cancer screening two days prior e. TSH-secreting pituitary adenoma

E Not D as recombinant TSH has very short half life

CLS Resp A tall 17yo male presents to hospital with mild chest discomfort on inspiration which came on while playing basketball. A pneumothorax measuring 2cm at the hilum is identified on plain CXR. He has had no previous episodes. Which of the following management options would not be appropriate? A. Aspiration at the bedside using cannula, 3-way tap and syringe B. Small bore intercostal catheter using Seldinger technique C. Small bore intercostal catheter using blunt dissection D. Observation E. Surgical referral for VATS pleurodiesis

E Of the interventional approaches - aspiration has the shortest LOS and failure rate is low Both ICC insertion techniques are reasonable although Seldinger technique is very common now and generally straightforward Surgery generally not be indicated for a first episode unless he was a pearl diver - in which case both sides would need to be done

Deltamed Onc Which of the following features in a pt with CRC is most suggestive of HNPCC? A. Right sided lesion B. MSI in tumour tissue C. multiple synchronous polyps D. Bro with colon ca age 55 E. uncle with germline MLH1

E Others may suggest but less specific

CLS Resp A 72yo man with CKD presents with a one week history of intermittent fevers, mild chest pain and dyspnoea. CXR is shown. Which of the following is not a useful prognostic factor for this man's current illness? A. Serum urea B. Age C. Pleural fluid appearance D. Serum albumin E. Serum CRP

E RAPID score

Dunedin Haem Which pair is incorrect? A. B cells - CD19 B. T cells - CD3 C. Granulocytes - CD33 D. Monocytes - CD64 E. Stem cells - CD45

E Stem cells CD34

Dun ID S. aureus resistance to vancomycin is mediated through the VanA gene and is becoming increasingly prevalent worldwide. Which mechanism BEST explains the rapid spread of this drug resistance? a. Spontaneous chromosomal mutation b. Vertical transmission of chromosomally encoded resistance c. Transduction d. Transformation e. Conjugation

E • Conjugation: Direct cell-to-cell contact considered main mode of horizontal gene transfer of antibiotic resistance genes. Plasmid exchange via a bacterial pilus. • Transduction: transfer to genetic material from one bacterium to another using a viral vector (i.e. a bacteriophage). • Transformation: free short fragments of DNA (through death and lysis of another bacterium) are taken up by naturally transferable bacteria

CLS Resp A 60yo male with dyspnea for investigation. A TTE performed showed a RVSP of 70mmHg. His right heart catheterization revealed a mPAP of 45mmHg and a PCWP of 25mmHg. What is the next best treatment? A. Sildenafil B. Bosentan C. ISMN D. Nifedipine E. Frusemide

E Elevated PCWP indicates elevated left heart pressures (=LVEDP) and therefore left heart disease is probably the explanation for the pulmonary hypertension Normal mean pulmonary artery pressure (mPAP) is 10-20mmHg Pulmonary arterial hypertension = mPAP >25mmHg (rest) TTE or TOE can approximate PA systolic pressure (PASP PASP=RVSP=tricuspid pressure gradient + CVP (in the absence of RV outflow obstruction or pulmonary stenosis) RVSP (>40mmHg) = estimated mPAP (>25mmHg) by using a conversion mPAP=0.61xRVSP + 2 TTE can over- or under-estimate PASP - up to 48% in one observational study More accurate than clinical history and examination but should be confirmed with RHC RHC is gold standard allowing confirmation and evaluation of cause • Measures mPAP - mild (25-40), moderate (41-55), severe >55 • Also PCWP=LVEDP (except in mitral stenosis, pulmonary venous obstruction) >/=15mmHg suggests left heart disease (but with false positive rate of 5%)

CLS Renal Hyperphosphatemia is a common problem in patient with end stage renal failure requiring dialysis. Which additional treatment most effective in maintaining normal serum phosphate? A. Calcitriol B. Low phosphate diet C. Low protein diet D. Calcimimetic agents E. Phosphate binders

E • PO4 binders - best option from choices • To take WITH food (including snacks!) • Compliance an issue esp. with pill burden • Caltrate/calsup, sevelamer (renagel), lanthanum (fosrenol), sucroferric oxyhydroxide (velphoro), alutabs • PO4 in most food - dietary restriction difficult and risk of malnutrition (hypoalbuminaemia - strong predictor of mortality) • Protein restriction - may lead to malnutrition; no definitive evidence of benefits in humans Calcitriol - 1,25 OH vitamin D reduces PTH secretion (directly or indirectly via relative hypercalcaemia) but increase GI absorption of Ca & PO4 (unwanted SEs) • Avoid relative hypercalcaemia, hyperphosphataemia and "low" PTH (adynamic bone disease) - vascular calcification • (Parathyroidectomy - for 2° hyperparathyroidism (SHPT) if severe esp. resistant to cinacalcet; does drop PO4 slightly) • Calcimimetics (cinacalcet) - see later • Extended hours haemodialysis - PO4 removal is TIME-dependent

Deltamed Haem 2 Which of the following statements regarding VWD is correct? A. Type 1 is characterised by undetectable levels of plasma VWF B. Type 2 VWD are mostly autosomal dominant inheritance. C. Patients with type 3 VWD do not commonly exhibit bleeding manifestations unless subject to severe haemostatic challenge. D. PFA‐100 is normal in patients with VWD. E. VWF levels vary significantly for physiologic reasons and repeat testing may be needed to confirm the diagnosis of VWD.

E.

Deltamed Onc Man with node positive sporadic CRC was treated 12m ago with surg + chemo. Being F/U with reg CEA and annual CTs. Detecting which of the following would be the strongest reason for following this pt up in this way? A. anastomotic recurrence b. separate primary malignancy C. minimal residual disease so that chemo can be commenced D. pancreatic cancer E. resectable secondaries to liver + lung

E.

Deltamed liver Which of the following is a contraindication to the use of Direct ActingAntiviral (DAA therapy) A. Active ongoing IVDU B. Decompensated cirrhosis C. Chronic schizophrenia D. End stage renal impairment E. Concomitant carbamazepine administration

E.

Deltamed renal 105.In the evaluation of a patient with suspected kidney disease, the use of a urinary dipstick (Multi-stick) may fail to detect significant quantities of which of the following? A. Albumin. B. Erythrocytes. C. Leucocytes. D. Nitrites. E. Non-albumin protein.

E.

What systolic BP target is acceptable in patients with renal impairment and proteinuria? A.160mmHg B.150mmHg C.140mmHg D.130mmHg E.125mmHg

E.

deltamed renal A 45 year old man presented with nephrotic syndrome and hypertension. A renal biopsy revealed focal and segmental glomerulosclerosis, thought to be primary in origin. His estimated glomerular filtration rate is 40 ml/min/1.73 m2 [normal 90- 150]. He has failed to respond to a prolonged course of prednisolone. Which of the following approaches to therapy offers the best chance of long-term renal survival: A. Fish oil capsules daily for 2 years B. Cholesterol control with a "statin". C. Dietary protein restriction D. Dietary salt restriction E. Blood pressure control

E.

R 27 17 y/o M. arthralgia, abdo pain, smoky urine. Palpable rash both legs. Urine = RBCs. Normal complement, blood tests. What's the most likely dx? A. EN B. cryoglobulinaemic vascultiis C. contact dermatitis D. EM E. HSP

E. Palpable purpura, ritis/ralgia, abdo pain and haematuria = HSP. Immune medaited. IgA deposition Skin bx = leucocytoclastic vasculitis with IgA staining Renal involvement in 30% of kids (more in adults). IgA deposition Cryo = palpable purpura, low complement levels

Deltamed obs TNF inhibitors are used for management ofinflammatory bowel disease and rheumatological conditions in pregnancy. The following statement is true: a. All TNF inhibitors cross the placenta equally b. Transmission to the fetus is equal across all 3 trimesters c. TNF inhibitors should be stopped in pregnancy and steroid doses increased to manage symptoms d. TNF inhibitors should be withheld in 1st trimester and restarted in the 2nd to decrease risk of congenital malformations e. Live vaccines should be avoided in the first 6 months of life in newborn

E. a - Depends on the Fc portion b. greatest in 3rd trim c. Greater risk of infections with higher pred doses > 10 mg d - No increase risk reported lowest risk of increasing disease activity e - Rotavirus , MMR at 12 months ok

Deltamed obs Which of the following is incorrect: a. Metabolism of anti epileptic drugs (AED)may significantly increase with pregnancy and dose adjustment is necessary b. Sodium valproate should be avoided in women of child bearing age c. Initiation of folic acid 5mg mg pre conception is associated with better outcomes in fetus d. Levels of AED rapidly return to normal post partum e. Lamotrigine induces metabolism of combined oral contraceptive

E. a Especially with lamotrigine levels normally need increasing b highest risk malformations c High dose folic acid 5 mg d. true e. The combined oral contraceptive induces metabolism of lamotrigine

deltamed renal Which risk factor best predicts development of renal failure? A. Age B. Smoking C. Hypertension D. Obesity E. GFR<90

E. organ failure predicts organ failure DM > age > HTN > smoking

Book D5 51 y/o man with 4m of gradually worsening rash on back of hands. Small blisters initially. Rash has worsened (espec with sun exposured). He is concerned about scarring marks. Non-smoker. 2-3 alcohol/day. Similar lesions on face. Raised ALT and AST. What is the most likely dx? A. Bullous pemphigoid B. Cutaneous lichen planus C. dermatitis herpetiformis D. eczema E. porphyria cutanea tarda

E. Not itchy. Porphyria cutanea tarda = acquired def of UROD = accumulation of uroporphyrinogen. Mild elevations in serum liver enzymes. Alcohol is an impt + common susceptibility factor Backs of hands, forearms, face, ears neck Marked increase in urine porphyrins with predominance of uroporphyrin. Usually normal erythrocyte porphyrins. Skin bx: subepidermal bullae Bullous pemphigoid = subepidermal blistering, ollder adults. Cutaenous LP - shiny, flat, polygonal violaceous papules Dermatitis herpetiformis: intensely itchy

Deltamed Genetics A recessive condition is known to have a carrier frequency of 1/8 in the community. What is the likelihood that a couple with unknown carrier status will have an affected child? A) 1/16 B) 1/32 C) 1/64 D) 1/128 E) 1/256

E. 1/256 1/8 x 1/8 x 1/4 = 1 /256

R 30 37 y/o with SLE receiving roids for 4yrs. Initially 60mg/day for 3m, then tapered over 18m. Currently 15mg/day R groin pain, difficulty walking. Pain at rest. Atrumatic. Most likely dx? A. osteoporotic NOF B. RA C. R trochanteric D. Gout E. AVN

E. AVN Steroid use + alcohol intake.MRI is most sensitive.

Dun Neuro A 25 year old woman presents with a 2 year history of lower back pain, anxiety and episodes of spasm affecting limb and truncal muscles. • Examination reveals increased tone generally, with brisk reflexes, normal strength and flexor plantars. MRI studies of brain and spine are normal, apart from showing lumbar lordosis. • B12 and TSH are normal. • CK is normal. • EMG show abnormal continuous muscle activity at rest in trunk and limbs. Which antibody is most likely to be relevantly positive? A. NMDAR B. LGI1 C. Thyroid TPO D. ANA E. GAD

E. Stiff person syndrome

Dun Rheum In Dermatomyositis, all of the following infer a greater risk of underlying malignancy except: A. Cutaneous necrosis B. Cutaneous leukocytoclastic vasculitis C. Older age at disease onset D. Dysphagia E. Interstitial lung disease

E. actually lower risk

A. antiScl70 B antidsDNA C. anti SSA or antiRo D antiCCP E. antihistone F. ANCA G. anti ribosomal P H. anti U1 RNP R32 - Hydralazine induced lupus, which Ab detected in <95% of cases? R33 - SLE, which can cause neonatal heart block? 34 - Systemic sclerosis. Increased risk of ILD? 35 - RA, AB associated with increased risk of progressive joint damage?

E. antihistone >95% of drug induced lupus (procainamide, hydralazine, chlorpromazine, quinidine). Also seen in 80% with idiopathic SLE. antidsDNA sbsent in the above agents but more common with TNFA and IFN C. SLE + sjogren. Associated with photosens, ILD< neonatal lupus, congenital heart block A. Diffuse cutaneous SSc. Higher risk of severe ILD. Assoc with disease activity D. increased risk of progressive joint damage, May predict erosive disease more than rheumatoid factor. Also predicts increased risk of RGP for those who are RF-

Dunedin gastro 78 yr lady. History NSTEMI 2012 (no stent), CVA 2006, recurrent TIAs & HPT. Medications: aspirin, ibuprofen, metoprolol, cilazapril, citalopram. Presents with history abdominal pain, and 24hrs hematemesis and melaena. Haemodynamically stable, Hb 75, urea 24.3 Cr 100 (e-GFR 67) PR melaena. Started on PO PPI. Aspirin and NSAID withheld. At next list this gastric lesion is seen at endoscopy (non bleeding vessel). You would advocate: a. No intervention, post endoscopy PPI infusion b. No intervention, post endoscopy PO PPI c. Endoscopic mucosal resection, post endoscopy PO PPI d. Adrenaline injection, post endoscopy PPI infusion e. Adrenaline injection + clip, post endoscopy PPI infusion

E. dual is best

Dunedin endo A 34 year old woman reports reduced libido and mild depression and her GP tests her prolactin level which is 960 mIU/L (N<500mIU/L). A repeat test is at a similar level. Her thyroid function is normal. She has regular periods and no history of galactorrhoea. She had a recent normal pregnancy and was able to stop breast feeding without persistent galactorrhoea. Which of the following tests should now be done? A: Pituitary MRI B: LH and FSH measurement C: pregnancy test D: Measurement of prolactin at 1:100 dilution E: Macroprolactin assessment

E. macroprolactin = spurious

Dunedin Rsep The following ABG is taken from a 40 year-old male with breathlessness and SpO2 of 86%: pH 7.38, ΡCO2 34, ΡO2 85, HCO3 29, lactate 1.0, SaO2 98% Which of the following is the most likely diagnosis? A. Compensated metabolic acidosis B. Hyperventilation C. Normal ABG D. Carbon monoxide inhalation E. Methaemoglobinaemia

E. mismatch between bedside sats and measured

Dunedin Sleep Which of the following statements regarding sleep disordered breathing is most true? A.Autoadjusting PAP is superior to CPAP for suppression of apnoea + hypopnoea B. ASV has been shown to reduce mortality in Cheyne-Stokes breathing associated with CHF C. BiPAP is preferable to CPAP for rx of OHS w/out severe noctural oxygen desat D. CPAP reduces mort in CSA E. Evidence from a non-randomised study suggests a reduction in CV mortality with CPAP + OSA

E. non-randomised

deltamed renal A 64-year-old man is receiving candesartan, hydrochlorothiazide and amlodipine at maximal doses for hypertension. A 24-hour blood pressure monitor shows the average BP to be 164/86 mmHg. The addition of which drug is likely to have the greatest effect in reducing the blood pressure? A. Hydralazine. B. Moxonidine. C. Perindopril. D. Prazosin. E. Spironolactone.

E. spiro REHOT study

Deltamed resp A 50yo woman is referred to you with asymptomatic HRCT abnormalities including some bilateral upper zone nodular interstitial changes and bilateral hilar and mediastinal lymphadenopathy.After obtaining a history, examination and basic investigations you suspect sarcoidosis.Which of the following tests test is most likely to confirm your clinical diagnosis? • Serum ACE level • Spirometry and TLCO • Bronchoscopy with bronchoͲalveolar lavage for elevated CD4:CD8 ratio • Bronchoscopy with transbronchial and endobronchial biopsy • Endobronchial ultrasound guided transbronchial needle aspiration of hilar lymph

EBUS is best Don't need bx if Lofgren Both CD4:CD8 supportive and TBBx

Deltamed renal 65 y.o. woman presents with vague symptoms of fatigue. The following biochemistry is noted: Potassium 5.5 mmol/L [3.5-5.5] Urea 48 mmol/L [<6.5] Creatinine 1010 ȝmol/L [<120] Calcium 2.1 mmol/L [2.1-2.5] Phosphate 2.3 mmol/L [0.8-1.4] BP 165/95 mmHg, no oedema. Apart from the fatigue she is asymptomatic, although some scratch marks are noted. She has 8.8 cm kidneys without obstruction. PHx of cholecystectomy and perforated appendix. What is the most appropriate next step? A. Urgent haemodialysis. B. Urgent peritoneal dialysis. C. Elective haemodialysis. D. Elective peritoneal dialysis. E. Urgent continuous venovenous haemodiafiltration.

Elective HD PD contraindicated given gut stuff

Deltamed Resp 20yo female with known asthma presents with fever, cough and sputum production. CXR demonstrates patchy opacification in left and right mid zones. White cell count is elevated at 11.5 with a mild eosinophilia. She was commenced on ceftriaxone, azithromycin and prednisolone. Repeat CXR three days later reveals complete resolution of the opacities. White cell count is 14 but eosinophilia has resolved. She most likely has: • Bacterial pneumonia • Extrinsic allergic alveolitis • Fungal chest infection • Viral pneumonia • Eosinophilic pneumonia

Eosinophilic pneumonia (rapid radiol clearance) Asthma present in 50% at some stage in illness. Focal changes on CXR. responds rapidly to pred but often recurs (chronic) Cf: acute: <1w, peripheral eos uncommon, CXR diffuse

Deltamed renal 58-year-old smoker presents with a new diagnosis of HT. His BP has been difficult to control (190/110) despite amlodipine 10 mg/day and prazosin 5 mg bd. On examination he has a loud abdominal bruit. His CT scan reveals a 1.5 cm left adrenal mass. Lab investigations reveal: Sodium 138 mmol/l [135-145] Potassium 4.8 mmol/l [3.5 - 5.5] Bicarb 23 mmol/l [22 - 28] Creat 105 umol/l [< 120] The most likely cause of his hypertension is: A. Conns syndrome B. phaeochromocytoma C. essential hypertension D. renal artery stenosis E. Cushings syndrome

Essential HTN

What is the appropriate antibiotic prophylaxis in asplenic/hyposplenic patient with penicillin allergy?

In patients with documented severe penicillin/cephalosporin allergy (e.g. anaphylaxis), roxithromycin (150mg orally/day) is recommended. Patients who have mild non-IgE-mediated reactions to penicillin can generally tolerate cephalosporins (e.g. cephalexin).

Dunedin endo Case: 32 year old male with infertility • Couple attends the fertility clinic after 12 months inability to conceive • On examination - Height 182cm, gynaecomastia, sparse facial hair - Small firm testes - Varicose veins • Learning difficulties at school • Lab results: FSH 40.2, LH 31.4, testosterone 5.8 nmol/L - Karyotype 47XXY - Semen analysis: azospermia Dx?

Klinefelters. Tall, low IQ, small firm testes Hypergonadotrophic, hypogonadism

Deltamed Resp A 73yo male with moderate COPD presents to clinic for review.He has symptoms on exertion but no history of AECOPD.He is on no inhaled therapy currently.He has no history of asthma and his peripheral eosinophil count is 0.1.Which of the following inhalers would be expected to have the greatest effect on AECOPD, hospitalization and symptoms. • LAMA monotherapy • ICS/LABA • ICS monotherapy • LABA/LAMA • LABA monotherapy

LABA/LAMA greatest effect

Deltamed Onc 83 year old women presents with a breast mass and bone metastases. Biopsy shows ER+/ Her2 + cancer. She has a history of renal impairment (GFR = 35ml/min) and CHF (LVEF = 42%). What treatment would you offer Mastectomy followed by adjuvant letrozole Radiotherapy to the bone metastases then mastectomy Chemotherapy and herceptin Letrozole Symptomatic care

Letrozole. Older. Can't have herceptin due to heart. mastectomy = big surgeyr + already mets

Deltamed Resp A boy with well controlled asthma presents with status asthmaticus.He is treated aggressively and becomes hypotensive andbradycardic. What is the likely culprit agent? • Magnesium • Aminophylline • Ipratropium • Salbutamol • Hydrocortisone

Magnesium. Rx with IV calcium gluconate aminophylline (tachy) Ipratropium, salb (tachy) Hydrocort (HTN)

DM PT Gen3 3. A non-consanguineous family with a rare genetic disease has been identified. The pedigree is shown below. What is the most likely mode of inheritance? A. Mitochondrial B. X-Linked recessive C. Autosomal dominant D. Autosomal recessive

See handout Evidence of reduced penetrance. Male to male transmission, so X-linked and mitochondrial inheritance not possible. Not Autosomal Recessive due to multiple family members affected in a non-consanguineous family. Answer = C (Autosomal Dominant)

CLS Resp Quiz This 39 year old lady with a history of recurrent sinus infections presents with chronic dyspnoeaand an abnormal chest CT. The below investigation results are obtained: Lung function: FEV1 60%, FVC 80%, FEV1/VC 64% ANA: 1:180 speckled ENA: negative Myositis panel: negative ACE: elevated ANCA: negative RF: negative Which of the following investigations should be performed next to diagnose this interstitial lung disease? Select one: a. Surgical lung biopsy b. EBUS-TBNA c. Serum immunoglobulin levels d. Bronchoalveolar lavage e. CT guided percutaneous lung biopsy

The answer is (c). This lady has Granulomatous Lymphocytic Interstitial Lung Disease (GLILD) associated with Common Variable Immunodeficiency (CVID). The history of recurrent sinus infections is the clue to this diagnosis. GLILD is a rare interstitial lung disease that can mimic sarcoidosis. A bronchoalveolar lavage may be required to exclude atypical infection complicating this condition

CLS Resp Quiz Which of the following therapies does not have proven efficacy in systemic sclerosis associated interstitial lung disease? Select one: a. Cyclophosphamide b. Mycophenolate c. Autologous stem cell transplantation d. Nintedanib e. Methotrexate

The correct answer is (e) The pivotal Scleroderma Lung Study (SLS) trials confirmed the efficacy of cyclophosphamide and mycophenolate. A number of recent trials have confirmed the efficacy of autologous stem cell transplantation in appropriately selected patients. The SENSCIS trial recently confirmed the efficacy of nintedanib on rate of FVC decline in SSc-ILD. Methotrexate has not been studied in SSc-ILD and would typically be avoided in SSc-ILD due to its risk of causing ILD.

CLS Resp Quiz What other features raise the possibility of shortened telomeres in a patient with pulmonary fibrosis? Select one or more: a. Liver cirrhosis b. Premature hair greying c. Opportunistic infections d. Emphysema e. Osteoporosis

The correct answers are (a) -(e) (all are correct) The clinical features of telomere shortening can involve multiple organs; the effects are in part dependent on the rapidity of tissue turnover Manifestations in rapid turnover tissues include: Premature greying of the hair (20-30 years of age) Aplastic anaemia, macrocytosis, thrombocytopenia, lymphopenia B, T and NK cell immunodeficiency resulting in opportunistic infections Manifestations in slow turnover tissues include: Pulmonary fibrosis Premature onset emphysema Cirrhosis Coronary artery disease Osteoporosis Additional manifestations include an increased risk of epithelial and haematological malignancies

CLS Resp Quiz Which of the following features are shared between Idiopathic Pulmonary Fibrosis (IPF) and Rheumatoid Arthritis associated Interstitial Lung Disease (RA-ILD)? Select one or more: a. Usual interstitial pneumonia (UIP) radiological pattern b. MUC5b genetic mutation c. Association with a smoking history d. Acute exacerbations e. Response to anti-fibrotic therapy

The correct answers are (a), (b), (c) and (d) The efficacy of anti-fibrotics in RA-ILD are not yet proven -the results of the TRAIL1 study are eagerly awaited. INBUILD [ahead of print] may provide some support for nintedanib in RA-ILD. RA-ILD differs from other CTD-ILD in its predisposition to a UIP radiological/histological pattern. A gain of function mutation in MUC5b is observed in ~50% of IPF patients. The same mutation has been found to increase the risk of ILD in RA. The risk of RA-ILD is increased by a history of smoking. RA-ILD acute exacerbations are similar to acute exacerbations of IPF and have a high risk for mortality

CLS Resp Quiz What full blood count and biochemistry abnormalities may be encountered in the context of telomere shortening? Select one or more: a. Anaemia b. Microcytosis c. Thrombocytopenia d. Lymphopenia e. Deranged liver function tests

The correct answers are (a), (c), (d) and (e) The most common blood test abnormalities in the context of telomere shortening are: Anaemia (28%) Macrocytosis (24-45%) Thrombocytopenia (9-55%) Lymphopenia (8%) Deranged liver function tests (4%)

CLS Resp Quiz A 55-year-old non-smoking male gentleman was referred with an abnormal chest radiograph and an 18-month history of progressive breathlessness. His previously unlimited exercise tolerance had diminished to 300 metres on level ground. He had no infective symptoms, haemoptysis, orthopnoea or paroxysmal nocturnal dyspnoea. He did not have any symptoms of an underlying connective tissue disease (for example Raynaud's phenomenon, joint or muscle ache, skin tightening, rash, sicca symptoms, mucosal ulcers, alopecia or gastro-oesophageal reflux symptoms). There was no family history of pulmonary disease but his teenage son suffered with chronic aplastic anaemia What is the relevance of this patient's family history? Select one or more: a. The absence of a family history of lung disease suggests that the aplastic anaemia affecting his son is not important b. Childhood aplastic anaemia has no relevance to adult respiratory disease c. Childhood aplastic anaemia may be related to underlying dyskeratosis congenita (DKC) d. The family may carry a genetic mutation in the telomerase complex

The correct answers are (c) and (d) The occurrence of childhood aplastic anaemia raises the possibility of an inherited bone marrow failure syndrome. Dyskeratosis congenita is caused by mutations in genes encoding proteins involved in the maintenance of telomere length (e.g. telomerase) and manifests with a triad of mucocutaneous features in infancy: nail dystrophy, patchy skin hyperpigmentation and oral leukoplakia. Aplastic anaemia complicates dyskeratosis congenita in the first or second decade of life. Telomere shortening as a result of similar mutations has been associated with familial pulmonary fibrosisand emphysema.

CLS Resp Quiz Which of the following therapies should be avoided in a patient with Idiopathic Pulmonary Fibrosis (IPF)? Select one or more: a. Pirfenidone b. Nintedanib c. Azathioprine d. N-acetylcysteine e. Mycophenolate

The correct answers are (c) and (e) Both pirfenidone and nintedanib have proven efficacy in IPF. N-acetylcysteine is not commonly used in IPF, however there may be efficacy in those who carry a mutation in TOLLIP. The combination of azathioprine, prednisolone and n-acetylcysteine was shown to be harmful in those with IPF (PANTHER). Similar data exists for mycophenolate in IPF. Immunosuppression is therefore strongly discouraged in patients with IPF. The only exception may be in those with acute exacerbations of IPF where high dose corticosteroids might be utilized.

CLS Resp Quiz What genetic phenomenon explains the occurrence of aplastic anaemia in the son and pulmonary fibrosis in the father? Select one: a. Autosomal dominant inheritance b. X-linked recessive inheritance c. Mosaicism d. Anticipation

The telomere complex mutations are inherited in an autosomal dominant fashion. Shortened telomeres are carried in germ cells to offspring such that telomeres start from a shortened length in successive generations. This leads to an anticipation effect with the onset of disease becoming earlier in successive generations. Phenotype transition can occur, as exemplified in this case, with offspring presenting with haematological manifestations rather than respiratory disease -this explains the interesting occurrence of pulmonary fibrosis in this patient developing after the onset of haematological disease in his son. The disease phenotype can also become more severe in successive generations. This phenomenon raises important issues for genetic counselling in affected families

CLS Resp Bronch Lecutre • Which of the following represents an absolute contraindication for performing bronchoscopy? - Hypercapnia - Obstruction of the superior vena cava - Uraemia ' - Uncorrected bleeding diathesis - Partial tracheal obstruction

Uncorrected bleeding diathesis

Deltamed Resp A 50yo man attends your clinic and you discuss his current smoking habit and strategies to quit.You outline options for smoking cessation pharmacotherapy for him.Which of the following comorbidities would preclude the use of combination nicotine replacement therapy? • Unstable ischaemic heart disease • Bipolar disorder • Schizophrenia • Suicidal ideation • Stable ischaemic heart disease

Unstable ischaemic heart disease Buproprion contraindicated in BPD Varencline contraindicated with suicidal ideation

Dunedin pharm 50kg person given gent 320mg IV OD. serum conc 1hr post infusion = 20mg/L. Serum conc 6hrs post = 5mg What is the best estimate of clearance? Cl = 0.693 x Vd // t 1/2 A 37mg/hr B. 12.5mg/hr C. 1 L/hr D. 3.5L/hr E. 3mg/L/hr

Vd = dose / peak = 320 / 20 approx 16 1/2 = 2.5 0.693 x 16 // 2.5 11 / 2.5 = 4.4 time 0 = X time 1hr = 20 time 2.5hr = 10 time 6hrs = 5

Deltamed Resp A 68yo exͲsmoker presents with gradually increasing dyspnea over the past 12 months.He has crackles on examination and is clubbed.He has no history of occupational exposures or clinical features of connective tissue disease.He is referred for HRCT which is reviewed in a multidisciplinary meeting and a diagnosis of idiopathic pulmonary fibrosis is reached. Which of the following features on HRCT is least likely to be present? • Honeycombing • Subpleural reticular opacity • Traction bronchiectasis • Basal reticular opacity • Widespread ground glass opacity

Widespread ground glass Traction bronchiectassis probably leastr specific of UIP pattern

CLS ID A 60-year-old woman with cochlear implants presents with a one-day history of headache, neck stiffness and sensitivity to light. On examination, she is febrile and has nuchal rigidity, with GCS 15/15 and no focal neurologic signs. Which of the following is true? a) This patient does not require a CT brain before undergoing lumbar puncture b) While awaiting CSF results, ceftriaxone and vancomycin should be used to cover Streptococcus pneumoniae, Neisseria meningitidis and Listeria monocytogenes c) If the CSF Gram stain shows gram-positive diplococci, high-dose benzylpenicillin should be used while awaiting culture and susceptibilities d) A CSF white cell count of <1000/μL rules out bacterial meningitis e) Dexamethasone is contraindicated in bacterial meningitis

a

Deltamed neuro A 59-year old woman was admitted to the ICU after undergoing successful repair of faecal peritonitis. She is intubated and ventilated and treated with antibiotics. After a week, she improves from her sepsis but on attempting extubation, she goes into hypercarbic respiratory failure. What is the most likely cause for this? a. Critical illness neuropathy b. Acute GBS c. Steroid myopathy d. Muscle relaxant overdose

a

CLS Quiz Resp Use of LTOT in chronic resp failure.... a. Definitively increases physical activity b. Reduces overall mortality when given nocturnally c. Definitively increases physical activity d. Should be administered for at least 15 hours per day to be effective e. All of the above

d

Deltamed Gastro A 48 yo male patient with recently diagnosed ileal Crohn's disease has been prednisolone dependent for a period of 6 months. He has a history of coeliac disease (compliant with gluten free diet) and a history of a melanoma excised 4 years prior. What therapy is best avoided due to safety concerns in this patient? A) Azathioprine B) Infliximab C) Mesalazine D) Vedolizumab

infliximab (melanoma, NHL) prine = non-melanoma

Deltamed genetics A recessive condition is known to have a disease incidence of 1/256 in the community. What is the carrier frequency for a person in the general population? A) 1 in 2 B) 1 in 4 C) 1 in 8 D) 1 in 16 E) 1 in 32

r2 x 1/4 = 1/256 squareroot of 1/256 = 1/16 1/16 x 2 = 1/8

Deltamed gastro A 25yo female presents with a 3 month history of loose bowel motions. She is opening her bowels up to 3x daily with some urgency and bleeding. Her blood tests are all within normal limits. A flexible sigmoidoscopy is performed which revealed 10 cm of continuous mild inflammation with erythema and loss of vascular pattern. Above this point the mucosa appeared normal. Biopsies are consistent with ulcerative proctitis. Which is the best therapy to use in this patient? A) Azithromycin for 3 days B) Mesalazine PO 4g daily C) Weaning prednisolone D) Mesalazine PR 1g BD

mesalazine PR for up to10-15cm proctitis

DM PT Resp6 6. The arterial blood gas below was taken from a patient on room air. pH 7.37 pO2 80 mmHg pCO2 44 mmHg Bicarbonate 34mEq/L What is the alveolar arteriolar oxygen gradient? A. 15 B. 20 C. 10 D. 25

pH = 7.37 PaO2 = 80 mmHg PaCO2 = 44 mmHg Bicarbonate = 34 mEq/L PAO2 = 150 - 44/0.8 PAO2 = 150 - (1.25 x 44) • PAO2 = 150 - 55= 95 • A = 95 • a = PaO2 = 80 A-a = 95 - 80 = 15

Deltamed Resp A 38yo man presents with 18 months of cough and mild dyspnoea.His GP has arranged chest imaging that has been reported as demonstrating eggͲshell calcification.What occupational exposure should be specifically sought? • Asbestos • Flour • Beryllium • Silica • Birds

silica

Deltamed endocrine Which of the following treatments do not inhibit osteoclast bone resorption? a) Alendronate b) Teriparatide (PTH 1-32) c) Zoledronate d) Raloxifene

teriparatide PTH (sumulates bone resportion later)

CLS Renal 73 y.o. ex-smoker has HT (average BP 150/70 on 24 hour BP on perindopril 10 mg daily, amlodipine 10 mg daily, metoprolol 50 mg bd for last 12 months), T2DM (on gliclazide MR 60 mg daily) and PVD (intermittent claudication). • eGFR 29, K 5.4, urine ACR 10.1 mg/mmol (all similar over last 2 years); HbA1C 6.4% • Renal US - symmetrically normal sized kidneys; 75% RAS in right kidney • No symptoms of heart failure. • Which one of the following is the most appropriate next step in his management? A. Addition of an angiotensin receptor blocker to perindopril (ACEi) for better BP control, reducing albuminuria and slowing the progression of his CKD B. Addition of a 4th non-ARB anti-hypertensive to improve his BP control (target <130/80), as well as statin C. Proceed to a CT renal angiogram to evaluate the ultrasonographic evidence of R RAS as it is more sensitive and specific than renal arterial Doppler D. Proceed straight to diagnostic renal angiogram and stent the R RAS if confirmed to be >70% E. Proceed to renal denervation procedure to improve the control of his resistant HT

• Answer: B • A) ARB in addition to ACEi => not too much benefit in lowering BP, does reduce albuminuria further; dual blockade => increased risk of hyperkalaemia & AKI with no evidence of reducing progression of CKD (ONTARGET & VA NEPHRON-D) • C) CT renal angiogram => overestimation due to vascular calcifications; risk of contrast nephropathy; unlikely to change management (see (D)) • D) No evidence for benefit of stenting (ASTRAL, STAR, CORAL) • E) No evidence for renal denervation (SYMPLICITY HTN-3) • B) Medical management for RAS in RCTs; BP targets (SPLINT); also statin for CKD (esp. diabetes) to reduce CV events (SHARP)

CLS Renal A 34 year old female, G0P0, diagnosed with PR3-ANCA positive (anti-GBM negative) GPA with both pulmonary and renal involvement 12 months ago, was given 4 months of oral cyclophosphamide as induction therapy. She was intolerant of both azathioprine (hepatotoxicity) and mycophenolate (severe diarrhoea) and was maintained in remission on weekly methotrexate and reducing doses of prednisolone (ceased 3 months ago). • She presents with new microscopic haematuria, increasing proteinuria, a rise in creatinine to 180 µmol/l (baseline 100 µmol/l) but no haemoptysis. A diagnosis of relapsed PR3-ANCA positive vasculitis has been made. In addition to reintroducing steroid, what treatment would you recommend to the patient? A. Further cyclophosphamide induction orally B. Further cyclophosphamide induction intravenously C. Plasmapheresis D. Induction with mycophenolate mofetil E. Induction with rituximab

• Answer: E • A) and B) are both effective to induce remission but: • Cyclophosphamide toxicity ++ when cumulative dose >36 g • Some evidence of increased long-term toxicity even below this dose • Higher risk of relapse with IV (although as effective to induce remission) compared with oral but lower risk of toxicity (as lower cumulative dose) • Fertility a big issue if she hasn't ruled out pregnancy in the future • Also cannot use azathioprine as maintenance Rx after 3-6 months of cyclophosphamide

Deltamed 8/2. A 45-year-old male presents with decreased exercise tolerance, petechiae and bruising, and unexplained fever of 38.3C. Blood tests reveal pancytopenia with frequent circulating blast cells. Coagulation profile shows hypofibrinogenaemia consistent with disseminated intravascular coagulopathy (DIC). Below is a circulating blast cell with Auer rod

E. APML Bilobed blast cell most commonly l15;17 leading to PML-RARA fusion median age 40y/o PML-RARA inhibits myeloid differentiation Rx: ATRA, arsenic trioxide

Deltmed5/1 A 43-year-old female is referred by her GP with an incidental finding of a 7mm right upper lobe lung nodule on computed tomography of the chest. Which of the following is the most correct regarding initial follow-up of this lung nodule? A. In non-smoker the nodule does not require any follow-up B. A repeat CT scan should be performed at 3 months C. In a non-smoker a repeat CT scan should be performed at 2 years D. A repeat CT scan should be performed at 6 months

1. ANS: D Nodule <30mm Mass >30mm Solid or Subsolid (ground glass or partsolid) Part-solid = higher likelihood of malignancy but less common than solid Incidental: size is independent predictor of malignancy Solid, <8mm = less likely to be malignat. >8 = higher malignnacy, better characterised by PET and more likely to be biopsiable Subsolid: malingnacy more common in >10mm Fleischner guidelines Low risk, <6m = nil F/U. High risk = option CT at 12m Low or high risk, 6-8mm = CT 6-12m, then consider at 18-24m (or just do if high risk) 8mm: CT at 3/12, PET/CT or tissue sampling Ground glass >6mm = 6-12m, then CT q2yrs until 5yrs Partsolid >6mm = CT 3-6m. If unchanged + solid remains then CT q1yrs for 5yrs

Deltamed 2/10. A 74-year-old woman is planned to undergo total hip replacement. She does not have fever or urinary symptoms. A urine culture is sent and grows Escherichia coli with the following susceptibility pattern: Ampicillin Resistant Amikacin Sensitive Amoxycillin/Clavulanic acid Sensitive Ceftriaxone Resistant Cefepime Resistant Ciprofloxacin Resistant Gentamicin Sensitive Piperacillin/Tazobactam Sensitive Tobramycin Sensitive Trimethoprim Resistant Meropenem Sensitive What is the most appropriate next step in management? A. Meropenem B. No treatment C. Amoxycillin/Clavulanic acid D. Gentamicin

10. ANS: B Asymtomatic bacteruria (ASB) is an extremely common condition. There are updated guidelines from the Infectious Diseases Society of America (https://www.idsociety.org/globalassets/idsa/practice-guidelines/2019-asymptomaticbacteriuria.pdf), which recommend screening for ASB in limited circumstances, namely patients in the first month after kidney transplant, neutropenic patients, patients undergoing endourologic procedures and in pregnancy. While many surgical units will screen patients for ASB pre-operatively due to concern about possible seeding to the planned implant, this risk has not been confirmed in prospective trials. Screening does not need to be performed and asymptomatic bacteruria does not need to be treated in these circumstances.

Deltamed 3/10 10. A randomised clinical trial of a new antiplatelet agent 'loclot' versus aspirin was undertaken in patients with established atherosclerotic cardiovascular disease. The primary outcome was major adverse cardiovascular event (MACE), a composite of cardiovascular death, myocardial infarction and stroke. After a median follow-up of 36 months, the 95% confidence interval for the hazard ratio for loclot versus aspirin was 0.93 to 0.99. Which of the following statements is the most correct? A. There is 95% confidence that the true hazard ratio lies somewhere in the range 0.93 to 0.99 B. Compared to aspirin, loclot reduces the likelihood of MACE at 36 months by between 1% and 7% C. The p value would be less than 1% D. The p value would be more than 5%

10: ANS: A B would be correct if "Compared to aspirin, loclot reduces the likelihood of MACE by between 1% and 7% over the course of a 36-month period." Hazard ratio: H (interv): H (control) Applied to whole PERIOD of follow up

Deltamed HF lecture 70yo male with PMHx HTN, OA, carpal tunnel syndrome and GORD. Presents with 6 months of fatigue, weakness and progressive exertional dyspnoea. Baseline bloods demonstrate normal renal function, electrolytes and FBE. BNP is 150ng/L (N<100ng/L). Echocardiogram shows LVEF 50%, moderate-severe LVH, biatrial dilatation, restrictive diastolic filling, a marked reduction in Global Longitudinal Strain, mild pulmonary hypertension and a trivial pericardial effusion. ECG is shown. Which step is most likely to lead to a diagnosis? 1. Coronary angiogram 2. Cardiac MRI and biopsy 3. Right heart catheterisation 4. Stress echocardiography

2. The combination of exertional dyspnoea, history of carpal tunnel syndrome, low voltage ECG and echo findings (LVH, severe diastolic dysfunction, trivial pericardial effusion, atrial dilatation and marked reduction in GLS) raises suspicion of cardiac amyloidosis. Cardiac MRI and cardiac biopsy are the best investigations. Other options are reasonable and may be helpful, but cardiac amyloidosis is the most likely diagnosis and no other option will confirm this.

Deltamed 3/2. A 42-year-old woman presents to you to discuss her breast cancer risk. Her mother was diagnosed with breast cancer at 35, her maternal aunt was diagnosed with breast cancer at 49 and her maternal grandmother was diagnosed with breast cancer at 60. Her mother underwent genetic testing - no BRCA mutations were found. Which of the following is the best assessment of her breast cancer risk? A. She is considered at population risk, but she should undergo genetic testing and only if a BRCA mutation were found would she be considered at high increased risk. B. She is at moderate increased risk, as she has only 1 affected first degree relative and testing has not ruled out a genetic cause. C. For now, she is considered at high risk, but she should undergo genetic testing to clarify her risk. D. She is at high increased risk, as she has 3 affected first- or second-degree relatives and testing has not ruled out a genetic cause.

2. ANS: D No BRCA mutation found but this doesn't exclude underlying genetic cause. Remains at high increased risk as 3 1st or 2nd degree relatives. No pt offering her BRCA testing as inconclusive testing in affected relative (AD) BRCA1 or 2: increased risk of breast cancer, ovarian, prostate, male breast, pancreatic

Deltamed HF lecture A 45 yo female with previous breast cancer presents with a new diagnosis of HFrEF (LVEF 45%). Which of the following agents are MOST likely to have a cardiotoxic effect leading to her left ventricular dysfunction? 1. paclitaxel 2. gemcitabine 3. trastuzumab 4. None of these are associated with cardiomyopathy

3. 1. Paclitaxel Associated with arrhythmia (bradycardia, atrial flutter, AF, atrial tachycardia) but HF usually results only when used with anthracyclines or trastuzumab 2. Gemcitabine Not thought to be cardiotoxic (or very rarely) 3. Trastuzumab Although cardiotoxicity is greatest when combined with an anthracycline (27% in some studies), there is a lower proportion of patients who develop LV dysfunction (rarely severe) in the absence of anthracycline use 4. None of these are associated with cardiomyopathy

Deltmed 7/3 3. A 63-year-old man with metastatic melanoma commences treatment with the anti-CTLA4 monoclonal antibody ipilimumab. Two months later a restaging CT with iv contrast reports a reduction in visceral metastasis. Two weeks thereafter he reports increasing fatigue and dizziness. Serum sodium is 133 nmol/L, compared to 139 nmol/L 2 weeks prior. What is the most important initial investigation? A. Serum cortisol B. Serum TSH C. Serum ACTH D. Urine osmolality

A Don't want to miss secondary ACTH def due to hypophysitis RFs: male, 9 weeks, enlarged pituitary, headache Thyroid > gonadal > adrenal > hyponat > GH low prolactin (ACTH pulsatile)

Deltamed 3/3. Which of the following statements is most correct with regards to the comparison between whole genome sequencing (WGS) and whole exome sequencing (WES)? A. WGS costs less, covers both introns and exons, but results in more incidental findings. B. WGS costs more, covers both introns and exons, but results in fewer incidental findings. C. WES costs less, covers exons only, but results in fewer incidental findings. D. WES costs less, covers both introns and exons, but results in more incidental findings.

3. ANS: C WES = whole exome sequencing, which means it covers the coding regions (exons) only. It costs less than WGS and results in fewer incidental findings. WGS = whole genome sequencing and covers both introns and exons. It is more expensive and likely to result in increased incidental findings.

Deltamed HF lecture 30 yo female with advanced non ischaemic cardiomyopathy is seen in outpatient clinic. She is on carvedilol 25mg bd, sacubitril/valsartan 97mg/103mg, spironolactone 50mg. She has NYHA II symptoms, with no hospital admissions in the past year. She is clinically euvolaemic and her ECG demonstrates sinus rhythm with a heart rate of 65/min with a LBBB (QRS duration 140ms). LVEF 25%. Renal function is preserved. Which is the best change to her management? 1. Start ivabradine 2. Add frusemide 3. Implant primary prevention cardiovertor defibrillator (ICD) 4. Implantation of biventricular pacemaker with ICD

4. 1. Start ivabradine Not indicated with resting heart rate <70/min 2. Add frusemide No, patient is euvolaemic. No outcome benefit for long term loop diuretics 3. Implant primary prevention cardiovertor defibrillator (ICD) She meets current criteria (LVEF <35%, NYHA II), however this is a controversial area in NICM! In any case this is not the BEST answer 4. Implantation of biventricular pacemaker with ICD This patient meets all the criteria for CRT (LVEF <35%, NYHA II, QRS >130ms). Evidence is best for those with LBBB, QRS >150ms, SR although Aust guidelines have removed the requirement for LBBB morphology, and QRS must be >130ms. Also consider in AF.

Deltamed2/5 A 60-year-old woman is diagnosed with follicular lymphoma and is planned to receive rituximab as part of her chemotherapy induction. Screening blood borne virus serology was performed. Her hepatitis B serology is shown below. HepBcAB +ve. surface Ag/surgface Ab -ve. Ab titre <10 DNA +ve What is the most appropriate management strategy to prevent hepatitis B reactivation? A. Hepatitis B vaccination B. Antiviral therapy with entecavir or tenofovir C. Monitor hepatitis B DNA throughout rituximab therapy and treat if viral load is increasing D. Monitor liver function tests during chemotherapy and commence antiviral therapy if liver function tests become abnormal

5. ANS: B Hepatitis B reactivation • Patients with serologic evidence of HBV infection (HBsAg-positive and/or antiHBc-positive) are at risk for HBV reactivation if they receive immunosuppressive therapy. The level of risk is influenced by the HBsAg and sAb status and the type of immunosuppressive agent to be used. Highest risks are associated with actively replicating disease and B-cell depleting agents such as rituximab, which increases risk of reactivation for >12 months High risk = B cell depletor, anthracyclines, >20mg pred for >4w. HBsAf and antiHBc. HBV DNA if +ve Mod: TNFa, CK, cytokine, high dose roids. HBsAg and DNA if +ve Low risk.

Deltamed 2/7. A 35-year-old man with HIV has been off antiretroviral treatment and presents for regular outpatient care. His CD4 count is 153. He has a past history of Stephens-Johnsons Syndrome following trimethoprim-sulfamethoxazole use. What is the most appropriate regimen for Pneumocystis jirovecii prophylaxis? A. Dapsone B. Clindamycin-primaquine C. Atovaquone D. Intravenous pentamidine

7. ANS: C There are multiple agents available for PJP prophylaxis. Bactrim should always be the first choice as it is the agent with the most data supporting its efficacy. Dapsone would typically be the next agent used, however the patient should be assessed for G6PD deficiency and care must be taken because patients with severe Bactrim reactions may have cross reactivity to the sulfur content of dapsone. Clindamycin-primaquine is a second-line regimen for treatment of PJP but is not used for prophylaxis. IV pentamidine is associated with significant adverse effects such as electrolyte disturbances and would be used as a last line agent. Atovaquone is generally well tolerated and would be the most appropriate choice here but is associated with much higher cost.

Delta med 8/10 A 75-year-old man presents with watery diarrhoea for 2 weeks. On presentation, his blood tests showed hypokalaemia, hypomagesaemia and an acute kidney injury. Faecal microscopy and culture are unremarkable. Clostridium difficile toxin is not detected. Colonoscopy is normal except for a diminutive sigmoid colon polyp which was resected. Random colonic biopsies reveal a diagnosis of microscopic colitis. What is next best treatment option? A. Budesonide B. Azathioprine C. Methotrexate D. Faecal microbiota transplantation

A Microscopic colitis - chronic, watery, non-bloody diarrhoea. Normal appearance at colonoscopy. Avoid RFs (smoking, NSAIDs), antidiarrhoeals, glucocorticoids

Deltamed 2/8. A 34-year-old man is receiving induction chemotherapy for acute myelogenous leukaemia. He is neutropenic and develops fevers without an obvious source then becomes progressively hypotensive, necessitating admission to the intensive care unit for pressor support. Blood cultures drawn at the time of the hypotensive episode rapidly flag positive with lactose-non fermenting Gram negative bacilli. What is the most appropriate empirical therapy? A. IV Piperacillin/Tazobactam and IV Gentamicin B. IV Ceftazidime C. IV Meropenem D. IV Ciprofloxacin and IV Vancomycin

8. ANS: A The stem is consistent with septic shock in the context of febrile neutropenia. All empirical regimens recommended for therapy in febrile neutropenia contain an antipseudomonal agent as Pseudomonas was traditionally responsible for up to 50% of Gram negative bacteremia in these patients, however it now makes up a much lower proportion for reasons which are not entirely clear. Nevertheless, the Gram stain findings would be consistent: a simple microbiological classification for Gram negative bacilli is to divide them into lactose fermenters (mostly Enterobacteriaceae such as E. coli, Klebsiella spp. etc) or lactose non-fermenters (Pseudomonas, Acinetobacter, Stenotrophomonas etc). This is easily seen on MacConkey plates which turn a pink colour with lactose activity. Given the severity of the presentation, combination therapy for Pseudomonas/Gram negative sepsis is warranted in the early period before susceptibility results become available. Once the results are available it would be appropriate to use monotherapy as combination therapy once susceptibility is known has not been shown to improve outcomes.

Deltamed 2/9 A 42-year-old woman has recently been diagnosed with rheumatoid arthritis and was commenced on prednisolone 20 mg daily and methotrexate. A CT scan was done to evaluate for possible pulmonary fibrosis and showed an isolated lung nodule. A CT-guided biopsy was performed that did not show malignancy but showed yeast-like fungal forms on Gomori-Methenamine stain. Cryptococcus neoformans is cultured from the tissue specimen. Which of the following is the next appropriate step in management? A. Perform lumbar puncture B. Start Amphotericin B / Flucytosine C. Start Fluconazole D. Perform surgical excision of nodule

9. ANS: A Cryptococcosis is a common invasive fungal infection in immunocompromised hosts. While the presentation is often more clinically apparent, either with chronic meningitis or disseminated disease, it can also be subclinical. Lung nodules are a recognized complication and can be mistaken for malignancy. The therapy for uncomplicated lung disease consists of fluconazole but immunocompromised patients should be evaluated for possible meningitis as this would require treatment with liposomal amphotericin B / flucytosine.

Deltamed 4/9 9. Which of the following clinical features is the least helpful in confirming or rejecting a diagnosis of giant cell arteritis (GCA)? A. Fever B. Jaw claudication C. Diplopia D. Normal ESR

9. ANS: A GCA MANIFESTATION • Fever • Occurs, but a poor discriminator • Countless other explanations • Jaw claudication • Classical GCA symptom • Diplopia • Atypical, but GCA can cause a 6th nerve palsy • GCA not diagnosed on the basis a a 6th nerve palsy though • Normal ESR • Normal acute phases occur in ~10% GCA pts • If the presentation is otherwise consistent, don't worry about this Positive LR: jaw claud, diplopia, temp headache, wt loss, anorexia/fatigue/fever beaded temp artery, prominent temp artery, absent temp artery pulse, tender temp artery, optic atrophy/scalp tenderness

Deltamed 6/9 9. Which of the following disease states is NOT responsive to immunoglobulin treatment? A. Chronic inflammatory demyelinating polyneuropathy B. Guillain Barre syndrome C. Length dependent sensorimotor axonal polyneuropathy D. Multifocal motor neuropathy with conduction block

9. ANS: C Up to date Overview of polyneuropathy Axonal neuropathies are generally not responsive to immunoglobulin therapy.

Deltamed 5/ 9. A 'Get Up and Go' test is commonly performed in assessing falls risk. It may help to identify deficits in all of the following EXCEPT: A. Leg strength B. Vestibular function C. Visual fields D. Balance

9. ANS: C • A get up and go (or timed up and go) test involves getting up from a chair, walking 3 metres, turning around and sitting back down again. It involves leg strength, balance (including vestibular function) but not particularly visual fields.

Deltmed 7/9 9. Which among the following does not affect serum potassium levels? A. ADH B. Aldosterone C. Thyroxine D. Adrenaline

A

Delta med 9/7 . The cancer immunity cycle consists of interactions between tumour cells and immune cells involving both the adaptive and innate immune system. This has been explored therapeutically. Which of the following describe the interaction between the tumour and immune cells? A. Costimulatory immune molecules (CD137, CD27, OX40 and ICOS) can also be modulated to induce anti-tumour immunity B. MHC Class I molecules interact with CD4+ cells C. PD-1 and CTLA-4 stimulate immune-mediated antitumour immunity. D. FOXP3+ T regulatory cells secrete TNF-gamma and IL-23 to cause perforin and granzyme release

A B is incorrect - MHC class I molecules interact with CD8+ T cells, C is incorrect - PD-1 and CTLA-4 are immune checkpoint inhibitors i.e. 'brakes' in immune activation. D is incorrect - Treg IL-10 and TGF-beta to suppress effector cell function.

Deltamed 7/7 A 43-year-old man received a deceased-donor kidney transplant, 3/6 HLA mismatch, ABOcompatible, and enjoyed immediate graft function. He received basiliximab, steroids, tacrolimus and mycophenolate. Four months after transplantation a rise in serum creatinine from 110 to 150umol/L was observed, tacrolimus levels were on target at 7ng/ml, spot urine ACR was normal, MSU was normal, US was normal, a test for anti-HLA antibodies was negative, a PCR for BK virus was moderately positive. He was submitted to a kidney biopsy which revealed normal glomeruli but moderate inflammation of the tubulo-interstitium with tubulitis and several large nuclei in tubular cells which stained positively for SV40 antigen. What would be the most appropriate management? A. Reduce tacrolimus and mycophenolate exposure to treat BK nephropathy B. Pulse methylprednisolone to treat acute cellular rejection C. Increase tacrolimus dose to reduce inflammation D. Switch to sirolimus (an mTOR-inhibitor) to reduce CNI-toxicity

A BK nephropathy rising creat Looks like acute rejection (infiltrate, tubulitis), but abn nuclei. SV40 staining. Reduce immunosuppression. 2nd line rxs with nil RCT evidence IVIG, cidofivir, leflunomide, quinolones

Deltamed 7/4 4. Regarding the regulation of pituitary hormones by hypothalamic factors, which of the following pituitary hormones is primarily regulated by tonic inhibition, rather than stimulated by releasing hormones? A. Prolactin B. Thyroid-stimulating hormone (TSH) C. Luteinising hormone (LH) D. Growth hormone (GH)

A PRL suppressed by dopamine. >5x ULN = hypogonadism >10x ULN = macroprolactinoma <10 = micro or other pituitary

Deltamed 7/6 6. A 21-year-old patient with T1Dm is reviewed in clinic. He was diagnosed at the age of 18 with T1DM. Since then his glycaemic control has been very good. The BP readings have been excellent. O/E There is no evidence of retinopathy or neuropathy. Which among the following statements is correct? A. Annual screening for albuminuria by spot urine ACR is superior to conventional dipstick for patients at risk of diabetic nephropathy B. Patients with T1DM have a higher risk of developing diabetic nephropathy compared to T2DM C. The absence of retinopathy effectively excludes diabetic nephropathy. D. A renal biopsy is usually required to establishing the diagnosis of diabetic nephropathy in T1DM.

A. ACR superior nephropathy in T1DM + T2DM equal Degree of albuminuria and creat best predictor of progression Nephropathy NEARLY always assoc with retinopathy but does not exclude

Delta med 9/2 In what part of the cell cycle does DNA replication occur? A. S B. G1 C. G2 D. M

A. synthesis = replication Mitosis G0 (cell cycle arrest) or G1 (growth) S (duplication of chromes) G2 (checks for errors)

Delta med 9/5 A 63-year-old man is diagnosed with urothelial carcinoma of the renal pelvis. He has a sister with endometrial cancer and older brother with colon cancer. What is the familial cancer syndrome most likely to be present in this family? A. Lynch syndrome B. Li Fraumeni syndrome C. BRCA2 mutation D. Familial adenomatous polyposis

ANS: A Lynch Syndrome HNPCC or Lynch syndrome is associated with DNA mismatch repair defect and increased risk of - colorectal - endometrial - stomach - small bowel - hepatobiliary - upper urinary tract urothelial cancers FAP = loss of APC gene = early onset colon cancer with 1000s polyps Li Fraumeni: high rate germline TP43 tumour suppressor mutation + assoc with childhood or young adult cancer (breast, sarcoma, leukaemia, adrenocortical)

Deltamed 9/6 6. Which of the following are not receptor/ligands forming an immune checkpoint? A. CD40 / CD-40L B. Galactin-9 / TIM3 C. PD-1 / PD-L1 D. CTLA-4 / CD80

ANS: A CD40L is an immune agonist. The others on this list are all immune checkpoint inhibitor ligand-receptors.

Deltamed 8/9. Cirrhosis related portal hypertension leads to multiple complications including hepatorenal syndrome. What mechanism of action of a medication would best treat this complication? A. Increase splanchnic vasodilation B. Increase splanchnic vasoconstriction C. Increase peripheral vascular resistance D. Reduce peripheral vascular resistance

B. Portal HTN = splanchnic art vasod = reduced effective arterial R = activation of symp + RAS = vicious cycle HRS characterised by: - AKI/increased creatinine in advanced liver disease with PHT - Absence of other apparent cause for AKI, i.e. diagnosis of exclusion - Benign urinary sediment, minimal proteinuria and very low rate of sodium excretion - Lack of improvement following volume expansion with albumin and withholding diuretics

Deltamed 1/3 Giovanni, 64, has a pacemaker and automatic implantable cardioverter defibrillator for his ischaemic cardiomyopathy. He presents to the emergency department in your rural hospital having been victim of a stabbing while withdrawing money from an ATM machine late at night. He has extensive deep lacerations requiring surgical intervention on his neck and chest. The surgeons ask you whether a magnet should be put over his PPM/AICD prior to going to theatre. What would this achieve? A. Will inhibit antitachycardic pacing, defibrillation and all pacing B. Will inhibit antitachycardic pacing, defibrillation and revert to asynchronous VOO backup pacing (ie at a rate of 60 beats per minute) C. Will inhibit defibrillation but not antitachycardic pacing or normal pacing D. Will inhibit defibrillation but not antitachycardic pacing and will revert to asynchronous VOO backup pacing (ie at a rate of 60 beats per minute)

ANS: B Usually programmable in modern Cardiac Implantable Electronic Devices (CIED) Pacemakers (PPM) • 60% of pacemakers will convert to a high-rate asynchronous mode • 80-100 bpm depending on battery life - used as battery longevity test • 25% will convert to asynchronous mode at the programmed lower rate • 15% will respond with 60-100 asynchronous beats. Implantable Cardioverter Defibrillators (ICD/AICD) • >90% remove their tachycardia detection capabilities • Usually leave pacemaker programming unchanged • May turn off pacing (old Guidant ICDs) • Important safety measure for inappropriate shocks • ED & CCU always have a magnet

Deltamed 9/3 In which of the following is CA199 falsely elevated? A. Pancreatic cancer B. Bronchitis C. Older age D. Urinary tract infection

ANS: B Bronchitis True elevation in pancreatic / biliary cancers. Also elevated with biliary obstruction of any cause.

Deltamed 3/7. 'Wondermycin' is newly developed antimicrobial given intravenously. It is highly lipid soluble and distributes very rapidly after dosing, and its excretion follows first-order kinetics. A 70kg man is given a single 10mg dose of wondermycin. One hour later, the plasma level was found to be 10mcg/L. Five hours after the single dose, the plasma level was 2.5mcg/L. What is the half-life of wondermycin? A. 1 hour B. 2 hours C. 4 hours D. 5 hours

ANS: B Concentration reduced 4-fold (2 times 2-fold) in 4 hours. Half-life = 2 hours. 10mcg/L @ 1 hr 5mcg/L @ 3 hrs 2.5mcg/L @ 5 hrs 1st order kinetics

Deltamed 4/7. Febuxostat is an alternative agent to allopurinol for the lowering of serum uric acid in patients with gout. Which of the following is true regarding Febuxostat? A. Febuxostat is superior to allopurinol for lowering uric acid B. Gout flares are common with initiation of febuxostat C. It is recommended in patients with ischaemic heart disease D. Worsening renal function was the most common reason for ceasing febuxostat

ANS: B FEBUXOSTAT • Non-purine selective inhibitor of xanthine oxidase In Australia only available for people who are intolerant, hypersensitive to allopurinol, or where there is a contraindication to allopurinol At doses of 80mg 70% of patients achieved a target uric acid of <0.36mmol/L It did outperform allopurinol in the pivotal trials, but allopurinol doses did not go above 300mg Daily. Gout flares are very common with initiation • Most common adverse event leading to discontinuation in the trials was abnormal liver function tests • Cardiovascular safety is still uncertain • The recent CARES study in NEJM showed that "while febuxostat was non-inferior to allopurinol with respect to rates of adverse cardiovascular events. All cause mortality and cardiovascular mortality were higher with febuxostat than with allopurinol."

Deltamed 2/3 A 53-year-old man is 130 days post bone marrow transplant and recently was treated for acute intestinal graft versus host disease and was placed on voriconazole as mould-active antifungal prophylaxis. He had a fall in the backyard and noted a skin lesion on his chest (see below). In the next several days he develops fever and cough requiring admission to hospital. Despite treatment with broad-spectrum antibacterials, his condition continues to decline, and computerised tomography of the chest is performed. A skin biopsy done at the site of the chest wound shows aseptate hyphae What is the most likely cause of his illness? A. C. glabrata infection B. Mucormycosis C. Aspergillosis D. Fusariosis

ANS: B Mucor • Mucor is a devastating condition seen in heavily IC patients and patients with poorly controlled diabetes and is known for its ability to invade through tissue planes. • Imaging shows a 'reverse halo' sign due to angioinvasion and infarction with haemorrhage • Following widespread use of voriconazole as mould-active prophylaxis, predominantly in haematology patiewnts, there were multiple reports of an increase in diagnoses of Mucormycosis, which is resistant to this drug (posaconazole is active) • Fusariosis is well known for causing skin lesions. However, the microbiology is key in this question as Fusarium and Aspergillus typically have septate hyphae and C. glabrata is a yeast. Voriconazole should also be at least partially active against all of these organisms, making a breakthrough infection less likely • Treatment for mucormycosis has traditionally involved surgery if feasible due to poor outcomes with medical therapy. Liposomal amphotericin 5 mg/kg is the typical initial antifungal regimen (as compared with 3 mg/kg used in most other indications)

Deltamed 5/7 A 94-year-old man is admitted from a high-level care nursing home. He has a history of dementia and is bed bound. Aspiration pneumonia has been diagnosed and he is unable to safely swallow tablets or thin fluids. There is no Advanced Care Directive in place. His family do not want antibiotic therapy but would like for him to be able to eat and drink. He deteriorates over the next 24 hours and becomes dyspnoeic and agitated. Oxygen saturation is 70% on 12L oxygen via a Hudson mask. His wife of 70 years would like him to have palliative care but his daughter, who is a lawyer, is refusing. Which of the following statements is most correct? A. Antibiotics should be commenced to 'buy time' until a decision can be agreed upon B. The wife 'trumps' the daughter in the hierarchy of decision-making C. The family can refuse palliative medications D. He should be referred to the High Dependency Unit for ventilatory support

ANS: B • Most states have heirarchies for surrogate decision maker in event of the patient lacking capacity to make medical decisions. - Usually the appointed POA - Then spouse, parents, children, carer • No patient/ surrogate decision maker can demand any treatment deemed futile or burdensome. Can refuse medical treatment. • Patient / SDM cannot refuse palliative care

DeltaMed Gastro Lecture 1 35 y/o man is referred to Gastro cos of epigastric pain not responding to PPI and fasting gastrin level of 650. Most likely reason for this finding is A. High calcium B. PPI C. gastrinoma D. pt not properly fasted

B. Common High calcium rare Gastrinoma possible but usually much higher >10,000 If not fasted = will not increase to that extent

Delta med2/4 A 62-year-old sheep farmer from Warrnambool presents with a 12-month history of intermittent discomfort in his right upper quadrant. He is afebrile and has not had any recent travel. Prominent hepatomegaly is noted on abdominal examination. Computerised tomography of the abdomen is performed (shown below What is the most likely diagnosis? A. Amoebic liver abscess B. Fascioliasis C. Hydatid disease D. Hepatic metastases

ANS: C Echinococcosis The clinical stem and imaging is characteristic of hydatid disease caused the tapeworm Echinococcus. • This infection is initially asymptomatic and can have a latency period of up to 50 years. • It typically causes liver cysts (with the right lobe being more frequently involved) and can also cause lung infection. • It can rarely affect other sites such as the CNS, kidneys, heart and bone. • Dogs are the definitive host and herbivores such as sheep and kangaroos are an intermediate host. • Diagnosis is usually made through a combination of imaging and serology. This CT shows a cyst in the right lobe of the liver with multiple daughter cysts. • Treatment can be difficult, and modalities include surgery for complicated disease (as pictured), percutaneous drainage and treatment with antiparasitic drugs with albendazole being the preferred agent. • Given the absence of a travel history, amoebic liver abscess is unlikely. Sheep are the definitive hosts of fasciola, which also involves the liver but would not cause this appearance on imaging. • Given the epidemiological clues and the imaging appearance, hepatic metastases are also unlikely

DeltaMed 2/1 1. Eculizumab is a novel biologic agent initially approved for use in paroxysmal nocturnal haemoglobinuria and atypical haemolytic uremic syndrome-associated thrombotic microangiopathy. The risk of infection with which of the following organisms is NOT significantly increased by eculizumab? A. Neisseria meningitidis B. Haemophilus influenza type B C. Mycobacterium tuberculosis D. Streptococcus pneumoniae

ANS: C Eculizumab blocks the cleavage and activity of complement factor 5, ultimately inhibiting complement-mediated cell lysis Deficiency in C5b-C9 (terminal complement) has a strong association with infections caused by Neisseria spp. and both disseminated gonococcal infection and meningococcal disease have been reported in the context of eculizumab use • There is also an increased risk of infection with other encapsulated organisms (e.g. S. pneumoniae and H. influenzae) but less than Neisseria spp. as they are prevented mostly by opsonisation rather than complement activation • Tuberculosis has not been associated with eculizumab, but risk is significantly increased with use of TNF-alpha inhibitors, making it wise to thoroughly evaluate patients for latent TB infection prior to initiation of these agents

Delta med 1/4 4. Which of the following may be a side effect of the cardiac medication ivabradine? A. Hypotension B. Peripheral neuropathy C. Visual changes D. Renal tubular acidosis

ANS: C Ivabradine Adverse Effects • 1-10% • Bradycardia (10%) Hypertension or increased blood pressure (8.9%) Atrial fibrillation (8.3%) Luminous phenomena (phosphenes) or visual brightness (2.8%) Post-marketing Reports • Syncope, hypotension • Angioedema, erythema, rash, pruritus, urticaria • Vertigo, diplopia, visual impairment • Torsade de pointes, ventricular fibrillation, ventricular tachycardia

DeltaMed 2/6 6. A 35-year-old lung transplant recipient with a history of cystic fibrosis is hospitalized with multifocal pneumonia requiring ICU admission. He has had several hospitalizations this year and repeated courses of antibiotics for recurrent Pseudomonas aeruginosa infection. The susceptibilities for the Pseudomonas isolate cultured on this admission show that it is resistant to meropenem, cefepime, ceftazidime, aztreonam and gentamicin. Which of the following agents will not have activity against this isolate? A. Ceftolozane-tazobactam B. Colistin C. Tigecycline D. Amikacin

ANS: C Pseudomonas aeruginosa is a formidable pathogen and is capable of developing resistance rapidly to multiple classes of antimicrobials, even on treatment. While there have been several new agents developed for other multi-drug resistant Gram-negative infections, many of these have low rates of susceptibility in Pseudomonas. Ceftolozane-tazobactam however is a novel agent that can be used as a last-resort. Colistin is a cationic antimicrobial peptide that has a significant adverse effect profile and is also used as a last resort. While the isolate is resistant to gentamicin, it may still retain susceptibility to other aminoglycosides such as amikacin. Tigecycline and other novel tetracyclines (eg. omadacycline) do not have activity against Pseudomonas.

Delta Med 1/2 Maria, 70, a diabetic lady tells you she has been experiencing chest discomfort and shortness of breath on exertion for the last 3 months. You arrange a stress echocardiogram. She exercises for 3 minutes according to the Bruce Protocol then develops her symptoms and the test is ceased. The ECG is relatively unchanged, but there is failure of the lateral wall to augment as much as the other myocardial walls. In which coronary artery territory does this imply ischaemia? A. Left main B. Left anterior descending C. Left circumflex D. Right coronary artery

ANS: C • Isolated lateral wall: • circumflex artery Frequently multisegment left main artery • anterior, lateral, and apical wall • left anterior descending artery • entire apex, including the lateral apex

Deltamed 5/6 6. A 94-year-old man is admitted from a high-level care nursing home. He has a history of dementia and is bed bound. Aspiration pneumonia has been diagnosed and he is unable to safely swallow tablets or thin fluids. There is no Advanced Care Directive in place. His family do not want antibiotic therapy but would like for him to be able to eat and drink. What is the most appropriate plan of management? A. Make him nil by mouth (NBM) until he is safe to swallow food and water B. Insert a nasogastric (NG) tube to allow administration of feeds C. Allow him to eat and drink as desired, despite aspiration risk D. Commence total parenteral nutrition (TPN)

ANS: C • The family have opted for a conservative approach to his treatment. Parenteral nutrition is not consistent with this approach. He is unlikely to ever be 'safe' to swallow (given we are not treating his pneumonia), thus answer A is not realistic.

10. Perioperative care in the older patient is an area of increasing focus. Which of the following statements is FALSE when considering anaesthesia in the older person? A. There is diminished ventilator response to hypoxia and hypercapnia, which can be further exacerbated by anaesthesia B. Decline in hepatic mass and function means there may be slower metabolism of drugs used in anaesthesia C. Impaired beta receptor responsiveness limits the ability to increase cardiac output by increasing heart rate D. Vascular stiffening in older age leads to a gradual decrease in blood pressure

ANS: D • Vascular stiffening leads to widened pulse pressure in the elderly and increased systolic blood pressure. • There is less vascular reactivity in older persons, which would contribute to hypotension with hypovolemia though.

Deltamed 3/5 Jenny and Steve consult the infertility clinic for 12 months of primary infertility. Following investigation, Jenny's karyotype demonstrates 45, XX + rob(14q21q). Which of the following statements is correct about the couple's ability to have a healthy child? A. The couple will not be able to conceive naturally. B. The couple may be able to conceive naturally but face an increased risk of having a child with Down syndrome. C. The result bares no impact on the couple's ability to have a healthy child and other causes for infertility should be investigated. D. The result suggests a high rate of miscarriages for this couple, but no further testing is possible

Ans B 45 XX = robertsonian translocation. q = long arm. (cf: 45 XO = turners) Increased risk of producing aneuploidy of 14 and 21. monosomy 21, monosomy 14 and trisomy 14 = miscarriage, but trisomy 21 = Downs 4% DOwns is due to transl. Not affected by mat age. Can come from Dad 21, 13, 18, 8 compatible with life

Deltamed 3/8. Drug X has a fraction excreted unchanged of 0.80. In a person with normal renal function, the usual dose is 100mg daily. What is the most appropriate renally-adjusted dose for a person whose renal function is quantitatively halved? A. 50mg BD B. 20mg daily C. 60mg daily D. 80mg daily

Answer C adj dose = (1 - Fu) + (Fu x pts egFR/eGFR)

Deltamed Haem 1 A 54‐year‐old man presented to Accident and Emergency with fevers and confusion. He is on amlodipine for hypertension and metformin for diabetes. His blood tests and blood film are shown: Haemoglobin: 88g/L (115 - 165) MCV: 100fL (80 - 99) White cell count: 12.1x109/L (4 - 11) Neutrophils: 8.8x109/L (2 - 8) Lymphocytes: 3.0x109/L (1 - 4) Monocytes: 0.3x109/L (<1.1) Platelets: 77x109/L (150 - 450) Renal function was normal ALP: 110 U/L (30‐100) ALT: 40 U/L (5‐35) GGT: 35 U/L (<37) Bilirubin: 57 mmol/L (<21) The next most appropriate managementstep is: A. Test for ADAMTS13 B. Start plasma exchange C. Send for direct Coombs' test D. Give platelet transfusion

Answer: B Rationale: The clinical history is consistent with thrombotic thrombocytopenic purpura (TTP) despite having normal renal function. This is a medical emergency and having results of ADAMTS13 should not preclude commencing therapy with plasma exchange. As the diagnosis is unlikely to be autoimmune haemolytic anaemia, a Direct Coombs' Test (DCT) is not helpful. Platelet transfusions are usually avoided in TTP

Deltamed Haem 1 A 24‐year‐old man presents with fatigue, shortness of breath and jaundice. His FBE, LFTs and blood film showed: Haemoglobin: 54g/L (135 - 165) MCV: 110fL (80 - 99) White cell count: 13.8x109/L (4 - 11) Neutrophils: 9.0x109/L (2 - 8) Lymphocytes: 4x109/L (1 - 4) Monocytes: 0.8x109/L (<1.1) Platelets: 440x109/L (150 - 450) ALP: 110 U/L (30‐100) ALT: 40 U/L (5‐35) GGT: 35 U/L (<37) Bilirubin: 60 mmol/L (<21) LDH: 880 U/L (120‐250) Haptoglobin: <0.1g/L (0.3‐2.0) 2 days later, his G6PD screen result was normal. What is the next most appropriate investigation? A. Test for congenital TTP by measuring ADAMTS13 B. Repeat the G6PD screen 3 monthslater C. Perform flow cytometry for hereditary spherocytosis D. Measure B12 and folate levels

Answer: B Rationale: The film features show oxidative haemolysis (with bite cells, blister cells and spherocytes). This is reflected in the FBE and biochemical markers. There is brisk reticulocytosis as evidenced by the polychromatic red cells and increased MCV. Reticulocytes (immature red blood cells) have increased G6PD. Therefore, G6PD levels may be artificially normal during an acute oxidative crisis. Repeat testing when well is recommended. Similarly, testing should not be performed until 3 months after any red blood cell transfusions.

Deltamed 4/1. Biological agents may be indicated in the treatment of moderate to severe asthma. Which of the following agents would be LEAST useful in this role? A. Omalizumab (anti-IgE) B. Ipilimumab (anti-CTLA4) C. Mepolizumab (anti-IL-5) D. Dupilumab (anti-IL-4 Receptor)

B. ipilimumab omalizu (IgE) - severe allergic asthma mepoliz (IL-5) - eosinophilic asthma dupulimumab (asthma, atopic dermatitis) AntiCTLA4: prevents T ell activation. B7 blocker. Licensed for RA

Deltamed Haem 1 A 24‐year‐old G1P0 14 week pregnant woman returns for follow up of her prenatal blood tests. She was previously diagnosed with iron deficiency anaemia as a child and is on regular oral iron supplementation. Her blood tests showed: Haemoglobin: 95g/L (115 - 165) MCV: 72fL (80 - 99) RCC: 5.8x1012/L (3.6 - 5.2) White cell count: 4.9x109/L (4 - 11) Platelets: 300x109/L (150 - 450) Iron: 22umol/L (9 - 32) Transferrin : 3.0g/L (2.0 - 3.6) Transf saturation: 39% (16 - 60) Ferritin: 80ug/L (26 - 388) Blood film showed microcytic anaemia with no red cell fragments. What is the next most appropriate management plan? A. Test soluble transferrin receptor B. Repeat renal function, liver function and iron studies C. Organise haemoglobin electrophoresis on herself and the baby's father D. Test B12 and folate levels

Answer: C Rationale: The normal iron levels and previous diagnosis of "iron deficiency anaemia" together with red cell parameters (low MCV, increased RCC) all suggest a haemoglobinopathy like thalassaemia. In view of her pregnancy, urgent confirmation of diagnosis, testing of the baby's father and genetic counselling should be offered. Repeat testing of iron studies, B12 or folate levels is unhelpful. Similarly, the lack of fragments or thrombocytopenia makes HELLP, pre‐eclampsia or TTP less likely.

Deltamed Haem 1 A 24‐year‐old woman with menorrhagia presented with fatigue. She had iron deficiency anaemia with a haemoglobin of 80g/L and MCV of 70fL. She was commenced on oral iron supplementation. 8 weeks later, a repeat blood test showed: Haemoglobin: 95g/L MCV: 72fL (80 - 99) RCC: 5.8x1012/L (3.6 - 5.2) White cell count: 4.9x109/L (4 - 11) Platelets: 300x109/L (150 - 450) Iron: 22umol/L (9 - 32) Transferrin : 3.0g/L (2.0 - 3.6) Transf saturation: 39% (16 - 60) Ferritin: 80ug/L (26 - 388) She reports compliance with oral iron supplementation despite the gastrointestinalside effects. What is the next most appropriate management plan? A. Test soluble transferrin receptor B. Perform a bone marrow biopsy C. Test haemoglobin electrophoresis D. Test serum transglutaminase antibody

Answer: C Rationale: There was only partial correction of anaemia but persistent microcytosis and increased red cell count despite correction of her iron deficiency. Another cause for microcytosis, specifically thalassaemia, should be investigated.

Deltamed Haem 1 A 40‐year‐old man with chronic liver disease presents with confusion and a fever of 38.2°C. He was treated empirically for spontaneous bacterial peritonitis. His FBE, LFTs and blood film showed: Haemoglobin: 72g/L (135 - 165) MCV: 105fL (80 - 99) White cell count: 14x109/L (4 - 11) Neutrophils: 10x109/L (2 - 8) Lymphocytes: 3.5.0x109/L (1 - 4) Monocytes: 0.5x109/L (<1.1) Platelets: 70x109/L (150 - 450) ALP: 250 U/L (30‐100) ALT: 200 U/L (5‐35) GGT: 420 U/L (<37) Bilirubin: 150 mmol/L (<21) LDH: 320 U/L (120‐250) Haptoglobin: <0.1g/L (0.3‐2.0) What is the most likely reason for the anaemia? A. Anaemia of chronic disease B. Iron deficiency anaemia C. Folate deficiency D. Haemolytic anaemia

Answer: D Rationale: The blood film shows many acanthocytes or spur cells. The most likely cause for anaemia is increased haemolysis in the context of chronic liver disease. The red cell features (and MCV) are not consistent with iron deficiency or folate deficiency.

Deltamed Haem 1 A 40‐year‐old man presents with fatigue and shortness of breath. His FBE, LFTs and blood film showed: Haemoglobin: 54g/L (135 - 165) MCV: 110fL (80 - 99) White cell count: 2.0x109/L (4 - 11) Neutrophils: 1.0x109/L (2 - 8) Lymphocytes: 0.8x109/L (1 - 4) Monocytes: 0.2x109/L (<1.1) Platelets: 50x109/L (150 - 450) ALP: 110 U/L (30‐100) ALT: 40 U/L (5‐35) GGT: 35 U/L (<37) Bilirubin: 40 mmol/L (<21) LDH: 320 U/L (120‐250) Haptoglobin: <0.1g/L (0.3‐2.0) What is the next most appropriate managementstep? A. Send for Direct Coombs' Test B. Commence plasma exchange C. Organise urgent bone marrow biopsy D. Measure B12 and folate levels

Answer: D Rationale: The FBE and blood film show hypersegmented neutrophil and macrocytic anaemia. Pancytopenia, irregularly shaped cells and mild haemolysis can occur in B12 deficiency. A careful review of the film and FBE parameters make TTP or autoimmune haemolysis less likely. A bone marrow biopsy is not indicated unless there is no haematinic deficiency.

Delta med 6/6 A 65-year-old man is admitted to intensive care following out-of-hospital VF arrest with prolonged bystander resuscitation. On examination, alerting is attempted with sternal rub. Which of the following features in response is in keeping with brain death? A. Arms adducted, extended, pronated; legs extended, feet plantar flexed, muscles tightened and held rigidly, with the head and neck arched backward. B. Asymmetric hyperextension and spasticity with head, neck and spinal column entering into a "bridging" or "arching" position C. Body stiff with bent arms, clenched fists, and legs held out straight, internally rotated, feet plantar flexed. The arms are bent in toward the body and the wrists and fingers are bent and held on the chest D. Facial grimacing

B. A - decerebrate C - decorticate D - cortical B- spinal reflex ACD - brain originating B - asymmetric opisothotonic posturing of trunk can originate from spinal reflex in absence of brain function

Deltamed 8/5. A 45-year-old female is being investigated for fatigue and deranged liver function tests. Her background is significant for alcohol excess, with >7 standard drinks per night. She is ex-IVDU, and previous hepatitis B infection, and has recently been diagnosed with diabetes. Blood investigations are belowFerritin 1300mg/L (30 - 300) Transferrin saturation 70% (15- 50%) HFE genotyping C282Y/H63D heterozygosity What is the most appropriate first line management? A. Iron chelation therapy B. Commence venesection targeting ferritin <50 C. Encourage alcohol abstinence D. Plasmapheresis

B. Compound heterozygote Target ferritin <50 to prevent end organ damage Alcohol reduction impt too if C282Y homo = regular phleb, fibroscan, family screening C282Y/H63D hetero = venesect to noraml SF + observe periodically

Deltamed gastro lecture 1 68 y.o man presents with abdo pain and wt loss. Gastroscopy normal. Small bowel series shows multiple small bowel diverticula Which of the following is least likely to be associated with this condition? A. weight loss B. low serum B12 C. Positive anti gliadin antibodies D. low serum folate E. +ve hydrogen breath test

C. Bacterial overgrowth

Deltamed 4/2. Manipulation of T-cell co-stimulators by biological agents is used in the treatment of malignancy but also for autoimmune disease. Which of the following agents is immunosuppressive in its action? A. Ipilimumab (Anti-CTLA4) B. Nivolumab (Anti-PD-1) C. Abatacept (CTLA4-Ig) D. Durvalumab (Anti-PDL-1)

C Check point Is = anti-CTLA4, antiPD1, AntiPDL1

Deltamed 6/4 . A 64-year-old man is brought by his family for concern around gait difficulties and worsening memory. He has gradually been slowing down in his walking, and walks with a slow, stooped posture. He has had increasing memory difficulties, most recently manifest getting lost driving to his local shopping centre, and with some concerns of his employer regarding work performance. Recently, he had an episode of extreme somnolence, and slept for most of the day, with no apparent cause. On examination, there is reduced animation in conversation. He has bilateral relatively symmetrical moderate rigidity. There is moderate predominantly right sided rest tremor in the hand. Gait is small paced and slowed. Which of the following features in this situation is more in keeping with a diagnosis of idiopathic Parkinson disease than Dementia with Lewy Bodies? A. An additional history of REM sleep behavioural disorder B. An additional history of visual hallucinations C. Asymmetrical tremor D. Fluctuation in cognitive function

C DLB: fluctuations early in course, blankout/lose consciousness, daytime drowsiness, disorg speech REM precursor to both PD: more likely to have tremor

DeltaMed 2/2 A 43‐year‐old with myasthenia gravis presents with sudden onset of fevers, chills and rigors on a background of increasing dysuria in the last 3 days. She has had multiple previous antibiotic treatment courses for recurrent urinary tract infection. She has normal renal function on presentation. Blood cultures from admission have grown an E. coli with the following susceptibility profile: Ampicillin R Cephazolin R Ceftriaxone R Ceftazidime R Ciprofloxacin R Gentamicin S Piperacillin/Tazobactam S Meropenem S Trimethoprim/Sulfamethoxazole S Which is the most appropriate initial treatment regimen? A. Gentamicin IV 5mg/kg daily B. Piperacillin/Tazobactam IV 4.5g 8-hourly C. Meropenem IV 1g 8-hourly D. Trimethoprim IV 1600/320mg 8-hourly

C ESBL - producing organisms Resistance to third generation cephalosporins but susceptibility to piperacillintazobactam and meropenem raises red flags about the possible presence of an extended-spectrum beta-lactamase (ESBL) The patient's history of myasthenia gravis, which precludes use of some antibiotics such as aminoglycosides The role of beta-lactam/beta-lactamase inhibitor combinations for treating EBSL infections has been hotly debated for some years with observational studies showing conflicting results regarding efficacy when compared to treatment with carbapenems In order to address this question, the Australasian Society for Infectious Disease Clinical Research Network (ASID-CRN) organized a multi-centre prospective randomized control trial that compared piperacillin-tazobactam to meropenem and found greater mortality with piperacillin-tazobactam (12.3% vs 3.7%) (MERINO trial)

Deltamed 7/8 8. A patient with a peritoneal dialysis catheter presents with abdominal pain and a cloudy bag. Peritoneal Fluid : WCC 10,000. An initial empirical therapy of intra-peritoneal Vancomycin and Gentamicin are given. The next day the culture report : Gm -ve and Anaerobic organism. Which among the following is the appropriate management? A. No change to therapy and await sensitivities B. Add intravenous Metronidazole C. Laparotomy and removal of the dialysis catheter D. Commence oral fluconazole and IV metronidazole

C Laparotomy is required to remove catheter and look for perf commonest, staph, epi mixed orgs suspicious for bowel perf espec anaerobes

Deltamed 5/2. An 80-year-old man with a history of COPD presents in respiratory distress. On examination, he is saturating at 90% on 4L via nasal prongs, respiratory rate 28, pulse 124 and blood pressure 88/64. He has wheeze at his left lung base with decreased air entry on the right side. His ECG shows atrial fibrillation. His arterial blood gas is shown below: pH 7.36 pO2 64 mmHg pCO2 38 mmHg Bicarbonate 20mEq/L What is the next step in your investigation and management? A. Give oral metoprolol 25mg B. Commence NIV C. Urgent chest X-ray D. Reduce oxygen flow

C Need to exclude pneumothorax Not in type II so doesn't need NIV Hypotension is relative contra to metoprolol

Deltamed 7/10 10. A 60-year-old man with background history of chronic hepatitis C following transfusion presents with increasing pedal edema over the past 3 months. O/E 150/90. Edema +++. Chest clear. Results: Hb : 134, WCC: 6.0, Plt: 180, Alb: 24, Creatinine: 160umol/l. Cholesterol:8.0. C3: Normal, C4: Low. Urine Blood ++, spot urine ACR: 600 mmol/mmol. He has no previous history of haematuria or renal impairment. Which among the following is the most likely diagnosis? A. IgA nephropathy B. Membranous nephropathy C. Mesangiocapillary (Meangiocapillary) Glomerulonephritis D. FSGS

C Nephrotic with active urinary sediment. low CD4 Low complement: cryo, hep B or C, lupus, MPGN, PIGN, subacute IE. Hep C related Cyoglobulinaemic (most common. generaliased vasculitis) Fever, neuropathy (mesangiocapl, low C4)

Deltamed 8/3 A 73-year-old male presents with lethargy, early satiety, nightsweats and recurrent infections. Examination reveals massive splenomegaly. Blood investigations showHaemoglobin 95g/L (135-160) MCV 98fL (80-100) Platelet 73x109 /L (150-400) Neutrophil 1.3x109 /L (2-7) Blood film is reported as: leucoerythroblastic film with tear drop poikilocytes, occasional circulating blast cells 1-2%. Bone marrow biopsy is performed, with reticulin stain as below.

C myelofibrosis Increased reticulin formation on BM bx JAK2 mutation is most commonly detected mutationt then CALR, MPL

Deltamed 7/2 Regarding denosumab, an antiresorptive agent used in the treatment of osteoporosis, which of the following is NOT correct? A. In patients with severe chronic kidney disease (stage 4-5) denosumab may cause hypocalcaemia even if serum 25-hydroxyvitamin D levels are > 50 nmol/L. B. Cessation of denosumab treatment may be associated with a risk of multiple and/or severe vertebral fractures C. Following a 5-year course of denosumab, bone mineral density remains stable for at least 3-4 years post cessation of denosumab treatment D. In randomised clinical trials, the incidence of atypical femoral fractures associated with denosumab treatment has been low (<1 %)

C. Denosumab not retained in bone. Treatment cessation = rapid increase in bone remodelling markers which can cause vertebral fractures post-cessation. Don't initiate drug holiday. Likely need post-cessation bisphos Mx of atypical fem fracture Stop antiresorptive ?teriparatide to accelerate healing Consider prophylactic pinning of contra side

Deltamed 5/3 A 28-year-old female presents for review of asthma diagnosed 5 years ago. She is currently using salbutamol via metred-dose inhaler twice a day for asthma symptoms. She wakes overnight twice per week on average with symptoms. She had a course of oral prednisolone 6 months ago for an exacerbation. She is a current smoker. What is the appropriate management? A. Smoking cessation and continue short acting bronchodilator when required B. Regular low dose inhaled corticosteroid and continue short acting bronchodilator when required C. High dose inhaled corticosteroid and down titrate when symptoms improve D. Add monteleukast

C. Freq nocturnal sx and daily SABA use Not well controlled = sx >2d/week, nighttime 1-3x/week. some limitation of normal activities, SABA >2 days/week, 70% peak flow. 2+ exacerbations/yr

Deltamed 6/3. A 63-year-old man presents with gradual onset over weeks of symmetrical aching and stiffness of the shoulders and hip-girdle. He reports stiffness after inactivity, especially at night, with great difficulty turning over in bed. There is some improvement in the morning after a hot shower, and with activity. He has charted a low-grade temperature ranging to 37.6°C. Over the last 7 days he has also noticed generalised moderately severe headache and malaise. On direct questioning he reports worsening difficulty with chewing and with swallowing, now about halfway into his meals. He has lost 3 kg weight since symptoms began. His past medical history includes hypercholesterolaemia, for which he takes pravastatin 20mg at night. Investigation shows ESR 55mm/hr. Full blood examination, renal and liver function are normal. Which of the clinical features is LEAST likely associated with this condition? A. Fever B. Gradual onset C. Swallowing Claudication D. Weight loss

C. Initial sx: New headache > PMR > fatigue/malaise > fever > jaw claud Present sx: headache > PMR > fever/malaise > jaw claud > fever Swallowing claud/dysphagia are rare

Deltamed 6/2 A 66-year-old woman with hypertension and type 2 diabetes presents with a 3-week history of headache and fever. The headache is worse when supine. She is admitted following acute onset of altered mental status. On examination she is febrile and minimally responsive. She has a stiff neck. Her MRI brain is shown: (abscess occipital lobe going into V) She is started on broad spectrum antibiotics and surgical drainage is arranged. Her family wish to discuss prognosis. Which of the following is appropriate to convey in this situation? A. Stupor in itself is not a poor prognostic feature, with appropriate emergency drainage and ongoing antibiotics. B. The patient's rapid clinical deterioration - a poor prognostic feature - is likely secondary to mass effect caused by the lesion. C. Ventricular rupture is a dire complication, with 80-100% mortality. D. With modern surgical and antibiotic therapies, the outcomes from cerebral abscess management in this situation are fair, with >70% chance of normal medium-term recovery.

C. No role for LP deterioration secondary to intraV rupture Gram positives 3rd gen cep, vanc, metro If >2.5cm then surgical drainage Poor prognosis: rapid deterioration, severe mental status changes, stupor/coma (60-100% mort), V rupture (80-100% mort)

Deltamed 8/4 A 24-year-old male presents with drenching night sweats, weight loss and enlarging cervical lymphadenopathy. He has had recent travel to Bali with no fresh water or animal exposure, though had close proximity to unwell contacts. He had frequent binge alcohol episodes. Peripheral blood lymphocytosis is noted, with blood film showing these cells. (Pathology Outlines) What is the most likely diagnosis? A. Chronic lymphocytic leukaemia B. Prolymphocytic leukaemia C. Burkitt lymphoma D. Acute Epstein-Barr Virus

C. The large, immature, vacuolated cells with deeply basophilic cytoplasm are concerning for Burkitt lymphoma. Reactive features (cytoplasmic scalloping, basophilic edge) are not prominent. CLL cells are small and mature with 'blocky' condensed chromatin. Highly aggressive High risk tumour lysis, high cure rate y8;14

deltamed 8/7 A 78-year-old pig farmer presented to the emergency department with one week of right upper quadrant pain and a syncopal event the morning of presentation. Apart from jaundice, his physical examination was unremarkable. Regular medications included ramipril and atorvastatin. He had also had a course of augmentin 5 months prior for an upper respiratory tract infection. On taking further history he acknowledged previous heavy alcohol consumption but denied intravenous drug use. plt 431 ALT 4798 AST 3783 ALP 159 bili 181 INR 1.9 CRP 62 Transjugular biopsy: panacinar inflammation with necrosis and prominent intrahepatic cholestasis. The most likely diagnosis is: A. Ischaemic hepatitis B. Epstein Barr Virus C. Hepatitis E D. Drug-induced cholestasis

C. Causes for hepatitis >1000 acute viral, drug/toxins, ischaemic, AIH, Wilson, Budd-Chiari Timeframe not consistent for augmentin or ischaemic (syncope several days before) RFs for HEV - enterically transmitted, age, alcohol hx, undercooked meat, animals Severe/acute failure assoc. with preg, malnutrition, liver disease Dx: IgM ABs initially, then HEV RNA

Deltamed 3/9 A validation test is undertaken of a new, point-of-care test that detects malignant cells in the sputum of people suspected of having lung cancer. The results of the new sputum test are compared against the 'gold standard' of bronchoscopy with biopsy. The sputum test has 90% sensitivity and 90% specificity for lung cancer. If the pre-test probability of lung cancer in the population being studied is 10%, what will the positive predictive value of the sputum test be? A. 1% B. 10% C. 50% D. 99%

C. PPV = sens x prev ___________________ (sens x prev) + ((1-spec) x (1- prev)) NPV = spec x ( 1 - prev) _______________________ (spec x (1-prev)) + (1-sens + prev)

Deltamed 5/4 For each chest X-ray image below, select the most likely clinical diagnosis. Option list: white out of entire L chest with trachea pulled towards A. endobronchial obstruction B. pleural effusion C. pneumonectomy D. pneumonia E. right lower lobe collapse F. right middle lobe collapse

C. pneumonectomy Pneumonectomy is one of the causes of a white out of the hemithorax. • The hemithorax is completely opacified with the trachea pulled towards the side of the abnormality. • Surgical clips may be identified at the lung hilum.

Deltamed 5/5 For each chest X-ray image below, select the most likely clinical diagnosis. Option list: flattened elevated R hemi, volume loss A. endobronchial obstruction B. pleural effusion C. pneumonectomy D. pneumonia E. right lower lobe collapse F. right middle lobe collapse

E RLL collapse Increased opacity at medial base of R lung R hilar depressed RHB still well visualised

Deltamed 4/6 6. A 36-year-old man with a long history of systemic lupus erythematosus complicated by lupus nephritis presents with increasing left hip pain. He is currently managed on Mycophenolate 1g BD, Hydroxychloroquine 200mg Daily and Prednisolone 5mg mane. On examination he has an antalgic gait and restricted abduction and internal rotation. His blood tests are normal, and an x-ray of his left hip is also normal. Considering the most likely diagnosis, what investigation is most useful to confirm the diagnosis? A. Blood cultures B. Doppler Ultrasound C. Joint aspiration D. MRI scan

D AVASCULAR NECROSIS • Can affect anyone but common between ages 30-60 Death of bone tissue secondary to impaired blood supply • Risk factors: - Trauma - Excessive ETOH - Bisphosphonate use - Medical conditions such as kidney transplant, cancer requiring radiation therapy, Pancreatitis, diabetes, Gaucher's disease, HIV, SLE, Sickle cell disease • Treatment - reverse any known triggers, but usually surgery

Deltamed 8/1 A 65-year-old male is admitted with a 2-day history of melena, on a background of ischaemic heart disease with recent angiogram showing disease not suitable for percutaneous intervention. He is on dual antiplatelet therapy. He has not had a transfusion before. His haemoglobin is 65g/L (135-160) on admission, and he reports shortness of breath. Two units of packed red blood cells are requested from blood bank. 10minutes into the first unit he reports severe pain at the infusion site, back pain, dark urine, increased shortness of breath and temperature rises from 36.5 to 39C. Blood investigations are consistent with haemolysis with a positive DAT. What is the likely cause of this reaction? A. Alloantibody formation B. Cold agglutinin disease C. IgA deficiency D. Mislabelled tube

D Acute haemolytic transfusion reaction from ABO incompatibility Immune med: ABO/Rh mismatch, red cell alloAbs (from prev preg or transfusion) Stop transfusion Induce diuresis

Deltamed 7/5 5. In a patient with thyrotoxicosis, which aetiology is LEAST compatible with the thyroid 99mTc-pertechnetate scan shown below: A. Recent iodine contrast exposure B. Struma ovarii C. Surreptitious thyroxine use D. hydatidiform mole

D Suppressed scan utake = DESH drugs, exogenous/excess iodine, subacute thyroidisits etc Hydatid mole = excess bHCG stimulates TSH = increased scan uptake Other increased: Graves, TMNG, adenoma, TSH-oma

Deltamed 8/6 A 73-year-old male from regional victoria was referred to an outpatient clinic with incidental bony lesions in his ribs, scapula and right proximal femur. He was a functionally independent gentleman living at home with his wife. Past medical history was significant for hypercholesterolemia, hypertension and obesity. Examination revealed few spider naevi on the upper chest, no hepatic flap or abdominal ascites. Laboratory results: Platelet count 135 x10^9/L [150-400] Alanine transaminase 38 IU/L [5-40] Total bilirubin 18 mg/dl [3-18] Albumin 35g/dl [35-46] INR 1.3 Serum alpha-fetoprotein 10 [<6] CT abdomen with contrast: nodular liver with features of portal hypertension, three arterially enhancing masses with venous washout in segment 2/3, the largest measures 5cm. What treatment should be offered to this patient? A. Radiofrequency ablation B. Transarterial chemoembolization C. Referral for transplantation D. Tyrosine kinase inhibitor therapy (Sorafenib 400mg oral twice daily)

D. Child Pugh A complicated by met HCC Oral chemo - pt must have preserved LFTs (CPA) and ECOG 1/2 to be candidate Ablation if early stage, resect if single + liver tx Chemoemb if intermediate stage, ECOG0 Advanced stage = systemic rx

Deltamed gastro lecture 1 56 y.o man presents with long hx of sx suggestive of GORD Gastroscopy = severe oesophagitis, nil stricture PPI commenced with complete relief of sx 6w later = furthe gastroscopy = metaplasia, nil dysplasia. Most appropriate next action is A. double PPI B. antireflux surgery C. repeat in 12m D. check autoimmune markers E. continue current rx

E.

Deltamed Haem 1 A 24‐year‐old woman with menorrhagia presented with fatigue. She had iron deficiency anaemia with a haemoglobin of 80g/L and MCV of 70fL. She was commenced on oral iron supplementation. 4 weeks later, a repeat blood test showed: Hb 86, plt 300, iron 8, transferrin 3 (normal), low transf sat 13%, ferritin 8 She reports compliance with oral iron supplementation despite the gastrointestinalside effects. What is the next most appropriate management plan? A. Test soluble transferrin receptor B. Perform a bone marrow biopsy C. Test haemoglobin electrophoresis D. Test serum transglutaminase antibody

D. The patient's iron deficiency persists despite oral iron supplementation. Soluble transferrin receptor, bone marrow biopsy or hepcidin will not assist in the management of her iron deficiency. They will confirm the results of the low ferritin. Provided a patient is adhering to oral iron supplementation, malabsorption of iron needs to be considered. Coeliac disease is a common cause for iron malabsorption.

Deltmed 7/1 A 41-year-old man presented with a 15-month history of fatigue, myalgias, diffuse bone pain, and tenderness. Pertinent laboratory studies revealed a serum calcium of 2.30 mmol/L (reference range 2.20-2.25), serum phosphate of 0.48 (reference range 0.78- 1.43), and 25-hydroxyvitamin of 60 nmol/L. Serum alkaline phosphatase was 318 U/L (reference range 41-119), and liver function tests were otherwise normal. His bone scan (C) and 68Ga-DOTATATE PET/CT are shown below: Lots of blastic lesions Which blood test is most likely to be diagnostic? A. Prostate specific antigen. B. Parathyroid hormone C. 1, 25-dihydroxyvitamin D D. Fibroblast growth factor 23

D. Tumour induced osteomalacia (small mesenchymal tumour secreting FGF23). Muscle weakness, low phosphate, high urine P, high ALP Ca, VitD, creat usually normal

Deltamed 3/1. Which of the following statements is most correct regarding X-linked dominant inheritance? A. Affected males are linked by unaffected females in the family B. There is no female to female transmission C. Affected males and females show the same phenotype D. Female carriers show a phenotype

D. XLD. F consistently shows phenotype. May be different to male as x2 Xs = milder. No male to male transmission

Deltamed 3/6 6. Drug X has a volume of distribution of 0.5L/kg, a fraction excreted unchanged of 60%, a half-life of 1 hour and is 80% protein bound. What is the clearance of Drug X in a 100kg man? A. 10mg/hr B. 2mg/hr C. 50L/hr D. 35L/hr

D. clearance = Vd x 0.693 _________ t 1/2 clearance = Cl (hepatic) + Cl (renal) + Cl (other)

Deltamed 6/5. A 42-year-old woman presents with a 2-month history of behaviour change and memory difficulties. She has been unable to continue in her usual work as a journalist, with insomnia, fatigue and difficulty processing cognitive tasks. She has been excessively and uncharacteristically anxious. In the last two weeks, she has had recurrent brief stereotyped episodes of staring with loss of contact, noted by her partner, of which she seems unaware, out to 5 per day. These last around 30 s with behavioural arrest, then return to normal function. Her MRI FLAIR is shown: temporal whiteness

D. paraneoplastic and autoimmune encephalitis anti-NMDA limbic encephalitis 50% have ovarian teratomas (may be bilat). CSF ab testing

Deltamed 6/7. At which level of the visual pathway is a lesion most likely to produce a symptomatic defect with binocular vision? A. Left retina B. Left optic nerve C. Optic Chiasm D. Left optic radiation

D. retrochiasmal - optic tract, optic radiations, occipital lobe'

Delta med 9/1 Which of the following is accurate about the positive predictive value of a screening test? A. If a screening test is of high sensitivity, it has high positive predictive value B. If a screening test is of high specificity, it has high positive predictive value C. A screening test of high specificity has high positive predictive value D. A screening test with high specificity can be of low positive predictive value

D. A screening test with high specificity can be of low positive predictive value. Sensitivity and specificity describe properties of the test. Predictive values depend on the prevalence of a disease within a population. Even with a highly specific test, if the disease is uncommon amongst those tested, a significant proportion of the positive results will still be false positives and therefore the positive predictive value will be low.

4. A 37-year-old man develops the rash shown, (widespread) on the day after completing a 5-day course of oral co-trimoxazole. Which of the following constitutes the most appropriate further investigation? A. Skin prick testing to Bactrim, with oral rechallenge with co-trimoxazole if negative B. Serum specific IgE (RAST) testing to Bactrim, with oral rechallenge with co-trimoxazole if negative C. Patch testing to Bactrim, with oral rechallenge with co-trimoxazole if negative D. No further investigation

D. Stevens-Johnson Syndrome T-cell mediated IgE testing (skin tests, RAST tests) no use Diagnosis made clinically ± biopsy • maybe a role for patch testing (research) MUST NOT rechallenge • irrespective of results of any test

Delta med 8/8. A 27-year-old non-smoking female with Crohns disease presented to a tertiary referral centre with fevers, shortness of breath and a non-productive cough that developed over 6 weeks. Medications included mesalazine 4.8g oral daily, azathioprine 100mg oral daily and after previously failing to respond to adalimumab, she was transitioned to infliximab infusions resulting in remission. On examination, she was febrile, chest auscultation revealed bronchial breath sounds in the right upper zone. CRP 147, WCC 16 What initial management should be instigated? A. Commence empiric antibiotic therapy and continue azathioprine and infliximab B. Commence empiric antibiotic therapy and cease mesalazine C. Commence empiric antibiotic therapy and cease azathioprine and infliximab D. Do not commence empiric antibiotic therapy and cease azathioprine and infliximab

D. Risk of atypical infections. Withhold IS until active infection treated/controlled

Deltamed 6/1 A 17-year-old male presents with upper limb weakness and has noticed he can't 'bulk up' in the same way his friends can. On examination, he has winging of the scapulae bilaterally, with markedly atrophic pectoral muscles, biceps and triceps. Deltoid and forearm muscles seem relatively spared. He has facial weakness which he had not registered, with difficulty closing eyes tightly, and unable to whistle or to smile broadly. There is no lower limb involvement. What is the most likely syndrome? A. Anterior horn cell B. Radiculopathy C. Polyneuropathy D. Myopathy

D. FSHD 3rd commonest MD after Duchenne, Becker + mytonic dystrophy. Face, scapular, Upper arm, lower leg, abdo weakness Often asymmetric If leg affected = foot drop. Onset in infancy to middle age. 90% have sx by age 20

Deltamed 4/5 A 73-year-old Chinese lady gave a two-month history of left leg swelling. 5 years earlier she had developed hypertension and was found to have bilateral renal obstruction; she was stented on both sides, treated with one month of steroids, and the stents were removed 6 months later. She was well until two months ago when she developed left leg swelling. An ultrasound and MRI of the left leg showed hypoechoic material involving the anterior aorta and progressing to the left common iliac vessels. The clinical diagnosis was confirmed by biopsy of the hypoechoic material. The biopsy might be expected to show an abundance of cells expressing which of the following immunoglobulin isotypes? A. IgG1 B. IgA C. IgG4 D. IgM

IgG4 Systemic disease manifested by infiltration of IgG4 lymphocytes/plasma cells into multiple tissues often with fibrotic/sclerotic lesions, polyclonal. Most = increased serum IgG4. Overlap with Sjogrens. Roid responsive. Autoimmune pancreatitis Mikulicz disease Kuttner tumour (chronic sclerosing sialeadenitis), interstitial nephritis, interstitial pneumonitis, sclerosing cholangitis, retroperitoneal fibrosis, mediastinal fibrosis, aortitis Obliterative phlebitis, storiform fibrosis

Deltamed HF lecture 26yo female previously fit, now 2 weeks "vaguely unwell". 2 days later develops PND. Presents to ED with acute pulmonary oedema, BP 85/50, HR 120/m, anuric, trop I 26ng/L (N <15ng/L). ECG is shown. saddle STE What is the most likely diagnosis? 1. Myocarditis 2. Acute myocardial infarction 3. Viral pericarditis 4. Hypertrophic cardiomyopathy

Myocarditis Biopsy showed lymphocytic myocarditis Acute myocardial infarction Demographic makes this less likely, history not typical Viral pericarditis highly unusual to become this unwell with viral pericarditis Hypertrophic cardiomyopathy Doesn't fit history

Deltamed 6/10 10. A 79-year-old man presents with onset while gardening of excruciating low back pain, worsening with transitioning between seated and standing positions. Radiologic evaluation shows a large osteophyte coming off the right anterolateral border of L5-S1. Which clinical sign is most likely to be abnormal in this setting? A. Absent ankle reflex B. Absent knee reflex C. Weak great toe dorsiflexion D. Weak hip flexion

NS: C Osteophytes at the L5-S1 junction can entrap the L5 nerve root. Great toe dorsiflexion is relatively specific to L5, which segment does not have an associated reflex: ankle reflex is S1, knee L4>L3. Hip flexion is L1/2

DeltaMed 1/5 5. Craig, 47, is retrieved by MICA from a construction site with crushing chest pain. His ECG is as below: He is sweaty and pale, with ongoing pain. His heart rate is 96 beats per minute and blood pressure 100/50. His chest is clear, and he has no murmurs auscultated. ECG - anteroinferior STEMI Which of the following would NOT be useful in his management prior to cath lab? A. Heparin infusion B. Ticagrelor C. Morphine D. GTN infusion

Occlusion of wrap around LAD (supplies ant + inf walls) ANS: D Inferior ST Elevation MI Inferior StEMI aspirin, P2Y12 (clopi/ticag), GP IIb/IIIa, anticoag, morphine (may reduce P2Y12) GTN if ongoing pain, useful in HTN or HF. Avoid if severe AS, PDE-I <24hrs, hypotension, RV infarction

Deltamed 3/4. Dorothy has an uncle (mother's brother) and a brother affected with Duchenne Muscular Dystrophy (DMD). Dorothy's daughter is pregnant. What is the risk to Dorothy's grandchild of developing DMD? A. 1 in 2 B. 1 in 4 C. 1 in 8 D. 1 in 16

X linked inheritance. Dorothy has 1/2 chance of being carrier, as does daughter (1//2). 1/2 chance baby is boy, and 1/2 of boys would be affected 1/2 x 1/2 x 1/2 x 1/2 = 1/16


Kaugnay na mga set ng pag-aaral

Pharmacology: Chapter 54: Drugs Acting on the Upper Respiratory Tract

View Set

Medial Femoral Muscles Table 6-5

View Set

9th Grade- Geography and History of the World: Africa

View Set

Medical Terminology-Surgical Procedures

View Set

Hematologic System Disorders and Drugs

View Set

Mech Eng 1761 - Final Review (Quiz Questions)

View Set